Flemings's Multistate Examination Workbook - Volume 2

Download as pdf or txt
Download as pdf or txt
You are on page 1of 470
At a glance
Powered by AI
The workbook provides sample questions and explanations for Civil Procedure, Constitutional Law, Criminal Procedure, Evidence, and Real Property to help students prepare for the Multistate Bar Exam.

The workbook covers 5 subjects: Civil Procedure, Constitutional Law, Criminal Procedure, Evidence, and Real Property.

The workbook provides over 500 sample multiple choice questions, explanatory answers, and test-taking techniques and methodology to help students learn how to effectively prepare for the Multistate Exam on their own pace.

LEARN TO

•••••♦ ♦••♦ ♦•••••♦ WRITE • • • ♦••••••••••••••♦


ITHE RIGHT IATAY; I

F7 1k9ifsi.giS
44,

MULTISTATE EXAMINATION
WORKBOOK
VOLUME II
A Collection of 536 Multiple Choice
Questions and Answers
for

CIVIL PROCEDURE
CONSTITUTIONAL LAW
CRIMINAL PROCEDURE
EVIDENCE
REAL PROPERTY
•••••••••••••••••••••••••••••••••••••••••••••
[FLEMING'S
FUNDAMENTALS OF LAW PL.!

MULTISTATE EXAMINATION
WORKBOOK
Volume II
ay
Jeff A. Fleming
Attorney at Law

Edited by
Susan P. Sneidmiller
Attorney at Law

Published by: FLEMING'S FUNDAMENTALS OF LAW


23166 Los Alisos Blvd., Ste. 238, Mission Viejo, CA 92691
CALIFORNIA TOLL FREE 1 (800) LAW EXAM
(714) 770-7030 • Fax (714) 454-8556

Distributed by: LEGAL BOOKS DISTRIBUTING


4247 Whiteside Street
Los Angeles, CA 90063
1 (800) 200-7110

All Rights Reserved (© 2005)


Copyright 1997, 2000, 2005 by Fleming's Fundamentals Of Law. Third Edition. All rights reserved. No part of this
book may be reproduced in any form or by any electronic or mechanical means, including photocopy, recording, or
any information storage and retrieval system, without permission from the publisher in writing. Printed in the U.S.A.
SUPERVISING WRITER AND EDITOR

Jeff A. Fleming
Attorney at Law

MANAGING WRITER AND EDITORS

Susan P. Sneidmiller
Attorney at Law

LAYOUT:

Donald F. Bayley II
IMAGEN Company Irvine, CA.

CONTRIBUTING WRITERS:

Professor Todd Brower


Standford University School Of Law, J.D.

Jarod Gross
University Of Santa Clara School Of Law, J.D.

Melody Lynne Jolly


Western State University College Of Law, J.D.

Professor Glenn Koppel


Harvard Law School, J.D.

Steve Liosi
Western State University College Of Law, J.D.

Professor Jeremy Miller


Tulane School Of Law, J.D.
CONTENTS - Volume II

THE RULES OF THE ROAD - An Introduction to the Multistate Method 3

CIVIL PROCEDURE - Question Breakdown 45


CIVIL PROCEDURE Questions 49
CIVIL PROCEDURE Answers 113

CONSTITUTIONAL LAW - Question Breakdown 149


CONSTITUTIONAL LAW Questions 153
CONSTITUTIONAL LAW Answers 191

CRIMINAL PROCEDURE - Question Breakdown 229


CRIMINAL PROCEDURE Questions 233
CRIMINAL PROCEDURE Answers 275

EVIDENCE - Question Breakdown 301


EVIDENCE Questions 305
EVIDENCE Answers 351

PROPERTY - Question Breakdown 379


PROPERTY Questions 383
PROPERTY Answers 441
RULE #1 — FAMILIARIZE YOURSELF WITH THE MBE TERRAIN

Material released by the NCBE indicates, in an almost lawyer-like disclaimer


fashion, that the MBE is:

1. Not a multiple-guess test, or a test of test-taking skills, but of legal skills and knowl-
edge, with a high correlation to performance on the essays;
2. Not a needlessly difficult, arcane or tricky test, but is designed to be a fair and un-
biased index of whether the applicant has the ability to practice law, regardless or
race or ethnicity;
3. Not a test in which time is a statistically significant factor, as the time allotted is
sufficient for 99 percent of the test takers;
4. Not inferior to essay and performance exams as a measure of minimum competency
to practice law, because the MBE can cover a greater breadth of subjects, can be
scored objectively and scaled to account for variations in difficulty from test to test;
and
5. Not getting easier from year to year.

Perhaps this last representation can go without dispute, but even the less gifted
advocates among us may, after a trip down the MBE road, be able to raise an is-
sue or two.

The landscape on the MBE road looks something like this:

A. TORTS (34 questions)

50% - Negligence, Proximate Cause, Damages (Apportionment), Defenses, Vicarious


Liablility
50% - Divided among remaining Tort areas

Questions here are based on the common law, the majority rule, and, increasingly,
on the Restatement 2d, Torts. The importance of Torts, and specifically of negli-
gence, cannot be overstated. The remaining emphasis is primarily on the intentional
torts and strict liability, with nuisance, defamation, privacy and misrepresentation
also being represented. You can count on at least one question from each of these
seven topics.

The specifications (i.e., subject matter outline) for the Torts MBE questions fea-
ture five main headings. Nuisance, defamation, privacy and misrepresentation
fall under a single heading of Other Torts. Products liability is segregated from

4 INTRODUCTION: THE RULES OF THE ROAD


D. REAL PROPERTY (33 questions)

75% - Interests in Land, Co-Tenancies, Landlord & Tenant, Ownership in Trusts,


Easements, Profits, Licenses, Covenants, Servitudes, Water and Land Rights,
Zoning, Fixtures (Article 9, U.C.C.), Adverse Possession, Deed, Conveyancing
by Will, Recording Act
25% - Land Sales Contract/Risk of Loss/Marketable Title/Mortgages

The common law, the modern law, the majority rule, and the U.C.C. (Article 9
— Fixtures) are tested in Real Property. The problems are, along with Contracts, the
longest and most complex on the MBE; unfortunately, most students would agree
that the concepts tested are anything but basic. Traditionally, the lowest percent-
ages come out of Real Property. On recent MBE's, there has been a significant
rise in testing on mortgages.

The 33 Real Property questions will draw on the areas of ownership, rights in
land and title for 75% of the questions, land sale contracts and mortgages for the
remainder. Newly tested subjects include zoning, conveyance by will and owner-
ship interests in trusts.

E. CONSTITUTIONAL LAW (33 questions)

50% State Action, Due Process, Equal Protection, Privileges and Immunities, Bill
of Attainder, Ex-Post Facto, Contract Clause, First Amendment (Speech,
Association, Press, and Religion)
17% - Federal/State Conflict
16% - Separation of Powers
16% - Procedure

The principles tested in Constitutional Law are derived from the U.S. Constitution,
the decisions of the U.S. Supreme Court, and, apparently, from obscure footnotes
in Nowak's hornbook on Constitutional Law. The good news is that the questions
in this area tend to be the most straight forward on the MBE, accounting for the
highest percentage scores.

The MBE includes 33 questions in Constitutional Law, approximately 50% of


which will test on "individual rights": due process, equal protection and first
amendment freedoms.

6 INTRODUCTION: THE RULES OF THE ROAD


F. EVIDENCE (33 questions)

33% - Hearsay
33% - Relevancy, Real and Demonstrative & Scientific Evidence, Authentication, Expert
Testimony, Privileges, Best Evidence Rule
33% - Impeachment, Rehabilitation, Opinion, Burdens and Presumptions, Judicial
Notice, Direct Examination, Cross-Examination

The questions arc based on the Federal Rules of Evidence. Throw out those old
outlines based on the common law, with one possible exception — From time to
time, you may see a federal court sitting in diversity jurisdiction. In those situa-
tions, apply state rules of procedure that may be outcome determinative, including
state rules of Evidence, as spelled out in the problem.

Note that if the questions testing on character evidence (from relevance and im-
peachment) are combined with the hearsay questions, the total is generally 60%
or more of the testing in Evidence. The questions tend to be the shortest, yet most
devious, on the MBE.

The MBE includes 33 questions in Evidence, one-third based on presentation of


evidence, one-third based on hearsay, and one-third based on relevancy, privileges
and writings.

The subjects of Crimes, Evidence and Torts tend to emphasize, and will likely con-
tinue to emphasize, the elements of the rule invoked by the facts; in Constitutional
Law, Contracts, and Real Property factual analysis and reading comprehension
is emphasized.

Unlike the essays, cross-over testing is the exception rather that the rule. This is
due, at least in part, to the fact that the questions are drafted by individual com-
mittees, each responsible for one of the MBE areas.

Although Civil Procedure is not considered a testable subject, a recent trend has
been to get to Torts by means of a motion for a directed verdict (known as a motion
for judgment as a matter of law), a motion to dismiss, or a motion for summary
judgment. For example, a question may require you to know that the doctrine of
res ipsa loquitur will raise an inference of negligence sufficient to turn the matter
over to the jury, and overcome a directed verdict motion.

INTRODUCTION: THE RULES OF THE ROAD 7


Prior MBE questions have tested the Erie doctrine in Constitutional Law, have used
a will or trust to get to an issue in Real Property, have used a federal court sitting
in diversity jurisdiction to get to an Evidence issue, or have used a contract to get
into a Real Property issue. Expect to see as many as three or four questions that
could cross-over between Evidence and Criminal Procedure. Remedies, includ-
ing damages and equitable relief, will arise in the context of in Contracts, Real
Property (e.g., specific performance), Torts, and, possibly, Constitutional Law
(e.g., injunctive relief).

It is highly unlikely that anything resembling a cross-over pattern will be seen on


the Baby Bar, as the examiner is unable to assume that you have any knowledge
beyond the Contracts, Crimes or Torts material covered in your first year of law
school.

8 INTRODUCTION: THE RULES OF THE ROAD


RULE #2 — PREPARE FOR THE LONG HAUL (or, You Have to Get out of the
Driveway before You Can Ride the MBE Fast Lane)

The foregoing subject breakdowns are to be used as study guides, along with
comprehensive outlines, such as the Gilbert series recommended by Fleming's
Fundamentals of Law in the MBE areas. It is recommended that one outline be
reviewed every one and one-half weeks for long term and review two outlines per
week for short term. Greater emphasis should be placed on any weak or highly
testable areas, combined with practice problems as discussed below.

The essentials of preparation for the MBE can be addressed in three words — prac-
tice, practice, practice.

In gearing up for the practice runs, the student must sort through a potentially
overwhelming assortment of materials. At the outset, a schedule should be pre-
pared that considers the outlines to be covered, as well as the practice problems
to be done. A series of goals, both long term (overall) and short term (daily), will
be helpful, but only if these goals are realistic and within reach.

A successful performance on the MBE requires more than the ability to regurgitate
rules of law and a rote memorization of the outlines. An examinee must be able to
master the rules., the exceptions, the exceptions to the exceptions, the footnotes in
the hornbooks, the Restatement comments and illustrations, the ability to function
in a stress-filled environment, the ability to think on one's feet, and the ability to
apply all of this in 1.8 minutes per question.

Unquestionably, success on the MBE is grounded on a solid foundation of the


black letter law. By itself, however, this is not enough. Each rule, as it is studied,
must be put into a factual context so that the application of the rule is understood
and the associative process is enhanced.

Generally, the commercial outlines, such as the Gilbert series, or the bar review
outlines, will be a sufficient source of the law needed. Despite all of our experi-
ence with last-minute cramming, merely reading the outlines will not be enough
to vanquish the MBE. The study of the law must be incorporated into an approach
that contemplates problem-solving as the goal, rather than rote memorization.

The recommended first step is to break down each subject area into its component
parts and to master each of these areas before moving on to the next. For example,

INTRODUCTION: THE RULES OF THE ROAD 9


one cannot learn Crimes without first mastering the general requirements of ac-
tus reus (the criminal act) and mens rea (the mental state) before moving on to
homicide and theft crimes.

While studying the outline materials, take the time to discern the necessary ele-
ments of each rule reviewed. List these elements in your mind, on a piece of paper
or on a flash card and then contextualize. This means putting the outline aside for
a moment to consider or imagine a set of facts that will call up the rule, element or
exception to be applied. For example, while reviewing contract offers, consider a
number of situations in which an offer may or may not exist, such as Do you want
to go to the movies? (merely an inquiry); If you pay for the movies, I'll pay for the
food (possibly an offer, despite its indefiniteness, if another would be justified in
believing that a power of acceptance has been created); or I'll pay you $5.00 an
hour to come over at 7:00 P.M. this Saturday and watch my kids while I go to the
movies (clearly an offer). If at first this seems difficult, there are a number of flash
card sets, such as those published by Law-in-a-Flash or PMBR that list examples
or hypotheticals that may prove helpful in helping initiate the process.

The next step is to turn to a source of practice questions, such as the Finz Multistate
Method book utilized by Fleming's Fundamentals of Law, starting with the table
of contents. You will see, for example, in the Criminal Law section, that following
the heading General Principles is a listing of 13 questions that test in that specific
area. Working questions in the subtopic immediately following its review will not
only help you place the rules in context, i.e., to apply the rules, it will also help
you to determine whether or not there is sufficient comprehension of the rules so
as to allow you to move on to the next area of testing. Again, it is vital to achieve
some sense of mastery in each area before moving to the next.

When taking the practice test questions, always time yourself, allowing no more
than 1.8 minutes per question. If you are aware of the time pressure from the very
beginning, then you will start to develop a rhythm or an almost instinctive aware-
ness of the passage of time as you work the problems.

10 INTRODUCTION: THE RULES OF THE ROAD


RULE #3 — KEEP YOUR EYES ON THE ROAD (or, the Zen of testing)

One of the attributes of the successful examinee is the ability to focus, to the exclu-
sion of all else, while in testing mode. This can be thought of as finding a quiet place,
both externally and internally, in which to perform. The distractions may be external,
such as the continued groans of a neighboring test taker; or the distractions may be
internal, such as wondering what impulse possessed you to sit for the exam.

The key is to block out everything but the test question, and to be able to do that for
an extended period of time. By doing test questions in blocks of no less than 17 at
a time, one half hour per set, you can consciously put yourself in the exam for that
period of time to the exclusion of all else. Consider continuing to practice the half
hour sets until you have mastered the ability to go for thirty minutes without think-
ing about anything but the test questions. Once you have mastered this ability, then
expand to one hour of testing, or 33 questions, and so on, until you can sit for three
hours without a significant break in concentration.

This does mean, however, that no matter how great the temptation, you cannot
look at the answer key until you have completed the set. When you do review the
answers, look at the explanations for both the correct and incorrect responses to
ensure a well-reasoned basis for your selections and to reinforce your grasp of the
rule being tested.

Some students will find it helpful to write out the rule on a flash card for those prob-
lems they got wrong. This, again, helps to reinforce your grasp of the rule, will help
you keep track of areas of difficulty, and will create a valuable study tool.

It is also important to sit for at least one simulated exam so as to test your ability
to focus over the long haul. Use the simulation as an opportunity to diagnose areas
of weakness, not only substantively, but technically or procedurally as well, i.e.,
Does the brain completely fade somewhere around question 75? Does it take 10 to
15 questions to establish a rhythm'? Does the brain think of nothing but lunch as the
clock nears the noon hour?

As the exam draws nearer, the emphasis should shill from outline review to problem
solving, with the outlines being used primarily as a refresher or supplement to the
problems. Again, the exam is testing on problem solving skills, not the ability to
regurgitate the black letter law. The final month should be devoted to doing practice
MBE's.

INTRODUCTION: THE RULES OF THE ROAD 11


RULE #4 — MAXIMIZE YOUR FUEL EFFICIENCY

As mentioned above, study time should be allotted to reflect the relative importance
of each subject area. Of the 200 questions on the current MBE, there are 34 each
in the areas of Contracts and Torts, and 33 each in the areas of Constitutional Law,
Criminal Law and Procedure, Evidence and Real Property.

Factor in the testing on the essays, where typically three out of the six one-hour
questions test on the same rules as the MBE, and that you cannot study law for
the performance essays, and the result is that at least 75% of the black letter law
needed to pass the California Bar Exam comes out of the MBE areas. This same
basic principle, that studying for the MBE will help on the essays, also applies
to the Baby Bar essay questions, but is limited to the topics of Contracts, Crimes
and Torts.

Emphasis in study time for the California Bar Exam should be placed on the MBE
areas. Generally, each MBE area should be studied three times as much as any
individual essay-only subject, such as Wills or Community Property.

As we saw in the subject breakdowns, above, within each MBE subject can be
found certain key areas, for example:

CONSTITUTIONAL LAW: .... Individual Rights And


The First Amendment 16 Questions

CONTRACTS: Formation (Including U.C.C.), Defenses ... 15 Questions

EVIDENCE: Hearsay And Character Evidence 15 Questions

TORTS: Negligence 17 Questions

PROPERTY . Establishing And Conveying Title 20 Questions

(Note: While Criminal Law has a somewhat emphasized area, the


general requirements and defenses will likely be tested directly
or indirectly on the bulk of questions, and should therefore be
considered key, both in study and in the review of the elements
of each criminal offense.)

12 INTRODUCTION: THE RULES OF THE ROAD


Obviously, the key areas require a greater expenditure of time. Again, the subject
breakdowns will be an important tool in emphasizing specific areas, and getting
the most efficient use of your study time.

INTRODUCTION: THE RULES OF THE ROAD 13


RULE #5 — KNOW WHERE YOU ARE HEADED (Before You Leave the
House)

In order to maximize your reading comprehension and initiate your analysis of


an MBE test question, it will help to know the area of testing, and, if possible, to
determine the rule of application, before you start to read the facts. The area being
tested will not be specifically identified for you.

The typical MBE question is composed of three parts — the root/stimulus (fact
pattern); the stem (call of the question); and the options (answer choices), as in
the following example:

Question 1 is based on the following facts:

Archie, a candidate for Student Body President at the exclusive


Riverdale Academy, was scheduled to debate the other presiden-
tial candidates before the assembled student body in the school's
auditorium. After the students had taken their seats, Ms. Grundy,
the faculty advisor, called Archie to the podium. Just as Archie was
crossing the stage to get to the podium, his opponent Reggie stuck
out his foot, causing Archie to fall off of the stage. As a result,
Archie was deeply humiliated, but suffered no physical harm.

1. If Archie asserts a claim against Reggie due to the humiliation


that he suffered, will Archie prevail?

A. Yes, if Reggie knew that Archie had to walk across the


stage in front of Reggie at the precise moment Reggie
stuck out his foot.

B. Yes, if Reggie carelessly failed to notice that Archie had


to walk across the stage in front of Reggie.

C. No, because Archie suffered no physical harm.

D. No, only if in sticking out his leg Reggie did not intend
to cause physical harm to Archie.

14 INTRODUCTION: THE RULES OF THE ROAD


The answer to this example can be found in the Rule #7 Section that follows.

Prior to reading the facts, read the stem. In the above example, the asserts a claim
language identified the area of testing — Torts. Now the root can be read with a
eye toward the appropriate cause of action. This is essentially the same as issue
spotting an essay. The facts are designed to trigger the application of a rule. The
examinee's job is to recognize those trigger facts and apply the appropriate rule.

Getting a sense of the ultimate destination helps to initiate the reasoning process
that goes into analyzing the facts. Again, this is the same process that goes into
issue spotting an essay, i.e., Do the facts support a particular outcome or result?
Have all elements been satisfied? Are there additional facts that must be evalu-
ated? Are there missing facts that must be considered before the outcome can be
determined? Is there an exception that will apply?

In the above Archie-Reggie example, once the area of law was established, we can
now consider the various rules which might apply, e.g., negligence, intentional tort
or strict liability. Which facts become significant? The facts state that as Archie
was crossing the stage, his opponent Reggie stuck out his foot. There is no men-
tion of whether or not it was intentional or accidental. It does not appear that any
of the categories of strict liability (products liability, wild animals, abnormally
dangerous activities) can be supported by the facts, so this area can be eliminated.
Negligence is possible, but given the lack of physical harm, we will need to con-
tinue the search for a more appropriate rule. The claim asserted will likely arise
out of the intentional torts, perhaps battery or intentional infliction of emotional
distress. The facts must be scrutinized to see if they will support the claim, or if
something more will be needed.

On occasion, the stem may go further in establishing the rule of application, e.g., if
Defendant is charged with murder..., or if Plaintiff asserts a claim in strict liability
due to the injuries caused by the defective product.... This may allow you to get
further along in the distillation process, but it does not mean that your analytical
skills will not be called into play.

In those problems where the root is followed by more than one stem, i.e., multiple
questions follow a single fact pattern, try to read each stern so as to get a general
sense of the issues or areas being tested. Later, when working through the responses,
you can deal with each stem independently.

INTRODUCTION: THE RULES OF THE ROAD 15


If the stem does not provide you with sufficient information to determine the area
of testing, glance at the options. This means allowing your eyes to drop down an
imaginary centerline through the potential responses, so that you recognize key
words. This should be enough to enable you to determine, at a minimum, the area
of testing. In the above example, you may have noted the words know and care-
lessly in the first two options, suggesting that the key, or correct answer, will be
coming out of intentional torts or negligence, and may possibly require a distinc-
tion to be made as to those two areas.

Consider the following example:

2. In a common-law jurisdiction, Defendant should be found


guilty of:

A. Burglary only

B. Arson only

C. Burglary and arson

D. Neither burglary nor arson

As you have likely observed, the stem did identify the rule of application, the
common-law rules. It did not identify the rule of law; however, a quick glance at
the options clarified the area (Criminal Law) and the crimes of burglary and arson.
Once you have identified the rule or rules of application, your job may merely be
the accounting of all essential elements.

In some examples, the stem essentially replaces or supplements the existing facts
by asking you to assume additional facts. In this situation, reading the entire stem
first may not be as desirable. It may be helpful to first read the last sentence or
phrase of the longer stem, so as to learn the objective of the problem, and then read
the additional facts contained in the stem. Consider the following example:

16 INTRODUCTION: THE RULES OF THE ROAD


Assume for the purposes of this question only that instead of
doing the job herself, Builder delegated the remodeling work to
Framington, who was an experienced cabinetmaker. Although
Framington had never worked on a job of this exact type, size and
complexity, she eigerly accepted the delegation because she owed
a sizable gaunbl in debt to Builder. Builder did not inform Owner
of the delegation. Builder then assigned the contract proceeds to
Karman to pay off her car loan.

Will Owner be required to accept Builder's delegation of du-


ties to Framington?

In this example. glancing at the last sentence identifies this stem as testing on a
delegation of contract duties. By having this knowledge, the important facts are
easily distinguished from those immaterial to the inquiry (e.g., that Framington
was inexperienced with the type of job versus Framington's experience as a cabi-
netmaker or the assignment to Karman). Note also that where the stem instructs
you to Assume for the purposes of this question only do not carry those additional
facts on to other questions and do not allow the new facts to influence your read-
ing of the basic fact pattern.

INTRODUCTION: THE RULES OF THE ROAD 17


RULE #6 — USE ROAD MAPS

Reading the stem, or glancing at the answers, to establish the objective or direc-
tion the problem is taking will help boost your reading comprehension. This can
be compared to using a road map to establish which direction to take in order to
reach your ultimate destination, i.e., the objective of the problem.

Reading comprehension is enhanced if you have a sense of direction, that is, an


idea of which facts are likely to be important and which facts are set forth as mere
distracters. Whole paragraphs may be skimmed if you know in advance that such
paragraphs will not have a bearing on the result. Parties are more easily identified,
issues are more easily narrowed.

Once the objective has been established, it becomes easier to recognize the details
that may be important in resolving the issues presented. For example, the fact that
the plaintiff was an anticipated or known trespasser, and thus a duty was owed,
may be determinative in Torts; or that the seller was an unemployed ballerina, and
thus not a merchant, may help resolve the issue as to whether or not a written op-
tion contract not supported by consideration is enforceable; or that the seller was
planning to sell, rather than presently intending to sell, may indicate that there is
no offer on which to base a contract.

As indicated above, a fact critical to the outcome may be buried within the details
of the fact pattern which, in essence, is identical to the essays. These details may
be seen in adjectives, verbs, adverbs, times, dates, places, quotations, personal
characteristics, names, or intentions. The question to ask is: What is the legal sig-
nificance of each fact or detail as it relates to our inquiry or call of the question?

In some problems, but not all, a simple, quickly executed diagram may be help-
ful to set out, for example, the chronology or the relationships that are central to
the question. This can be done right in the test booklet, or on the scratch paper
provided by the proctors. Consider the following example:

Question 4 is based on the following facts:

Buss is the owner in fee simple absolute of Lakeracre, a 13,000


acre ranch in Kern County, in the state of Golden. On May 2 ,

18 INTRODUCTION: THE RULES OF THE ROAD


despondent over the playoff loss of her beloved Magicians
basketball team to their arclirivals. the Missiles, Buss eon ON cd
Lakeracre to Johnson for the price of $150.000.00 by means of a
quitclaim deed. Johnson did not immediately record this convey-
ance. On June 16, Buss conveyed Lakeracre by warranty deed to
Nelson in consideration of $250.000.00 debt owed by Buss to
Nelson, which Nelson forgave. Nelson recorded this conveyance
immediately. On July l . Johnson recorded his quitclaim deed.

On July 10. Johnson conveyed Lakeracre to his friend Threatt


as a gift.

Threats recorded the conveyance immediately. On July 15, Nelson


conveyed Lakentcre to Peeler by quitclaim deed for the price of
$300.000.00. Peeler recorded immediately.

4. Assume for the purposes of this question only that Golden


has a recording statute that reads as follows:

No conoeyance is goad as against 0 subse-


quent purchaser Jar valuable considenaion
and without notice, unless such con vevoncc
be recorded prior to subsequent put chaYe.
-

If all necessary parties are joined in a quiet title action. title


will he found in Nkhich of the following of the following par
ties?

This problem can be diagrammed to strip the facts down to the essentials. The
key is to identify the bona fide purchasers. and when each recorded relative to
the other, so as to establish the absence or presence of notice in this race notice
jurisdiction. A diagram may look something like the following:

B to J/BFP B to N, N rec J rec J to T/gift, T rec. N to P/BFP, P rec

INTRODUCTION: THE RULES OF THE ROAD 19


Of course, the diagram can be distilled further by using arrows instead of to, or
abbreviating recorded to the letter R, so long as there is no confusion as to mean-
ing or a party.

The areas that are most likely to call on diagramming skills would be Contracts and
Real Property. It is recommended that you practice diagramming, if that technique
is to be employed, wherever appropriate in the practice questions. The objective
to distill the information into a useful format as quickly as possible, keeping in
mind that if you spend more than 1.8 minutes with each stem following the root,
that time must be made up elsewhere.

Some students may wish to highlight or underline the facts. This can either be
helpful, or a waste of time. In order to highlight effectively, there must still be
some awareness of the relative importance of the material highlighted. You may
recall how much material you highlighted in your casebooks during the first weeks
of law school. Soon, as you grew more sensitive to the relevant facts and law,
less and less was being highlighted. This was a product of learning how to read a
case. In much the same way, you will learn how to recognize the vital facts, and
disregard the immaterial.

In reading the root of the Archie-Reggie example, above, it may not be as impor-
tant to note that Archie was running for office as it was to note that the accident
occurred in front of the assembled student body, thus increasing the probability
that Archie would be humiliated. This is not to completely discount the Archie's
purpose for being in the auditorium, or that Reggie was his opponent. The fact
that Archie was there to participate in a debate makes it clear that the audience
would likely watch Archie's every move. The facts are not clear on Reggie's in-
tent in sticking out his leg; while it might appear that he intended to trip Archie,
we have to be careful in making an unwarranted assumption. In this example, we
appear to be missing a critical fact, and cannot make up for it with imagination.
The resolution of this issue is likely the key to the problem, and will be dealt with
in the answer choices.

It is important that you follow the facts and do not fight or respond emotionally to
the facts that are given. This means taking the facts at face value, without reading
too much into the facts. In this regard, you may need to stifle some of the natural
inclinations of an advocate, as arguing or assuming facts may do little except waste
time or lead to an incorrect response. Where the facts state that the arresting officer

20 INTRODUCTION: THE RULES OF THE ROAD


had a valid warrant, do not attempt to challenge the warrant. Where the facts state
that the police made a random stop, do not go on to consider the possible excep-
tions to the warrant requirement. Where the facts state that a deed is valid and has
been properly executed, there is no need to ask if the description is sufficient.

Some rules may seem unfair, perhaps even illogical, in applying the facts. For
example, Why shouldn't a physician be required to act to render assistance to an-
other who may be seriously injured as a result of the physician's inaction, where
there is no risk to the physician, and despite the fact that the victim was placed
in peril by another'? Why shouldn't a party he allowed to prove up a subsequent
remedial measure to prove negligence or culpability, when such remedy is logi-
cally connected to the dangerous condition causing injury? The MBE is not the
place to argue policy, and attempting to do so will exact a very high toll. Again,
do not respond with emotion, but rather a reasoned approach to applying the law
to the facts.

If a straight forward interpretation is available, do not over-analyze or seek out


an unnecessarily convoluted interpretation. A reasonable, logical inference may,
however, be required. Ensure that the inferred fact logically and necessarily fol-
lows from the information given. Making assumptions, creating new facts, or
working with conclusions that are not warranted by the existing facts will, in most
instances, generate an incorrect response.

Constitutional Law, Contracts and Real Property are where the subjects that are
more demanding on reading comprehension, given the nature of the subject matter
and the tendency that the examiners have to use longer, more complex fact patterns
with multiple parties to test in these areas. Fact patterns in these areas will often
require greater scrutiny, and therefore require more time.

Testing in Crimes, Evidence and Torts tends to emphasize elements and command
of the black letter law. As a result, the fact patterns tend to be shorter, and may
require less in the way of reading comprehension, and more in the way of substan-
tive knowledge of the rules of law.

When reading the facts, regardless of the subject, keep in mind that the examiner is
testing on lawyering skills. Cases very often turn on details, so an effective reading
of the facts will often depend on the examinee's ability to discern the operative
from the inoperative, the relevant from the immaterial, and the substantial from

INTRODUCTION: THE RULES OF THE ROAD 21


the inconsequential. Also, you must be as flexible and as adaptable as conditions
may demand. This is known as thinking on one's feet. These are qualities that lead
to a successful performance.

As with all the methods and techniques discussed in this introduction, including
the following materials on selecting a response, success is dependent on practic-
ing until the approach is virtually unconscious. Techniques must be adjusted to
fit each individual's approach, and that only comes through practice. You must
sort through each suggestion, and incorporate only that which has produced the
greatest result with the practice questions.

Arguably, if the work done in reading the facts and the call of the question has been
successful, then there should be little effort expended in reaching your destination,
i.e., selecting a correct answer. In the most basic terms, we are usually seeking the
best legal basis on which to support a particular or given outcome.

22 INTRODUCTION: THE RULES OF THE ROAD


RULE fI7 — DON'T GET SIDETRACKED

On occasion, the correct answer may appear obvious. Certainly, if the answer is
obvious, do not fight it. In other instances, however, it will seem as though the
examiner has taken great pains to hide the ball. The incorrect answers, referred to
as distracters by the examiners, are aptly named.

Consider the Archie-Reggie example (Question 1) above. It would appear that Reg-
gie acted intentionally to trip Archie: however, the facts say nothing about Reggie's
state of mind, and we cannot afford to guess at what Reggie was thinking. The
facts state that Archie was not injured, so the (B) response, based on negligence,
is improbable. The (C) response can be ruled out, because an action for battery or
intentional infliction of emotional distress can be maintained even in the absence
of actual physical harm. The (I)) response can also be eliminated, as it limits li-
ability to intent to cause physical harm; however, a cause of action can be based
on the offense to Archie's dignity' or the intent to cause emotional distress. The
(A) response, therefore, is the key. or correct response. By using the word know
it addresses the missing element at intent.

Typically, the person drafting the question is or has been a law professor. As a
result, he or she has ample experience with the phrasings that are likely to lure an
examinee into an incorrect response. There must be something attractive in a dis-
tracter. It may be an inaccurate or incomplete statement of the law. It may trigger
some faint memory of an arcane rule. It may be a concept that has been drilled into
you. Or, more likely. the distracter may reflect an incomplete understanding of a
rule that has traditionally led to confusion,

In the Archie-Reggie example, the examiner may have been counting on the dif-
ficulty some students may have in distinguishing negligence and the intentional
torts, or the requirement of physical harm that accompanies a negligence action.

One concept that routinely turns up in distracters is promissory estoppel. Those same
people who taught you to love and cherish the concept, law professors, now turn it
against you. In the absence of a promise and a reasonable expectation on the part
of the promisor that the promise will induce some action or inaction, the doctrine
does not apply. Yet students have routinely attempted to use promissory estoppel
as an all-purpose substitute for consideration, despite the absence of a promise, or
the presence of a more appropriate response that has very real consideration.

INTRODUCTION: THE RULES OF THE ROAD 23


In other situations, the distracter may prey on an examinee's confusion as to the
identity of the holder of a right, or when the right arises. In a problem where the
government is maintaining an action in inverse condemnation, an examinee must
realize the action is one properly maintained against the government. In a prob-
lem where a defendant seeks to assert the last clear chance doctrine, an examinee
must realize that only a plaintiff can assert the doctrine. Where there is a claim of
privilege against self-incrimination, be sure that there is a statement in issue, and
that the statement is testimonial in nature.

Other distracters may seek to create whole new, incorrect concepts. Only a student
with sufficient familiarity with the hearsay exceptions under the Federal Rules
will know that there is no exception for a recent sense impression. Bargained-for
reliance, a concept that withers in daylight, might get a second glance from the
examinee who is unsure of his or her knowledge of Contracts. Keep in mind that
after reading outlines in each of the subject areas, and doing thousands of practice
questions, if there was such a thing as bargained-for reliance you would recognize
the concept.

This is not intended to mean that you will necessarily be familiar with all potential
phrasings. In an area such as negligence we cannot expect to see 17 problems that
may be answered Liable, if negligent. Some synonyms are in order, such as reck-
lessly, carelessly, unreasonably, without due care, and so on. This does not mean,
on the other hand, that new, special duties can be created, e.g., in a negligence
action, the defendant owes the highest or an utmost duty of care.

Ultimately, the best cure is a thorough substantive review, followed by judicious


amounts of practice questions. Adequate preparation will thwart the intentions of
the examiner seeking to trap the unwary examinee.

24 INTRODUCTION: THE RULES OF THE ROAD


RULE #8 — REMEMBER TO READ THE TRAFFIC SIGNS

A clear understanding of the language in the stem and the options will be critical
to achieving a sense of direction, or, in other words, the objective of the problem
and arriving at the correct response.

The language in the stern may cast the examinee in a specific role, which in turn
will have a bearing on which response is the most appropriate. For example, a stem
that asks for the best hasis on which to support a judge's decision is asking you,
in essence, to act as that judge's law clerk. A question asking for the strongest or
weakest argument is calling for an advocate. On the other hand, a stem seeking
the most probable outcome may require a neutral approach in analyzing the most
appropriate response.

Consider the following example to determine how much information is contained


in the stern:

Question 5 is based on the following facts:

Dennis and Joey operate a private courier service in the city of


Metropolis. They entered into a contract with the law Iirrn of
Ketcham and s Wilson for the delivery of the firm's promotional
calendars to clients throughout the city. Prior to commencing the
deliveries. Dennis suggested to Joey that the two of them throw
away the calendars, pocket the money paid for the deliveries, and
spend the rest of day at the local race track. Joey said nothing.
Dennis then proceeded to dispose of the calendttrs.

. It Dennis and .loey arc charged with conspiracy to commit


larceny in a common law jurisdiction, which of the following
constitutes Joey's weakest defense'?

A. Joey did not have the required intent.

B. Joey did not form an agreement with Dennis, either ex-


plicitly or as to tt criminal objective.

INTRODUCTION: THE RULES OF THE ROAD 25


C. Joey did not act to take the calendars out of Ketcham and
Wilson's possession.

D. Joey was feigning agreement.

In the above example, the question stem is identifying the following: (1) the area
of law, (2) the specific rule being tested, (3) the law of the jurisdiction, (4) the
defendant to focus on in applying the rule, and (5) the role to assume in respond-
ing to the question. Actually, very little is needed from the root in this example.
The criminal charge has been set out, so in order to respond, apply the rule of
conspiracy.

The fact that the question seeks the weakest defense should not dramatically affect
your analysis. This question may be more easily solved by working it in reverse;
in other words, re-phrase the call to read Which of the following constitutes Joey's
strongest defenses? In this example, the (A), (B) and (D) responses are all negating
an element of conspiracy, i.e., the specific intent to commit larceny or the agree-
ment between two or more actors, and as such, are very strong defenses. Option
(C) appears to be dealing with the crime of larceny, and not the inchoate crime
of conspiracy, and would not be an adequate defense, is therefore the key in this
example. Note that in the areas of Crimes and Torts the strongest, best defenses
are those which negate an element of the charge or claim.

The same care must be used in analyzing the options. Consider the following
example:

Question 6 is based on the following facts:

Peters was skiing in the exclusive resort of Deer Park, an area


which he was not familiar with, and inadvertently skied out of
bounds. As he attempted to get back on the ski trail, a blizzard
started, making it impossible for him to find his way back. As
darkness descended, Peters started to fear that he would suffer
from hypothermia unless he found shelter. As he was making his

26 INTRODUCTION: THE RULES OF THE ROAD


way through the waist-high snow, Peters spotted a ski cabin owned
by Denver. Despite seeing a sign which read Private Property
— Keep Out!, Peters immediately inLide his way to the cabin and
was able to gain entry by forcing the door open. The cabin was
unoccupied.

Once inside the cabin, Peters started a fire in the fire place. Without
realizing there was a defect in the fire place screen which allowed
sparks to escape. Peters went to sleep. A tire broke out which
destroyed the cabin and caused serious injury to Peters.

6. If Peters asserts a claim against Denver for his injuries. will


he he successful?

A. Yes, if Den\ er was aware of the defective fire place


screen.

B. Yes, only if Denver could have discovered the defective


fire screen by a reasonable inspection.

C. No, unless Denver had reason to anticipate a skier might


break into the cabin.

D. No, because Peters assumed the risk that a lire might


break out v, hen he went to sleep.

In this example, notice that each option has a result, followed by a suggested basis
on which to support that result. This is the most common formulation of answers
on the MBE. Look closer, and you will notice that each outcome is joined to the
basis by a conjunction, which can he referred to as the modifier, or operative term.
Each different modifier has its own connotation or usage.

The modifier in option (A), if, suggests that the outcome must follow as a result
of the statement of law of fact given, and no other outcome is possible. The de-
termination that must be made is whether the logical connection is sufficient to
justify the result. In a problem where there is one missing element in order for the
plaintiff to prevail, and that missing element is preceded by an if modifier, then no

INTRODUCTION: THE RULES OF THE ROAD 27


other result is possible. This would be a very strong choice. The if modifier may
be followed by a new fact, in which case the inquiry is whether no other result
logically follows the new information. In the above example, we do not need to
question whether or not Denver actually knew of the defect. The inquiry is whether
or not Denver would necessarily be liable, given this awareness. Your knowledge
of Torts will tell you that absent some duty owed, this result does not necessarily
follow, and this answer may be rejected.

The only if modifier in option (B) suggests that the basis which follows the outcome
is the exclusive means of arriving at the outcome, and that there is no other way to
logically arrive at the outcome. The word only is a word of restriction or limitation.
If there is another way to arrive at the outcome, then this modifier would indicate
an incorrect response. In this example, the duty to inspect (owed to invitees) would
not be the exclusive basis on which to predicate Denver's liability to Peters, and
therefore must be rejected.

Option (C) features the unless modifier, which requires a reversal of the analysis
of the only if modifier in options (A). In other words, the outcome stated can only
be avoided by the statement which follows. The unless modifier should be treated
as the logical opposite of only if. Your knowledge of Torts will tell you that Denver
will prevail if Peters is unable to satisfy all of the elements of negligence. All of the
elements needed to find for Peters have been supplied except one — duty. Another
way to view this option is to rephrase it to Peters will recover only if Denver had
reason to anticipate Peters presence in the cabin. This logically follows, because,
based on the facts, duty to the trespassing Peters will only arise where Denver has
reason to know or anticipate his presence in the cabin; therefore, this is the key
response.

The modifier in option (D), because, can be used interchangeably with the conjunc-
tions since and as. Here the inquiry is two-fold. First examine the basis as a correct
statement of law or fact. If it is correct, then ask if the outcome stated logically and
necessarily follows from the basis. In this example, there is nothing in the facts
that indicates Peters voluntarily and unreasonably encountered the known risk of
a defective fire place screen. As a result of this, this option must be rejected.

The analysis of the language above may seem awkward, even somewhat convoluted
at first. After working through a substantial number of practice questions, you will
find that familiarity and comfort with the language increases at a rapid pace.

28 INTRODUCTION: THE RULES OF THE ROAD


RULE #9 — BE ALERT FOR CHANGING ROAD CONDITIONS

The three complete examples above followed a fairly standard format. The root
was followed by the stem. which in turn was followed by options which each
listed an outcome followed by a proposed basis on which to support the outcome.
The method used to arrive at the correct response in the explanations above was
the process of elimination. Once we eliminated the false statements, whatever
remained became our selection.

Another way to view the exam is as a multiple true-false test, rather than multiple
choice. That means looking at each option, independently, as a true-false question.
It will be helpful to mark a T or an F next to each option as you read through them.
Alternately, you might use a system of pluses or minuses, or crossing out the false
statements. Selecting the correct option then becomes a function of looking for a
T, a plus, or an option which has not been crossed out.

But what about the options with a question mark next to them? What if you can
only narrow it down to two choices?

If you are able to narrow it down to two choices, one of which is correct, you
are delinitely on the right track. You will have increased the odds of getting the
problem correct by l 00c,4. This is one reason why you may wish to indicate your
second choice as you work through the practice questions. As you read through
the explanatory answers, you will seek to develop a sense of why the examiner
prefers one choice over another.

Remember that the examiner, at least in the problem types we have reviewed so
far, is putting you on the hunt for the best legal basis on which to support a given
outcome. Consequently, the an sw er which is narrower or more specific to the result
is usually preferred; the same is true when comparing a legal basis with a factual
basis. The legal basis is generally preferred. For example, Not guilty, because
the facts do not state that Defendant had an improper motive does not present as
good a legal basis as Not guilty, because Defendant lacked the intent to steal in
exculpating the defendant in an example testing on larceny.

Be on the alert for those answer choices that are underinclusive or overinclusive
in stating the basis for the outcome. For example, the option stating Constitu-
tional, because the 1st Amendment only protects words is likely to be wrong in a
Constitutional Law problem, because certain acts of expressive conduct may be

INTRODUCTION: THE RULES OF THE ROAD 29


within the scope of 1st Amendment protection. If you find that the rationale in an
option eliminates a right which ought to be included, or encompasses a right or
result which does not follow from your understanding of the law, then such option
is to be avoided.

The examiners use a number of variations on the problem-types illustrated above,


such as where the result is the only thing contained in the option. Consider the
following example:

Question 7 is based on the following facts:

During the night, Dexter, a drug addict, broke into Gordon's


apartment, a reputed drug dealer, with the intention of stealing
Gordon's supply of rock cocaine. Although Dexter did not find any
cocaine, he did find some methedrine, an illegal stimulant. Dexter
took the methedrine with him when he left the apartment.

7. In a common law jurisdiction, Dexter is guilty of:

A. Burglary only

B. Larceny only

C. Burglary and larceny

D. No crime

Although the options are not the compound type that we have looked at, the ap-
proach is similar, in that we must still go through each possible response and deem
it either true or false. We do this by reviewing the elements of each charge, and
determining whether or not the facts will support the charge. Where there are mul-
tiple charges (or outcomes), deal with the elements of each individual charge.

In this example, we are in a common-law jurisdiction, and the facts state that
Dexter broke in to Gordon's apartment at nighttime with the intent to steal drugs.
We do not have to struggle with the elements of breaking and entering because

30 INTRODUCTION: THE RULES OF THE ROAD


they arc there in the facts. The examiner is hoping for an emotional response, that
somehow stealing illegal drugs is not morally wrong. But the facts make it clear
that Dexter had the intent to commit a larceny at the time of breaking and enter-
ing, therefore Dexter has committed burglary. He has also taken the drugs, which
are the personal property of another, with the intent to steal; therefore Dexter has
also committed larceny. and choice (C) is correct. Keep in mind that each crime
requires satisfaction of an element the other does not, so there is no double jeop-
ardy prohibition.

Another problem type incorporates the facts into the options themselves, which
may be referred to as a squib option format, as in the following example:
-

In which of the following fact situations will the contract


most likely be en forceable despite the absence of a written
agreement?

A. Homeowner and Cable Company have a dispute over the


placement of Cabie'S use of the utility poles on Homeown-
.

er's land. In an effort to settle the matter. Cable offers to


purchase an easement from Homeowner, and Ho m eowner
accepts.

B. Vendor operates vending machines throughout the city


of Sierra Visu.t. Supplier offers to sell S5,000.00 worth
of snack foods to Vendor for only $3,000.00. Vendor ac-
cepts.

C. Agent hears Singer perform in a nightclub and on the spot


offers her a two-year recording contract. In reliance on the
offer, Singer quits her job at the nightclub, and accepts
the offer.

D. Owner asks Builder to submit a bid for the construction


of addition to Ow ner's house. Builder offers to build the
addition for S10,000.00. Owner accepts.

INTRODUCTION: THE RULES OF THE ROAD 31


Problems of this type can generally be attacked with a two-step approach. Ask
yourself the following: What is the rule being tested (as identified by the stem)?
And, How is it to be applied, i.e., what is the task to perform? In this example,
the stem indicates that the rule being tested is the Statute of Frauds (requiring a
writing in order to enforce certain types of agreements), and the task is to classify
each contract to see whether it falls within the Statute. Options (A), (B), and (C)
each reflect an agreement that requires a writing under the Statute of Frauds — a
contract for the sale of an interest in land, a contract for the sale of goods for the
price of $500 or more, and a contract that is not to be performed within one year
from the making thereof, respectively. Only (D), involving a construction contract,
clearly falls outside of the Statute, and is the key response.

Answer choice (C) may present a problem for some students, as they might want
to argue the element of reliance. The examiners are counting on this, expecting
some students will waste time wanting to argue. If you have an arguable response,
versus a clear cut response, do not waste time. Answer the question, and then
completely erase it from your mind. One of the pitfalls of the exam is that certain
problems are so challenging that a student will continue to work it in the back of
the head long after a response has been marked. Resist this temptation, as it will
only slow you down, frustrate you and destroy your effectiveness.

Another problem type the examiners have used is the tiered question, as in the
following example:

Question 9 is based on the following facts:

Thanatos operates a factory in the city of Olympia which manu-


factures air conditioners. One of the ingredients used in the
manufacturing process is freon gas, which when exposed to the
welding torches necessary to the manufacturing process creates
the highly toxic phosgene gas. The phosgene gas is pulled out of
the inside of the factory by means of large exhaust fans, which
are vented directly to the atmosphere. No other means are avail-
able to Thanatos for the removal of the phosgene gas. Morpheus,
who lives next to the factory, inhaled the fumes and was severely
injured.

32 INTRODUCTION: THE RULES OF THE ROAD


. If Morpheus asserts a claim against Thanatos for her injuries.
which of the following must he established if she is to be
successful'?

Thanatos operated the factory in a negligent man-


ner.

The Calory constituted a public nuisance.

The toxic gas released by Thanatos caused her inju-


ries.

A. III only

B. I Lttid. III only

C. II . and III only

D. I, II. and 111

The most efficient, effective way to work this problem is to treat it in the same way
that we have already seen, that is, to treat it as a compound true-false question.
The stem is asking which of the following is an absolute requirement, regardless
of the cause of action. We can cross out or write an F next to statement I and state-
ment II, because Thanatos is involved in an abnormally dangerous activity giving
rise to strict liability. i.e., disbursing highly toxic gases into the atmosphere in a
populated area. Morpheus can therefore prevail even in the absence of proof of
negligence or nuisance. Option (A) is therefore correct.

Yet another problem type is the complex case precedent. In this problem type,
a series of case precedents are set out, with the stems being fact patterns, and the
options designating which of the precedents will be controlling, as in the follow-
ing example:

INTRODUCTION: THE RULES OF THE ROAD 33


Questions 10 -12 are based on the following facts:

Read'the summaries in the four cases (A— D) below. Then decide


which is the most applicable as a precedent to each of the cases
in the questions that follow, and indicate each choice by marking
the corresponding space on your answer sheet.

(A) Defendant, while in the process of robbing a liquor store,


dropped his revolver. The gun discharged, and the bullet struck
and killed a customer in the store. Defendant fled, but is later ap-
prehended and charged with murder. At trial, Defendant testifies
that the killing was accidental, and requests that an instruction
for manslaughter be given. Held: Guilty of murder.

(B) Defendant, angered over not being invited to her next-door


neighbor's housewarming party, discharged her gun into their
common wall while the party was ongoing. The bullet struck a
lighting fixture, and the shards of glass in turn struck and killed a
partygoer. Defendant was apprehended and charged with murder.
At trial, Defendant testifies that she never intended to harm anyone
and requests that an instruction for manslaughter be given. Held:
Guilty of murder.

(C) Defendant, while in the Mojave Desert region of Southern


California, decided to practice target shooting with her new rifle.
Taking aim at a cactus, she discharged a round. The bullet struck
a rock, ricocheted off of another rock, and struck and killed a
person standing 1.0 yards behind Defendant. Defendant is ap-
prehended and charged with murder. At trial, Defendant testifies
that she did not know that the victim was present, and requests
that an instruction for manslaughter be given. Held: Guilty of
manslaughter.

34 INTRODUCTION: THE RULES OF THE ROAD


(D) Defendant w as away on a two-week lecture tour. On her
return home. she discovered her husband having sex with her
sister. Defendant then removed a revolver from her briefcase
and fired it in the general direction of her husband and sister. The
sister is struck by the bullet and dies. Defendant is apprehended
and charged with murder. At trial, Defendant testifies that she
only intended to shoot her husband, that the shooting °flier sister
was accidental, and she only discharged the gun because she had
lost control and was unable to regain her composure. Defendant
also requested an instruction for manslaughter. Held: Guilty of
manslaughter

10. Defendant, intending to collect on an insurance claim, sets


lire to his witrehouse. The fire spreads to an adjoining build-
ing, which apparently had been abandoned several years
earlier. A vagrant who was living in this adjoining building
wits sleeping at the time and was overcome by the smoke.
As a result. the vagrant was unable to escape in time and
was killed in the ensuing blaze. A statute in the jurisdiction
defines arson as the malicious burning of any structure.

11. Defendant, while driving his car at the posted speed limit on
the Golden State Freeway, began to be perturbed at the close
proximity of the car, driven by Victim, immediately behind
him. In an effort to send a message to the tailgating driver,
Defendant abruptly slammed on his brakes. Victim attempted
to avoid the impact by swerving his car, but instead struck
the concrete center median barrier and Victim was killed.

12. Defendant and Victim were fellow employees ()FIDE a corn-


munici.itions software developer. One night the two of them
began to argue over which long distance telephone service
the company should choose. Victim then reached into her
desk drawer. and started to pull out something that appeared
metallic. Defendimt believed thin Victim was pulling out it
gun and was about to shoot her, so Defendant reached into
her purse and pulled out a gun and shot Victim. Victim, who
was merely taking out a letter opener. died.

INTRODUCTION: THE RULES OF THE ROAD 35


The temptation will be to match the facts in the stems with the facts in the prec-
edents, which will usually result in confusion and one or more incorrect answers.
In the above example, the facts in the question cases did not closely resemble the
precedents. The key is to distinguish each precedent as to the legal basis as it relates
to the result, use your knowledge of the law to determine the result in each stem,
and then match result for result. In the above example, this means identifying the
following:

A. Murder/Felony murder rule — robbery

B. Murder/Depraved heart murder

C. Manslaughter/involuntary — act of criminal recklessness

D. Manslaughter/voluntary — adequate provocation

Now, it becomes a task of matching result and basis with each of the ensuing fact
patterns. This means that the correct answer to Question 10 is (A), because the
accidental killing arose as a foreseeable consequence of the commission of an
inherently dangerous felony, arson. Question 11 was an example of involuntary
manslaughter involving criminal recklessness, and not depraved heart murder, be-
cause the necessary showing of the Defendant's awareness of the high probability
of death was not present; thus, option (C) is correct. Finally, Question 12, is an
example of voluntary manslaughter because of mistaken justification — Defendant
had a mistaken belief in an imminent threat of death or serious bodily injury. The
result must be matched with the result in one of the precedents; consequently,
option (D) is correct. Thankfully for many students, this question type does not
appear with frequency; when it does, usually in Criminal Law/Procedure, you may
recognize that with a workable approach, these questions will not seem overly
difficult.

In the supposition stem problem type, the stem will ask which of the facts con-
tained in the options, if true, will best support a particular result. The goal here is to
treat each of the options as true, combine it with the existing facts, and determine

36 INTRODUCTION: THE RULES OF THE ROAD


if it logically and necessarily leads to the desired outcome, as in the following
example:

Question 13 is based on the following fact situation:

On Mav 28. Pitcher and Dante. a dealer of vintage cars, entered


into a w ritten. signed agreemeat whereby, Dante ot L to give
Pilchcr 30 days in which to decide whether or not to purchase
Dante's rare BMW' 507 automobile for $100,000.00. Only 50
BMW 507s were imported to the United States, and only live are
currently known to exist. On June 1, Pitcher tendered a cashier's
cheek for $100,000.00. Dante refused to convey title to the auto-
mobile. Pitcher brings an action for specific performance.

13. Which of the following facts, if true, would provide Dante


with his strongest defense?

A. Pitcher did note give any form of consideration for the


option to purchase the BMW 507.

B. Pitcher had entered into an agreement with another dealer


for the purchLise of a different BMW 507 after May 28,
and had communicated that fact to Dante.

C. At the time of com►nencing the action, Pilcher knew that


another BMW 507 was available for S150,000.00.

D. Both parties thought the agreement was for the purchase


of DLinte's Mktserati

Again, as each option is examined, it is assumed that it is true. Option (A) does
not have legal significance, as the facts state that Dante was a dealer in this type of
automobile. Under the U.C.C, a merchant who gives assurance in a signed writ-
ing that an offer will not be revoked cannot later claim lack of consideration as a
defense to an option contract. Option ( B) appears to state that Pi Icher revoked his

INTRODUCTION: THE RULES OF THE ROAD 37


offer; however, this does not follow from the facts, as Pilcher is an offeree who
has not yet rejected Dante's offer. Option (C) cuts against the remedy of specific
performance, yet it is incomplete in that it does not state that the two automobiles
were alike in all practical respects. Option (D), although it flies in the face of the
facts, is the key response, because if mutual mistake is present, the contract is
necessarily voidable at the election of either party. Remember, in this problem
type each additional fact must be treated as true.

In the three-one split problem type, there will be one option that stands alone as
to result. Do not attach any special significance to this, as it probably has a one in
four chance of being correct. Work it through as you would any other example,
but pay special attention to the modifiers, particularly one that would affect the
result such as unless. If one option does not include a basis or rationale, select it
only if the facts provide sufficient support for the outcome. Consider the follow-
ing example:

Question 14 is based on the following fact situation.

Di vac, a skater with the Ice Follies, a traveling troupe of ice skat-
ers, stopped into the Fern Bar for a drink after performing one
evening. After having several drinks, Divac began to speak with
V lade, who was seated next to him at the bar. The topic turned
to the greatest hockey goalkeeper of all time, and the disagree-
ment was so fierce that Divac took a swing at Vlade. The contact
was slight, but it was enough to knock Vlade off of his bar stool.
Unbeknownst to Divac, Vlade was an undercover officer for the
federal Drug Enforcement Agency, and was at the Fern Bar in
order to make a drug buy.

Subsequently, Divac was arrested and charged with assaulting a


federal officer. At trial, Divac testified that he was unaware that
Vlade was a federal officer, and that if he had known he would
,

have never struck him.

38 INTRODUCTION: THE RULES OF THE ROAD


14. If the jury believes Divac, it should find him:

A. Guilty.

B. Not guilty, because he lacked the specific intent to assault


a federal officer.

C. Not guilty, unless assault of this type is considered a gen-


end intent crime.

D Not guilty, because Divac was mistaken as to Vlade's


identity.

In the above example, the only requirement as to mens rea is that Divac intended
the assault. Divac's awareness of Vlade as a federal officer is immaterial. No men-
tal awareness of the jurisdictional element of this type of offense is necessary to
convict. Note that option (B) can be eliminated on this basis. Option (C), which
has the modifier unless, can be eliminated on the basis that the facts indicate that
Divac intended the assault, and so the classification does not have a bearing on
guilt. Option (D) is incorrect, because mistake is no defense on these facts. Again,
only after eliminating all other choices can we properly arrive at the correct an-
swer, option (A).

Other problem types may appear from time to time. It is the ability to think on
your feet and apply the law to the facts in a lawyer-like fashion that will allow
you to vanquish each problem, and, ultimately, the MBE.

INTRODUCTION: THE RULES OF THE ROAD 39


RULE #10 — ENSURE THAT SPEED IS ADEQUATE TO PASS

Be aware that a number of problems do not appear to test your minimum compe-
tency to practice, but rather are designed to screw with your mind. As the exam
is a test against time, you cannot afford to get bogged down on any one problem.
If the answer proves elusive, mark down a response, and move on. If the facts
are lengthy, and there is only one problem, consider skipping it to conserve time,
especially if it is testing on arcane or obscure material. For example, if the prob-
lem has two full columns of facts, and a single question testing on planned unit
developments and the mutuality of equitable servitudes in Real Property, Burby
himself might skip it. Before moving on, though, glance at the options to see if
any can be eliminated as clear false statements of law, so as to improve the odds
of a guess.

Always be sure to mark down an answer so as to avoid skewing the Scantron. If


you have skewed the answer grid, say around problem 15, you cannot afford to
discover this as the proctor is calling time. You must always be aware of time and
speed, so that you do not run into the situation where no time remains, and yet
you are still trying to fill in the bubbles.

Avoid changing answers unless you specifically see where you have misread a
problem, or later remember the formulation of a rule that applies in a earlier prob-
lem; however, if you are still thinking about an earlier problem, the concentration
is not there, and speed is likely lagging. If you spend enough time practicing, your
first guess will probably be correct anyway.

Remember, no one gets them all, not even the law professors who take the exam.
In fact, the examiners will typically throw out two to three questions on each exam
due to ambiguity or inconsistency. Of the 200 questions on the bar exam MBE,
most examinees will get at least 100 correct. The key to success is getting the next
40 to 60 problems correct.

Always, always remain positive. Indicate to yourself that you are ready to pass by
doing the things that you, and only you, know you need to do.

Treat the exam like a game, and do it for the sake of the game. Don't think about
the result or the stakes, just enjoy the ride.

40 INTRODUCTION: THE RULES OF THE ROAD


THE RULES OF THE MBE ROAD, IN REVIEW:

1. Familiarize Yourself With The MBE Terrain

2. Prepare For The Long Haul

Keep Your Eyes On The Road

4. Maximize Your Fuel Eficiency

5. Know Where You Are Headed

6. Use Road Maps

7 Don't Get Sidetracked

K. Remember To Read The Traffic Signs

9. Be Alert For Changing Road Conditions

10. Ensure That Speed Is Adequate To Pass

Good luck on the MBE!

INTRODUCTION: THE RULES OF THE ROAD 41


42 INTRODUCTION: THE RULES OF THE ROAD
CIVIL PROCEDURE - QUESTION BREAKDOWN

1. Personal Jurisdiction Diversity of Citizenship

2. Personal Jurisdiction — Service of 15. Venue


Summons
16. Removal — Convenience of the Parties
3. Third Party Claim — Derivative Claim and Witnesses

4. Third Party Claim — Joinder of Claims 17. Personal Jurisdiction — Quasi-In-Rem

5. Subject Matter Jurisdiction — 18. Supplemental Jurisdiction


Diversity of Citizenship
19. Subject Matter Jurisdiction
6. Personal Jurisdiction — Minimum
Contacts 20. Impleader — Personal Jurisdiction

7. Personal Jurisdiction — Long Arm 21. Joinder of Claims


Statute
22. Personal Jurisdiction — Diversity of
8. Personal Jurisdiction — Quasi-In-Rem Citizenship
Jurisdiction
23. Supplemental Jurisdiction
9. Removal
24. Joinder — Impleader
10. Personal Jurisdiction — Minimum
Contacts 25. Joinder — Necessary and
Indispensable Parties
11. Personal Jurisdiction — Minimum
Contacts 26. Class Action — Adequate
Representation
12. Venue — Waiver
27. Class Action — Damages
13. Joinder
28. Discovery — Interrogatories
14. Subject Matter Jurisdiction —
29. Discovery — Interrogatories

CIVIL PROCEDURE 45
30. Discovery - Admissibility of Fairly Litigated
Deposition
48. Res Judicata - On the Merits
31. Discovery - Work Product
49. Res Judicata
32. Discovery - Witness
50. Collateral Estoppel - Mutuality Rule
33. Discovery - Production of Documents Exception

34. Class Action - Collateral Estoppel 51. Pleadings

35. Right to a Jury Trial 52. Discovery - Disclosures

36. General Appearance 53. Discovery - Experts

37. Pretrial Disposition - Voluntary 54. Discovery


Dismissal
55. Discovery - Witnesses
38. Pretrial Disposition - Summary
Judgment 56. Summary Judgment

39. Pleadings - Leave to Amend 57. Res Judicata

40. Attack on the Verdict 58. Summary Judgment

41. Jury 59. Summary Judgment

42. Appellate Review 60. Discovery - Witness Statement

43. Appellate Review 61. Discovery - Work Product

44. Joinder 62. Discovery

45. Pretrial Disposition - Summary 63. Discovery - Work Product


Judgment
64. Discovery - Work Product
46. Joinder - Cross-Claims
65. Subject Matter Jurisdiction -
47. Summary Judgment - Fully and Diversity of Citizenship

46 CIVIL PROCEDURE
66. Subject Matter Jurisdiction - 85. Subject Matter Jurisdiction -
Diversity of Citizenship Diversity of Citizenship

67. Supplemental Jurisdiction 86. Jurisdiction - Person/Subject

68. Impleader 87. Res Judicata

69. Supplemental Jurisdiction 88. Personal Jurisdiction - Due Process

70. Counterclaims 89. Subject Matter Jurisdiction

71. Counterclaims 90. Res Judicata

72. Subject Matter Jurisdiction - Federal 91. Collateral Estoppel


Question
92. Subject Matter Jurisdiction
73. Class Action
93. Subject Matter Jurisdiction
74. Class Action - Notice
94. Erie
75. Class Action - Representative
95. Res Judicata
76. Class Action - Notice
96. Collateral Estoppel
77. Discovery - Relevancy
97. Mutuality Of Estoppel
78. Discovery - Automatic Disclosure
98. Judgment As A Matter Of Law
79. Judgment As A Matter Of Law
99. Jurisdiction
80. Discovery - Physical Examination
100. New Trial Motion
81. Discovery - Physical Examination

82. Summary Judgment - Burden Of


Persuasion

83. Renewed Judgment As A Matter Of Law

84. Juror Misconduct

CIVIL PROCEDURE 47
48 CIVIL PROCEDURE
Q
CIVIL PROCEDURE 1. Mack served Dr. Feelgood pur-
suant to California's Long Arm
0 0
0<
-
QUESTIONS statute, which authorizes the
exercise of personal jurisdic-
tion to the extent permitted by
Questions 1 - 5 are based on the the Fourteenth Amendment. Dr.
following facts: Feelgood made no pre-answer
motions and, in his answer, de-
Mack is an interstate truck driver. He sud- nied the allegations in Mack's
denly fell ill while driving his rig through complaint. He also asserted the
Phoenix, Arizona, after returning from a affirmative defense that the court
delivery in Houston, Texas. He was on his lacked personal jurisdiction over
way to his home in Los Angeles. Mack him. Dr. Feelgood then moved
pulled off the interstate onto Main Street to dismiss for lack of personal
in downtown Phoenix and pulled into the jurisdiction. Which of the fol-
first doctor's office that he saw nearest to lowing is the most correct ruling
the interstate. It was Dr. Feelgood's office by the court on Dr. Feelgood's
on Main Street. Dr. Feelgood had a sign motion?
that was visible to motorists just after they
exited the off-ramp and entered Main Street. A. The court has personal jurisdic-
After being treated by Dr. Feelgood, Mack tion, because Dr. Feelgood failed
got back on the interstate and returned home to assert lack of personal juris-
to Los Angeles. When he arrived home, diction in a pre-answer motion;
Mack became violently ill and brought suit thus, he waived this defense.
against Dr. Feelgood for malpractice in fed-
eral district court in Los Angeles, claiming B. The court has personal jurisdic-
$100,000 in damages. tion because Dr. Feelgood did
not confine his response just to
attacking personal jurisdiction
but also addressed the merits of
the suit, which is deemed to be
a general appearance and, thus,
consented to personal jurisdic-
tion by Dr. Feelgood.

CIVIL PROCEDURE 49
C. The court does not have personal A. The court cannot exercise per-
jurisdiction over Dr. Feelgood sonal jurisdiction over Dr. Feel-
because he is just a local Phoenix good, because he did not pur-
doctor who intends to serve only posefully reach into California
the market in and around the to receive a benefit. He came to
neighborhood where his office California because his son hap-
is located. pened to be lying in a hospital
bed in California.
D. The court has personal jurisdic-
tion over Dr. Feelgood because B. The court cannot exercise per-
his location near an interstate sonal jurisdiction over Dr. Feel-
off-ramp, with a sign visible by good, because California has no
motorists after they exited the interest in adjudicating a dispute
off-ramp, constitutes purposeful arising out of medical services
availment of interstate business rendered in Arizona.
by Dr. Feelgood.
C. The court can exercise personal
2. Assume, for purposes of this question jurisdiction over Dr. Feelgood,
only, that the court granted Dr. Feel- because Dr. Feelgood was served
good's motion to dismiss in Question 1. with a summons in Los Ange-
Two months after Mack's federal court les.
suit was dismissed, Dr. Feelgood's son
was seriously injured in an automobile D. The court can exercise personal
accident while he was vacationing jurisdiction over Dr. Feelgood,
in southern California. Dr. Feelgood because California has an over-
rushed to Los Angeles to be at his son's whelming interest in opening its
bedside in the hospital. While at the courts to one of its citizens who
hospital, Mack had Dr. Feelgood served was injured by medical malprac-
with a summons in connection with tice.
a second suit (for malpractice claim-
ing $100,000 in damages) that Mack
had filed in the federal district court
in Los Angeles. Dr. Feelgood made a
pre-answer motion to dismiss on the
grounds that the court cannot assert
personal jurisdiction under the Due
Process Clause. Which of the following
is the best ruling on this motion?

50 CIVIL PROCEDURE
Q
3. Assume, for purposes of this question, C. The third-party action is im-
that the court denied Dr. Feelgood's proper, because Dr. Feelgood's
motion in Question 2. Dr. Feelgood complaint does not assert a de-
filed a timely answer denying all of rivative claim against Drugco.
the allegations in Mack's complaint.
Dr. Feelgood also served a summons D. The third-party action is proper,
and third-party complaint on Drugco, a because Dr. Feelgood's claim
pharmaceutical company incorporated against Drugco arises out of the
in California whose principal place of same transaction or occurrence
business is in California. Dr. Feelgood as Mack's main claim against Dr.
claimed that Mack became violently ill Feelgood.
upon his return to Los Angeles because
of a drug - Cyclopropane - which Dr. 4. Assume, for purposes of this question,
Feelgood administered to Mack. Dr. that the court granted Drugco's motion
Feelgood alleges that Cyclopropane is in Question 3. Dr. Feelgood then served
defective under the law of products li- another timely summons and third-
ability, and that Drugco is solely liable party complaint against Drugco making
to Mack. Drugco moved to dismiss the the following claims against Drugco: 1)
third-party action. Which of the fol- if Dr. Feelgood is held to be liable to
lowing is the best ruling on Drugco's Mack for having administered Cyclo-
motion? propane to Mack, then Drugco is liable
to Dr. Feelgood for all or part of Mack's
A. The third-party action should claim against Dr. Feelgood; and 2) a
be dismissed, because Drugco slander claim for $80,000 arising out
and Mack are both California of a statement by Drugco to a Phoenix
citizens, and Drugco's impleader newspaper reporter that Mack's injuries
destroyed diversity of citizen- were caused by Dr. Feelgood's lack of
ship. rudimentary sanitary practices in con-
nection with administering injections;
B. The third-party action is proper, and 3) a claim for a $60,000 unpaid
because Dr. Feelgood is assert- fee for Dr. Feelgood's participation in
ing a derivative claim against a testing program involving an experi-
Drugco, claiming that, if Dr. mental drug (Fixall) manufactured by
Feelgood is passively liable to Drugco. On Drugco's motion to dismiss
Mack for administering the drug all of these claims, which is the correct
Cyclopropane to Mack, then ruling?
Drugco, the active tortfeasor,
must indemnify Dr. Feelgood.

CIVIL PROCEDURE 51
Q
A. Claim 1 is proper, because it is a 5. Assume, for purposes of this question,
derivative claim against Drugco; that the court denied Drugco's motion
Claim 2 is proper because it in Question 3. It dismissed none of the
arises out of the same transaction three claims in the third-party com-
as Mack's main claim; Claim 3 plaint. On the fourth day of trial, the
is improper because it does not judge discovered that two weeks after
arise out of the same transaction Dr. Feelgood filed the second third-
as Mack's claim. party complaint with the court, Drugco
had changed its state of incorporation
B. Claim 1 is proper, because it is a from California to Arizona. The judge
derivative claim against Drugco, asks you, his law clerk, whether the
but Claims 2 and 3 are improper court can still exercise subject matter
because they are not derivative jurisdiction over claim 3 in the third-
claims against Drugco. party complaint. Which of the following
responses is most nearly correct?
C. Claim 1 is proper, because it is a
derivative claim against Drugco; A. Dismiss claim 3, because the
Claim 2 is proper under F.R.C.P. court cannot exercise supple-
18; but Claim 3 is not proper mental jurisdiction over that
under F.R.C.P. 18. claim since it does not relate to
either claim 1 or to Mack's main
D. Claim 1 is proper, because it is a claim.
derivative claim against Drugco;
and both Claims 2 and 3 are B. Do not dismiss claim 3, because
proper under F.R.C.P. 18. any subject matter jurisdiction
objection relating to this claim
is, by now, waived.

C. Do not dismiss claim 3, because


it was properly joined under Rule
18 and supplemental jurisdic-
tion is intended to support the
policies underlying the liberal
federal joinder rules.

52 CIVIL PROCEDURE
Q
D. Do not dismiss claim 3, because Questions 6 - 9 are based upon the o
citizenship of the parties, for following facts: <
diversity purposes, is determined cl
c
at the time the third-party com- Dave owns and operates a BurgerQuick
plaint was filed and, at that time, restaurant under a franchise agreement with
diversity was complete. the BurgerQuick Corporation. BurgerQuick
Corporation franchises fast-food restaurants
nationwide and advertises the BurgerQuick
trade-name nationally. BurgerQuick Cor-
poration is a New York corporation with its
headquarters in New York City. Although
Dave resides in San Diego, Dave's restaurant
is located in Santa Ana, California just off
the 17th Street exit ramp of the Interstate
5 freeway. Pat, a New Yorker, was touring
southern California and was en route from
Los Angeles to San Diego when she spotted
Dave's BurgerQuick sign from the freeway.
She pulled off the freeway, pulled into
Dave's restaurant, had a "Big Quick" with all
the fixings. She then proceeded on her way
to San Diego. When she reached San Diego,
Pat suffered severe stomach cramps. She at-
tributed the cramps to food poisoning from
the "Big Quick" she had consumed. When
she returned to New York City, she filed a
personal injury suit against Dave in a federal
district court in Manhattan for $80,000. Pat
served Dave under New York's Long Arm
statute. New York law requires a defendant
to enter a special appearance to challenge
personal jurisdiction.

CIVIL PROCEDURE 53
Q
6. Dave filed an answer denying that his D. The court can, under the Due
food caused injury to Pat. Dave also as- Process Clause, exercise person-
serted a defense of lack of personal ju- al jurisdiction over Dave because
risdiction. Dave then moved to dismiss Dave deliberately entered into
Pat's complaint for lack of personal a long-term franchise contract
jurisdiction (under the Due Process with a New York franchiser.
Clause of the U.S. Constitution). Which Further, Dave reaps the benefits
of the following is the most nearly cor- of national advertising by the
rect ruling? franchiser of the BurgerQuick
trade name.
A. The court can exercise personal
jurisdiction over Dave, since 7. Assume, for purposes of this question,
Dave has waived his lack of that Dave timely amended his motion
personal jurisdiction defense to add a second ground for challenging
because, under Erie, New York personal jurisdiction, namely, lack of
State's "special appearance" law authorization by the New York Long
governs the issue of how to chal- Arm Statute. The relevant section of
lenge personal jurisdiction in a the Long Arm Statute provides:
diversity case in federal court.
"As to a cause of action arising from
B. The court cannot, under the any of the acts enumerated in this sec-
Due Process Clause, exercise tion, a court may exercise personal
personal jurisdiction over Dave jurisdiction over any non-domiciliary
because Dave did not intend to who commits a tortious act within the
serve the national or New York state causing injury to person or prop-
market with his burgers, fries and erty within the state if he regularly does
shakes; Dave's contact with New or solicits business, or engages in any
York is merely unilateral. other persistent course of conduct, or
derives substantial revenue from goods
C. The court can, under the Due used or consumed or services rendered,
Process Clause, exercise per- in the state."
sonal jurisdiction over Dave
because Dave did intend to serve Which of the following are plausible
the national or New York market arguments for Dave to make under New
with his burgers, fries and shakes York's Long Arm statute?
by deliberately choosing to lo-
cate his restaurant alongside an I. Dave did not cause injury
interstate off-ramp with his sign to person or property
visible from the interstate. within New York State.

CIVIL PROCEDURE
II. Dave does not derive 8. Assume, for purposes of this question,
0 0
substantial revenue from that in addition to serving Dave with 0
m< -
goods used or consumed a summons pursuant to New York's 0 -
C.
or services rendered in Long Arm Statute, Pat, for the purpose
New York State. of securing quasi-in-rem jurisdiction,
had the sheriff seize a Certificate of
III. Dave does not regularly Deposit that Dave maintained in the
do or solicit business in Bank of New York in New York City.
New York State. Dave opened the account in New York
City because of the favorable interest
IV. Dave did not purpose- rates. Assume that New York has a
fully avail himself of the limited appearance statute observed by
benefits and protections the federal court. This statute allows
of New York law. a defendant whose New York prop-
erty has been seized for jurisdictional
V. Dave did not transact purposes to enter an appearance for
business within New the limited purpose of defending the
York State. property on the merits without being
deemed to have appeared generally. On
A. I, II and III only. a motion by Dave to dismiss for lack of
quasi-in-rem jurisdiction under the Due
B. I, H, III and V only. Process Clause, which of the following
arguments are not plausible under the
C. All of the above. Supreme Court's opinion in Shaffer v.
Heitner?
D. None of the above.
A. The court cannot exercise quasi-
in-rem jurisdiction, because
Dave's property is completely
unrelated to the cause of action
asserted by Pat.

B. The court can exercise quasi-in-


rem jurisdiction, because prop-
erty owned by Dave was seized
within the territory of New York
State.

CIVIL PROCEDURE 55
LLI
ce C. The court can exercise quasi- III. Removal is improper
w in-rem jurisdiction, because because Dave did not file

0 ce
0 Dave deliberately purchased a the petition for removal
cL. Certificate of Deposit in a New in the proper federal ju-
York bank and he can defend that dicial district.
property by entering a limited
appearance. A. I only.

D. The court cannot exercise quasi- B. II only.


in-rem jurisdiction, because
Dave has no other contacts with C. III only.
New York and the Certificate of
Deposit, viewed as Dave's only D. II and III only.
contact with New York, is not
indisputably and permanently
located in New York.

9. Assume, for purposes of this question,


that Pat had filed suit against Dave in a
California Superior Court in San Diego
(which, in the federal court system,
is located in the Southern District of
California). Dave filed a petition to
remove the action in the federal district
court in Santa Ana, which is located in
the Central District of California. On
motion by Pat to remand, which of the
following arguments are correct?

I. Removal is proper be-


cause the federal court
has diversity jurisdiction
over the action.

II. Removal is improper


because Dave is a citizen
of California.

56 CIVIL PROCEDURE
Q
- co
Questions 10 -14 are based upon 10. CompuChip made a pre-answer motion
00
the following facts: under F.R.C.P. 12(b)(2) to dismiss the <
claim against it for lack of personal c
Dirk owns a business in Fullerton. This jurisdiction under the Due Process
business assembles custom "special order" Clause. Which of the following is (are)
motorcycles. Paul, one of Dirk's customers a plausible argument(s) for either Paul
and a Nevadan, asked Dirk to put together a or CompuChip?
"souped-up" motor bike that would "really
burn rubber." Dirk knew of a "special" fuel I. As in the Asahi case,
injection system microchip that enables the whether or not Com-
motorcycle in which it is installed to accel- puChip purposefully
erate three times faster than commercially availed itself of the Cali-
manufactured bikes. The "special" micro- fornia market, it would
chip is not sold in the U.S. because it is il- be fundamentally un-
legal under the laws of most states, including reasonable and unfair
California. However, it is manufactured and for a court in California
sold only in Japan by CompuChip, Inc., a to exercise personal ju-
Japanese corporation. Dirk asked a friend of risdiction because the
his to buy ten "special" chips for him while interests of the plaintiff
in Tokyo on a business trip. Dirk's friend and the forum in the as-
purchased the chips and told CompuChip sertion of jurisdiction by
that they were going to be installed in mo- the federal court in Cali-
tor bikes in California. Aside from the one fornia is slight and the
sale to Dirk's friend, CompuChip has sold burden on CompuChip
no other "special" chips in the U.S. Dirk as- in defending thousands
sembled the motorcycle requested by Paul, of miles from Japan and
installing the "special" computer chip. Paul in a foreign legal system
was seriously injured in an accident with is great.
his new motorcycle. He brought a products
liability suit for $100,000 in damages against II. CompuChip did not pur-
Dirk and CompuChip, Inc. in federal district posefully avail itself of
court in Santa Ana, California, claiming that the California market
the special computer chip was defective. because it performed
Both Fullerton and Santa Ana are located no action that was pur-
within the Central District of California. posefully directed at the
forum state. Therefore,
the court cannot exer-
cise personal jurisdiction
over CompuChip.

CIVIL PROCEDURE 57
Q
III. CompuChip did purpose- 11. For purposes of this question only,
fully avail itself of the assume that the court granted Compu-
California market by Chip's pre-answer motion in Question
deliberately placing the 10. Which of the following additional
ten "special" computer facts would have arguably changed the
chips into the stream of result?
commerce when it rea-
sonably knew that such A. Although CompuChip has never
chips would flow to Cali- intentionally marketed its "spe-
fornia where they would cial" high acceleration com-
be installed as a compo- puter chips in the U.S., it does
nent in motorcycles to be regularly and deliberately market
sold in California. "ordinary" computer chips for
installation in fuel injection sys-
IV. Even though Compu- tems in motor cycles.
Chip did not purpose-
fully avail itself of the B. Evidence reveals that Compu-
California market, the Chip knew of the defect in the
court can exercise per- design of its "special" computer
sonal jurisdiction over chip, that there was a feasible,
CompuChip because it though more expensive, alter-
is fair and reasonable in native design, but CompuChip
light of the great interest deliberately refused to adopt the
to both the plaintiff and alternative design.
the forum state in having
Paul's claim adjudicated C. CompuChip has extensive real
in a California court. estate holdings in California for
investment purposes.
A. I only.
D. CompuChip entered into a con-
B. II only. tract with a California company
to design certain computer soft-
C. IV only. ware to go with CompuChip's
PC computers.
D. I and III only.

58 CIVIL PROCEDURE
12. For purposes of this question only, 13. Assume, for purposes of this ques-
assume that the court denied Compu- tion only, that CompuChip's motions
Chip's pre-answer motion in Question in Questions 10 and 12 were denied.
10. CompuChip subsequently moved Dirk timely amended his answer to
to dismiss Paul's claim against it on assert two claims against codefendant
grounds of improper venue. Which of CompuChip: (1) a claim for indemnity
the following rulings is most nearly in the event that a judgment is entered
correct? against both defendants, holding them
jointly and severally liable to Paul, and
A. Venue is proper, because both Paul collects the entire amount from
defendants reside in California. Dirk; and (2) a claim for $80,000 in
In addition, the computer chip in damages to Dirk's business, arising
question was incorporated into out of the failure of certain "ordinary"
Paul's motorcycle in Fullerton. computer chips purchased by Dirk
from CompuChip to perform as war-
B. Venue is improper, because ranted. CompuChip moved to dismiss
CompuChip resides in Japan and the second of these two claims. Which
Dirk resides in California. of the following rulings is most nearly
correct?
C. Venue is proper, because Paul
resides in Orange County. A. The second claim should be dis-
missed, because the court cannot
D. CompuChip has waived its venue assert supplemental jurisdiction
objection. over a claim that is neither re-
lated to Paul's main claim nor to
Dirk's proper cross-claim against
CompuChip for indemnity.

B. The second claim should be dis-


missed, because it is an improper
cross-claim. It does not arise out
of the same transaction as Paul's
main claim.

C. The second claim should not be


dismissed, because it is properly
joined. The first claim asserted
by Dirk against CompuChip is a
proper cross-claim.

CIVIL PROCEDURE 59
Q
D. The second claim should be dis- D. CompuChip waived its objection
missed on the grounds that the to subject matter jurisdiction,
court cannot exercise personal because it omitted that objec-
jurisdiction over CompuChip. tion from its pre-answer motion
before the district court.
14. Assume that Paul's lawsuit against Dirk
and CompuChip proceeded through
trial culminating in a judgment for
$40,000 in damages against both de-
fendants imposing upon them joint and
several liability for injury to Paul. For
the first time on appeal (to the Court
of Appeals), CompuChip attacked the
validity of the judgment on the grounds
that the district court lacked subject
matter jurisdiction. Which of the fol-
lowing rulings by the Court of Appeals
is most nearly correct?

A. CompuChip cannot challenge the


validity of the judgment against
it on grounds of lack of subject
matter jurisdiction, because
alienage jurisdiction supports
Paul's claim against Compu-
Chip.

B. The judgment against each


defendant is invalid for lack of
diversity jurisdiction, since the
damage award against each de-
fendant was only $30,000.

C. The judgment against each


defendant is invalid for lack of
diversity jurisdiction, since Paul
and Dirk are both Californians.

60 CIVIL PROCEDURE
Questions 15 -18 are based upon D. Venue is proper, because Doug
the following facts: could be found in the Southern
District of California and there
Pam, who lives in San Diego, brought suit was no district in which the ac-
in the federal district court for the Southern tion could otherwise have been
District of California in San Diego against brought.
Doug and Diana. Doug also resides in San
Diego, and Diana, a U.S. citizen, resides in 16. Assume, for purposes of this (and suc-
Istanbul, Turkey. Pam served Diana under ceeding) questions, that the court denied
California's Long Arm statute. Pam's com- Diana's venue motion in question 15.
plaint alleged that Doug and Diana conspired Diana then moved to transfer the suit to
in Istanbul to violate the Sherman Antitrust Turkey under Title 28, U.S.C. section
Act. Diana's answer raised the defense of 1404 "for the convenience of parties
improper venue. She subsequently moved and witnesses" and "in the interest of
to dismiss the suit on grounds of improper justice." Which of the following argu-
venue. ments for either plaintiff or defendant
is the least plausible?
15. Which of the following rulings is most
nearly correct? A. A Turkish court would be likely
to apply Turkish law and, al-
A. Diana has waived her venue though Turkey has a body of law
objection, because she failed to that is roughly the equivalent of
raise it in a pre-answer motion. U.S. antitrust law, it would be
somewhat more difficult for Pam
B. Venue is improper, because both to win a judgment against Diana
defendants do not reside in Cali- under substantive Turkish law.
fornia and a substantial part of Therefore, a Turkish court is not
the events giving rise to the claim a viable alternative forum and
did not occur in California. the suit against Diana cannot be
transferred.
C. Venue is proper, because both de-
fendants are subject to personal B. The action against Diana should
jurisdiction in the Southern Dis- be dismissed under forum non-
trict of California at the time that conveniens, because all the evi-
the action was commenced. dence is in Turkey and most of
the documents are in Turkish.

CIVIL PROCEDURE 61
Q
ce C. The action against Diana cannot 17. Diana also timely asserted the defense
— C3 be transferred from a U.S. court of lack of personal jurisdiction over her
>
Uv
0 to a Turkish court because the and moved to dismiss on that ground.
ce U.S. court has no authority in The court granted her motion and dis-
a.
Turkey. missed Pam's claim against Diana find-
ing insufficient contacts between Diana
D. The action against Diana should and California that relate to the antitrust
not be dismissed under forum claim. Pam, thereupon, located a bank
non-conveniens, because Pam's account maintained by Diana with the
claim against Doug would re- Bank of America in San Diego where
main in the federal court in Cali- Diana had deposited money she inher-
fornia thereby splitting the two ited from her Aunt Cecily. Diana has
related claims into two lawsuits made no other deposits or withdrawals
in two countries. from that account since the original
deposit of the inheritance money. Pam
attached Diana's bank account for
the purpose of securing quasi-in-rem
jurisdiction. Assume that California
offers Diana the right to enter a limited
appearance. Diana timely moved to
challenge the exercise of quasi-in-rem
jurisdiction, requesting the court again
to dismiss Pam's claim against her on
jurisdictional grounds. Which of the
following arguments for either plaintiff
or defendant is the least plausible?

A. It is fair to exercise quasi-in-


rem jurisdiction, because Diana
deliberately chose to open the ac-
count in a California bank. Any
judgment entered against Diana
will be limited to the money in
that account.

62 CIVIL PROCEDURE
<
'CP
B. It is unfair to exercise quasi-in- 18. Assume, for purposes of this question,
rem jurisdiction, because the that the court denied Diana's motion
O
sole contact between Diana and in Question 17. Two months later, Pam
California appears to be the bank amended her complaint to include ad- rrl
account. Pam's cause of action ditional claims against Doug and Diana
does not relate to that bank ac- under the state common law of business
count. torts (based upon the same conduct by
Doug underlying Paul's antitrust claim).
C. The court can exercise quasi-in- After two years of discovery, Doug and
rem jurisdiction based upon the Diana each moved for summary judg-
attachment of the bank account, ment to dismiss Pam's Sherman Act
because the bank account was claim. The court granted this. Doug then
seized within the territory of the moved to dismiss the state tort claim
State of California. against him on the ground that the court
lacks subject matter jurisdiction to hear
D. It is unfair to exercise quasi-in- that claim. Which of the following rul-
rem jurisdiction, because Diana ings is most nearly correct?
should not reasonably anticipate
that, as a consequence of opening A. The court lacks subject matter
a bank account in California, she jurisdiction over the state law
would have to defend an antitrust claim, because Pam and Doug
suit having nothing to do with are both California citizens.
California in a court located in
California. B. The court must exercise supple-
mental jurisdiction over the state
law claim against Doug.

C. The court may, in its discretion,


exercise supplemental jurisdic-
tion over the state law claim
against Doug.

D. The court cannot exercise sup-


plemental jurisdiction over the
state law claim against Doug
because the Sherman Antitrust
claim was dismissed.

CIVIL PROCEDURE 63
Questions 19 - 23 are based upon I. The fact that Pauline
the following facts: moved back to Califor-
nia three months after
Pauline was involved in an automobile col- filing suit and bought an
lision with Dan and Dotty in Las Vegas, expensive home there
Nevada on June 1, 1990. At the time of shows that Pauline was
the collision, Pauline, a native Californian, not a Nevadan at the
lived in Hollywood and was struggling to time she filed suit. Thus,
"make it in the movies." Dan, also a native diversity was not com-
Californian, lived in Fullerton, California. plete.
Dotty was a citizen of Nevada. Two weeks
after the collision, on June 14, 1990, Pauline II. The fact that Pauline and
terminated the lease on her Beverly Hills Dan were both Califor-
apartment and rented an apartment in Las nians at the time of the
Vegas, where she commenced working as a collision deprives the
blackjack dealer in a local casino. A month court of diversity juris-
later, on July 14, 1990, Pauline filed a per- diction.
sonal injury suit against Dan for $200,000
in federal district court in Los Angeles, and III. The fact that Pauline
served process on Dan a week later. Three terminated her Beverly
months after filing suit (October 14, 1990), Hills lease and was living
Pauline won $14 million in the Nevada State and working in Las Ve-
Lottery. She subsequently quit her casino gas at the time she filed
job, rented out her house in Las Vegas and suit shows that Pauline
moved back to Bel Air, California where she was a Nevadan at that
bought an expensive home. She started her time. Thus, diversity was
own movie company (so she could star in complete.
her own pictures.) She has remained there
ever since. IV. Because the collision
occurred in Nevada, Pau-
19. On November 1, 1990, Dan moved to line's cause of action has
dismiss Pauline's claims against him no substantial connection
on the grounds that the court lacked with California. Thus,
subject matter jurisdiction. Which of the the federal court in Los
following arguments are not correct? Angeles does not have
subject matter jurisdic-
tion over this claim.

A. I only.

64 CIVIL PROCEDURE
B. I and II only. B. The court cannot exercise per-
sonal jurisdiction over Dotty, a
C. I, II and III only. non-resident of California, be-
cause Dan's contribution claim
D. II and IV only. against Dotty arose out of Dot-
ty's driving a car in Nevada, not
20. Assume, for purposes of this question in California.
(and succeeding questions), that Dan's
motion in Question 19 was denied. C. Dan improperly impleaded
Dan subsequently served a timely Dotty, because it is solely the
third-party complaint and summons on prerogative of the plaintiff which
Dotty while she was visiting her aunt of the several joint tortfeasors to
for a day in Banning, California. Dan's sue.
third-party complaint stated a claim for
contribution among joint-tortfeasors, a D. Dotty was properly impleaded
state-law tort, alleging that Dotty was and is subject to personal juris-
50% at fault. In response, Dotty served diction, because she was served
on Dan a third-party answer in which with process in California.
she denied any responsibility for the
collision and also included a defense 21. Assume, for purposes of this and suc-
of lack of personal jurisdiction. Dotty ceeding questions, that Dotty's motion
then moved to dismiss the third-party in Question 20 was denied. Dan subse-
complaint. Which of the following rul- quently amended his third-party com-
ings is most nearly correct? plaint to assert a claim against Dotty
for $30,000 worth of property damage
A. Because the sole basis for subject to his car caused by Dotty's negligent
matter jurisdiction over Dan's driving. Dotty moved to dismiss this
third-party claim is diversity new claim. Which of the following rul-
jurisdiction, Erie requires that ings is most nearly correct?
the court apply California's state
court "special appearance" rule. A. Dan's second claim for property
Applying such rule, Dotty has damage is improperly impleaded
waived her defense of lack of because it is an independent, and
personal jurisdiction. not a contingent, claim.

B. Dan's second claim for property


damage is not supported by di-
versity jurisdiction because the
amount in controversy is only
$30,000.

CIVIL PROCEDURE 65
Q
C. Dan's second claim for property D. Pauline's claim against Dotty
damage is properly joined under is supported by supplemental
F.R.C.P. 18. Further, Dan's two jurisdiction, because her claim
claims together meet the amount against Dotty is sufficiently re-
in controversy requirement. lated to her claim against Dan.

D. Although Dan's second claim for 23. Assume, for purposes of this question,
property damage falls below the that Dotty's motion in Question 22 was
amount in controversy require- granted. Subsequently, Dotty asserted,
ment, the court can adjudicate in the appropriate manner, a negligence
the claim because of supplemen- claim in the amount of $40,000 directly
tal jurisdiction. against Pauline for personal injury
arising out of the same auto accident.
22. On August 1, 1991, Pauline amended Pauline moved to dismiss this claim.
her complaint to add a personal injury Which of the following rulings is most
claim for $200,000 directly against Dot- nearly correct?
ty. Dotty moved to dismiss this claim on
the ground that the court lacked subject A. Dotty's claim is supported by
matter jurisdiction. Which of the fol- supplemental jurisdiction, be-
lowing rulings is most nearly correct? cause this "Rule 14" counter-
claim is sufficiently related to
A. Pauline's claim against Dotty Pauline's claim against Dan.
should be dismissed, because at
the time Pauline filed her claim B. Dotty's claim is supported by
against Dan, Pauline, like Dotty, supplemental jurisdiction, be-
was a Nevada citizen. cause prior to the Judicial Im-
provements Act, case law ruled
B. Pauline's claim should be dis- that federal courts will assert
missed, because the California ancillary jurisdiction over "com-
court cannot exercise personal pulsory" counterclaims.
jurisdiction over Dotty who is a
Nevada citizen. C. Dotty's claim is not supported
by supplemental jurisdiction,
C. Pauline's claim is supported by because it would violate the
diversity jurisdiction, because principle of the Kroger case by
at the time Pauline amended opening the door to wholesale
her complaint, she had become, evasion of the complete diversity
once again, a California citizen requirement.
and Dotty remained a Nevada
citizen.

66 CIVIL PROCEDURE
D. Dotty's claim is supported by A. Impleader of the captains is prop-
00
diversity jurisdiction, because er, because the liability of the -
<
at the time Dotty asserted this captains arises out of the same -
C
claim, Pauline had become, operative facts as Shrimpboats,
once again, a California citizen Inc.'s refund claim against the
and Dotty remained a Nevada U.S. Government.
citizen.
B. Impleader of the captains is prop-
24. Shrimpboats, Inc. owns seven com- er, because either Shrimpboats,
mercial fishing vessels. Each of these Inc. or the captains are liable as
boats is operated by a captain and two "employers" for the crewmen's
crewmen. The I.R.S. assessed and employment taxes.
collected employment taxes for the
crewmen from Shrimpboats, Inc., tak- C. Impleader of the captains is
ing the position that Shrimpboats, Inc. improper, because the U.S.
was the "employer" of the crewmen. Government's claim against the
Shrimpboats, Inc. thereupon sued the captains is not a contingent or
U.S. Government for the refund of the derivative claim but, rather, an
employment taxes. The U.S. Govern- independent claim based upon
ment served a third-party complaint and the captains' liability to the U.S.
summons on each of the captains alleg- (under the tax code).
ing that, if Shrimpboats, Inc. was able
to prove that it was not the "employer" D. Impleader of the captains is
of the crewmen and, thus, not liable proper, because, otherwise, the
for their employment taxes, it would U.S. will be exposed to the risk
automatically follow that the captains of inconsistent judgments if it
were the "employers" of the crewmen has to assert its claim against
and would be liable to pay the U.S. the the captains in a separate lawsuit
very employment taxes the U.S. would with a separate trial.
have to refund to Shrimpboats, Inc. The
captains moved to dismiss the third-
party complaint against themselves.
Which of the following rulings is most
nearly correct?

CIVIL PROCEDURE 67
LU
ce 25. Pete underwent surgery in which a C. The motion should be granted,
> LU "plate and screw" device was implanted because it is prejudicial to the
00 in his lower spine. The surgical proce- defendants to have the two
0- dure caused Pete greater discomfort separate lawsuits being carried
than did his previous condition. The on simultaneously. Medco's de-
device was manufactured by Medco, fense will be that the plate was
Inc. The surgery was performed by Dr. not defective but that the doctor
Zorba at St. Nowhere Hospital. Pete and hospital were negligent. On
brought a state court suit against Dr. the other hand, the doctor's and
Zorba and the hospital for malpractice the hospital's defense will be that
and negligence. Pete also filed suit in they were not negligent, but that
federal district court against Medco, the plate was defective.
Inc. alleging defective design and
manufacture of the device. Medco filed D. The motion should be denied, be-
a motion: (1) to require Pete to join the cause plaintiffs have an absolute
doctor and the hospital as defendants right to decide whom to sue.
arguing that they are "necessary par-
ties" under F.R.C.P. 19(a); and (2) if
Pete is unable to join the doctor and
hospital, the court must dismiss Pete's
suit arguing that doctor and hospital are
"indispensable" parties under F.R.C.P.
19 (b). Which of the following rulings
is most nearly correct?

A. The motion should be denied,


because joint tortfeasors are not
"necessary" or "indispensable"
parties.

B. The motion should be granted,


because the public's interest in
judicial economy will be harmed
by multiple lawsuits where one
federal suit can resolve the con-
troversy.

68 CIVIL PROCEDURE
Q
Questions 26 - 34 are based upon C. Class representation by Pete
00
the following facts: would be inadequate, because -
<
he claims only $75,000 in dam- CV -
C
On June 1, 1992, a Global Airways jetliner, ages.
Flight 101, crashed at San Diego airport,
seriously injuring most of the 700 passengers D. Class representation would be
on board. On August 1, 1993, Pete, a partner inadequate, because Pete's serv-
with a large firm specializing in air-disaster ing as class counsel and as class
litigation, was one of the lucky passengers on representative would present a
board Flight 101 who suffered only a broken conflict of interest.
arm and a concussion. Pete commenced a
class action against Global in federal dis- 27. At the certification hearing, the court
trict court in San Diego claiming $75,000 must categorize Pete's class action as a
in individual damages and an aggregate of (b)(1)(A), (b)(1)(B), (b)(2) and/or (b)(3)
$40 million in damages for the class. Pilot class action. On this issue, which of the
error was alleged as the proximate cause of following classifications is most nearly
the crash. Pete, who himself has extensive correct?
experience in air-disaster litigation, named
himself as the class representative and hired A. This is a "prejudice" (b)(1)(A)
his law firm as counsel for the class. Pete class action because the pros-
headed up the litigation to prosecute the class ecution of individual damage
action against Global. suits by class members could
prejudice Global by imposing
26. At the class certification hearing on the on Global inconsistent results.
issue of the adequacy of class represen- Some plaintiffs could win and
tation by Pete, which of the following some could lose.
arguments is most nearly correct?
B. This is a "prejudice (b)(1)(B)
A. Class representation by Pete class action because some class
would be adequate, because Pete members who are the first to
has extensive experience as an bring their individual suits and
attorney in air-disaster litiga- who win large damage awards
tion. against Global could bankrupt
the airline leaving no funds re-
B. Class representation by Pete maining to pay later judgments
would be adequate, because in favor of other plaintiffs.
Pete's law firm gives the class
access to substantial financial
and clerical resources.

CIVIL PROCEDURE 69
C. This is an "injunctive" (b)(2) B. This interrogatory requires the
class action because Global has attorney for the class to evalu-
acted on grounds generally ap- ate facts in light of his legal
plicable to the class. It flew all knowledge and, therefore, is
class members as passengers on objectionable as calling for
the same plane, thereby making that attorney's mental impres-
appropriate declaratory relief sions which are protected under
with respect to the class as a F.R.C.P. 26(b)(3).
whole.
C. This interrogatory is objection-
D. This is a "damage" (b)(3) class able because it is overbroad.
action because all members of
the class claim to have been D. This interrogatory calls for the
injured by Global in a similar opinion of experts in air disas-
way. ters who have been employed
by the class to prepare the case
28. Assume, for purposes of this and suc- in anticipation of litigation and,
ceeding questions, that the court grant- therefore, calls for informa-
ed class certification. Global served tion protected under F.R.C.P.
on Pete, as class representative, a set 26(b)(4).
of interrogatories. One asked: "State
the factual basis for your contention in 29. To which of the following information
paragraph 3 of the class action com- is Pete not entitled under the mandatory
plaint that Global's pilot of Flight 101 ("required") disclosure rules?
flew the plane in a negligent manner."
Pete objected to this interrogatory and A. Any liability insurance policy
Global moved to compel an answer. held by Global for inspection and
Which of the following is the most cor- copying.
rect ruling on Global's motion?
B. The identity of any trial ex-
A. This interrogatory is allowed by perts.
F.R.C.P. 33.
C. The identity of each individual
likely to have discoverable in-
formation relevant to issues
alleged with particularity in the
pleadings.

D. The identity of each trial wit-


ness.

70 CIVIL PROCEDURE
Q
30. Pete examined the captain of the plane 31. Immediately after Flight 101 crashed,
by oral deposition. In response to a Global asked Prof, a professor of aero- 0
<
-
question asking whether he had been nautical engineering who lived near San c
drinking prior to take-off, the captain Diego Airport, to investigate the wreck-
replied: "Yes." Pete wants to know what age. Global sent Prof to the crash site
use he can make of this portion of the for the purpose of determining whether
captain's deposition transcript at trial. he would make a good trial witness.
Which of the following answers is most Prof spent two hours investigating the
nearly correct? wreckage and wrote a five-page report
concluding that pilot error was the sole
A. The deposition is inadmissible cause of the crash. Prof was paid an
for any purpose under the hear- agreed honorarium of $200 and was
say rule. never contacted by Global again. Pete,
as class representative, demands a copy
B. The deposition is admissible of Prof's report to Global through the
only to impeach the captain in appropriate method. Global objects
the instance that he testifies that and Pete moves to compel production
he had nothing to drink prior to of the report. Which of the following
take-off. rulings on Pete's motion is most nearly
correct?
C. The deposition is admissible
to help prove plaintiffs' case of A. The report should be produced,
negligence as well as to impeach only if Pete can show a substan-
the captain in the instance that tial need for it and an inability
he testifies that he had nothing to get the substantial equivalent
to drink prior to take-off. of the report's contents without
undue hardship.
D. The deposition is admissible to
conclusively establish that the B. The report should be produced,
captain had been drinking prior because its contents are relevant
to take-off thereby removing that to the subject matter and it was
issue from the case and prevent- not prepared in anticipation of
ing the captain from testifying to litigation.
the contrary.
C. The report should be produced,
only if Pete can show excep-
tional circumstances.

CIVIL PROCEDURE 71
LLI
ce D. The report should not be pro- A. Global's motion should be grant-
w duced under any circumstances, ed, but only if Pete can show
00 because Prof is an informally substantial need for Mack's de-
ce consulted expert. position and that Pete is unable
without undue hardship to obtain
32. Pete served a notice of deposition on the substantial equivalent of the
Mack, an employee of Global who information known to Mack by
works in Global's Accident Preven- other means.
tion Department. Mack has a degree
in mechanical engineering and was the B. Global's motion should be de-
person who first examined the wreck- nied, because Mack is an or-
age of Flight 101 immediately after the dinary percipient witness with
crash to determine the cause of the crash respect to the facts sought to be
as part of his accident prevention duties. discovered by Pete on deposi-
Two months later, Mack was assigned tion.
to Global's legal team as an expert
whose duties were to re-examine the C. Global's motion should be de-
wreckage to assist Global's attorneys nied, but only if Pete can show
in the preparation of Global's defense exceptional circumstances for
to Pete's lawsuit. Mack will not testify deposing Mack.
for Global at trial. Global moved for a
protective order that Mack's deposition D. Global's motion should be grant-
not take place. Global argued that Mack ed, because the facts Pete seeks
is a non-testifying expert who Global to discover on Mack's deposition
specially employed in preparation for are irrelevant to the subject mat-
trial and, therefore, Pete needs to make ter of the lawsuit.
a showing of "exceptional circumstanc-
es" under which it is impracticable for 33. Whit, a mechanic employed by the San
Pete to obtain Mack's facts and opinions Diego Airport, witnessed the crash of
by other means. Pete responds that he Flight 101. Knowing that Global would
wants to question Mack solely about likely be facing a lawsuit by the injured
the facts acquired by Mack during his passengers, Whit sent Global's lawyers
initial examination of the wreckage a signed note recounting what he had
immediately after the crash. Which of observed. Pete seeks to discover this
the following rulings is most nearly note and Global objects. On a motion by
correct? Pete to compel production of the note,
which of the following rulings is most
nearly correct?

72 CIVIL PROCEDURE
Q
- CP
A. The note is discoverable, because A. If the case law of the Federal
00
it is relevant to the subject mat- Court of Appeals for the Ninth -
<
ter. Circuit clearly forbids plaintiffs -
C
who opt out of a class actions
B. The note is not discoverable, from using collateral estoppel
because Whit's status as an ex- offensively but California state
pert mechanic protects the note, case law does allow such plain-
under F.R.C.P. 26(b)(4). tiffs to use collateral estoppel
offensively, the California court
C. The note is not discoverable, could not grant Opie's motion.
because it is a written witness
statement that is protected as B. The Due Process Clause forbids
ordinary work product. Opie from invoking collateral es-
toppel against Global in reliance
D. The note is not discoverable, upon the class action judgment.
because it is protected under the
attorney-client privilege. C. If the California state court
determined that Opie had been
34. The class action went to trial and the motivated to opt-out in order to
plaintiff class won. Judgment was en- "wait and see" whether the class
tered for the class. Opie had previously action turned out to be success-
opted-out of the class action. However, ful, the court would likely deny
in response to the class action notice Opie's motion.
that he had received from Pete, Opie
brought a personal injury suit against D. If the federal courts still applied a
Global in California superior court for strict mutuality of estoppel rule,
damages arising out of the air crash. the California court could not
After judgment had been entered for grant Opie's motion.
the class in the federal class action
suit, Opie moved for partial summary 35. A bill is pending before Congress to
judgment requesting the court to order deter sexual harassment in the work-
that, based upon the federal class action place. You clerk for a U.S. Senator and
judgment, it is conclusively established an issue has arisen whether juries or
in Opie's suit that Global was liable in judges should determine fact issues in
negligence to Opie (leaving only dam- trials involving lawsuits brought under
ages to be determined). Which of the the new Act. Which of the following
following statements is not correct? versions of the statute will violate the
Seventh Amendment?

CIVIL PROCEDURE 73
Q
I. The statute creates a new IV. The statute creates a com-
private right of action prehensive federal regu-
for sexual harassment, latory program intended
allows both injunctive to stop sexual harass-
relief and damages, and ment in the workplace
provides that all issues and permits the federal
of fact will be tried by a government to bring suit
judge without regard to against violators before
the remedy sought. the federal Sexual Ha-
rassment Agency seek-
II. The statute creates a new ing the imposition of
private right of action fines. All actions brought
for sexual harassment, under the statute shall
allows both injunctive be tried before the fed-
relief and damages, and eral Sexual Harassment
provides that all ques- Agency.
tions of fact shall be
tried by a jury without A. I only.
regard to the remedy
demanded. B. II only.

III. The statute creates a new C. II and III only.


private right of action
for sexual harassment, D. II, III and IV only.
allows both injunctive
relief and damages, and
is silent on the issue of
who shall be the trier of
fact, judge or jury.

74 CIVIL PROCEDURE
-v
Questions 36 - 43 are based upon D. The default judgment should not
0
the following facts. be set aside, because Sporter's
m-
o <
conduct clearly indicated his C7-
C. 1-
On April 1, 1992, Hack, an unsuccessful intent to avoid complying with
and unknown composer of popular songs, orderly judicial procedures.
brought a copyright infringement suit in
federal court against the world-famous 37. Assume, for purposes of this and suc-
composer, Cole Sporter. Hack alleged that ceeding questions, that the court set
many of Sporter's most famous songs - Be- aside the default judgment and that
gin the Levine, My Heart Belongs to Laddie Sporter served an answer to the com-
and DeLightful DeLorian - were copied by plaint denying the operative allegations
Sporter from Hack's tunes. of the complaint. Shortly thereafter,
Hack filed a notice of dismissal and
36. Following the service of the sum- Sporter moved to quash and vacate
mons and complaint, and at Hack's request, the notice of dismissal. Which of the
Sporter engaged in settlement discussions following statements is most nearly
with Hack for six weeks before Hack, with- correct?
out indicating any previous intent to do so,
filed with the court a motion for default A. The motion should be denied, be-
judgment. The court granted Hack's motion. cause Sporter had not yet moved
Sporter then moved to set aside the default for summary judgment.
judgment under F.R.C.P. 60(b). Which of the
following rulings is most nearly correct? B. The motion should be granted,
because Sporter had previously
A. The default judgment should answered the complaint.
be set aside, because Sporter's
act of entering into settlement C. The motion should be denied,
negotiations shows that he was because the action had not pro-
not in default. ceeded to the point where Sporter
would be prejudiced.
B. The default judgment should
be set aside, because Sporter's D. The motion should be granted,
act of entering into settlement because plaintiffs never have
negotiations constitutes an ap- an absolute right to voluntarily
pearance by Sporter. dismiss their own actions.

C. The default judgment should not


be set aside, because Hack prop-
erly sought the default judgment
before the judge rather than the
clerk.

CIVIL PROCEDURE 75
Q
38. Assume, for purposes of this and suc- 39. Assume, for purposes of this and
ceeding questions, that the court granted succeeding questions, that the court
Sporter's motion to quash in question denied Sporter's motion for summary
37. Sporter moved for summary judg- judgment. The court held a pre-trial
ment. He attached to his moving papers conference. The pre-trial order that
the sheet music of Sporter's songs resulted from the conference specified
that he allegedly copied and Hack's copyright infringement under the Fed-
songs from which Sporter allegedly eral Copyright Act as Hack's only cause
copied. Hack responded by submitting of action. On the third day of trial, Hack
his complaint. The court believes that wanted to amend the pre-trial order to
Hack's songs and Sporter's songs are add certain state common law claims.
so dissimilar that they do not indicate Which of the following statements is
copying by Sporter. Which of the fol- most nearly correct?
lowing rulings is most nearly correct?
A. Hack has a right to amend the
A. The motion should be granted, pre-trial order to add another
because the songs are so dis- related cause of action.
similar that the court does not
believe they indicate copying by B. Hack may amend the pre-trial
Sporter. order provided he can show
exceptional circumstances for
B. The motion should be denied, doing so.
because Sporter did not meet his
movant's burden. C. Leave of court to allow plaintiffs
to amend the pre-trial order shall,
C. The motion should be granted, as a matter of policy, be freely
because Hack did not meet his granted.
respondent's burden.
D. Leave of court to amend the pre-
D. The motion should be granted, trial order shall be granted only
because the absence of affidavits to prevent manifest injustice.
or deposition testimony means
there are no credibility issues to
resolve.

76 CIVIL PROCEDURE
-D
40. Assume, for purposes of this and suc- C. The court should grant the mo-
00
ceeding questions, that the court did not tion, because Hack's testimony is o
m<-
approve amendment of the pleadings not credible as a matter of law. -
c
to add state common law claims. The
case went to trial. An essential element D. The court should deny the mo-
of Hack's infringement case is to show tion, because the jury should
that Sporter had access to Hack's songs. have the opportunity to deter-
Hack's sole evidence on this element mine the credibility of Hack's
was Hack's own testimony that Sporter testimony.
broke into his house on the night of June
15, 1989 and took pictures of Hack's 41. Assume, for purposes of this and suc-
musical manuscripts with a special ceeding questions, that the court denied
"spy" camera. In rebuttal, Sporter testi- Sporter's motion in Question 40. The
fied that he was 1000 miles away from jury rendered a verdict in favor of
Hack's home on the night of June 15, Sporter. Hack moved for a new trial.
1989. In addition, four disinterested Hack submitted the affidavits of three
witnesses each testified for Sporter that jurors each stating: (1) that each had
they had heard from someone with per- observed that Hack had shifted uncom-
sonal knowledge that Sporter had been fortably in his seat at plaintiff's counsel
1,000 miles away from Hack's house table throughout the trial; and (2) that
on the night in question. At the close Hack looked scared— which caused
of all the evidence, Sporter moved for each of the three jurors to believe that
a judgment as a matter of law. Which Hack was lying on the stand. Which
of the following rulings is most nearly of the following rulings is most nearly
correct? correct?

A. The court should grant the mo- A. The motion should be denied,
tion, because Sporter's case, because, under F.R.E. 606(b), no
consisting of corroborated and juror may testify to the effect of
disinterested testimony, over- anything upon his or any other
whelms Hack's weak case on the juror's mind.
issue of access.
B. The motion should be granted,
B. The court should deny the mo- because what the jurors observed
tion, because Sporter did not in the courtroom amounted to ex-
make the motion before the close traneous prejudicial information
of Hack's case. improperly brought to the jury's
attention.

CIVIL PROCEDURE 77
Q
LLI
C. The motion should be denied, be- D. The objection is reviewable,
LLI cause the jurors' three affidavits because this is an "exceptional
0 contained no evidence of jury circumstance" where justice
0
ce
o_ misconduct. might otherwise not result.

D. The motion should be denied, 43. On Hack's appeal, referred to in Ques-


because even if the jurors had tion 42, Sporter, as appellee, asked the
engaged in jury misconduct, that Court of Appeals to reverse the district
misconduct could not be deemed court's denial of Sporter's request for
prejudicial. attorney's fees. Which of the following
rulings by the Court of Appeals is most
42. Assume, for purposes of this and suc- nearly correct?
ceeding questions, that Hack's new trial
motion was denied. Hack appealed rais- A. The error is not reviewable,
ing for the first time the objection to the because Sporter is only an ap-
admission into evidence of the hearsay pellee.
testimony of the "four disinterested
witnesses" referred to in Question 40. B. The error is reviewable, because
Which of the following rulings by the Sporter had previously raised the
Court of Appeals is most nearly cor- error in the district court.
rect?
C. The error is reviewable, because
A. The objection is not reviewable, Sporter is not attacking the trial
because Hack did not raise it in judge on the underlying merits
the trial court. of the court's judgment in favor
of Sporter (on Hack's copyright
B. The objection is not reviewable, claim).
because the trial judge commit-
ted the error during the middle of D. The error is not reviewable, be-
the trial; therefore, it is interlocu- cause it is harmless error.
tory.

C. The objection is reviewable,


because if the testimony was
hearsay, it could have prejudiced
the verdict.

78 CIVIL PROCEDURE
Questions 44 - 47 are based upon A. If Pat v. Dan and Dave had been
the following facts: brought in a state court following
the traditional common law ap-
Pat, a Californian, Dan, a Nevadan, and proach to counterclaims, but Dan
Dave, an Arizonan, were involved in a v. Pat had been brought in fed-
three-way collision. On January 1, 1990, Pat eral court, Pat's summary judg-
sued Dan and Dave for negligence demand- ment motion should be granted
ing $55,000 in property damage to her car. because Dan's claim arises out
Dan and Dave each denied the allegations of the same transaction or oc-
in Pat's complaint and Dan and Dave each currence as Pat's claim against
asserted Pat's contributory negligence as Dan.
an affirmative defense to Pat's claims. Dan
cross-claimed against Dave for property B. If Pat v. Dan and Dave had been
damage in the amount of $500. Dave as- brought in a state court fol-
serted contributory negligence as an affirma- lowing the traditional common
tive defense to Dan's cross-claim. Assume, law approach to counterclaims,
for purposes of the foregoing questions, that Pat's summary judgment mo-
contributory negligence is an absolute bar tion should be granted because
to recovery. Dan's claim arises out of the
same transaction or occurrence
44. After a trial, the jury rendered a gen- as Pat's claim against Dan.
eral verdict for Dan and Dave on Pat's
claims, and a general verdict for Dan C. If Pat v. Dan and Dave had been
on Dan's cross-claim against Dave. brought in a state court fol-
Judgment was then entered on each of lowing the traditional common
these verdicts on June 1, 1990. Dan then law approach to counterclaims,
brought a negligence suit against Pat Pat's summary judgment motion
for personal injury claiming $5,000 in should be granted because Dan's
damages. Pat's answer asserted Dan's claim arises out of the same set
contributory negligence against Pat as of operative facts as Dan's con-
an affirmative defense to Dan's claim. tributory negligence affirmative
Pat moved for summary judgment to defense in Pat v. Dan.
dismiss Dan's claim relying on the June
1, 1990 judgment in Pat v. Dan and D. If Pat v. Dan and Dave had been
Dave. Under the traditional common brought in a state court fol-
law, which of the following rulings is lowing the traditional common
correct? law approach to counterclaims,
Pat's summary judgment motion
should be denied.

CIVIL PROCEDURE 79
ce 45. Assume that the court denied Pat's mo- 46. Assume that Pat v. Dan and Dave had
— tion for summary judgment in Question been brought in federal district court
>w
0 44. Dan moved for partial summary (properly invoking diversity jurisdic-
judgment on Pat's affirmative defense. tion). Dan sued Dave in state court for
He requested that the court find conclu- personal injury claiming $60,000 in
sively that Dan was not contributorily damages. Assume the res judicata law
negligent based upon the June 1, 1990 followed by the jurisdiction in which
judgment in Pat v. Dan and Dave. the state court sits (in Dan v. Dave)
Which of the following rulings is most applies the "primary rights" definition
nearly correct? of "cause of action." Dave moved for
summary judgment to dismiss Dan's
A. Dan's motion should be denied claim in reliance on the June 1, 1990
because the general verdict for judgment in Pat v. Dan and Dave.
Dan against Pat, on Pat's claim Which of the following rulings is most
against Dan, prevents Dan from nearly correct?
proving that the jury actually
determined that Dan was not A. Dave's motion should be granted,
negligent. because Dan violated the cross-
claim rule of F.R.C.P. 13(g) by
B. Dan's motion should be granted failing to join his personal injury
because the jury in Pat v. Dan claim against Dan with his prop-
and Dave actually and necessar- erty damage claim.
ily determined that Dan was not
negligent. B. Dave's motion should be granted,
because Dan's claim for personal
C. Dan's motion should be denied injury is part of the same cause
because the jury's findings of of action as Dan's cross-claim
fact in Pat v. Dan and Dave are against Dave for property dam-
unreliable. Their verdicts for age.
Dave against Pat (on Pat's claim)
and for Dan against Dave (on C. Dave's motion should be denied,
Dan's cross-claim) are hope- because there are no compulsory
lessly irreconcilable. cross-claims in federal court.

D. Dan's motion should be denied D. Dave's motion should be de-


because Pat did not have a full nied, because Dan sued Dave
and fair opportunity to litigate in state court, which follows the
the issue of Dan's negligence. "primary rights" definition of
"cause of action" for res judicata
purposes.

80 CIVIL PROCEDURE
47. Assume, for purposes of this question, Questions 48 - 50 are based upon
that the court denied Dave's summary the following facts:
judgment motion in Question 46. Dan
moved for partial summary judgment Pat claims ownership of Blackacre, located
requesting the court to find that it has in Wyoming, through adverse possession.
been conclusively established, by virtue Paul is the owner of record and has a deed
of the June 1, 1990 judgment in Pat v. bearing his (Paul's) name on file in the
Dan and Dave, that Dave was negligent. County Recorder's office. Don came onto
Which of the following rulings is most Blackacre without Pat's or Paul's permis-
nearly correct? sion and cut down timber which Don hauled
away and sold.
A. Dan's motion should be denied,
because the general verdict on 48. Pat brought a trespass action against
Dan's cross-claim against Dave, Don in federal district court in Ne-
in Pat v. Dan and Dave, prevents vada. Don's answer asserted, by way
Dan from showing what the jury of defense, that the Nevada statute of
actually determined. limitations on trespass had run. Don
then brought a pre-answer motion to
B. Dan's motion should be granted, dismiss Pat's case on statute of limita-
because the jury actually and tions grounds, which the court granted.
necessarily determined that Dave Pat brought another trespass action
was negligent and Dave had a against Don in the federal district court
full and fair opportunity to liti- in Wyoming where the statute of limita-
gate that issue. tions on trespass is long enough to allow
Pat to seek to enforce her claim in a
C. Dan's motion should be denied, Wyoming court. Don's answer asserted
because Dave did not have a full the following by way of defense against
and fair opportunity to litigate Pat's claim: (1) that Pat did not own
the issue of Dave's negligence. Blackacre; (2) that Don did not come
upon Blackacre and cut the timber, and
D. Dan's motion should be denied, (3) that Pat's claim is barred by res judi-
because of the doctrine of mutu- cata. Don then moved to dismiss Pat's
ality of estoppel. suit on grounds of res judicata. Which
of the following is the most likely ruling
of the court (for either Pat or Don)?

CIVIL PROCEDURE 81
LU
ce A. The court should grant Don's 49. Assume, for the purpose of this and suc-
>

motion, because Pat has previ- ceeding questions, that the court denied
00 ously sued Don on the same Don's motion in question 48. After a
cause of action in the Nevada trial, in which evidence was admitted on
court and the Nevada federal both of Don's defenses (i.e., that Pat did
court rendered a final judgment not own Blackacre and that Don did not
on the merits. come onto Blackacre), the jury rendered
a general verdict for Don and the court
B. The court should deny Don's mo- thereupon entered judgment for Don
tion, because the Nevada federal on June 1, 1990. Pat then sued Don for
court's judgment was not ren- conversion in a Wyoming federal court
dered after a hearing based upon to recover the value of the trees which
evidence and, therefore, was not Don allegedly cut down and sold. Don
a judgment on the merits. asserted, in his answer, the defense
of res judicata and moved to dismiss
C. The court should deny Don's mo- Pat's suit on that ground. Which of the
tion, because the Nevada federal following is the most likely ruling (for
court's judgment is not deemed either Pat or Don)?
to be on the merits as long as
Wyoming allows Pat to take A. The court should grant Don's
advantage of its longer statute of motion, because the conversion
limitations. and trespass are part of the same
transaction or occurrence.
D. The court should grant Don's
motion, because the Nevada B. The court should deny Don's
federal court has already made a motion, because conversion and
final determination that Pat's ac- trespass are different transac-
tion is barred by Nevada's statute tions.
of limitations.
C. The court should deny Don's
motion, because conversion and
trespass are based upon different
wrongful acts by Don.

D. The court should deny Don's mo-


tion, because Pat's conversion
and trespass claims involve sub-
stantially the same evidence.

82 CIVIL PROCEDURE
Q
50. Assume, for purpose of this question, Questions 51 - 55 are based on the
that the court granted Don's motion following facts:
in question 49 and dismissed Pat's
conversion suit against Don. Paul, Paul was driving his two year old "Flexus"
the record owner of Blackacre, then automobile on the Riverside Freeway when
brought a trespass suit against Don in the engine suddenly quit and the car slowed
federal court in Wyoming. Don moved down to a stop before Paul could steer the
for partial summary judgment, asking vehicle to the shoulder of the road. Conse-
the court to find that it has been con- quently, Paul's car was rear-ended, causing
clusively established, by the June 1, injury to the car and to Paul. Paul brought
1990 judgment in Pat v. Don, that Don a products liability suit in federal district
did not come upon Blackacre. Which is court against the Flexus Motor Company, the
the most nearly correct ruling (for either manufacturer of Paul's car. The complaint
Paul or Don)? alleged, in pertinent part, that Paul's car
was defectively designed and manufactured,
A. The court should grant Don's without specifying the nature of the defects,
motion because the jury, in Pat and that said defects were the proximate
v. Don, has already determined cause of injury to Paul and his car.
that Don did not come upon
Blackacre. 51. Flexus did not make a pre-answer mo-
tion, but filed and served a timely an-
B. The court should deny Don's mo- swer to Paul's complaint. In addition to
tion because Don cannot show denying the allegations of Paul's com-
that the jury, in Pat v. Don, actu- plaint, the answer stated, as a defense,
ally determined that he did not that the allegations regarding the car's
come upon Blackacre. defects were vague and conclusionary
and, because they were central to Paul's
C. The court should deny Don's products liability claim, the complaint
motion because, whether or not should be dismissed. On a motion by
the jury actually determined that Flexus to dismiss the complaint, which
Don did not come upon Black- of the following statements is most
acre, Paul was not a party in the nearly correct?
Pat v. Don suit.
A. The motion should be granted
D. The court should deny Don's because "defect" allegations,
motion, because the doctrine of going to the heart of Paul's prod-
mutuality of estoppel prevents ucts defect claim, should at least
Don from invoking collateral notify Flexus of the particular
estoppel against a stranger to the parts of the car alleged to have
Pat v. Don suit. been defective.

CIVIL PROCEDURE 83
Q
B. The motion should be denied 52. Assume, for purposes of the rest of this
because the "defect" allegations series of questions, that Flexus' motion
are adequately pleaded. in question 51 was denied. Shortly
thereafter, the parties met as required
C. The motion should be denied by Rule 26(f), in part, to arrange for
because Flexus should have the "disclosures" required under Rule
attacked the sufficiency of the 26(a)(1). At this conference, Paul in-
allegations in Paul's complaint sisted that Flexus had an obligation,
through a pre-answer motion under Rule 26(a)(1), to "disclose" to
under Rule 12(b)(6). Paul all persons possibly involved in,
and all documents affecting, the design,
D. The motion should be denied manufacture, and assembly of the same
because Flexus waived any ob- model and year of Flexus car Paul was
jection to the sufficiency of the driving. Flexus adamantly refused and
complaint when it responded to asked Paul to identify more particularly
the complaint's allegations by the parts of Paul's car that were defec-
denying them. tive. Paul argued that it was too early
in the case to do so. Paul then moved
for sanctions against Flexus under
Rule 37(a) for failure to comply with
its "disclosure" obligations. Which of
the following statements is most nearly
correct?

A. Paul's motion should be granted


because Paul is not required,
early in his case, to know the
particulars of his case against
Flexus.

B. Paul's motion should be denied


because Flexus did not violate its
"disclosure" obligations under
Rule 26(a).

C. Paul's motion should be granted


because Flexus did violate its
"disclosure" obligations under
Rule 26(a).

84 CIVIL PROCEDURE
'CP
D. Paul's motion should be denied 54. Six months later, Paul narrowed the de-
00
because Paul's discovery request fect that he alleges caused the engine in
called for irrelevant informa- C7
<
his car to quit to a defect in the design of
tion. the fuel injection system. Immediately
thereafter, Flexus sent to Paul a list
53. Paul sought to depose Egbert, an expert identifying all Flexus employees who
who was retained by Flexus to provide had been involved with the design of
expert testimony on behalf of Flexus the fuel injection system in question and
at trial. After being retained by Flexus, describing by category and location all
Egbert inspected the wreckage of the documents in the possession, custody
automobile after it had been towed and control of Flexus relating to the
away by the police. Flexus moved for design of that fuel injection system. On
a protective order to bar the deposition motion against Flexus for sanctions,
of Egbert. Which of the following state- Paul asserts that Flexus was obligated
ments is most nearly correct? by the "disclosure" requirements of
Rule 26(a) to provide Paul with actual
A. Flexus' motion should be granted copies of these documents, not merely
because Paul should hire his own a description. Which of the following
expert to inspect the wreckage. statements is most nearly correct?

B. Flexus's motion should be A. Flexus is not obligated, under


granted because Paul failed the "disclosure" rules, to pro-
first to serve the required inter- vide copies of the documents in
rogatories described in Rule question as long as it provides a
26(b)(4)(a)(i). description of the documents by
category and location.
C. Flexus' motion should be granted
because the facts and opinions of B. The "disclosure" rules only al-
Flexus' expert are protected by low Flexus to provide Paul with
the work product immunity. a description of the documents
in question, but not copies; to
D. Flexus' motion should be de- obtain copies, Paul must serve
nied. on Flexus a formal request under
Rule 34.

CIVIL PROCEDURE 85
Q
C. By providing Paul with a de- C. Flexus' motion should be granted
scription of the documents in because the identity of Paul's
question, pursuant to the "disclo- trial witnesses is relevant in-
sure" rules, Flexus has waived formation and is not protected
its right to object to production from discovery as Paul's trial
of those documents on the basis strategy.
of privilege and work product
protection. D. Flexus' motion should be granted
because Paul is obligated to pro-
D. Flexus is obligated, under the vide this information to Flexus
"disclosure" rules, to provide under the "disclosure" rules.
copies of the documents in ques-
tion.

55. Twenty days before the commencement


of trial, Flexus had not received a list of
Paul's trial witnesses. Flexus told Paul
by telephone and letter that he was ob-
ligated by Rule 26(a) to "disclose" this
information but Paul adamantly refused.
Flexus moved for sanctions under Rule
37(a) and included a certification that
it had attempted to confer with Paul in
an effort to secure disclosure without
court action. Which of the following
statements is most nearly correct?

A. Flexus' motion should be denied


because Paul's trial strategy
constitutes mental impressions
and, as such, are protected from
discovery.

B. Flexus' motion should be denied


because Flexus should have
sought this information by serv-
ing a formal interrogatory.

86 CIVIL PROCEDURE
Q
Questions 56 - 59 are based on the 56. Before filing an answer, Joe immedi-
following facts: ately moved for summary judgment to
dismiss the claim against him on the
Pete and Joe moved into a rental apartment, grounds that the signature on the lease
owned by Larry Lessor, on January 1, 1992. purporting to be his is a forgery and that
The rent was $500 a month, to be paid on the he did not sign the lease. In support of
first of each month. On December 1, 1991, his motion, Joe submitted the follow-
Larry had mailed an original and a copy of ing evidence: (1) his affidavit swearing
the lease to Pete and Joe with a space for that he did not sign the lease; (2) the
the signatures of Pete and Joe, respectively. affidavits of 10 handwriting experts
On December 15, 1991, Larry received the who, based upon a comparison of the
original by mail and it contained signatures signature on the lease with signatures
purporting to be both Pete's and Joe's. On on 25 checks drawn on Joe's account
June 1, 1992, Larry brought suit against Pete (each of which "cleared"), each affirm
and Joe claiming Pete and Joe failed to pay that the signature on the lease is a plain
rent for March and April and demanding forgery; (3) an affidavit by Joe's prior
$1000, the rent for those two months. landlord that he was familiar with Joe's
signature and that the signature on the
lease is a forgery and (4) the copy with
no signature in the space reserved for
Joe's signature. In response, Larry sub-
mitted no affidavits or other evidence
but argues in his Memo of Points and
Authorities that the court should deny
the motion because he may be able to
show, at trial, that Joe signed the lease.
There were no eyewitnesses to Joe's
signing of the lease. Assume the rules
and case law relating to summary judg-
ment are those applicable in federal
court. Which of the following states the
most appropriate disposition of Joe's
motion (for either side)?

A. Joe's motion should be granted


because Larry did not meet his
respondent's burden.

CIVIL PROCEDURE 87
B. Joe's motion should be denied A. Pete's motion should be denied
because Joe did not meet his because he did not submit any
movant's burden. evidence to support his movant's
burden.
C. Joe's motion should be denied
because he did not previously B. Pete's motion should be denied
serve and file an answer before because each monthly obligation
moving for summary judgment. to pay rent constitutes a separate
"cause of action."
D. Joe's motion should be denied
because Joe did not negate every C. Pete's motion should be granted
possible way that Larry might because Larry split his cause of
prove that Joe had signed the action.
lease.
D. Pete's motion should be granted
57. Assume, for the remainder of this series because Larry did not meet his
of questions, that the court granted Joe's respondent's burden.
motion for summary judgment and dis-
missed Larry's claims against Joe. Pete 58. Assume, for purposes of this series of
answered Larry's complaint, admitting questions, that the court granted Pete's
failure to pay the March and April rent summary judgment motion referred
but asserting as a defense Larry's failure to in Question 57. Larry, immediately
to repair the heater which provides heat thereafter, commenced a second suit
to the apartment and hot water. Assume against Joe for the "May" rent. Joe
that, if proved, this would state a valid moved for summary judgment citing
defense to payment of rent under appli- the summary judgment in his favor in
cable substantive law. At trial, evidence Larry v. Joe. Which of the following
was presented on both sides of the issue statements is most nearly correct?
of the repair of the water heater and the
judge, as trier of fact, found that the A. The court should grant Joe's
heater had been repaired and found for motion because Larry did not
Larry. Judgment was entered for Larry include Joe in his suit against
on August 1, 1992. On August 15, 1992, Pete for the May rent, and Larry,
Larry brought another suit against Pete having lost his suit against Pete
for alleged failure to pay the May rent. for the May rent, cannot now
Pete moved for summary judgment cit- come after Joe.
ing the prior judgment in Larry v. Pete.
Which of the following statements is
most nearly correct?

88 CIVIL PROCEDURE
B. The court should grant Joe's B. Larry's motion should be granted
motion because, in the prior suit because (assuming Joe is found
against Joe, Larry only asserted to have signed the lease and that
a claim for the March and April he is jointly liable as a cotenant
rent, and omitted the May rent. to Larry for the full amount of
the rent) Joe is in privity with
C. The court should grant Joe's Pete and is, therefore, bound by
motion because the court, in the the judgment for Larry entered
prior suit against Joe, found that August 1, 1992.
Joe had not signed the lease.
C. Larry's motion should be denied
D. B and C. because mutuality of estoppel
prevents Larry from invoking
59. Assume, for purposes of this series of collateral against a stranger to
questions, that the court denied Joe's the Larry v. Pete suit.
motion referred to in Question 58. Joe
answered Larry's complaint, denying D. Larry's motion should be denied
the allegations therein and asserting, because the issue of the state of
as a defense, that the water heater had the water heater in May is not
broken down again in May and that necessarily identical with the
Larry failed to repair it. Larry moved state of the water heater in March
for partial summary judgment to strike and April.
Joe's defense regarding the water heater
citing the prior judgment for Larry en-
tered August 1, 1992. Which of the fol-
lowing statements is the best answer?

A. Larry's motion should be denied


because Joe did not have his day
in court on the issue of the bro-
ken water heater.

CIVIL PROCEDURE 89
Q
ce
LU
Questions 60 - 64 are based upon D. Pat's motion should be granted
because Sam is not a lawyer.
>0

the following facts:
U0
ce
a_ Pat slipped on the floor in a Plucky's Su- 61. Lou, Plucky's lawyer, tracked down
permarket in Anaheim Hills. Pat brought Wally, another witness to Pat's acci-
suit against Plucky's in federal district court dent and got a written statement from
(assume jurisdiction exists) for personal Wally. Pat properly served on Plucky's
injury. a discovery demand for Wally's state-
ment. Plucky's objected and Pat moved
60. Pat's friend, Joe, who accompanied to compel production. Which of the
Pat to the market and claims to have following statements is most nearly
seen what happened, gave a written correct?
statement to Sam, the store manager,
immediately after Pat slipped on the A. Wally's statement is not discov-
supermarket floor. Neither Pat nor erable because it is protected by
Joe had a copy of Joe's statement. Pat the attorney-client privilege .
properly served on Plucky's a discov-
ery demand for Joe's statement. After B. Wally's statement is not discov-
Plucky's objected to producing Joe's erable because it is protected by
statement on work product grounds, Pat the work product immunity.
moved to compel production. Which of
the following statements is most nearly C. Wally's statement is not discov-
correct? erable because it is irrelevant.

A. Pat's motion should be granted D. Wally's statement is not discover-


because Pat has an unqualified able because it is confidential.
right to discover Joe's state-
ment.

B. Pat's motion should be denied


because Joe has an unqualified
right to get a copy of his state-
ment from Plucky's.

C. Pat's motion should be denied


because neither Pat nor Joe
has an unqualified right to get
a copy of Joe's statement from
Plucky's.

90 CIVIL PROCEDURE
Q
62. In response to a different "slip and fall" D. Ed's memo is not discoverable
incident that occurred a year prior to because it is protected by the
Pat's accident, Plucky's President re- "subsequent remedial measures"
tained Ed, a safety expert, to examine rule.
Plucky's Anaheim Hills store for poten-
tial safety hazards and to make recom- 63. Pat seeks to discover a report by a city
mendations for preventing future ac- Health Department officer regarding
cidents. Ed drafted a memo containing fruits and vegetables which he found on
his findings and recommendations and the floors of all Plucky's Supermarkets
sent it to Plucky's President. Although — including the Anaheim Hills store.
Pat's case has not yet reached the trial Plucky's objected and Pat moved to
phase, because the parties are still in compel production of the report. Which
discovery, Plucky's lawyer knows that of the following statements is most
Ed's memo would not be admissible nearly correct?
evidence at trial under the evidence
law known as the "subsequent reme- A. The report is not discoverable
dial measures" exclusionary rule. This because it is irrelevant.
rule bars the admission into evidence
at trial of measures taken by a party to B. The report is not discoverable
remedy a dangerous situation to prove because it is confidential.
negligence. The rule is concerned that
parties would be deterred from taking C. The report is not discoverable
such remedial measures. Pat properly because it is protected by the
served on Plucky's a discovery demand work product immunity.
for Ed's memo. Plucky's objected
and Pat moved to compel production. D. The report is discoverable.
Which of the following statements is
most nearly correct? 64. In anticipation of Pat's lawsuit, Sam,
the store manager, investigated Pat's
A. Ed's memo is not discoverable "slip and fall" incident and wrote a
because it contains the mental report which he sent to Lou, Plucky's
impressions of an expert. lawyer. On oral deposition of Sam by
Pat, Pat's lawyer asked Sam the follow-
B. Ed's memo is not discoverable ing question: "Describe the condition
because the purpose of the memo of the floor where Pat fell at the time
is to avoid future litigation. you arrived at the scene of Pat's ac-
cident." Plucky's counsel objected and
C. Ed's memo is discoverable. Pat moved to compel Sam to answer.
Which of the following statements is
most nearly correct?

CIVIL PROCEDURE 91
Q LLI
Ce A. The information requested by Pat Questions 65 - 71 are based on the
>w

from Sam is discoverable. following facts:
Uv
0
ce B. The information requested by Pat Owner, a New Yorker, contracted with Gen-
from Sam is not discoverable be- eral, a California general contractor, to build
cause it is protected by the work a house for Owner in Anaheim, California.
product immunity. Owner had previously hired Archie, a New
York architect, to draw up the plans for the
C. The information requested by house. Owner gave these plans to General
Pat from Sam is not discoverable who relied upon them in building the house.
because it is confidential. When the house was completed, Owner
found that the house started to sink. Owner
D. The information requested by brought suit in federal court for $80,000
Pat from Sam is not discoverable in damages, jointly and severally, against
because it is protected by the at- (1) General, for defectively constructing
torney-client privilege. the house, and (2) the City of Anaheim for
negligently allowing the reservoir adjacent
to Owner's house to leak water, causing the
ground underneath the house to give way.

65. The two defendants moved to dismiss


the complaint for lack of subject matter
jurisdiction, contending that the amount
in controversy was insufficient since
$80,000 split between two defendants
was only $40,000 per defendant. There-
fore, each claim lacked the requisite
jurisdictional amount. Which of the
following statements is most nearly
correct?

A. The court has subject matter ju-


risdiction because the two claims
arise out of the same transaction
and occurrence.

B. The court has subject matter ju-


risdiction because the court can
aggregate the two claims.

92 CIVIL PROCEDURE
C. The court has subject matter ju- D. The court should deny the mo-
00
risdiction because the damages tion because a municipality can -
<
were pleaded against the defen- always sue a citizen in federal 0 -
C
dants jointly and severally. court.

D. The court does not have subject 67. General asserted a cross-claim for con-
matter jurisdiction. tribution against the City of Anaheim.
The City of Anaheim moved to dismiss
66. Assume, for purposes of the remainder the cross-claim. Which of the following
of this series of questions, that the court statements is most nearly correct?
denied the motions in Question 65.
The City of Anaheim counterclaimed A. The court should grant the mo-
against Owner for $10,000 which it tion to dismiss because a claim
claimed was a special city tax assess- for contribution or indemnity
ment for the building of the reservoir can only be joined by means of
which Owner should have paid (but impleader.
didn't) when he purchased the land
upon which his house was built. Owner B. The court should grant the
filed and served a Reply which asserted, motion to dismiss because the
as an affirmative defense, that the tax cross-claim is not supported by
assessment violated his rights under the an independent basis of subject
14th Amendment to the U.S. Constitu- matter jurisdiction.
tion. Owner moved to dismiss the City's
counterclaim. Which of the following C. The court should deny the mo-
statements is most nearly correct? tion to dismiss because joinder
is proper.
A. The court should grant the mo-
tion to dismiss because of lack D. The court should deny the mo-
of subject matter jurisdiction. tion to dismiss because the court
can exercise supplemental juris-
B. The court should deny the mo- diction over the cross-claim.
tion to dismiss because federal
courts can exercise supplemental
jurisdiction over compulsory
counterclaims.

C. The court should deny the mo-


tion because the court has federal
question jurisdiction over the
counterclaim.

CIVIL PROCEDURE 93
Q
68. Assume, for purposes of the remain- 69. Assume, for purposes of the remain-
der of this series of questions, that the der of this series of questions, that the
court denied the motion in Question court granted the motion in Question
67. General served a timely third-party 68. General served a timely third-party
complaint and summons on Archie, complaint and summons on Fred, a
the architect, claiming that Archie's California sub-contractor, who was
defective plans caused the house to hired by General to lay the foundation
sink. As a result, General had to spend for Owner's house. The third-party
$60,000 more on the project than he had complaint contained two claims: In
anticipated which caused General to Claim #1, General alleged that the
go bankrupt. Archie moved to dismiss foundation was defectively laid and
the third-party complaint. Which of the this caused the house to sink. In Claim
following statements is most nearly cor- #2, General alleged that Fred failed to
rect? repay a loan General had made to Fred
in connection with another project. Fred
A. The court should grant Archie's moved to dismiss both claims. Which of
motion because impleader is the following statements is most nearly
improper. correct?

B. The court should grant Archie's A. The court should dismiss Claim
motion because the impleader #2 but not Claim #1.
of Archie, a New Yorker, would
destroy diversity. B. The court should dismiss Claim
#1 but not Claim #2.
C. The court should deny Archie's
motion because impleader is C. The court should dismiss both
proper. Claims.

D. The court should deny Archie's D. The court should not dismiss
motion because the court can ex- either Claim.
ercise supplemental jurisdiction
over General's third-party claim
against Archie.

94 CIVIL PROCEDURE
70. Assume, for purposes of the remainder 71. Assume that, after a trial, judgment
00
of this series of questions, that the court was rendered in favor of Owner against
<
dismissed only Claim #2. Assume, fur- both General and the City of Anaheim CP
ther, that Fred, the sub-contractor, was a and appeals by each defendant failed.
New Yorker, not a Californian. Owner Owner collected the full amount of his
moved for leave to amend his complaint judgment from the City of Anaheim.
to add a claim against Fred alleging that The City of Anaheim sued General for
Fred's defective laying of the founda- contribution, claiming reimbursement
tion contributed to the sinking of the in proportion to General's degree of
house. Fred opposed Owner's motion. fault for the sinking house. General
Which of the following statements is moved to dismiss the City's suit. Which
most nearly correct? of the following statements is most
nearly correct?
A. The motion should be denied
because Fred will be prejudiced A. The motion to dismiss should
in having to defend against two be granted because the City of
claims (General's and Owner's) Anaheim should have counter-
instead of only one claim (Gen- claimed for contribution in the
eral's). previous suit by Owen against
General and the City of Ana-
B. The motion should be granted heim.
because Owner's claim against
Fred arises out of the same B. The motion to dismiss should be
transaction and occurrence as denied because cross-claims are
his claims against General and never compulsory.
the City of Anaheim.
C. The motion to dismiss should be
C. The motion should be denied, denied because when the City
otherwise, diversity will be de- of Anaheim failed to assert its
stroyed. contribution claim in the previ-
ous suit, General failed to object
D. The motion should granted be- at that time, thereby waiving its
cause Fred has waived his sub- objection.
ject matter jurisdiction defense.
D. The motion to dismiss should
be granted because cross-claims
which arise out of the same
transaction and occurrence are
compulsory.

CIVIL PROCEDURE 95
Q
Questions 72 - 76 are based on the C. The court has subject matter ju-
following facts: risdiction because Joe and I.B.N.
are diverse parties since I.B.N. is
Joe, a New Yorker, bought stock in the I.B.N. also a Delaware corporation.
Corporation, a Delaware Corporation with
its principal place of business in New York D. The court has subject matter
City, on the basis of glowing reports in an jurisdiction because the claims
I.B.N. stock prospectus about a revolution- arise under federal law.
ary new invention. Joe bought the stock at
$100 a share. When the Wall Street Journal 73. Under which category or categories
reported that I.B.N.'s reports regarding this of class action does Joe's class action
invention were false, the price of the stock fit?
plummeted to $1.00 per share. Joe brought a
federal class action against I.B.N., on behalf A. "Prejudice" class action [Rule
of himself and as representative of all others 23(b)(1)].
who purchased I.B.N. stock in reliance on
the misleading prospectus, alleging violation B. "Injunctive" class action [Rule
of the federal Securities and Exchange Act 23(b)(2)].
of 1934. Joe alleged $10,000 in damages
and none of the 100,000 class members C. "Damage" class action [Rule
individually had more than $6,000 in dam- 23(b)(3)].
ages. The aggregate class damages exceeded
$3,000,000. D. Both "Prejudice" and "Damage"
class actions.
72. I.B.N. moved to dismiss Joe's claim and
those of the class on the grounds that 74. Which statement best defines the notice
the federal court lacked subject matter requirement applicable to Joe's class
jurisdiction. Which of the following action?
statements is most nearly correct?
A. The best notice practicable under
A. The court has no subject matter the circumstances.
jurisdiction because the damages
of Joe and the class members B. The best notice practicable under
may not be aggregated. the circumstances, including
individual notice to each class
B. The court has no subject mat- member.
ter jurisdiction because Joe and
I.B.N. are both New York citi-
zens.

96 CIVIL PROCEDURE
Q
C. The best notice practicable under 76. Joe learns that mailing notice to the
the circumstances, including class will cost $300,000. Joe has rep-
individual notice to all class resented to the court that he has no
members who can be identified more than $10,000 to pay for notice to
through reasonable effort. the class. Which of the following best
describes how a court will handle this
D. Publication of the class action situation?
notice in newspapers of wide
circulation. A. Joe will have to pay the entire
$300,000 himself.
75. Assume for this question that Joe is also
an attorney and, in addition to class rep- B. The court will order I.B.N. to pay
resentative, intends to serve as counsel the $280,000 if it determines,
to the class. I.B.N. contends that this after a preliminary hearing, that
fact makes Joe an inadequate class Joe is likely to win the class ac-
representative. Which of the following tion on the merits.
statements is most nearly correct?
C. The court will scale down the
A. The court will find that Joe's sta- number of class members who
tus as class attorney makes Joe will receive individual notice and
an inadequate class representa- authorize notice to the remain-
tive. ing members by publication in a
newspapers of wide circulation.
B. The court will find that Joe's sta-
tus as class attorney makes him D. The court will reduce the size
an adequate class representative of the class until the cost of
because it gives him an even mailing individual notice equals
greater stake in the successful $10,000.
outcome of the suit.

C. The court will find Joe's status


as class attorney makes him an
adequate class representative
because it will save the class the
cost of hiring a separate attor-
ney.

D. The court will find Joe's status


as class attorney to be irrelevant
to whether Joe will adequately
represent the class.

CIVIL PROCEDURE 97
Questions 77 - 79 are based on the D. The report is protected by F.R. C P.
following facts: 26(b)(4) [facts and opinions of
experts].
Paul, an Oregonian, and Don, a Californian,
were each independent truckers who were 78. Donna sought to depose Eggbert, an
driving their respective trucks on a two-lane expert in accident reconstruction, who
highway and were approaching each other has been identified by Paula, pursuant
from opposite directions. Their side-mir- to disclosure requirements, as a wit-
rors struck each other, causing each truck to ness she has retained to provide expert
swerve off the highway killing each driver. testimony in the case. Donna wants to
Paula, Paul's wife and executrix for Paul's depose Eggbert and has served upon
estate, brought suit in federal court against him a notice of deposition. Donna has
Don's estate, represented by Don's wife not yet served any interrogatories on
Donna as executrix, for wrongful death, Paula, nor has she received any reports
alleging that Don's truck had veered into prepared by Eggbert on his proposed
Paul's on-coming lane, thereby causing trial testimony. Paula moved for a pro-
the mirrors to come into contact. Assume tective order to prevent Donna from
diversity of citizenship exists. There were proceeding with Eggbert's deposition.
no eyewitnesses to the accident. Which of the following statements is
most nearly correct?
77. A year before he died, Don thought it
would be prudent to draft a will. Don A. The protective order should be
asked his accountant to review Don's granted because Donna has not
assets and draft a report so that Don yet received any reports prepared
would know the extent, nature and by Eggbert regarding his pro-
worth of his estate. Paula served a posed trial testimony.
request to produce that report. Donna
objected. Which of the following is B. The protective order should be
the strongest objection Donna could granted because Donna has not
make? yet served, and received answers
to, interrogatories regarding
A. The report is irrelevant. Eggbert's proposed trial testi-
mony.
B. The report contains confidential
information.

C. The report is protected by the


work product immunity.

98 CIVIL PROCEDURE
Q
C. The protective order should be C. Donna's motion should be de-
00
granted because the facts and nied because, when there is a
opinions held by an expert,
<
conflict in the evidence, the jury c
like Eggbert, who has been re- should resolve that conflict, not
tained or specially employed in the court.
preparation for trial, can only be
discovered upon a showing of D. Donna's motion should be grant-
exceptional circumstances. ed.

D. The protective order should be


denied.

79. At trial, Paula's only evidence on liabil-


ity is the expert testimony of Eggbert
who testified that, based upon the cir-
cumstantial evidence he viewed at the
scene of the accident, including skid
marks, the position of the trucks and
the condition of the wreckage, it was
his opinion that Don's truck could well
have veered into Paul's lane. On cross-
examination, Eggbert also testified that
it was equally possible that Paul's truck
could have veered into Don's lane. Be-
fore Paula had an opportunity to put on
her damage evidence, Donna moved for
judgment as a matter of law on the issue
of causation. Which of the following
statements is most nearly correct?

A. Donna's motion is premature


and, therefore, should not be
granted because Paula has not
yet put on her whole case.

B. Donna's motion should be de-


nied because the credibility of
Eggbert is for the jury to deter-
mine, not the court.

CIVIL PROCEDURE 99
Questions 80 - 84 are based on the B. The court should deny the mo-
following facts: tion because Willy is not a party
nor in the custody nor a person
Pam and Del collided at an intersection at in the custody or under the legal
2:00 a.m. Believing that Del was driving in control of a party.
the course of his employment for Acme, Inc.,
Pam, an Arizonan, brought a personal injury C. The court should grant the mo-
suit in federal court against Acme, Inc., a tion because the condition of
California corporation, alleging that Del ran Willy's eyesight is relevant to
a red light at the intersection. (Assume di- the case.
versity jurisdiction.) Acme, Inc. denied that
Del ran the light and alleged that Pam ran the D. The court should deny the mo-
red light. As a result of the accident, neither tion because Willy's ex-wife is
Pam nor Del recall the circumstances leading biased.
up to the collision. Pam's only evidence to
prove Del caused the accident is Willy who is 81. Pam moved, under F.R.C.P. 35(a), to
prepared to testify to the following: that, for compel Del to submit to a neurological
10 minutes before the collision, he had been and a psychological examination based
following Del's vehicle and that, at each red upon an affidavit from Pam's attorney
light, Del ran the light. However, Del turned that his review of court files shows that
off the street prior to the intersection where Del has been convicted of reckless driv-
the collision occurred and, thus, did not wit- ing four times. Which of the following
ness the collision. statements is most nearly correct?

80. Acme moved, under F.R.C.P. 35(a), to A. The court should grant the mo-
compel an eye examination of Willy, tion because Del is a person who
submitting an affidavit from Willy's is under the control of Acme,
ex-wife in which she affirmed that Willy Inc., a party defendant.
suffers from serious night blindness.
Which of the following statements is B. The court should deny the motion
most nearly correct? because Pam has not shown that
Del's nervous system and mental
A. The court should grant the mo- state are "in controversy."
tion because the affidavit from
Willy's wife shows that the C. The court should grant the mo-
condition of Willy's eyes are "in tion because Del's ability to
controversy." drive a vehicle safely is centrally
relevant to the case.

100 CIVIL PROCEDURE


—co
D. The court should deny the mo- 83. Assume, for purposes of the remainder
0 0
tion because the affidavit of of this series of questions, that Pam's
<
Pam's attorney is not based upon summary judgment motion in Question CP
personal knowledge. 82 was denied. At trial, Pam's liability
case consists of (1) Willy 's testimony
82. Pam moved for summary judgment as described in the base facts above and
submitting in support of her motion the (2) the testimony of two bishops (newly
affidavit of Willy that states, under oath, discovered by Pam) who swear that they
what he is prepared to testify to at trial saw Del run the red light. Acme, Inc.'s
(as set forth in the base facts above). rebuttal case consists of one witness, a
In response, Acme, Inc. only submitted twice-convicted perjurer, who swears
its answer, in which it denied that Del that he saw Pam run the red light. After
ran the red light and alleged that Pam the jury returned a verdict in favor of
did. Which statement most correctly Acme, Inc., Pam moved for a judgment
expresses how the court should rule on as a matter of law in her favor and, in
Pam's summary judgment motion? the alternative, for a new trial. Which of
the following statements most correctly
A. The motion should be denied expresses how the court should rule on
because Pam did not meet her Pam's motion for judgment as a matter
movant's burden. of law?

B. The motion should be granted A. Pam's motion should be denied


because Acme, Inc. did not meet because an offensive motion for
its respondent's burden. judgment as a matter of law can
never be granted where the mov-
C. The motion should be denied ant has offered oral testimony,
because there is a two-sided fact since credibility is always a jury
issue regarding causation which issue.
should go to the jury.
B. Pam's motion should be granted
D. The motion should be granted because Pam met her burden of
because courts never grant offen- production and Acme, Inc. did
sive summary judgment motions not.
based on affidavit testimony.
C. Pam's motion should be denied
because Pam did not make a mo-
tion for a judgment as a matter
of law before the close of all the
evidence.

CIVIL PROCEDURE 101


D. Pam's motion should be grant- D. The reversal was proper because
ed because Pam's case "over- Juror Joan's affidavit was inad-
whelmed" Acme's case. missible.

84. Assume, for purposes of the remainder


of this series of questions, that the court
granted Pam's motion for judgment as
a matter of law. The court also condi-
tionally granted Pam's motion for a
new trial. Her motion was based upon
an affidavit of a juror, Joan, who swore
in an affidavit, that she voted against
Pam because she was biased against
Pam based on Pam's race. Acme, Inc.
appealed the granting of the judgment a
matter of law and the new trial motion.
The Court of Appeals reversed the grant
of the judgment as a matter of law and
also the grant of the new trial motion.
Which of the following statements best
expresses the correctness of the Court
of Appeals reversal of the lower court's
new trial order?

A. The reversal was improper


because appellate courts never
reverse the exercise of discretion
of the lower court on new trial
motions.

B. The reversal was improper


because the lower court's con-
ditional grant of a new trial was
not a "final decision."

C. The reversal was improper be-


cause bias is an improper basis
for determining a jury verdict.

102 CIVIL PROCEDURE


Q
Questions 85 - 91 are based on the C. The court does not have diversity
00
following facts: jurisdiction because Pam cannot
<
prove to a legal certainty that she CI -
C
Darren was a student at Hower U., a college will recover the full $75,000 in m
located on the Nevada side of Lake Tahoe, pain and suffering damages.
10 miles from the Nevada-California border.
Darren's parents live in Anaheim, Califor- D. The court does not have diver-
nia. One night, Darren had been drinking sity jurisdiction because Darren
all evening at Beers, a bar located on the is also a California citizen like
California side of the border, and 20 miles Beers.
from Hower U. Beers advertised itself in the
student newspaper at Hower U. as a "way 86. Assume, for purposes of the remain-
cool" student hangout. The bar was very der of this series of questions, that the
popular with Hower U. students. Totally court denied Beers' pre-answer motion
"lit," and headed back to the dorm, Darren in Question 85. Beers then answered
got into a collision near the campus, inside Pam's complaint, denying, in part, (1)
Nevada, with a Jaguar automobile owned that Darren was drunk when he left the
and operated by Pam, whose home is in Las bar and (2) that Darren caused the col-
Vegas. Pam brought a personal injury suit, lision. Beers' answer also affirmatively
in a federal district court in Nevada, against alleged, as affirmative defenses, (a) that
Beers for $105,000, including $75,000 for Pam's negligence caused the collision
pain and suffering alleging that Beers con- and (b) that the court does not have
tinued to serve alcohol to Darren knowing personal jurisdiction over Beers. Beers
he was drunk and was, therefore, responsible then moved to dismiss Pam's complaint
for the collision and Pam's injuries. for lack of personal jurisdiction. As-
sume Nevada has a long arm statute
85. Beers made a pre-answer motion to dis- which limits tort actions in Nevada
miss Pam's suit on the ground that the courts against non-residents to actions
court lacked subject matter jurisdiction arising out of tortious conduct (not just
over her claim. Which of the following injury) occurring in Nevada. Which of
statements is most nearly correct? the following statements is most nearly
correct?
A. The court has diversity jurisdic-
tion. A. The court can exercise personal
jurisdiction over Beers because
B. The court does not have diversity it solicited business in Nevada.
jurisdiction because Beers is also
a Nevada citizen, like Pam, since
it "does business" in Nevada by
soliciting business in Nevada.

CIVIL PROCEDURE 103


B. The court cannot exercise per- B. The court should not grant the
sonal jurisdiction over Beers motion because the 7/1/92 judg-
because Beers serves customers ment was not "on the merits."
(including Darren) exclusively
in California. C. The court should not grant the
motion because Pam's second
C. The court cannot exercise per- suit against Beers asserts a differ-
sonal jurisdiction over Beers ent right than her previous suit.
because Pam's action arises out
of tortious conduct occurring in D. The court should not grant
California. the motion because the 7/1/92
judgment is not "valid" since
D. The court can exercise per- that judgment was for less than
sonal jurisdiction because Beers $75,000.
waived its personal jurisdiction
defense. 88. Assume, for purposes of the remainder
of this series of questions, that the court
87. Assume, for purposes of the remainder granted Beers' motion in Question 87
of this series of questions, that the court and entered judgment for Beers on Sep-
denied Beers' motion in Question 86. tember 1, 1992. Pam filed suit against
Beers then abandoned defense of the Darren in a federal district court in Cali-
suit, ignoring all discovery orders. The fornia demanding $80,000 in damages
court finally entered a default judg- for injury to her Jaguar automobile,
ment for Pam for $30,000 on July 1, serving him personally while he was
1992. Two months later, Pam brought home visiting his parents in Irvine.
another suit against Beers arising out Darren made a pre-answer motion to
of the same auto collision with Darren, dismiss Pam's complaint on the ground
demanding $80,000 in damages for that the court lacked personal jurisdic-
injury to her Jaguar automobile. Beers tion over him. Which of the following
moved to dismiss Pam's suit based upon statements is most nearly correct?
the July 1, 1992 judgment in Pam's
previous suit against Beers. Which of A. The court should deny Darren's
the following statements is most nearly motion because Pam's cause
correct? of action arose out of Darren's
drinking in California.
A. The court should grant the mo-
tion. B. The court should grant Darren's
motion because the collision
arose directly out of Darren's
driving in Nevada.

104 CIVIL PROCEDURE


C. The court should grant Darren's 90. Assume, for purposes of the remainder
motion because most of the evi- of this series of questions, that the court
dence relating to the cause of the denied Darren's motion in Question 89.
collision, including witnesses, is Darren moved to dismiss Pam's action
located in Nevada. based upon the September 1, 1992 judg-
ment in the second Pam v. Beers suit.
D. The court should deny Darren's Which of the following statements is
motion because Darren was most nearly correct?
served in California.
A. The court should deny Darren's
89. Assume, for purposes of the remainder motion because neither res judi-
of this series of questions, that the court cata nor any joinder rule required
denied Darren's motion in Question Pam to join Darren as a co-defen-
88. Darren moved to dismiss Pam's dant alongside Beers.
suit on the ground that the court lacked
subject matter jurisdiction. Which of B. The court should deny Darren's
the following statements is most nearly motion because Darren waived
correct? his res judicata defense by failing
to include it in his pre-answer
A. Darren's motion should be de- motion.
nied because Darren is a Califor-
nia citizen and Pam is a Nevada C. The court should grant Darren's
citizen. motion because Pam should not
be allowed to "hedge her bets"
B. Darren's motion should be grant- by suing a series of defendants,
ed because Darren is a Nevada one-by-one, on the same prop-
citizen like Pam. erty claim until the supply of
defendants runs out.
C. Darren's motion should be grant-
ed because Darren is a citizen of D. The court should deny Darren's
the forum state, California. motion because the 9/1/92 judg-
ment was not "on the merits."
D. Darren's motion should be de-
nied because he waived his sub-
ject matter jurisdiction defense.

CIVIL PROCEDURE 105


91. Assume, for this question, that the court Questions 92 - 96 are based on the
denied Darren's motion in Question following facts:
90. Pam moved for partial summary
judgment, based on the July 1, 1992 Jacques and Jill are graduate students at a
judgment in Pam's favor in the first Pam private university in Pomona, California.
v. Beers suit, on the issue of Darren's Jacques is a French citizen. Prior to her com-
liability for causing the collision. Which ing to California to study, Jill lived with her
of the following statements is not cor- parents in Phoenix, Arizona. Jill, however,
rect? left on very bad terms and vowed never to re-
turn to her parents' home in Arizona. Jacques
A. Pam cannot invoke collateral es- and Jill met while students at the university
toppel because the Due Process in Pomona, fell in love, married and rented
clause prevents Darren from a condo in Pomona. After one year of mar-
being bound by the 7/1/92 judg- riage, they began arguing a lot and Jill sus-
ment. pected Jacques of having many extra-marital
affairs. One evening, Jacques and Jill were
B. Pam cannot invoke collateral returning from a movie. Jacques was driving
estoppel because no facts were and Jill was a passenger and the two began
actually determined in the first arguing violently. Their car collided with a
Pam v. Beers suit. car driven by Don, a Californian. Jacques
and Jill sued Don in federal district court
C. Pam cannot invoke collateral in Santa Ana, California. Jacques asserted a
estoppel because the 7/1/92 judg- claim for loss of Jill's consortium, demand-
ment was not "on the merits." ing $40,000 in damages, including $10,000
in punitive damages, and Jill asserted a
D. The mutuality of estoppel rule claim for her personal injuries, demanding
is inapplicable to resolve the is- $100,000 in damages, including $60,000
sue whether Pam can invoke the in punitive damages. Their complaint de-
benefits of collateral estoppel. manded a jury trial on both claims.

92. Don moved to dismiss both Jacques and


Jill's claims for lack of subject matter
jurisdiction. On the issue of "complete
diversity," which of the following state-
ments is most nearly correct?

A. The court has alienage jurisdic-


tion over Jacques' claim but must
dismiss Jill's claim because she
is a citizen of California.

106 CIVIL PROCEDURE


B. The court has alienage jurisdic- D. Jill's claim independently meets
tion over both Jacques and Jill's the amount in controversy re-
claims because Jill takes on the quirement and Jacques' claim
domicile of her husband. will be saved by the exercise of
pendent party (i.e., supplemen-
C. The court does not have diversity tal) jurisdiction.
jurisdiction over either Jacques
or Jill's claims because Jill's 94. Assume, for purposes of the remainder
California citizenship destroys of this series of questions, that the court
complete diversity. denied Don's motion with respect to
both Jacques and Jill's claims. Don filed
D. The court has diversity juris- and served his answer to the complaint,
diction over both Jacques and which included an affirmative defense
Jill's claims because Jacques is to Jacques' claim that Jacques was con-
a French citizen and Jill is an tributorily negligent. Assume that con-
Arizona citizen. tributory negligence is an absolute bar
to recovery. Don moved to have Jacques
93. In Don's motion to dismiss for lack of and Jill's claims tried to a judge, in-
subject matter jurisdiction, Don also stead of a jury, citing a California state
claims Jacques and Jill's claims do not statute that requires all personal injury
satisfy the jurisdictional "amount in claims asserting punitive damages to
controversy" requirement. On the issue be tried by the court rather than a jury.
of "amount in controversy," which of The statute was enacted in response to
the following statements is most nearly skyrocketing punitive damages jury
correct? verdicts. Plaintiffs opposed Don's mo-
tion, asserting their 7th Amendment
A. Jacques' claim does not meet the right to a jury trial of all issues relating
amount in controversy require- to their damage claims. On the "jury
ment while Jill's claim does. trial" issue, which of the following
statements is most nearly correct?
B. Because Jacques and Jill can
aggregate their claims, both A. The "Erie" doctrine requires the
their claims meet the amount in court to apply the California stat-
controversy requirement. ute because that statute embodies
important substantive interests.
C. Neither Jacques' claim nor Jill's
claim meet the amount in contro-
versy requirement.

CIVIL PROCEDURE 107


B. The Supreme Court's ruling in 95. For purposes of the remainder of this
Hanna v. Plumer does not require series of questions, assume that the
the federal district court to apply court denied Don's motion in Ques-
the 7th Amendment because it tion 94. After a jury trial, during which
is not a "Federal Rule of Civil Jacques offered minimal evidence of
Procedure." damages on his consortium claim and
Don offered substantial evidence that
C. The Supremacy Clause of the plaintiffs' marriage was in trouble, the
U.S. Constitution requires the jury rendered the following verdict:
federal district court to apply the (1) On Jacques' claim, in favor of Don.
7th Amendment to Jill's jury trial (2) On Jill's claim, in favor of Jill for
issue. $80,000. Judgment was entered upon
the verdict on April 1, 1993. Jacques
D. The federal district court can ap- filed a California state court suit against
ply the California statute because Don for his personal injuries caused
there will be no outcome differ- by the collision with Don's car. Don's
ence between a judge vs. a jury answer included an affirmative defense
trial. that Jacques was contributorily negli-
gent. Don moved to dismiss Jacques'
claim based upon the April 1, 1993
judgment in Jacques and Jill v. Don. As-
sume the California law of res judicata
defines "cause of action" according to
the "primary rights" test. Which of the
following statements is most nearly
correct?

A. The court should deny Don's


motion because, applying the
primary rights test, Jacques did
not split his cause of action.

B. The court should grant Don's


motion because, applying the
"same transaction test," Jacques
split his cause of action.

108 CIVIL PROCEDURE


C. The court should deny Don's C. Jacques should not be collater-
motion because Jacques' loss ally estopped on the issue of
of consortium claim in the prior contributory negligence because,
(federal) suit against Don de- although the state court knows
rives from Jill's injury whereas that the federal court "actually
Jacques' claim in the current determined" that Jacques was
(state) suit asserts Jacques' own contributorily negligent, it can-
injuries. not know whether such deter-
mination was "necessary" to the
D. The court should grant Don's judgment.
motion because Jacques was in
privity with Jill in the prior fed- D. Jacques should not be collater-
eral court suit. ally estopped on the issue of
contributory negligence because
96. Assume, for this question, that the he did not have a "full and fair
court denied Don's motion in Ques- opportunity to litigate" this is-
tion 95. Don again moved to dismiss sue.
Jacques' claim based upon the April 1,
1993 judgment, in the prior federal suit.
Don's contention, this time, was that
Jacques is collaterally estopped from
proving that he was not contributorily
negligent. Which of the following state-
ments is most nearly correct?

A. Jacques should be collaterally


estopped on the issue of con-
tributory negligence because
the federal court "actually and
necessarily determined" that
Jacques was contributorily neg-
ligent.

B. Jacques should not be collater-


ally estopped on the issue of
contributory negligence because
the state court cannot tell wheth-
er the federal court "actually
determined" that Jacques was
contributorily negligent.

CIVIL PROCEDURE 109


Q
Questions 97 - 100 are based on the 97. DeWinter moved for summary judg-
following facts: ment, based upon the September 1,
1992 judgment of acquittal in the
On January 1, 1992, DeWinter was charged California murder case that had been
with the murder of his wife, Rebecca. On brought against him. In this motion,
April 1, 1992, DeWinter stood trial in Cali- DeWinter asked to court to find that
fornia on this charge. Evidence introduced the judgment of acquittal conclusively
by the prosecution showed, inter alia, (1) establishes, for purposes of his civil suit
that during the year prior to Rebecca's against Floyd's, that DeWinter did not
death, DeWinter had been carrying on a murder his wife. Which of the following
passionate affair with Cecily, (2) that, dur- statements is not correct?
ing that time, DeWinter had purchased an
"accidental death" life insurance policy A. Mutuality of estoppel, assuming
on Rebecca's life for $1,000,000 and (3) that it was the applicable law,
that, three months after Rebecca's death, would not permit DeWinter to
DeWinter and Cecily were secretly mar- use collateral estoppel against
ried. DeWinter offered, in his defense, the Floyd's, a stranger to the prior
testimony of Cecily who stated that she was criminal proceeding.
present when Rebecca died and she saw
Rebecca jump off a cliff into the rocks be- B. The Due Process clause prevents
low. The prosecution claimed that Rebecca DeWinter from using collat-
was pushed by DeWinter but could produce eral estoppel against Floyd's,
no eyewitnesses of its own. DeWinter was a stranger to the prior criminal
acquitted and a judgment of acquittal was proceeding.
entered on September 1, 1992.
C. DeWinter cannot use collateral
On October 1, 1992, DeWinter filed a death estoppel because the burdens of
benefits claim with Floyd's of London, the persuasion in the prior criminal
insurance company from which DeWinter suit and in DeWinter's civil suit
had purchased the "accidental death" life are different.
insurance policy on Rebecca's life. Floyd's
refused to pay, claiming that Rebecca's D. In order to acquit DeWinter in
death was not an accident but that she had the criminal trial, the jury must
been murdered by DeWinter. On December have actually and necessarily
1, 1992, DeWinter brought suit in federal determined that there was a rea-
court in Los Angeles to recover on Rebecca's sonable doubt as to whether he
life insurance policy. DeWinter is a British murdered Rebecca.
subject who has been domiciled in Santa
Monica, California since 1983. Floyd's is a
California corporation with its headquarters
in Los Angeles.

110 CIVIL PROCEDURE


98. Assume, for the remainder of this se- 99. Assume, for the remainder of this se-
00
ries of questions, that the court denied ries of questions, that the court denied 0 -
m<
DeWinter's motion in Question 97. DeWinter's motion in Question 98. The CI
r-
On the third day of trial, DeWinter jury rendered a verdict for DeWinter. x3

and Cecil)/ each testified that they saw Two days later, Floyd's moved to dis-
Rebecca accidentally fall off the cliff to miss the suit on grounds that the court
her death. In its case-in-chief, Floyd's lacked subject matter jurisdiction to
produced no eyewitnesses of its own enter judgment on the verdict. Which of
to Rebecca's death. At the close of the following statements is most nearly
all the evidence, DeWinter moved for correct?
judgment as a matter of law, which
was opposed by Floyd's. Which of the A. Floyd's motion should be denied
following statements is most nearly because, by waiting until after
correct? the jury verdict to challenge sub-
ject matter jurisdiction, Floyd's
A. The court should grant the mo- has waived this defense.
tion because Floyd's failed to
meet its burden of production B. Floyd's motion should be denied
to offer testimonial evidence to because the court has alienage
contradict directly DeWinter's jurisdiction.
eyewitnesses.
C. Floyd's motion should be granted
B. The court should deny the mo- because complete diversity does
tion because courts never grant not exist since both DeWinter
an offensive judgment as a mat- and Floyd's are California do-
ter of law where the movant's miciliaries.
case relies heavily on testimonial
evidence. D. Floyd's motion should be granted
because DeWinter's claim does
C. The court should grant the mo- not "arise under" federal law.
tion because DeWinter met its
burden of production.

D. The court should deny the mo-


tion because DeWinter has not
shown conclusively that Re-
becca's death was accidental.

CIVIL PROCEDURE 111


100. Assume for this question, that the D. Even if the court believes Juliet's
court denied Floyd's motion in Ques- affidavit, the court should deny
tion 99. Three weeks after entry of Floyd's motion because it is un-
judgment, Floyd's moved for a new timely.
trial on grounds of jury misconduct.
Floyd's offered in support of its mo-
tion the affidavit of Juliet, a juror, who END OF QUESTIONS
affirmed that Jack, another juror, had
told the jury, during its deliberations,
that he had done his own "snooping
around" and had been told by certain
employees at Floyd's (who spoke with
Jack, not knowing he was a juror) that
Floyd's knew that DeWinter didn't kill
Rebecca and that Rebecca was a noto-
rious "klutz" who probably slipped and
accidentally fell off the cliff. Which of
the following statements is most nearly
correct?

A. If the court believes Juliet's af-


fidavit, the court should grant
Floyd's motion because evidence
of extraneous prejudicial infor-
mation improperly brought to
the jury's attention is admissible
in federal court to impeach the
verdict.

B. Even if the court believes Juliet's


affidavit, the court should deny
Floyd's motion because there is
no showing that Juror Jack's mis-
conduct prejudiced the verdict.

C. If the court believes Juliet's af-


fidavit, the court should grant
Floyd's motion because evidence
of extrinsic misconduct is admis-
sible to impeach the verdict.

112 CIVIL PROCEDURE


CIVIL PROCEDURE 2. C is correct. California, like virtually
all other states, has a statute that au-

ANSWERS thorizes its courts to exercise personal


jurisdiction over persons served within
the state. Such exercise of personal
1. C is correct. Nothing in the facts indi- jurisdiction is probably valid under the

A
cate that Dr. Feelgood is targeting a na- Territoriality Rule, which a plurality of
tional market merely because his office the Supreme Court affirmed in Justice
is located near an interstate off-ramp. A Scalia's opinion in Burnham v. Superior

<_
is incorrect because Dr. Feelgood has Court (1990). A is incorrect because 00
the option of challenging personal juris- the Territoriality Rule, rather than the
diction either in a pre-answer motion or minimum contacts rule, probably gov-
in his answer. B is incorrect because the erns the issue of personal jurisdiction
federal rules do not adopt the "special on these facts. Further, even under the
appearance" procedure for challenging minimum contacts rule, one can argue
personal jurisdiction (see F.R.C.P. 12). that it is "fair" to subject Dr. Feelgood
D is incorrect because Dr. Feelgood did to personal jurisdiction (see Justice
nothing to lure motorists from other Brennan's concurring opinion in Burn-
states to his office in Phoenix. Rigg ham). B and D are incorrect because the
just happened to exit the freeway near- Territoriality Rule probably governs
est Dr. Feelgood's office, and, further, this issue and, even if the "minimum
Rigg just happened to notice his sign, contacts" rule is applicable, B and D do
not while traveling on the interstate, not address the threshold requirement
but after exiting the interstate on Main of purposeful availment.
Street.

CIVIL PROCEDURE 113


3. C is correct. Dr. Feelgood's third-party 5. D is correct. At the time the third-party
complaint does not assert a derivative complaint was filed, Dr. Feelgood and
claim against Drugco. Dr. Feelgood's Drugco were completely diverse par-
third-party complaint denies all liability ties. Subsequent changes in citizenship
to Mack and shifts all liability to Drug- do not automatically divest the court

A
co. A proper third-party claim would of diversity jurisdiction. A is incorrect
allege: "If Dr. Feelgood is held liable because supplemental jurisdiction is
to Mack, then Drugco is liable to Dr. not an issue. The court has diversity
Feelgood." A is incorrect because diver- jurisdiction over all three claims be-
sity of citizenship exists between Mac tween Dr. Feelgood and Drugco. B is
(California) and Feelgood (Arizona) incorrect because the defense that the
and between Feelgood (Arizona) and court lacks subject matter jurisdiction
Drugco (California); thus, each claim can be raised anytime during the life of
is supported by diversity jurisdiction. B the original lawsuit. C is incorrect, first,
is incorrect for reasons stated in answer because supplemental jurisdiction is not
C. D is incorrect because the impleader an issue and, second, because even if
rule (F.R.C.P. 14) says nothing about supplemental jurisdiction were an issue,
"same transaction or occurrence." the court cannot exercise supplemental
jurisdiction over unrelated claims.
4. D is correct. Claim 1 is a derivative
claim, and proper under F.R.C.P. 14(a);
claims 2 and 3 are proper because, once
Dr. Feelgood has properly asserted
a claim against Drugco, F.R.C.P. 18
permits Dr. Feelgood to assert as many
other claims against Drugco as he wants
whether or not those claims are related
to the derivative claim.

114 CIVIL PROCEDURE


6. B is correct. Dave just happened to 8. B is correct. It contains the only argu-
serve a New Yorker (Pat) who just hap- ment that is not plausible under Shaffer
pened to pull into Dave's restaurant. v. Heitner. This is because the argument
Dave's decision to locate his Burger- in answer B is based upon the Ter-
Quick restaurant just off the exit ramp ritoriality Rule and Shaffer v. Heitner
of an interstate does not translate into eliminated the "in rem" wing of the Ter-
purposeful availment of the national ritoriality Rule. Arguments contained
market. Dave did nothing to draw cus- in answers A, C and D are all plausible
tomers from other states. Furthermore, because they each speak to the issue of
the brand name recognition that Dave whether it is "fair" to sue Dave in New
gets from operating a BurgerQuick York based upon his "contacts" with
franchise does not translate into pur- that state.
poseful availment of the national mar-
ket because travelers from other states 9. D is correct. Choices II and III are the
are not coming to Santa Ana just to eat only correct choices. Choice II is cor-
at Dave's particular BurgerQuick. The rect because a case is not removable
benefit to brand name recognition is where original jurisdiction is based
that hungry travelers (including those solely on diversity and a defendant is a
from California) who want to stop for citizen of the forum state. Choice III is
fast food will be more likely to eat at correct because a civil action can only
a BurgerQuick than a "mom and pop" be removed to a federal district court
operation. A is incorrect because Dave for the district and division embracing
did not waive his personal jurisdiction the place where such action is pending
defense under F.R.C.P. 12(b), which, which, under these facts, is the Southern
under Hanna v. Plumer, governs the District of California, not the Central
issue of how to challenge diversity ju- District. A is incorrect because Dave is
risdiction in federal court regardless of a citizen of the forum state of Califor-
the basis of subject matter jurisdiction. nia.
C is incorrect for the same reason B is
correct. D is incorrect because, unlike
the cause of action in the Burger King
case, Pat's tort claim does not arise out
of Dave's contract with BurgerQuick.

7. A is correct. Choices I, II and III each


apply elements that are relevant under
New York's long arm statute and apply
them correctly to the facts.

CIVIL PROCEDURE 115


10. B is correct. CompuChip did not pur- 11. A is most nearly correct. CompuChip
posefully avail itself of the California should be prepared to defend in the
market. In Asahi Metal Industry v Su- U.S. lawsuits arising out of motorcycle
ity opinion stated that "a defendant's computer chips. Although Paul's suit
awareness that the stream of commerce does not directly arise out of the delib-

A may or will sweep the product into the


forum state does not convert the mere
act of placing the product into the
erate marketing of ordinary computer
chips, it significantly "relates" to those
purposeful contacts. B is incorrect be-
stream into an act purposefully directed cause it is completely irrelevant to the
toward the forum State." Even apply- issue of whether CompuChip intended
ing Justice Brennan's "stream of com- to serve the California market in its
merce" theory, purposeful availment "special" chips. C is incorrect because
is lacking because, in Asahi, the sales Paul's cause of action is completely
of the component parts to the manu- unrelated to CompuChip's real estate
facturer of the finished product were holdings, and cannot constitute suffi-
characterized as "regular and exten- cient contacts for personal jurisdiction.
sive." This is clearly not the case with D is incorrect because this contract is
CompuChip which, aside from the ten unrelated to Paul's cause of action. This
"special" chips, had never sold any in contract would be a relevant contact
the U.S.. CompuChip just happened to with California if, for example, Corn-
sell these 10 chips to Dirk's friend who puChip failed to pay the California
unilaterally brought them to the U.S.. A company for its design services.
is incorrect because, unlike Asahi, the
claim arose directly out of an injury to
a plaintiff in California and the plaintiff
is a Californian. C is incorrect because
the Supreme Court has not abandoned
the threshold requirement of purposeful
availment. D is incorrect for reasons
stated above.

116 CIVIL PROCEDURE


12. The best answer is D. Whether or not 14. C is correct. The presence of a non-di-
venue is improper, CompuChip previ- verse defendant destroys the "diversity"
ously brought a pre-answer motion jurisdiction (i.e., 28 U.S.C. section
under Rule 12(b) and omitted from 1332) that supports the claims against
that motion an objection to venue. The both Dirk and CompuChip. A is incor-
venue objection is, therefore, waived rect for the reason stated in connection
under Rule 12(h)(1) even if venue with answer C. B is incorrect because
was improperly laid. Therefore, B is the only damage figure that is relevant
incorrect. A might be correct but only for amount in controversy purposes is
if the court finds that CompuChip had the "ad damnum" figure in the com-
sufficient contacts with the Central plaint. D is incorrect because objection
District of California to be subject to to subject matter jurisdiction may be
personal jurisdiction in that district or raised at any time during the lifetime
if the court determined that a substantial of the original lawsuit.
part of the events giving rise to Paul's
claim occured in the Central District of 15. D is correct. There is no judicial dis-
California. C is incorrect because it is trict in which venue would otherwise
irrelevant, under the current venue law, be proper [under section 1391(b)(1) or
where the plaintiff resides. (2)], and Dirk ("any defendant") may
be "found" in the Southern District. A
13. C is correct. As a matter of joinder, once is incorrect because Diana has a choice
Dirk properly cross-claimed against whether to assert her improper venue
CompuChip, Rule 18 allowed Dirk to defense in her answer or by pre-answer
join as many other claims he may have motion. B is incorrect because it does
against CompuChip whether related or not take account of section 1391(b)(3)
not. There is no problem with subject which is meant to address the situation
matter jurisdiction because alienage where sections 1391(b)(1) and (2) do
jurisdiction supports a claim between not yield a judicial district in which
an American citizen and an alien (here, venue is proper. C is incorrect because
Japanese) corporation. A is incorrect the underlying rule statement contained
because supplemental jurisdiction is in this answer is incorrect [see section
not required (see supra). B is incorrect 1391(b)].
because it ignores Rule 18. D is incor-
rect because personal jurisdiction is no 16. A is the best answer. In order for Turkey
longer an issue once the court denied to be a non-viable alternative forum,
CompuChip's motion to dismiss for its law must not effectively offer any
lack of personal jurisdiction. remedy to Pam. Under the facts, Turk-
ish law is just not as favorable to Pam as
is U.S. antitrust law. Choices B, C and
D each present plausible arguments.

CIVIL PROCEDURE 117


17. C is correct. It is the only answer that 18. C is correct. The court has the constitu-
does not contain a plausible argument. tional and statutory power to exercise
The argument contained in C is based supplemental (formerly known as
upon the old Territoriality Rule. After "pendent") jurisdiction over the state
Shaffer v. Heitner, the Territoriality law claim between non-diverse parties
Rule can no longer be applied to deter- because Pam had originally filed a com-

A mine the constitutionality of quasi-in-


rem jurisdiction under the Due Process
Clause. Each of the other answers is
plaint which contained a colorable (i.e.,
non-frivolous) Sherman Act claim over
which the court had federal question ju-
plausible because each is framed in risdiction. That claim was dismissed for
terms of "fairness" to the defendant- want of sufficient proof (not because the
owner of the res that was seized for complaint's allegations failed to state
jurisdictional purposes. a colorable claim under the Sherman
Act). Pam's state tort claim is so related
to the antitrust claim so that they form
part of the same case or controversy
under Article III of the U.S. Constitu-
tion. Thus, the court has the power to
hear the state claim [28 U.S.C. section
1367(a)] even though the federal claim
has been dismissed. However, the court
also has the discretion under section
1367(c)(3) to decline to exercise this
power. A is incorrect because it fails
to consider the supplemental jurisdic-
tion statute. B is incorrect because
the supplemental jurisdiction statute
permits the court to decline to exercise
this power in circumstances outlined
in section 1367(c). D is incorrect. The
Sherman Act claim was not dismissed
because of a failure of the complaint to
state a colorable claim under the Sher-
man Act, but because of insufficient
evidence.

118 CIVIL PROCEDURE


19. D is correct. The argument in choice II 20. D is correct. First, the impleader was
is incorrect because complete diversity proper because Dan's claim against
is not determined at the time of the ac- Dotty for contribution is a contingent
cident but at the time the complaint is claim for reimbursement. Second, the
filed. The argument in choice IV is not California court can exercise personal

A
correct because subject matter jurisdic- jurisdiction over Dotty because she
tion has nothing to do with where the was personally served with process in
cause of action arose. It is, however, California (see Burnham). A is incor-
relevant to the issue of personal juris- rect because Dotty's attack on personal
diction. Choice I is correct because the jurisdiction was procedurally proper 0
court can consider behavior by the party under F.R.C.P. 12(b). Further, Hanna 0
m< -
subsequent to the filing of the complaint v. Plumer requires the federal court to CP
if it is relevant to that party's intent at apply all federal rules of civil procedure
time of filing the complaint. The facts that are on point over a conflicting state
that Pauline moved to Nevada two rule. B is incorrect because it ignores
weeks after the collision and a month the Territoriality Rule as a valid basis
before filing suit in federal district court for exercising personal jurisdiction
and that she returned to California three under the Due Process Clause. C is
months after filing suit is circumstantial incorrect because Dan is not trying
evidence that she really did not intend, to shift liability over to Dotty thereby
at the time of filing, to remain in Nevada effectively making her the defendant.
for the foreseeable future. Choice III Rather, he is asserting that, if he is held
is correct because these facts provide liable to Pauline, Dotty has to pay her
a basis for a reasonable inference that proportional share of whatever Dan
Pauline was abandoning her California pays to Pauline.
domicile and acquiring a Nevada domi-
cile.

CIVIL PROCEDURE 119


21. C is correct. First, regarding the join- 23. A is correct. The Supplemental Ju-
der issue, once Dan properly asserted risdiction statute permits the federal
(through impleader) a contribution district court to exercise supplemental
claim against Dotty, Rule 18 allows jurisdiction over a claim that lacks an
him to assert as many other claims as independent basis of subject matter

A
he may have against Dotty. Second, jurisdiction but is sufficiently related
regarding the subject matter jurisdiction to a claim that is supported by an inde-
issue, Dan can aggregate his two claims pendent basis of subject matter jurisdic-
against Dotty to satisfy the $75,000 tion. Dotty's personal injury claim is
amount in controversy requirement. sufficiently related because it arises out
A is incorrect because it ignores Rule of the same auto accident as Pauline's
18. B is incorrect because it ignores main claim against Dan. B is incorrect
the aggregation rules. D is incorrect for two reasons. First, the Supplemental
because supplemental jurisdiction is Jurisdiction statute's language should
not required since Dan can aggregate. be cited rather than prior case-law and,
second, Dotty's claim against Pauline
22. C is correct. In determining whether is not a compulsory counterclaim since,
complete diversity exists between Pau- prior to Dotty's assertion of that claim,
line and Dotty, the court looks to the Pauline and Dotty were not opposing
date Pauline amended her third-party parties. C is incorrect because Kroger
complaint against Dotty. A is incorrect involved a claim by the plaintiff against
because the court does not look to the the third-party defendant, not a claim
date Pauline filed her complaint against by the third-party defendant against
Dan to determined whether diversity the plaintiff. D is incorrect because this
existed between Dan and Dotty. B is answer ignores the problem that Dotty's
incorrect because the court has already claim falls below the $75,000 amount
ruled that it can exercise personal ju- in controversy.
risdiction over Dotty (see question 20,
supra). D is incorrect because supple-
mental jurisdiction is not required since
diversity jurisdiction supports (indepen-
dently) Pauline's claim against Dotty.

120 CIVIL PROCEDURE


24. C is correct. The U.S. Government is 26. D is most nearly correct. The potential
not asking, in its third-party complaint, conflict of interest between Pete, as
for reimbursement from the captains for the class representative, and Pete, as
whatever refund the Government has lawyer for the class, would be too great
to pay to Shrimpboats, Inc. Rather, the for class representation to be adequate.
Government is asserting an independent While the other answers contain poten-
claim against the captains based upon
their alleged employment tax liability
to the U.S. (if the court determines that
tially viable arguments, the best answer
is D. A
'V
X:3
Shrimpboats, Inc. does not owe that 27. D is correct. The class members are 00
tax liability to the U.S.) A is incorrect passengers on the flight who all claimed —
<
because the impleader rule [F.R.C.P. to have been injured as a result of the ci
C r=
X1
14(a)] does not state anything about pilot's negligence. The only relief the m

"same operative facts." B is incor- class is seeking is damages. A is incor-


rect because it is possible that neither rect because, even if some passengers
Shrimpboats, Inc., nor the captains are were to win against Global while oth-
liable to the Government. The court ers were to lose, this does not impose
might deem the crewmen to be self- incompatible standards of conduct on
employed. D is incorrect for the reasons Global. B is incorrect because there is
stated in C. no evidence that a host of individual
damage suits would bankrupt Global. C
25. A is correct. Medco is, in effect, argu- is incorrect because the class is seeking
ing that the two other joint tortfeasors damage relief, not injunctive or declara-
should have been joined by Pete in tory judgment relief.
the federal court suit against Medco.
Joint tortfeasors are not necessary or 28. A is correct. F.R.C.P. 33(c) states: "An
indispensable parties under Rule 19. It interrogatory otherwise proper is not
is up to the plaintiff to determine which necessarily objectionable merely be-
defendants to sue in which actions. B cause an answer to the interrogatory
is incorrect because it fails to apply the involves an opinion or contention that
elements of F.R.C.P 19(a). C is incor- relates to fact or the application of
rect because Medco, in the absence of law to fact...." Therefore, all the other
the Dr. Zorba and the hospital, will still choices are incorrect.
be able to assert its defense that they
were the cause of plaintiff's injuries.
D is incorrect because plaintiffs do not
have an absolute right to decide whom
to sue.

CIVIL PROCEDURE 121


29. C is correct because it misstates the 31. D is correct. Global was obviously
applicable legal standard for initial dis- using Prof's services to see whether
closure. As part of "Initial Disclosures," he would be able to testify favorably
under F.R.C.P. 26(a)(1)(A), a party must for Global at trial. Two hours worth of
disclose the identity of "each individual investigation, a five-page report and a
likely to have discoverable information $200 "honorarium" does not indicate a

A that the disclosing party may use to


support its claims or defenses, unless
solely for impeachment, identifying
thorough investigation. Global appar-
ently found out all they needed to know.
They never contacted Prof again. A is
the subjects of the information." A is incorrect because it states the rule for
incorrect because this information is overcoming the work product immu-
required to be disclosed as part of initial nity. B is incorrect because the contents
disclosure under F.R.C.P. 26(a)(1)(D). of Prof's report are relevant to the sub-
B is incorrect because this informa- ject matter (i.e., the cause of the crash)
tion is required to be disclosed under and, also, the purpose of Prof's inves-
F.R.C.P. 26(a)(2)(A) ("Disclosure of tigation was apparently motivated by
Expert Testimony"). D is incorrect litigation. Global wanted to determine
because this information is required to Prof's suitability as a trial witness. C is
be disclosed under F.R.C.P. 26(a)(3)(A) incorrect because it assumes that Prof
("Pretrial Disclosures"). was "retained or specially employed"
by Global.
30. C is correct. That portion of the cap-
tain's deposition transcript containing 32. B is correct. Pete seeks to depose Mack
his "yes" response would be admissible to discover facts that Mack learned as
as a "vicarious" party admission. It is an ordinary percipient witness (i.e.,
also admissible to impeach the captain's what Mack discovered as he checked
trial testimony that he had nothing to out the plane for take-off). Mack did not
drink before take-off (as a "prior incon- learn these particular facts in prepara-
sistent statement"). Therefore, A and B tion for trial but pursuant to his accident
are incorrect. D is incorrect because the prevention duties as an employee in
captain's deposition testimony, while Global's Accident Prevention Depart-
admissible (under the party admission ment. A is incorrect because it states
rule) as evidence of the truth of his the standard for overcoming the work
statement that he had been drinking, product immunity. C is incorrect for the
does not constitute a judicial admission, reason that B is correct. D is incorrect
which removes the fact admitted out of because the facts learned by Mack at
the case altogether. the crash site are clearly relevant to the
cause of the crash.

122 CIVIL PROCEDURE


33. A is correct. Whit's eyewitness obser- 34. B is correct. It is the only choice that
vations of the crash are clearly relevant states an incorrect answer. It is the
and no rule protects his note to Global's old-fashioned, and fast-eroding, com-
lawyers. B is incorrect because Whit mon law rule of "mutuality of estop-
did not acquire his facts and opinions pel," not the Due Process Clause, that
in anticipation of litigation. While he would prevent Opie, a stranger to the
did write the note to Global's attorneys federal class action suit, from invoking
with the likelihood of a lawsuit against collateral estoppel. The Due Process
Global in mind, he was not retained Clause is inapplicable because Global
by Global to observe or investigate the did, in fact, have its day in court on the
crash. C is incorrect because the note issue of pilot error as the cause of the
is neither the work product of Global crash. A is incorrect because it states
nor its attorneys. It was a note volun- a correct answer. The California state
tarily written and sent to Global by a court must give the same enforcement
third party. No effort was expended scope to the federal class action judg-
by Global or its counsel to secure this ment that the federal courts give to that
note. D is incorrect because Whit was judgment. C is incorrect because it,
not communicating with either his own too, contains a correct answer. Federal
or his employer's attorneys. law, under Parklane Hosiery, gives the
federal court discretion to refuse to
allow a stranger to invoke collateral
estoppel if the stranger stayed out of
the earlier suit because of a "wait and
see" attitude. D is incorrect because it
is a correct answer: California must
give the same enforcement scope to a
federal court judgment as the federal
courts give that judgment. Thus, if the
federal courts still applied a strict mu-
tuality rule (which, of course, they do
not), California courts could not allow
Opie to invoke collateral estoppel.

CIVIL PROCEDURE 123


35. A is correct. This version of the statute 36. B is correct. Although Sporter was "in
violates the Seventh Amendment be- default" for having failed to answer or
cause it deprives a party of his right to otherwise defend against the suit, his
have all issues relating to the damage act of engaging in settlement negotia-
claims tried to a jury. The other versions tions throughout the six-week period
of the statute — II, III and IV — are all indicates that he did not intend to write-

A constitional; therefore, B, C and D are


incorrect. The terms of the statute in
version II provide for jury trials in both
off Hack's suit. Therefore, Sporter 's
settlement activities would be deemed
an "appearance" entitling him to notice
damage and injunctive actions. Such a before default judgment is entered. A
statute would not violate the Seventh is incorrect because Sporter did not
Amendment because there is no right timely answer or otherwise defend. C
to a judge (or bench) trial in a formerly is incorrect because it does not address
equitable claim. The terms of the statute the crucial "notice" issue. D is incorrect
in version III do not address the jury because it misstates the facts.
trial issue, but a court would apply the
Seventh Amendment to require a jury 37. B is correct. The plaintiff [under F.R.C.P.
trial of all issues pertaining to the dam- 41(a)] has a right to voluntarily dismiss
age claim (if a jury trial were timely his own suit in federal court until the
demanded). The statute in version IV defendant answers the complaint or
provides for a judge, not a jury, to try moves for summary judgment, which-
issues in suits brought by the federal ever occurs first. Here, Hack filed his
government for the imposition of fines notice of dismissal after Sporter served
to vindicate public rights. Such suits, his answer. Hence, the notice should
tried before an administrative tribunal be quashed. A is incorrect because it
before an administrative law judge, ignores the fact that Sporter served his
have been held not to require juries answer before Hack filed his notice
under the Seventh Amendment. of dismissal. C is incorrect because it
does not apply the correct terms of Rule
41(a). D is incorrect because it contains
an incorrect statement. Plaintiffs do
have a limited right to voluntarily dis-
miss their actions under Rule 41(a).

124 CIVIL PROCEDURE


38. B is correct. Sporter did not meet 40. D is correct. There is a clear conflict
his movant's burden because, from in the evidence that the jury must be
Sporter's moving papers, there is still allowed to resolve. Despite the doubt-
a material issue of fact for the jury to ful credibility of Hack's story, the jury
resolve, i.e., whether the plaintiff's - not the judge - is assigned the job of
songs and the defendant's songs are determining credibility. If they believe
so similar the jury could infer copy- Hack's eyewitness account, they can
ing by the defendant. How the judge
would find if he were the trier of fact
reasonably find that Sporter had access
to, and copied, Hack's songs. Therefore,
A
is irrelevant. A is incorrect for reasons A and C are incorrect. B is incorrect 00
stated in connection with answer B. C
is incorrect because Hack did not have
because F.R.C.P. 50 permits, but does
not require, the motion for judgment c
<
a respondent's burden, since Sporter as a matter of law to be made after the
failed to satisfy his movant's burden. opposing party has been fully heard
D is incorrect because whether or not with respect to a material issue. Rule
the two songs were similar enough to 50(b)(2) states that "motions for judg-
warrant the inference of copying by ment as a matter of law may be made at
Sporter is a material issue of fact that any time before submission of the case
the jury must resolve at trial. to the jury."

39. D is correct. It is the only choice which 41. C is correct. The two observations of
correctly states the law under F.R.C.P. Hack's behavior in the courtroom are
16(e), i.e., "The order following a final relevant to Hack's credibility which is a
pretrial conference shall be modified crucial issue in the case. It is an issue for
only to prevent manifest injustice." the jury to resolve. The jury did noth-
ing improper in basing its credibility
determination on their observations of
Hack in the courtroom. Therefore, B is
incorrect. The jurors' observations of
Hack were not improper. A is incorrect
because the jurors' affidavits refer, in
part, to activity outside the heads of
the jurors, i.e., observations which they
made of Hack's behavior, and not just
the effect of such observations on the
thought processes of those jurors. D is
incorrect because Hack's credibility is
very crucial. He claims to have person-
ally witnessed Sporter copying Hack's
music.

CIVIL PROCEDURE 125


42. A is correct. To be reviewable by an ap- 44. D is correct. Under the traditional (and
pellate court, an error by the court must still majority) common law view, there
be objected to at the trial court level. are no compulsory counterclaims even
Otherwise, that objection is waived. if the defendant's counterclaim arises
Therefore, C is incorrect. B is incor- (as does Dan's) out of the same opera-
rect because it is nonsense to speak of tive facts as an affirmative defense that

A "errors" as interlocutory. One speaks


of "decisions" as being interlocutory or
final for purposes of the final decision
had been asserted by the defendant in
the prior suit. A is incorrect because
the applicable law would not be the
rule. Hack's appeal was taken from a federal compulsory counterclaim rule,
final judgment. D is incorrect because but rather the traditional common law
Hack's situation does not fit the strin- rule of the state court that rendered
gent requirements of the "exceptional the judgment in Pat v. Dan and Dave.
circumstances" doctrine summarized Moreover, choice A incorrectly states
by the 5th Circuit in Carson Products that traditional common law rule. B is
v. Califano. incorrect because it, too, incorrectly
states the traditional common law
43. A is correct. Sporter is asking the Court view. C is incorrect because it applies
of Appeals for more than he received the emerging minority common law
from the trial court. Therefore, he view, not the traditional common law
should have cross-appealed. As appel- approach.
lee, this is not the kind of attack Sporter
can make on the trial court's rulings.
Therefore, B, C and D are incorrect.

126 CIVIL PROCEDURE


45. B is correct. In order for the jury to find 46. B is correct. Dan chose to assert his
for Dan on Dan's cross-claim against cross-claim against Dave for property
Dave, it had to make an actual deter- damage ($500) to his car but "split his
mination of fact that Dan was not con- cause of action" when he omitted his
tributorily negligent and that Dave was claim ($60,000) for personal injury
negligent. If Dan had been contributor- arising out of the same auto accident
ily negligent, the jury could not have transaction as the property damage
awarded a verdict to Dan on his cross- cross-claim. A is incorrect because
claim. Therefore, A is incorrect. C is cross-claims in federal court are only
incorrect because the verdicts for Dave permissive, never compulsory [see
against Pat and for Dan against Dave F.R.C.P. 13(g)]. C is incorrect because
are not hopelessly irreconcilable. The it ignores the fact that when Dan chose
jury must have found that Pat was con- to assert his cross-claim against Dan
tributorily negligent so that she cannot for property damage, he should have
recover against either Dan or Dave. But, included his personal injury claim under
the jury must have found that Dan was the federal common law of res judicata,
also not negligent, and that Dave was, which adopts the "same transaction"
so that Dan can recover against Dave. test for defining the scope of the cause
D is incorrect because there is nothing of action. D is incorrect because the
in the facts to cast doubt on whether Pat state court in Dan v. Dave will apply
had a full and fair opportunity to litigate the law of the jurisdiction (here, fed-
the issue of Dan's negligence. After all, eral) that rendered the judgment as to
she was the plaintiff and she sued Dan the enforcement effect to be given that
for $55,000. judgment. Here, the federal common
law of res judicata does not apply the
"primary rights" test.

CIVIL PROCEDURE 127


47. B is correct. Dan's original property 49. A is the correct answer because the test
damage cross-claim was for only $500 for determining whether Pat split her
in damages. His subsequent personal in- "cause of action" is the "same transac-
jury claim was for $60,000 in damages. tion" test (since Pat brought her previ-
Dave can hardly claim that he did not ous trespass suit against Don in a fed-
have adequate incentive to fully litigate eral district court) and the cutting down

A the issue of his (and Dan's) negligence


since Dave was also defending himself
against Pat's $55,000 claim against Dan
of the timber occurred at the same time
as the trespass and one would (for the
sake of judicial economy) expect to try
and Dave. Given the realities of litiga- both the trespass and conversion claims
tion, this gave Dan plenty of incentive in the same suit. B is incorrect for the
to show that Dan was negligent and reasons stated in answer A. C and D
Dave was not. A is incorrect for rea- are incorrect because they are based
sons given in the answer to question upon the wrong tests for determining
45 supra. C would have been correct the scope of the "cause of action" as-
if Dave had not also been defending serted by Pat in her trespass suit against
himself against Pat's $55,000 claim. Don.
D is incorrect because Dave was not a
"stranger" to the earlier Pat v. Dan and
Dave lawsuit.

48. C is the correct answer because a dis-


missal on statute of limitations grounds
is not deemed to be "on the merits"
if the plaintiff can find a jurisdiction
where the action is not barred by the
local statute of limitations (as in Pat's
case). Therefore, answers A and D
are incorrect. Answer B is incorrect
because a judgment, under appropriate
circumstances, can be deemed to be "on
the merits" although not literally "on the
merits," in the sense of an adjudication
based upon the evidence presented at a
hearing.

128 CIVIL PROCEDURE


50. C is most nearly correct because a per- 51. B is the correct answer. Notice pleading
son who was neither a party to a suit nor rules require only a short and plain state-
in privity with a party cannot be bound ment of the claim showing the pleader is
by the judgment in that suit. B would entitled to relief. Particularized facts are
have been the correct answer were it not required. Paul should have the right

A
not for the fact that Paul was neither to conduct discovery into the particular
a party, nor in privity with a party, to defects, if any, and Flexus will then be
Pat's suit against Don. A is incorrect be- able to discover (e.g., through conten-
cause it cannot be determined from the tion interrogatories) the details of Paul's
7:3
jury's general verdict for Don whether conclusory contentions. A is incorrect 00
the jury actually found that Pat did not for the reasons that B is correct. C is -
<
incorrect because Rule 12(b)(6) permits 0
own Blackacre (and therefore cannot c
recover against Don) or that Don did not Flexus to attack the sufficiency of the
come upon Blackacre (and, therefore, complaint in its answer if it had not pre-
Pat cannot recover against Don even viously made a pre-answer motion. D is
if Pat owned Blackacre). D is incor- incorrect because, unlike state "special
rect because the mutuality of estoppel appearance" practice, Federal Rule 12
doctrine does not prohibit collateral allows a defendant to assert the insuf-
estoppel from being invoked against a ficiency of the complaint as a defense
stranger (the Due Process clause does in the answer (along with denials).
that), but prohibits a stranger from in-
voking collateral estoppel. (Further, the 52. C is the correct answer. Flexus's refusal
mutuality rule is no longer an absolute to disclose was based upon its argument
bar to a stranger using collateral estop- that Paul's complaint did not allege with
pel in federal court). particularity the parts of Paul's car that
were defective. Rule 26 (a)(1) no longer
requires that disputed facts be alleged
with particularity in order to trigger
disclosure obligations. Flexus has a
duty to disclose to Paul the identity of
each individual likely to have discover-
able information that Flexus may use to
support its defenses to Paul's action. A
and B are incorrect for the reason C is
correct. D is incorrect because the infor-
mation requested is plainly relevant.

CIVIL PROCEDURE 129


53. D is the correct answer. Rule 26(b)(4)(A) 56. A is the correct answer. Under the
provides that all experts who will be more liberal federal standard for grant-
testifying witnesses at trial may be de- ing summary judgment after Celotex,
posed. There is no requirement that the most federal courts would probably
discovering party serve interrogatories reject Larry's argument that, by trial,
beforehand. These former prelimi- he may be able to show that Joe signed

A nary requirements were eliminated by


amendments to Rule 26(b)(4) effective
in December 1993. A is incorrect be-
the lease. Joe has presented too much
corroborating evidence not to shift the
burden onto the respondent, Larry, to
cause Rule 26(b)(4) recognizes Paul's show that he will be able to meet his
legitimate need to depose Egbert in burden of production on the issue of
order to prepare for Egbert's cross-ex- Joe's signature at trial. B is incorrect for
amination at trial. B is incorrect for the the reasons why A is correct. C is incor-
reasons that D is correct. C is incorrect rect because a defendant can make a
because the question of the protection summary judgment motion before filing
of the facts and opinions of an expert an answer (See Rule 56). D is incorrect
is a Rule 26(b)(4) issue, not a 26(b)(3) because, after Celotex, a movant does
work product issue. The work product not have to negate every possible way
rule does not cover "facts." the respondent might meet their burden
of production on the material issue.
54. A is the correct answer. Rule 26(a) gives
Flexus a choice either to identify the 57. C is the correct answer. This is a res
documents in question or to provide judicata question. When Larry filed
Paul with actual copies of those docu- suit, on June 1, 1992, Larry's "cause
ments. Therefore, B and D are incorrect. of action" did not include his claim for
C is an incorrect statement of the law the May rent which was then due and
under Rule 26(a). owing. Since Larry only asserted his
claims for the March and April rents,
55. D is the correct answer. Rule 26(a)(3) but not the May rent, Larry "split his
requires Paul to disclose his trial wit- cause of action" and is, therefore, pre-
nesses at least 30 days before trial cluded from asserting his claim for the
without the need for Flexus to serve a May rent in a subsequent action. B is
discovery request. Therefore, B is in- incorrect for the reasons that A is cor-
correct. A and C are incorrect because rect. A and D are incorrect because these
the issue here is not one of discovery, answers are inappropriate given the fact
under Rule 26(b), but of automatic that the real issue is not summary judg-
disclosure under Rule 26(a). ment per se, but res judicata.

130 CIVIL PROCEDURE


58. D is the correct answer. C is correct 60. B is the correct answer. Anon-party has
because a summary judgment is a judg- an unqualified right to get a copy of his
ment "on the merits" which precludes own witness statement and a party has
Larry from re-litigating the issue of an unqualified right to get a copy of
whether Joe signed the lease. In grant- his own witness statement. But, a party
ing summary judgment, the judge does not have an unqualified right to get
actually and necessarily determined, a copy of a non-party's witness state-
based upon documentary evidence and
as a matter of law, that Joe did not sign
ment. Pat will have to ask Joe to request
from Plucky's a copy of Joe's witness
A
-o
the lease. B also supports dismissing statement and to give it to Pat. A and 0 0
Larry's current suit against Joe (for
the May rent) for the same reason that
C are incorrect for the reasons stated
supporting B. D is incorrect because it
<
C7

the court dismissed Larry's prior suit assumes that the witness statement is
against Pete (for the May rent), i.e. not even "work product" because Sam
Larry should have included his claim is not a lawyer. As long as Sam obtained
for the May rent in his suit against Pete the statement "for" Plucky's, a party de-
and Joe commenced June 1, 1994. A is fendant, in anticipation of litigation, the
incorrect because res judicata does not statement falls within the work product
require a plaintiff to join as defendants rule.
all persons liable to that plaintiff.
61. B is most nearly correct because Lou
59. D is the best answer because the condi- obtained the witness statement from
tion of the water heater from month-to- Wally presumably in anticipation of
month may change (it may have worked litigation. A is incorrect because Wally
in March and April, but only failed was not Lou's client. C is incorrect be-
in May) so that the issue whether the cause Wally was an eyewitness to the
water heater was functioning in March accident. D is incorrect because there is
and April is not identical with whether no basis for a claim of confidentiality
it was functioning in May. Because the (e.g., that Wally would somehow be
issue is not identical, choices A, B and harmed by disclosure of his statement)
C are not the best answers. and, in any event, Wally should have
moved for a protective order for confi-
dentiality protection.

CIVIL PROCEDURE 131


62. C is the correct answer. The subsequent 63. D is the correct answer because (1)
remedial measures exclusionary rule the presence of refuse on the floor of
operates only at trial to bar the admis- Plucky's Anaheim Hills supermarket is
sion of evidence, not at the discovery relevant to the cause of Pat's accident,
phase. The issue here is "discover- (2) the dominant purpose behind the
abilty," not "admissibility" (of evidence report's preparation was to record the

A at trial). The general scope of discovery


rule [26(b)(1)] states that information
is not necessarily beyond the scope
results of a health investigation. While
it is true that there is a dual purpose
here, i.e., to promote sanitary conditions
of discovery simply because it will (the mission of the Health Department)
be inadmissible as evidence at trial and to prepare for a possible suit by the
(as long as it is reasonably calculated Health Dept. against Plucky's for viola-
to lead to the discovery of admissible tions of the Health Code, the question
evidence). A is incorrect because Ed's is: what was the dominant purpose? If
memo was not prepared in anticipation the former, then the report is not pro-
of litigation. B is incorrect because the tected. In any event, the report is not
work product rule protects documents protected as work product, even if the
prepared to prosecute or defend a latter purpose was dominant, because
lawsuit, not to prevent one. D is incor- the report was not prepared by any of
rect because the subsequent remedial the parties to the Pat v. Plucky lawsuit
measures rule operates only at trial to for use in litigating that suit. Therefore,
exclude evidence. A and C are incorrect. B is incorrect
because there is nothing in the facts to
suggest that the Health Dept. report is
confidential.

64. A is the correct answer. Although Sam's


report itself is protected by work prod-
uct immunity, the underlying facts that
Sam learned during his investigation
of the accident are not. The same is
true for the attorney-client privilege.
There, only the "communication" itself
(here, Sam's report) is protected from
disclosure, not the underlying informa-
tion. Thus, B and D are incorrect. C is
not the best answer mainly because, to
invoke the court's discretionary protec-
tion, Plucky's should have moved for a
protective order.

132 CIVIL PROCEDURE


65. C is the correct answer. The jurisdic- 67. D is the correct answer. The issue here
tional "amount in controversy" require- is whether the court can assert supple-
ment is met since the claim against each mental jurisdiction over General's
defendant was brought under the joint cross-claim, since the cross-claim is
and several liability theory thereby ex- not supported by either diversity or

A
ceeding the required $75,000 amount . federal question jurisdiction (state tort
The damages against each defendant are claim for contribution between two
not $40,000 but $80,000 each. A and B Californians). Under prior case law,
are incorrect because they assume that federal courts could assert ancillary
the damages against each defendant are jurisdiction over proper cross-claims. 00
$40,000. A is also incorrect for the ad- General's cross-claim for contribution m<
ditional reason that "same transaction is "proper" under Rule 13(g). The court C7

or occurrence" is an inapplicable rule can assert "supplemental jurisdiction"


statement. D is incorrect for the reasons over the cross-claim because the claim
why C is correct. for contribution is clearly "related" to
Owner's main claim. Therefore, A and
66. A is the correct answer. The City's coun- B are incorrect. C is incorrect because
terclaim is not supported by "diversity" it addresses only the joinder issue and
jurisdiction because the jurisdictional ignores the subject matter jurisdiction
amount in controversy is not satisfied. issue.
Nor is the counterclaim supported by
"federal question" jurisdiction because,
under the "well-pleaded complaint
rule," the federal question must be
raised by an essential element of the
City's counterclaim, not by Owner's
response to that counterclaim. Further-
more, the court cannot assert supple-
mental jurisdiction over the counter-
claim because it is not related to the
subject matter of Owner's main claim
(in other words, it is not a compulsory
counterclaim). Therefore, B and C are
incorrect. D is incorrect because it has
no basis in law.

CIVIL PROCEDURE 133


68. A is the correct answer. Impleader is 69. A is the correct answer. Both Claims
improper because General's third-party #1 and #2 need supplemental jurisdic-
complaint asserts an "independent," tion because they are each state claims
instead of a "derivative" claim against between two Californians. The court
Archie. The claim is not derivative be- can exercise supplemental jurisdiction

A
cause General cannot be held liable, un- over Claim #1 because it is sufficiently
der the law, for the misdeeds of Archie "related" to Owner's main claim (and,
- because General did not hire Archie; under prior case law, federal courts have
Owner did! General's third-party claim exercised supplemental jurisdiction
is independent of Owner's main claim over properly impleaded claims). How-
because General is not seeking indem- ever, even though Claim #2 is properly
nity from Archie in the event General joined under Rule 18, the court cannot
is found liable to Owner but, rather, exercise supplemental jurisdiction over
General is seeking to be compensated this claim because it arises out of an-
for damage to its business (i.e., bank- other project and is, therefore, unrelated
ruptcy). Thus, even if General is found to the subject matter of Owner's main
not liable to Owner, General can still claim. Therefore, B, C and D are incor-
recover against Archie for destroying rect.
General's business. Thus, C is incor-
rect. B is incorrect because General is a
Californian and Archie is a New Yorker
and the amount in controversy exceeds
$75,000. D is incorrect because there is
no supplemental jurisdiction issue since
the third-party claim is supported by
diversity jurisdiction.

134 CIVIL PROCEDURE


70. C is the correct answer. The supplemen- 71. A is the correct answer. When General
tal jurisdiction statute codifies the U.S. cross-claimed against the City for con-
Supreme Court's holding in the Kroger tribution in the previous suit, the City's
case by stating that federal courts can- claim for contribution should have
not exercise supplemental jurisdiction been asserted and joined by means of a

A
over a claim (like Owner's against Fred, compulsory counterclaim. B is incorrect
both New Yorkers), which is dependent because the City's contribution claim
solely on diversity jurisdiction, where would have been asserted against Gen-
the exercise of supplemental jurisdic- eral as an "opposing party" (not merely _o
XI
tion would undermine (or be inconsis- a "co-party") and, therefore, should be 0 0
tent with) the requirements of complete characterized as a "counterclaim," not -■•
<
CP
diversity. Here, Owner could not have a "cross-claim." C is incorrect because C r -

initially sued both General and Fred in no rule requires General to object to the m

its original complaint because diversity City's failure to assert its compulsory
would have been incomplete. A is in- counterclaim. D is incorrect for the
correct because there is no basis in law same reason why B is incorrect and for
for this answer. B is incorrect because the additional reason that cross-claims
it only addresses the joinder issue, and are never compulsory.
not the jurisdictional. Because Owner's
claim against Fred does, indeed, arise 72. D is the correct answer. The court has
out of the same transaction and occur- federal question jurisdiction over all the
rence as Owner's claim against General, security and exchange violation claims
that fact merely permits Owner, as a which arise under federal law. The
joinder issue, to assert a claim directly general federal question statute does
against Fred as a third-party defendant not require a jurisdictional "amount in
brought in by General. Despite proper controversy." Therefore, A is incorrect
joinder under Rule 14(a), however, because aggregation is not an issue
the supplemental jurisdiction statute since "amount in controversy" is not
(codifying Kroger), does not permit the an issue. B is incorrect since federal
court to assert supplemental jurisdiction question jurisdiction alone provides
over this claim. D is incorrect because an independent basis of subject matter
subject matter jurisdiction can be chal- jurisdiction; diversity is not required.
lenged anytime during the litigation and C is incorrect for the additional reason
even on appeal. that it misstates the diversity rules.

CIVIL PROCEDURE 135


73. C is the correct answer because the only 76. A is the correct answer. As set forth
relief sought was damage relief and the in Oppenheimer Fund, Inc. v. Sanders
only thing the class members have in (1978) 437 U.S. 340, that the cost of
common is that they claim to have been notice shall be borne by the representa-
damaged in a similar way. A is incorrect tive plaintiff, not by the defendant and

A
because the interests of neither class judicial attempts to improvise beyond
members nor I.B.N. will be prejudiced the express letter of the rule have been
in a way specified in Rule 23(b)(1). B is rejected by the U.S. Supreme Court
incorrect because the class is not seek- (See Eisen v. Carlisle & Jaquelin).
ing injunctive relief. D is incorrect for Therefore, B, C and D are incorrect.
the reason that A is incorrect.
77. A is the correct answer because Don's
74. C is the correct answer because it accu- wealth is not relevant to the subject
rately tracks the notice rule [Rule 23(c)] matter of Paula's lawsuit. It has no
specifically applicable to 23(b)(3) bearing on the issues of liability or
damage class actions. A and B omit recoverable damages; solely on the
some part of this notice rule. D is not defendant's ability to pay a judgment.
authorized by Rule 23(c). As such, the information in the report
is not reasonably calculated to lead to
75. A is the correct answer. Joe's dual status the discovery of admissible evidence.
as class representative and as attorney Since the information in the report is
for the class poses a potential conflict of irrelevant, A is the best answer and,
interest. As counsel for the class, Joe's consequently, B, C and D are not. C is
interest is in maximizing his attorney's incorrect for the additional reason that
fees so that, for example, as class the report was not prepared in anticipa-
representative, he would not oppose tion of litigation but, rather, to help Don
a settlement agreement that provided prepare his will. D is incorrect for the
for big attorney fees at the expense of additional reason that the report was not
respectable damages for the class. B, C prepared by the accountant in anticipa-
and D are, therefore, incorrect. tion of litigation.

136 CIVIL PROCEDURE


78. A is the correct answer. Rule 26(a), re- 79. D is the correct answer. While it is true
garding automatic "disclosure" obliga- that determination of witness credibility
tions of parties, requires that an expert is the jury's job, even if the jury believes
who has been retained by a party to Eggbert's testimony, the jury could only
provide expert testimony at trial prepare "speculate" (i.e., flip a mental coin),

A
a detailed report regarding that pro- based solely upon that testimony, that
posed trial testimony. Rule 26(b)(4)(A) Don caused the accident by crossing
unconditionally permits the deposition into Paul's lane. This is true since the
of experts who are designated to pro- expert testified only that Don's truck
vide trial testimony, but requires that
such deposition not take place before
"could well have" veered into Paul's
lane, not that it is likely that he did so.
OC
the report [required by Rule 26(a)] is Also, on cross-examination, Eggbert
received by the deposing party. B is conceded that it was equally possible
incorrect because Rule 26(b)(4)(A) no that Paul's truck veered into Don's lane.
longer requires the service of special Therefore, B is incorrect. C is incorrect
interrogatories as a pre-condition to a because, as already explained, the jury
motion to depose the expert witness. could not rationally find in favor of
C is incorrect because the rule it cites Paula, who has the burden of produc-
applies only to retained or specially tion on liability; therefore, there is no
employed non-testifying experts. D is conflict in the evidence. A is incorrect
wrong for the reason why A is correct. because, under current Rule 50, a mo-
tion for a judgment as a matter of law
can be made as soon as the respondent
has been fully heard on the material
issue that is the subject of the motion.

80. B is the correct answer. Although the


affidavit of Willy's ex-wife indicates
that the condition of Willy's eyes are
"in controversy," Willy cannot be
compelled to submit to an eye exami-
nation because he is only a non-party
witness (neither a party nor a person in
the custody or under the legal control
of a party.) Therefore, A, C and D are
incorrect.

CIVIL PROCEDURE 137


81. B is the correct answer because Del's re- 82. A is the correct answer. Since Pam has
cord of reckless driving could be the re- made an offensive summary judgment
sult of any number of reasons and does motion, she has the heavy movant's
not point specifically to poor reflexes or burden of showing conclusively that
some form of mental illness. Therefore, Del ran the red light which Willy's
Pam has not met the "in controversy" affidavit fails to do. While Willy's af-

A requirement of Rule 35. A is incorrect


because an employee is not deemed to
be within the custody or legal control
fidavit shows that, had the light at the
intersection in question been red for
Del, he probably would have run it, the
of his employer. C is incorrect because odds are only 50-50 that the light was
it only establishes the relevance of any red for Del . Had it been green, then
information relating to Del's ability to Del's contempt for red lights would not
drive a vehicle but does not address have been the cause of this collision.
the "in controversy" requirement. D is Therefore, Willy's affidavit does not
incorrect because Pam's attorney's affi- provide a basis for a reasonable infer-
davit is based upon personal knowledge ence that Del caused the accident, let
of the contents of the court files. alone conclusively establish that fact.
B is incorrect because Acme does not
have a respondent's burden because
Pam failed to meet her movant's bur-
den. C is incorrect because there is not
a two-sided fact issue on causation
because Pam did not present enough
evidence to support a rational inference
that Del caused the collision. D contains
an incorrect statement of the law.

138 CIVIL PROCEDURE


83. C is the correct answer. As a prerequisite 84. D is the correct answer. Federal Rule of
for a renewed motion for judgment as Evidence 606(b) bars any inquiry into
a matter of law, the movant must have the thought processes of jurors (which
previously made a motion for a judg- would include "bias"). The same is true
ment as a matter of law by the close of for the Mansfield Rule and the Iowa
all the evidence. Pam did not do this. Rule (which would consider "bias"
While it is true that, even if she had so intrinsic misconduct and, therefore, not
moved, her renewed motion would still
have been denied because the jury has
provable). A is incorrect because there
is a recent trend toward close appellate
A
the right to believe the twice convicted scrutiny, and some reversals, of orders 00
perjurer over the two bishops, this was
not one of the choices given in the
granting new trials. B is incorrect be-
cause the trial judge's order granting the
<
question. A is an incorrect statement of motion for judgment as a matter of law
the law. B is incorrect because, in order is a "final judgment" and Rule 50 ex-
for Pam to win her motion, she must pressly states that the conditional grant
show that she did more than merely of a new trial motion does not change
meet her burden of production, but that the situation. C is incorrect because it
she proved her case conclusively. Only fails to take account of D.
then would the burden of production
have shifted to Acme. D is incorrect 85. A is the correct answer. Complete di-
(1) because the term "overwhelmed" versity exists because Pam is a citizen
is properly used only to describe the of Nevada (her home is in Las Vegas)
situation where plaintiff has barely and Beers is a California citizen. The
met her burden of production and the amount in controversy is satisfied
defendant's evidence is so strong that because Pam claims $105,000 in dam-
it "overwhelms" plaintiff's case enti- ages and Beers cannot prove to a legal
tling defendant to a defensive motion certainty that she cannot recover that
for judgment as a matter of law; and amount. B is incorrect because solicit-
(2) because there was a conflict in the ing business in Nevada does not make
evidence, thus, creating a two-sided Beers a Nevada citizen. C is incorrect
credibility issue for the jury. because Pam does not have to prove to a
legal certainty that she will recover her
damages including pain and suffering.
It is the defendant who has to prove to a
legal certainty that Pam cannot recover
over $75,000 in order to convince the
court that it does not have diversity
jurisdiction. D is incorrect because Dar-
ren is not a party to the suit.

CIVIL PROCEDURE 139


86. D is the correct answer. When Beers 87. A is the correct answer. Res judicata
made its pre-answer motion to dismiss precludes Pam from asserting her claim
on subject matter jurisdiction grounds, it for property damage against Beers in a
should have also included its objection second suit because the default judg-
to personal jurisdiction. See F.R.C.P. ment against Beers is a valid (the court

A
12(h). Since it did not, it has waived its found that it had personal jurisdiction
personal jurisdiction defense and it is, and there was no appeal on that issue)
therefore, too late to assert that defense final judgment "on the merits" (a de-
as an affirmative defense in its answer. fault judgment is deemed to operate
A is incorrect for two reasons: Although as though it were "on the merits") and,
Beers purposefully availed itself of the applying the "same transaction" defi-
Nevada market by soliciting Nevada nition of "cause of action" applicable
student business, which satisfies the in federal court, Pam split her cause
Due Process clause, (i) Nevada's long of action. B is incorrect because a
arm statute does not authorize its courts default judgment is deemed to operate
to exercise personal jurisdiction in this as though it were "on the merits." C is
case because the tortious act of serving incorrect because the "rights" test is
Darren occurred outside Nevada and (ii) inapplicable in federal court. D is in-
Beers waived its personal jurisdiction correct because the fact that a plaintiff
defense, as shown above. B is incorrect who, in good faith, claims over $75,000
because Beers did solicit business from in her complaint receives a verdict for
Nevada students and Beers waived its less than $75,000 does not divest the
personal jurisdiction defense. C would court of subject matter jurisdiction to
have been correct but for the fact that enter judgment on the verdict.
Beers waived its personal jurisdiction
defense.

140 CIVIL PROCEDURE


88. D is the correct answer. Under the Ter- 90. A is the correct answer. The res judicata
ritoriality Rule, personal service upon issue presented here is whether Pam
Darren in the forum state of California should have asserted a claim for damage
is a valid basis under the Due Process to her car against Darren alongside her
clause for the California court to exer- claim for car damage against Beers. Res

A
cise personal jurisdiction over Darren. judicata does not require a plaintiff to
A is incorrect because, under a mini- join every tortfeasor against her in the
mum contacts analysis, Pam's claim same lawsuit. B is incorrect because
against Darren arises out of his driving the defense of res judicata does not
inside Nevada, not California. B is in- have to be included (indeed, it cannot 7:3
00
correct for the reason that D is correct be included) in a pre-answer motion. C <
and, also, because Darren is a California C7 -
is incorrect for the reason why A is cor- Cr -

X3
citizen. C is incorrect: (i) because it rect. D is incorrect because the 9/1/92
focuses upon "fair play and substantial judgment dismissing Pam's suit against
justice" or "litigation fairness" rather Beers was an involuntary dismissal that
than purposeful availment and, (ii) in is deemed to be "on the merits." See
any event, personal jurisdiction over F.R.C.P. 41(b).
Darren is constitutional under the Ter-
ritoriality Rule and because Darren is 91. C is the correct answer. C is the only
a California citizen. choice that is not correct. A is a correct
statement because Darren was not a
89. A is the correct answer. Darren keeps party nor in privity with a party in Pam
his California citizenship until he v. Beers. B is a correct statement be-
changes it for a new one. Although cause the 7/1/92 judgment was a default
Darren is a college student in Nevada, judgment; no facts were determined. D
this does not necessarily constitute an is a correct statement because mutuality
intent by Darren to change his domi- only applies to the assertion of collat-
cile to Nevada. Since Pam is a Nevada eral estoppel by a stranger. Pam is no
citizen, complete diversity exists. B is stranger to her suit against Beers. C is
incorrect for the reason that A is cor- incorrect because, as stated previously,
rect. C is incorrect because it is based a default judgment is deemed to operate
upon an incorrect rule of law. Diversity as though it were "on the merits."
jurisdiction is not affected by whether
a defendant is a citizen of the forum
state. This answer confuses diversity
jurisdiction with removal jurisdiction.
D is incorrect because subject matter
jurisdiction is not waived during the
life of the original lawsuit.

CIVIL PROCEDURE 141


92. D is the correct answer. The federal 93. D is the correct answer. In 2005, the U.S.
court has "alienage" jurisdiction over Supreme Court (Exxon Mobil Corp. v.
the claim by Jacques, a citizen of a Allapatah Services) adopted a literal
foreign state (i.e., France), against Don, interpretation of the Supplemental Ju-
a citizen of California, and "diversity" risdiction statute allowing supplemental

A
jurisdiction over the claim of Jill, an jurisdiction over claims by plaintiffs
Arizona citizen, against Don, a Califor- that do not exceed the $75,000 amount
nian. Jill is still an Arizonan because her in controversy requirement so long
LU
student status in California does not in- as at least one co-plaintiff or named
ce
dicate her intention to live in California class representative has a jurisdiction-

> for the foreseeable future. Therefore, A ally sufficient claim Therefore, A is
0 ce
0 is incorrect. (A is also incorrect because, incorrect. B is incorrect because the
even if Jill were deemed to be a Cali- aggregation rule otherwise applicable
fornia citizen, that would undermine in this situation has been superseded
subject matter jurisdiction over Jacques' by the Supplemental Jurisdiction stat-
claim as well and the court would, ute. C is incorrect because Jill's claim
therefore, have to dismiss both claims.) does meet the jurisdictional amount in
B is incorrect because the principle that controversy.
a woman takes on the domicile of her
husband is not applicable to the ques-
tion of "state domicile" for purposes of
the diversity statute. C is incorrect for
the reason why D is correct.

142 CIVIL PROCEDURE


94. C is the correct answer. Hanna v. 95. B is the correct answer. In determin-
Plumer states that, under the Supremacy ing the res judicata effect of the fed-
Clause, when there is federal law (U.S. eral court judgment entered 4/1/93, the
Constitution, federal statute or F.R.C.P.) California court must apply the federal
on point, that law governs over any definition of "cause of action," which

A
conflicting state law. Therefore, since is the "same transaction" test. Applying
the 7th Amendment to the U.S. Con- that test, Jacques should have joined his
stitution guarantees Jacques and Jill's personal injury claim alongside his loss
right to trial by jury of their damage of consortium claim since both claims
claims, this right cannot be undercut arose out of the auto collision. There- 00
in federal court by the California state fore, A is incorrect, since it applies the <
law on punitive damages. Therefore, "primary rights" test. C is incorrect CP
A is incorrect because, under the Su- because it relies on the theory-oriented
premacy Clause, the 7th Amendment primary rights test. D is incorrect be-
prevails over the California state law cause husband and wife are not per se
despite the important substantive state in privity with each other.
interests behind the state law. B is incor-
rect for reasons stated in the discussion 96. B is the correct answer. In the prior
of choice C. D is incorrect for three federal court suit, the jury did not
reasons: (i) the outcome determinative state its findings upon which it based
test is irrelevant to the application of its verdict for Don on Jacques' loss of
the Supremacy Clause; (ii) even if one consortium claim. The jury could have
could apply the outcome determinative found that Jacques was contributorily
test, there could well be a difference negligent. But it could well have found
between a judge and a jury trial of the that Jacques failed to prove damages,
punitive damage issue (the California since his damage evidence was "mini-
legislature thought this was so); and (iii) mal" and Don had offered substantial
applying the outcome determinative evidence that Jacques' marriage was
test, assuming there were no difference "in trouble." Therefore, it cannot be
in outcome, the result would be to ap- determined whether the jury actually
ply the federal rule rather than the state determined that Jacques was contribu-
rule. torily negligent. Therefore, A and B are
incorrect. D is incorrect because there
is no indication in the facts that Jacques
did not have a "full and fair opportunity
to litigate."

CIVIL PROCEDURE 143


97. A is the correct answer. A is the only 98. D is the correct answer. DeWinter has
choice that is "not correct." DeWinter the burden of production on the issue of
is seeking to use the prior criminal the cause of Rebecca's death; therefore,
judgment of acquittal bind Floyd's who DeWinter's motion for judgment as a
was a stranger to that criminal pro- matter of law is being used offensively.

A
ceeding. This is prohibited by the Due For DeWinter to win his motion, it is
Process Clause, not by the mutuality not enough that he meet his burden of
of estoppel rule. Mutuality of estoppel production, but he must prove conclu-
prevents the use of collateral estoppel sively that Rebecca fell off the cliff.
by a stranger to the previous suit. B is This he failed to do since the testimony
true for reasons just explained. C is true of DeWinter and his new wife, Cecily,
because the only finding the jury made are potentially biased and interested
in acquitting DeWinter is that there was and, therefore, the jury may well choose
a "reasonable doubt" that he murdered to disbelieve them. Even without con-
his wife. They could still have believed, tradictory witnesses, credibility is an is-
by a preponderance of the evidence, sue for the jury to decide. Furthermore,
that he killed her. In DeWinter's suit no inferences may be drawn against
against Floyd's, the applicable burden Floyd's for failing to produce any con-
of persuasion is "preponderance of the tradictory witnesses since there were no
evidence." D is true for reasons just other eyewitnesses to Rebecca's death.
explained. The burden of production did not shift
to Floyd's on the issue of Rebecca's
death. For the above reasons, A and C
are incorrect. B is also incorrect because
sometimes courts have been known, in
rare cases, to grant offensive motions
for judgment as a matter of law based
upon testimonial evidence.

144 CIVIL PROCEDURE


99. B is the correct answer. Since DeWinter,
the plaintiff, is a British subject and
Floyd's, the defendant, is a California
citizen, the federal court has alienage
jurisdiction to enter judgment upon the
jury's verdict. A is incorrect because
subject matter jurisdiction can be chal-
lenged even on appeal. C is incorrect
because DeWinter is not a California
domiciliary. In order to be a domiciliary
of a state of the U.S., one must also be a
U.S. citizen. D is incorrect because the
court, having alienage jurisdiction, does
not need federal question jurisdiction to
enter judgment.

100. D is the correct answer. A new trial mo-


tion must be made no later than 10 days
after entry of judgment. Here, Floyd's
waited three weeks. Had the motion
been timely, A would have been cor-
rect. Had the motion been timely, B
still would have been incorrect because
the judge could reasonably infer from
the content of Juliet's affidavit that it
could well have swayed the verdict. C
is incorrect because it states the wrong
rule of law, i.e., the Iowa Rule instead
of Federal Rule of Evidence 606(b).

ND OF ANSWERS

CIVIL PROCEDURE 145


146 CIVIL PROCEDURE
CONSTITUTIONAL LAW - QUESTION BREAKDOWN

1. Due Process 18. Freedom of Expression

2. Establishment Clause - Religion 19. Freedom of Speech - Public Forum

3. Establishment Clause - Religion 20. Freedom of Speech - Public Forum

4. Freedom of Religion 21. Public Forum

5. Freedom of Speech 22. Freedom of Expression

6. Procedural Issues 23. Freedom of Expression - Non-Public


Forum
7. Freedom of Speech - Standard
Review 24. Freedom of Speech - Standard of
Review
8. Freedom of Speech - Standard
Review 25. Violation of Civil Rights § 1983

9. Procedural Issues - Ripeness 26. Tenth Amendment - State Powers

10. Fourth Amendment - Exclusionary 27. Pre-Emption Doctrine


Rule
28. Procedural Issues - State Supreme
11. Commerce Clause Court Review

12. Freedom of Speech 29. Procedural Issue - U.S. Supreme


Court Review
13. Freedom of Speech - Prior Restraint
30. Freedom of Association - Standard of
14. Freedom of Speech - TPM Review

15. §1983 U.S. Code 31. Substantive Due Process - Abortion


Rights
16. Due Process
32. Substantive Due Process - Abortion
17. Commerce Clause Rights

CONSTITUTIONAL LAW 149


33. Substantive Due Process - Abortion 50. First Amendment - Right to an Open
- Informed Consent Trial

34. Substantive Due Process - Abortion 51. First Amendment - Pretrial Hearing
- Notice to Husband
52. First Amendment - Open Trial for
35. Substantive Due Process - Minors Minors
Right to an Abortion
53. Freedom of Press
36. Substantive Due Process - Abortion
Rights 54. Freedom of Religion - Protected
Conduct
37. Commerce Clause
55. Freedom of Religion
38. Overbreadth Doctrine
56. Substantive Due Process
39. Overbreadth Doctrine
57. Freedom of Press
40. Constitutionality of Statute
58. Substantive Due Process - Right to
41. First Amendment - Free Speech and Privacy
Press Clause
59. Establishment Clause - Secular
42. Freedom of Speech Purpose

43. Separation of Powers - Congressional 60. Establishment Clause - Secular


Branch Purpose

44. Equal Protection 61. Free Exercise of Religion

45. Equal Protection 62. Free Exercise of Religion

46. Procedural Issues 63. Freedom of Religion

47. Procedural Issues - Ripeness 64. Free Exercise Clause

48. Just Compensation Clause 65. Free Exercise - Teachings in Private


Schools
49. First Amendment - Right to a Open
Trial 66. Free Exercise Clause

150 CONSTITUTIONAL LAW


67. Symbolic Speech 85. Equal Protection

68. Unprotected Speech 86. 1st Amendment/4th Amendment/5th


Amendment
69. Freedom of Speech - Conduct
87. Commercial Speech
70. Freedom of Speech - Obscenity
88. Treaties
71. Freedom of Speech - Content
89. Commercial Speech
72. Symbolic Speech
90. Congressional Power
73. Freedom of Association
91. Procedural Issues
74. Separation of Powers - Congressional
Power 92. Procedural Due Process

75. Judicial Appointment 93. Freedom of Speech

76. Speech and Debate Clause 94. State Action

77. Speech and Debate Clause 95. Equal Protection - Illegitimacy


Extraordinary Acts
96. Equal Protection - Standard of
78. Executive Powers - Pardons Review

79. Executive Powers - Pardons 97. Due Process/Just Compensation

80. Judicial Jurisdiction - Foreign 98. Title of Nobility


Ambassadors
99. Freedom of Association
81. Sovereign Immunity/10th/11th
Amendment 100. Freedom of Association

82. Congressional Power 101. Freedom of Speech - Defamation

83. Substantive Due Process -


Homosexuality

84. Substantive Due Process

CONSTITUTIONAL LAW 151


152 CONSTITUTIONAL LAW
CONSTITUTIONAL LAW 2. If Quail is able to file the case in federal
court rather than in state court, on which

QUESTIONS constitutional clause will he rely?

A. First Amendment Freedom of


Questions 1 — 4 are based on the Religion.
following facts:
B. First Amendment Freedom of
Junior attends Riverdale Public High School. Speech.
Riverdale is an affluent community, and
the Riverdale High School is known for C. Fourteenth Amendment Due
its innovative course offerings. Among the Process.
Riverdale courses are Eastern Philosophy,
Comparative Religion and Comparative D. None of the above.
Philosophy. In order to facilitate understand-
ing between all people and to integrate the 3. Will Quail's state suit prevail?
Riverdale offerings into all aspects of the
school, each year an official of one of the A. Yes, because the prayer exces-
world's major religions or philosophies is sively entangles a religion with
asked to lead a prayer/affirmation before the public schooling.
season's first football game. Quail, a would-
be law student, enrolled in Riverdale's pre- B. Yes, because Quail's own reli-
law program, files suit to enjoin an Islamic gion is being interfered with.
monk from offering a benediction at the
opening football game this year. C. No, because all religions are be-
ing treated equally.
1. If Quail files the suit against the state,
on what constitutional clause will he D. No, because Quail's claim is not
rely? in earnest.

A. First Amendment Non-Establish- 4. Assume for this question only, that it


ment of Religion. is not Riverdale high which offers the
benediction, but is instead the River-
B. First Amendment Freedom of dale School Board which offers such,
Speech. before their weekly meetings (where
no students are present). Which of the
C. Fourteenth Amendment Due following propositions are correct?
Process.
I. The First Amendment is
D. None of the above. irrelevant.

CONSTITUTIONAL LAW 153


II. There may be fewer First Questions 5 —10 are based on the
Amendment protections following facts:
than in Question 1.
The state of Columbia has made it a misde-
III. The purpose of the prayer meanor for anyone to solicit voters within
needs to be discerned. 75 feet of a polling place during election
day hours. Supporters of Libertarian Party
A. I only. Presidential candidate, Andre Marrou, frus-
trated at the lack of media coverage given
B. II and III only. their candidate and believing the public
would embrace their platform — but for the
C. II only. governmental hassling — approach voters
within 50 feet of the polls on election day.
D. III only. Their approach is moderate. Libertarian Don
Baggle approaches people entering the polls,
with the following soliloquy: "Hi. My name
is Don. Would you like to know what the
Libertarians stand for?" If the response is
positive, Don gives a five-minute descrip-
tion. If the response is negative, Don offers
a flyer.

Undercover officer, Wil Marsh, arrests Don


while he is in the process of offering a pam-
phlet. Subsequent to the arrest, one ounce of
marijuana is found on Don's person.

5. If Don challenges his misdemeanor ar-


rest, for soliciting voters within 75 feet,
on what constitutional clause or clauses
will he rely?

A. First Amendment Freedom of


Religion.

B. Fifth Amendment Due Process.

C. First Amendment Freedom of


Speech.

D. None of the above.

154 CONSTITUTIONAL LAW


6. If Don's claim is heard, it will be be- C. No, because there was no rational
cause which of the following threshold basis.
requirements have been met?
D. No, because there was a compel-
I. Standing. ling state interest.

II. Ripeness. 9. Assume for this question only, that


although the state has had this law
III. Mootness. on the books for 50 years, they have
never enforced it — although several
IV. Don is a U.S. Citizen. individuals have violated it. In order to
attempt to gain more political coverage
A. II only. for the Libertarians, Don sues the state
to have the law overturned. If this suit
B. II and IV only. is dismissed, what will be the likely
reason?
C. I, II, III and IV.
A. Mootness.
D. I and II only.
B. Bad faith of Don.
7. Provided that Don's claim of a constitu-
tional violation is heard, which standard C. Ripeness.
of review will be utilized?
D. Independent state grounds.
A. Strict scrutiny.
10. Assume for this question only, that the
B. Reasonable basis. Columbia law is deemed unconstitu-
tional, can Don be prosecuted for the
C. The Intermediate Standard. drug possession?

D. None of the above. A. Yes, because they are entirely


different laws.
8. Will Don's claim prove to be success-
ful? B. Yes, because of the Drug Ex-
ception to the Warrant require-
A. Yes, because the state had a ra- ment.
tional basis.
C. Yes, because the arrest was le-
B. Yes, because there was no corn- gal.
pelling state interest.

CONSTITUTIONAL LAW 155


D. No. Questions 12 —14 are based on the
following facts:
11. With the state of present technology,
the manufacture of nuclear-based en- The city of Simmee is predominantly Cauca-
ergy has an unfortunate side-effect sian, middle-class and blue-collar. Because
— the production of unusable and toxic of a recent jury trial verdict which was un-
nuclear waste. Because of the world popular with minority communities, Simmee
and nation's dwindling supply of oil, became a very popular spot for non-violent
the state of X has encouraged in-state civil rights demonstrations. The Simmee
business to embark on the production of Board of Directors, acting at the direction of
nuclear energy. In order to give this en- Simmee's Mayor Ringer passed an ordinance
couragement "teeth," a 20% discount is requiring all of those who wish to parade or
given to in-state corporations regarding demonstrate to apply for a permit and pay
the disposal of their nuclear waste. a fee based on the approximate cost to the
city (regarding extra police, clean-up, etc.).
State X, itself, owns all of the nuclear- The Simmee ordinance directed the Board to
disposal plants. (With that policy, it make an accurate tabulation of such.
ensures competent disposal).
Free Freedom Lovers (FFL), a group with
Y Corp, a corporation in State Y (a state socialist learnings, applied for a permit to the
bordering State X) is in the business of Simmee Board. The permit was granted, but
producing nuclear energy. Much of the a $500 fee "to cover costs" was levied. FFL
energy it produces is sold to the citizens never had their demonstration, but instead
and businesses in State X. requested a state court to restrain and enjoin
the Simmee procedure.
If Y Corp challenges State X for not
giving it a 20% discount on disposal, 12. On what basis is the FFL suit most likely
its challenge will: based?

A. Prevail, because of the Com- A. Freedom of Association.


merce Clause.
B. Freedom of Speech.
B. Prevail, because of the Due Pro-
cess Clause. C. Equal Protection.

C. Prevail, because of the Equal D. Due Process.


Protection Clause.
13. Which of the following propositions are
D. Fail, because of the sovereignty true?
of state regulation.

156 CONSTITUTIONAL LAW


I. The Simmee licensing Questions 15 — 16 are based on the


rocedure was a prior
p following facts:
restraint.
The city of Dickens, because of its many,
naturally-occurring rivers, is always build-
II. The Simmee licensing
ing bridges. Unfortunately, the city fails to
rocedure was a reason-
p
able time, place and man- either train or warn its employees about the
dangers of this job. Bill Richards is on the
ner restriction.
city payroll and dies in a bridge-related ac-
cident. Bill's widow, Sharon, sues the city
III. The city's procedure was

not "state action." based on §1983 U.S. Code.

A. I only. 15. For a § 1983 claim to succeed, there


must be:
B. II only.
A. A tort on the part of the city.
C. III only.
B. A crime on the part of the city.
D. II and III only.
C. A violation of constitutional
14. If the Simmee procedure is declared rights by the city.
unconstitutional, it will be because of
which of the following reasons: D. None of the above.

I. It was not narrowly tai- 16. If Sharon claims that Bill's substantive
lored to meet the city's due process rights were abridged, her
interest. suit will:

II. It vested too much power A. Prevail, because of the negli-


in the Board. gence of the city.

A. I and II only. B. Prevail, because there was a


§ 1983 tort.
B. I only.
C. Fail, because the city was not
C. II only. negligent.

D. Neither I nor II. D. None of the above.

CONSTITUTIONAL LAW 157


17. The fictional state of Perotia has pro- Questions 18 — 25 are based on the
hibited all waste-disposal businesses in following facts:
the state from accepting waste products
O produced out of the state. There is no John Wayne Airport, of Orange County Cali-
evidence that the refused waste is toxic. fornia, has just adopted a law, by the power
Bushco, a waste-disposal business in of the Orange County Port Authority, ban-
Perotia, challenges the Perotia regula- ning the solicitation of funds, the distribution
tion. Will Bushco prevail? of literature, or the sale of written material
within the common areas of its airport.
A. No, because a state can refuse to
allow pollution. The National Rifle Association (NRA),
angered by the rising tide of gun regulation
B. Yes, because of the Commerce has targeted the Wayne Airport as a forum to
Clause. make their case that the Second Amendment
is losing its all-important meaning.
C. No, because this does not appear
to be economically-based. NRA volunteer Duke Hinkins hands out a
free small cactus inside the common areas of
D. None of the above. the Wayne Airport. Then he asks for a small
contribution to "defray the costs of growing
and potting the cacti." Duke also offers to
sell two small books. One is on the Old West
and the other is on President Jefferson's
views on the Second Amendment. If the air
traveler is either unresponsive or if, at the
outset, the air traveler seems hurried, Duke
simply and only offers a free colorful flyer
entitled, "GUNS ARE FREEDOM."

On July 4, Duke is arrested by Officer Bart;


after offering Bart the cactus and asking
for a donation. Undeterred and back at the
airport, On July 6, Duke is again arrested,
but this time by Officer Cary, for attempting
to sell her the two books. On July 8, Duke
is back at the airport, and is again arrested,
but this time by Officer Darby, for offer-
ing her the free pamphlet. A few days later
Duke, through his lawyers, seeks to enjoin
the prohibition.

158 CONSTITUTIONAL LAW


Q C)
0
18. On what grounds should Duke chal- C. Yes, because the arrest was in a =
Cr)
lenge the Orange County ordinance? privately-owned airport. 57. =1
-I
. -i
0
A. Second Amendment Right to D. No. =
>
Bear Arms.
21. Will Duke's July 6 arrest be considered
B. First Amendment Freedom of valid?
Speech.
A. Yes, because he was soliciting
C. First Amendment Freedom of funds.
Association.
B. Yes, because he was a public
D. The Commerce Clause. nuisance.

19. Will the public areas in the airport be C. Yes, because the arrest was in a
considered a "public forum"? privately-owned airport.

A. Yes, because the public is al- D. No, because it chilled the exer-
lowed. cise of a fundamental right.

B. Yes, because airports have, by 22. Will Duke's July 8 arrest be considered
common custom, become a valid?
forum for the marketplace of
ideas. A. Yes, because he was soliciting
funds.
C. No, because many airports are
privately-owned. B. Yes, because he "assumed the
risk," having been arrested sev-
D. No, because there is a difference eral times prior.
between an airport and a public
park, street or sidewalk. C. Yes, because there were other
ways for Duke to have his ex-
20. Will Duke's July 4 arrest be considered pression heard.
valid?
D. No, because it was an unreason-
A. Yes, because he was soliciting able restriction on his freedom
funds. of expression.

B. Yes, because cacti are potentially


hazardous in air flights.

CONSTITUTIONAL LAW 159


23. Should the trial court use the compelling III. Officer Cary.
interest test in evaluating the Orange
County ordinance? IV. Officer Darby.

A. Yes, because the First Amend- A. I, II, III and IV.


ment was implicated.
B. I and III only.
B. Yes, because the Second Amend-
ment was implicated. C. I and IV only.

C. Yes, because this was a "public D. I only.


forum."

D. No, because the airport was truly


not a public forum.

24. Should the trial court use the "reason-


ableness" test regarding the July 8 ar-
rest?

A. Yes, because this was not a pub-


lic forum.

B. No, because the airport was a


public forum.

C. No, because there is no "reason-


ableness" test.

D. None of the above.

25. For this question only, assume that it is


held that Duke's constitutional rights
were, in part, violated. Duke will have
a successful § 1983 suit against:

I. The Orange County Port


Authority.

II. Officer Bart.

160 CONSTITUTIONAL LAW


Questions 26 — 27 are based on the 26. On what grounds will they most likely
following facts: base this challenge?

The year is 1999 and the U.S. has turned to A. Commerce Clause.
the use of nuclear energy, due to the dwin-
dling supply of its own oil, the Arab Oil B. Tenth Amendment.
Embargo of 1996, and the development of
nuclear fusion energy (which produces only C. Ninth Amendment.
low level nuclear waste).
D. Just Compensation.
However, many of the states have set up
barriers to the disposal of low level nuclear 27. Will the North California challenge
waste in their state. succeed?

The effect of this state action is to create A. Yes, as to all clauses of the Act.
a crisis situation regarding disposal of this
waste. B. Yes, as to clauses 1 and 2, only
of the Act.
After much debate Congress has reacted to
the problem by enacting the Comprehensive C. Yes, as to clause 3, only, of the
Waste Disposal Act of 1999. There are three Act.
clauses:
D. No.
1) States will receive federal mon-
etary incentives for providing for
such disposal;

2) States providing for such dis-


posal can surcharge states which
do not; and

3) If there are no waste programs


developed, states must take title
to their own waste and must pay
all "damages" if they do not take
title to their own waste.

The state of North California has challenged


the constitutionality of the Act.

CONSTITUTIONAL LAW 161


Q
0 Questions 28 — 30 are based on the A. Yes, because the Au Natural
Party was treated unfairly.
following facts:

0 The fictional state of Island requires, for B. Yes, because there is a "case or
a new political party to be placed on the controversy."
electoral ballot, a showing of "significant"
public support. This has been interpreted C. No, because the issue is moot.
by the Island Supreme Court to require for
state-wide election either petitions signed D. None of the above.
by no more than 1% of the voters, or a 5%
showing in a previous election. 30. Assume for this question only, that the
U.S. Supreme Court has accepted re-
The Island city of Corruptia, however, has view of Norris' case on the question of
a more stringent requirement — 5% of the the constitutionality of the Island ballot
eligible voters must sign a petition prior to access rules. Will he prevail?
a party being placed on the ballot. The "Au
Natural" Party has met the state require- A. Yes, if the state cannot prove that
ments and desires to run a candidate for the rules on new political parties
Mayor of Corruptia. However, the Corruptia are necessary to serve a compel-
City Council has refused to allow Bob Nor- ling government interest.
ris ballot access — since he has not met their
requirements. B. Yes, if the state can show that the
rules on new political parties are
28. If Norris appeals to the Island State rationally related to a permissible
Supreme Court, should he prevail? government interest.

A. Yes, because of the Freedom of C. No, because of the Tenth Amend-


Association. ment.

B. Yes, because of that court's prior D. None of the above.


ruling.

C. No, if the court relies on inde-


pendent state grounds.

D. None of the above.

29. For this question only, assume Norris


has lost his appeal to the Island Su-
preme Court. Must the U.S. Supreme
Court accept review of his case?

162 CONSTITUTIONAL LAW


Questions 31 — 36 are based on the Abortion Clinic XYZ and Mary Coe chal-
following facts: lenge the Act. The Act is upheld by the
Northeast State Supreme Court. The appeal
The fictional state of Northeast, based on a is now pending before the federal courts.
conservative swing in their citizens' poli-
tics, has enacted a comprehensive Abortion 31. Will Clause 1 be upheld?
Control Act. The Act is comprised of the
following clauses: A. Yes, because of Roe v. Wade.

1) An abortion is generally not to B. Yes, because it has now been


be allowed when the fetus is vi- held that a fetus is a human be-
able. ing.

2) A woman must receive informa- C. No, because of the Right to


tion at least 24 hours prior to the Life.
abortion regarding alternatives to
abortion, including adoption. D. Yes, but for none of the above
reasons.
3) A woman must give informed
consent to the abortion 24 hours 32. Will Clause 2 be upheld?
prior to the abortion, unless her
health is imminently in danger. A. No, because it is a pre-viability
regulation contrary to Roe v.
4) A married woman must furnish Wade.
proof at least 24 hours prior to
the abortion that her husband has B. No, because abortion is a funda-
been informed. mental privacy right.

5) An unmarried minor must ei- C. Yes, because a state is permitted


ther furnish proof of one of her to regulate abortion, provided
parent's consent to the abortion, the state does not entirely outlaw
or judicial-approval of her abor- it.
tion.
D. Yes, because the regulation does
6) Those medical personnel and not place an undue burden on the
clinics performing abortions right to abortion.
must keep records indicating
that clauses 1-5, above, were 33. Will Clause 3 be upheld?
complied with.

CONSTITUTIONAL LAW 163


A. Yes, but only as to the "medical- D. This is an open issue under pres-
emergency" provision. ent jurisprudence.

B. Yes, but only as to the "informed" 36. Will Clause 6 be upheld?


provision.
A. Yes, except as to the record-keep-
C. Yes, but only as to the "consent" ing required regarding Clauses 1
provision. and 5.

D. Yes. B. Yes, except as to the record-keep-


ing required regarding Clauses 4
34. Will Clause 4 be upheld? and 5.

A. Yes, because husband consent is C. Yes, except as to the record-


required. keeping required by Clause 5.

B. Yes, because such does not D. Yes, except as to the record-


place an undue burden on the keeping required by Clause 4.
husband.
37. SpecialtyX is a mail order business
C. Yes, because such does not place located in State A. It has targeted a
an undue burden on the woman mailing to all grocery stores in State
who seeks to terminate the fe- B. State B has decided to impose a use
tus. tax on the property sold by SpecialtyX.
SpecialtyX's only contact with State B
D. No, because this is an undue is via the mailings and subsequent sales.
burden on the woman. Is the State B tax unconstitutional?

35. Will Clause 5 be upheld? A. Yes, because there are no mini-


mum contacts.
A. Yes, because the minor need not
be insulated from those who may B. Yes, because there is a Com-
wish to counsel her against the merce Clause violation.
abortion.
C. No, because there is a substantial
B. Yes, but only if such occurs in the nexus.
second trimester of pregnancy.
D. No, because there is not a sub-
C. No, if such occurs in the first stantial nexus.
trimester of pregnancy.

164 CONSTITUTIONAL LAW


Questions 38 — 39 are based on the 39. Will Skin's post-arrest threat to Omar
following facts: suffice to find him guilty under the
Clintonia Hate Crime Law?
Late one evening Skin, a teenager, and two
of his friends trespassed onto the front yard A. Yes, because all of the elements
of Omar, a refugee from Iraq. Omar now of the crime appear to have been
lives in the fictional state of Clintonia. Skin met.
built a Cross and burned it in Omar's yard.
Skin's two friends escaped, but Skin was B. Yes, because these were fighting
apprehended. As Skin was apprehended and words.
Omar watched, Skin yelled to Omar, "I'll kill
you, you bloody Arab, if you stay here!" C. No, because there was no mens
rea in the crime.
Skin is prosecuted in Clintonia, under their
new "Hate Crime Law." The law, in pertinent D. No, because the law was facially
part, reads as follows: unconstitutional.

"It is a class 1 misdemeanor, to burn a cross,


display a swastika or state any racial epithet
for the purpose of expressing racial or ethnic
hatred in a public place or on private prop-
erty which is not in the lawful possession of
the suspect."

38. Will Skin's burning of the Cross on


Omar's lawn suffice to find him guilty
under the Clintonia Hate Crime Law?

A. Yes, because all of the elements


of the crime appear to have been
met.

B. Yes, because Skin's acts were


constitutionally unprotected.

C. No, because there was no mens


rea in the crime.

D. No, because the law was over-


broad.

CONSTITUTIONAL LAW 165


Questions 40 — 42 are based on the 40. Famed football star, Mike Toggle, was
following law: convicted of rape in Bronsonia. In order
to set the record straight, he contracted
Sick and tired of the profiteering of criminal with Violence Press to write a book. In
convicts and their attorneys, the state of the book, Toggle denied having raped
Bronsonia has enacted the following law, the victim, although he did admit to rap-
§208: ing a female when he was 16-years-old.
Toggle brought up this point to indicate
1) An individual is permitted to how guilty he felt at that time and,
write about his/her life. But if thus, why he would never commit this
the writing describes crimes crime. Violence Press paid Toggle an
for which the individual has advance of $5,000 and, to date, $2,333
been convicted; or crimes for in royalties. Which of the following
which the individual is formally propositions are true?
charged; or crimes which the in-
dividual admits to, but for which I. Violence Press violated
he/she has not been formally §208.2.
charged, all proceeds, profits and
royalties shall be turned over to II. Toggle violated §208.1.
the Bronsonia Victim's Fund.
A. I only.
2) A publisher, co-author or lawyer
who participates in the publish- B. II only.
ing of material delineated in
Clause 1, above, must furnish C. I and II.
all proceeds, profits and royalties
which would have been payable D. Neither I nor II.
to the author directly to the Bron-
sonia Victim's Fund. 41. Rotty has been convicted of several
"contract killings." Famed writer Jack
3) Violation of either Clause 1 or Junkins decides to write his story. He
2 shall be punished as a Class 3 interviews Rotty over a period of two
misdemeanor and violators shall months. If Bronsonia prosecutes Junk-
be fined $100,000 in addition to ins under §208, they will:
the amount of profits, proceeds
or royalties not furnished to the A. Succeed, because he profited
Bronsonia Victim's Fund. from crime.

B. Succeed, because he wrote about


crime.

166 CONSTITUTIONAL LAW


C. Succeed, because he profited 43. Due to migration to the Southern and
from another's crime. Western states, the fictional state of
Northwest has experienced a marked
D. Fail, because Junkins was only decrease in population — particularly as
morally guilty. compared to the growth of population
in the Southern and Western states. In
42. Which of the following propositions are order to facilitate proportional represen-
true? tation in the U.S. House of Representa-
tives, Congress has increased the seats
I. The Bronsonia statute is available in these states, and decreased
unconstitutional since it the seats in Northwest, but not beyond
is not narrowly tailored that accounted for by increased or de-
to further a compelling creased population. If Northwest chal-
state interest. lenges this reduction, it will:

II. The Bronsonia statute is A. Succeed, since states must be


unconstitutional since it given equal representation.
is overbroad.
B. Fail, because states must be
III. The Bronsonia statute is given equal representation.
a kind of prior restraint
on the press and speech. C. Fail, because the facts seem to
indicate an equal apportionment
A. I, II and III only. method was used.

B. I and II only. D. None of the above.

C. I and III only.

D. None of the above.

CONSTITUTIONAL LAW 167



Questions 44 — 45 are based on the C. I and IV only.
following facts:
D. I, II, III and IV.
The year is 1993. As of 1954, in the land-
mark decision of Brown v. Board of Edu- 45. If Elizabethia's school administrators
cation, 347 U.S. 483, the fictional state of are presently found to be in good faith,
Elizabethia has had its primary, secondary are the U.S. Supreme Court and its
and post-secondary schools under the super- lower federal courts discharged from
vision of the local Federal District Court. supervising desegregation?
The supervision was based on Elizabethia's
former pattern of de jure segregation. In A. Yes, provided admission policies
that era, Elizabethia assigned students to are race-neutral.
school solely depending on the race of the
student. As of 1993, all enrollments are open B. Yes, because there is no more
at all schools. However, of the five state-run that can be accomplished.
universities, three remain overwhelmingly
Caucasian and two remain overwhelmingly C. Yes, because of state's rights.
of African-descent.
D. No.
44. Federal jurisdiction was established
in Brown v. Board of Education under
which of the following reasons and/or
theories:

I. Separate but equal


schools segregated by
race, are inherently un-
equal.

II. Privileges and Immuni-


ties violation.

III. Equal Protection viola-


tion.

IV. Due Process violation.

A. I and II only.

B. I and III only.

168 CONSTITUTIONAL LAW


Questions 46 — 48 are based on the 47. If Norris files a federal suit, because
following facts: he had intended to buy a mobile home
park, and now wonders about the wis-
The city of Marblestone, in the fictional state dom of his potential investment, will
of Alfredia, has enacted a rent-control ordi- his case be heard?
nance. The ordinance is specifically aimed
at controlling the rent in mobile home parks. A. No, because the case is not
The policy of the rent control ordinance ripe.
is to protect the lower-middle classes and
elderly — who are the most frequent tenants B. Yes, because there was a govern-
of the parks. The ordinance at issue not only mental taking.
"freezes" the rent for those now tenants, but
also sets a maximum rent per site. C. Yes, because there was a de-
nial of the Just Compensation
46. Daniel files suit in federal district court, Clause.
to overturn the ordinance. Daniel files
this suit because he is against govern- D. None of the above.
mental regulation of the private sector.
If Daniel's suit is not heard, it will be 48. If the owner of a Marblestone mobile
because of which of the following rea- home park, who can substantiate that
sons: she has been prevented from raising
rent in her park, files a federal suit to
I. He lacks standing. overturn the ordinance based on an
abrogation of the Just Compensation
II. The case is not ripe. Clause, this suit will:

III.There is no case or con- A. Fail, because there was no "tak-


troversy. ing."

IV. There is no federal ques- B. Fail, because the Just Compensa-


tion. tion Clause is not applicable to
the states.
A. I and II only.
C. Fail, because the case is not
B. I, II and III only. ripe.

C. I, II, III and IV. D. Succeed.

D. I, II and IV only.

CONSTITUTIONAL LAW 169


Questions 49 — 53 are based on the 51. Assume for this question only, that there
following facts: is a pre-trial admissibility hearing re-
garding the introduction of certain nude
Famous movie director Billy Eucalyptus and photographs allegedly taken by Billy of
his wife Marci Sparrow are about to enter Marci's adult daughter (by another mar-
trial regarding their contested divorce. riage.) Can the judge close this pretrial
admissibility hearing?

judge so close the trial? have a right to be there.

A. Yes, since both parties consent- B. Yes, because it is pre-trial.


ed.
C. If there is a compelling reason to
B. Yes, since trials never need be do so, and if the press and public
public. are allowed to see a transcript of
the hearing soon after its comple-
C. Yes, since divorce trials can be tion.
closed.
D. None of the above.
D. None of the above.
52. Assume for this question only, that after
50. If only Billy consents, can the judge the dissolution of marriage proceed-
preclude the press and public from the ing, Billy is formally charged with the
trial? sexual assault of Marci's 15-year-old
daughter. Billy moves to keep the press
A. No, since both parties must con- out of the trial — based on his claim that
sent. the minor daughter's life will be ruined.
Will Billy's request be granted?
B. Yes, since trials never need be
public. A. Yes, if a transcript of the trial is
made public soon after the trial.
C. Yes, since divorce trials can be
closed. B. Yes, if the minor consents.

D. None of the above. C. Yes, since it is a minor who is


involved.

D. No.

170 CONSTITUTIONAL LAW


53. Assume for this question only, that Billy Questions 54 — 58 are based on the
moves that the dissolution of marriage following facts:
trial be open to television transmission.
The court's denial of this motion: Jessica Rabindra purports to follow an an-
cient Delphic Religion, which she, herself,
A. Violates the First Amendment. in psychic trance, has rediscovered. The
religion follows the so-called "goddess"
B. Violates the Sixth Amendment. perception of cosmic reality. She has formed
a church called, "The Church of the Astral
C. May be a permissible state rule. Goddess." The Church is located on a one-
acre secluded ranch in the fictional state of
D. None of the above. North Californ.

Membership in the Church is voluntary. But


each member, to be admitted, undergoes a
series of purification rites wherein the mem-
ber is purged of a chauvinistic view of the
world and is "put in touch with the power
of the earth's creative energy." Among the
aspects of the purification are an abstinence
from eating animal flesh, one day of fasting
per week and a voluntary sexual ritual with
Jessica.

The sexual ritual with Jessica, for males, in-


volves the performance of sexual intercourse
and cunnilingus with her. The sexual ritual
for females includes only the performance
of cunnilingus with her.

Jessica has written a book about her religion,


and this book has attracted many people to
join her church. Church dues are $400 per
month.

In the state of North Californ, the following


provisions are present in their penal code:

CONSTITUTIONAL LAW 171


Q
§ 1111: It is a misdemeanor to offer A. There were other activities of the
sexual intercourse or non- church besides sex and the pay-
intercourse sexual relations ment of money was not required
for hire. to have sex with Jessica.

§ 1112: It is a felony to own and B. She was the only priestess.


operate a house of prostitu-
tion. C. Other activities occurred in her
temple.
§1114: Cunnilingus and fellatio
are misdemeanors. D. She was exercising the Freedom
of Religion.
§2009: Solicitation occurs when
one person induces another 56. If Jessica is prosecuted for a violation
to participate in a crime, of § 1114, her prosecution will likely:
intending that this person
so participate. A. Succeed, because there is no
right to extra-marital sexual pri-
54. If Jessica is charged under § 1111, and vacy.
offers a First Amendment Freedom of
Religion defense, she will most likely: B. Fail, because the acts were pri-
vate and consensual.
A. Succeed, because she was exer-
cising a religion. C. Fail, because this crime does not
include sex-for-hire.
B. Fail, because the Freedom of
Religion does not protect against D. Fail, because there was no genital
prosecution for prostitution. penetration.

C. Succeed, because the adults con- 57. If Jessica is prosecuted for a violation
sented. of §2009, based on the inducement to
join her church in her book, her best
D. Fail, because her religion was defense will be:
bogus.
A. Freedom of Religion.
55. Jessica's best argument against a pros-
ecution under § 1112 is: B. Freedom of Speech.

C. The penumbra right to privacy.

D. Freedom of Association.

172 CONSTITUTIONAL LAW


58. If Jessica is prosecuted for initiating her Questions 59 — 60 are based on the
own husband into the religion, under following facts:
§ 1114, which defense will be the most
likely to succeed? The fictional state of Waywaynortheast re-
quires all businesses to close on Sundays.
A. Right to Privacy. An exemption is given to health care pro-
viders.
B. Right to Marry.
59. If Rex Bookstore challenges this law,
C. Freedom of Religion. they will:

D. None of the above. A. Succeed, because of the Freedom


of the Press.

B. Succeed, because of the Com-


merce Clause.

C. Succeed, because of the Estab-


lishment Clause.

D. None of the above.

60. If Ralph's Kosher Minimarket chal-


lenges this law on First Amendment
Establishment Clause grounds, they
will:

A. Succeed, because the law is for


the purpose of enforcing the
Christian Sabbath.

B. Succeed, because it impedes


their Free Exercise of Religion.

C. Succeed, because it is a taking,


without Just Compensation.

D. None of the above.

CONSTITUTIONAL LAW 173


Q
Questions 61— 62 are based on the 63. Grade school student, Jeremiah, refuses
following facts: to salute the flag in his second grade
class. He is disciplined by his teacher,
The fictional state of Rightwing has changed Ms. Thomas; and sent to the principal,
the motto on their license plates to "JUST Dr. Rehnquist, to serve 2 hours of deten-
SAY NO." Ted Paine is offended by this tion. He had refused to salute the flag
since he believes it is contrary to the prin- based on his upbringing as a Jehovah's
ciples of his Asian religion to demand others Witness, that one's allegiance is owed
behave in a certain way. So, Ted alters his only to God. His mother, who happens
plate to read "JUST SAY NOT." The Righ- to be a lawyer, sues the school board,
twing State Police arrested Ted for defacing seeking an injunction of forced flag-
public property (since the plates technically saluting and also seeking unspecified
belong to the state). Ted is convicted. damages. Will Jeremiah's suit pre-
vail?
61. What is the likely ground of Ted's ap-
peal suggested by the closest Supreme A. No, because this was not truly
Court precedent? prejudicial to his religion.

A. First Amendment Free Speech. B. No, because the Constitution


protects family values, such as
B. First Amendment Freedom of saluting the flag.
Religion.
C. Yes, because the Constitution
C. Fourth Amendment Search and protects family values such as
Seizure. a person's right not to be forced
to appear to support state-spon-
D. First Amendment Freedom of sored orthodoxy.
Association.
D. Yes, because the teaching of the
62. Will Ted's appeal be successful? Jehovah's Witnesses was essen-
tially correct.
A. No, because he did, in fact, de-
face public property.

B. No, because it was conduct.

C. No, because he could still as-


sociate with members of his
religion.

D. None of the above.

174 CONSTITUTIONAL LAW


Questions 64 — 65 are based on the I. Prevail, because of the
following facts: parental right of priva-
cy.
The agrarian, back-to-nature sect called the
Followers of Landrew, believe that a child, II. Prevail, because, in ef-
as of age 14, should not go to public school fect, the district was
and should begin to work the fields as part of violating the Establish-
their farm life. Archon, the parent of Jedidiah ment Clause of the First
(who is 14) takes him out of the local public Amendment.
junior high school. However, the school
district has a rule that children must attend III. Prevail, because the
school until their sixteenth birthday. school district was vio-
lating the Free Exer-
64. Archon is ordered to re-enroll Jedidiah cise Clause of the First
in school, and is threatened with con- Amendment.
tempt of court if he does not. If Archon
challenges this order, he will: A. I and II only.

A. Fail, because it is Jedidiah who B. II and III only.


will lose out.
C. I and III only.
B. Succeed, based on the Free Ex-
ercise Clause and Privacy. D. I, II and III.

C. Fail, because there is a compel- 66. The religion of Funkamental, now


ling interest in the education of established for 50 years, has fallen on
minors. hard times. In the eyes of Ron, Hub,
the present president, has allowed the
D. Succeed, because the school assets and membership of the religion to
district lacks jurisdiction. drastically fall. But Hub will not give up
his presidency. So, Ron files suit in the
65. Assume for this question only, that in state court to order Hub to stand for a
response to the practice of the school democrat election. Will Ron prevail?
district, the Followers of Landrew form
their own private school, which teaches A. No, because of the Free Exercise
the basics. If the school district enacts Clause.
a new rule to the effect that all students
must attend public school until the age B. No, because of the Establishment
of 18 and the Followers of Landrew Clause.
challenge this, the Followers of Lan-
drew will:

CONSTITUTIONAL LAW 175


C. No, because there is no case or Questions 67 — 71 are based on the
controversy. following facts:

D. Yes, if the facts establish Ron's The city of Puritania in the fictional state
claim to be true. of Hypocritia, troubled by the decay in the
moral fabric of its society, has enacted a
statute barring nudity in places open to the
public. The statute punishes such as a misde-
meanor. Nudity is defined as the "exposure
of female or male genitalia or the exposure
of female nipples or areola, when such is
done for the purpose of appealing to the
prurient interest."

67. On its face, is the Puritania statute un-


constitutional?

A. Yes, because there can be sym-


bolic speech aspects to both male
and female nudity.

B. Yes, because it is overbroad.

C. Yes, because it is vague.

D. None of the above.

68. The "Go For It" Bar, open only to those


over the age of 21, has a stage where
women dance topless. One evening the
owner, Paddy, and the offending fe-
males are arrested by the police. If they
appeal their conviction, they will:

A. Fail, because the Puritania law is


overbroad.

B. Succeed, because the Puritania


law is overbroad.

176 CONSTITUTIONAL LAW


C. Fail, because their conduct is C. Be convicted, because her acts
constitutionally unprotected. were in public.

D. Succeed, because the conduct D. Not be convicted, her conduct


was symbolic speech. was not lewd and lascivious.

69. One afternoon, Murphy Moe is arrested 70. Chipmunks, a club catering to adult fe-
for breast-feeding her infant child in a males, offers erotic dancing by men. In
frequented public park. Murphy hap- the dances, the men do a "strip tease", in
pens to be a very beautiful woman. Her which they remove all of their clothing
act of breast-feeding included exposure but for a tight bikini-type bottom. The
of her areola and one breast or the brief is so tight that outlines of the male
other (while switching her baby from genitalia are visible. In fact, many of the
breast-to-breast). Murphy was wearing men who work at Chipmunks arrange
a button down shear blouse, and tightly- "dates" with the customers after hours,
fitting, high-cut shorts. A group of men during which they perform sex for hire
would frequently, in a clandestine way, contrary to another prostitution law of
watch Murphy each afternoon during Puritania.
their lunch break.
The police are aware of the prostitu-
After her arrest, and after being given tion, but are unable to find any females
proper Miranda warnings, Murphy ad- who are willing to testify against the
mits that she knew several men were men. Thus, Chipmunks, and several
watching her. She also admits that she of the male dancers, are prosecuted
knew her actions were likely to sexually under the Puritania indecent exposure
arouse some of the men — that in fact, statute, above. Will the prosecution be
she wanted, "to show men the beauties successful?
of having babies and because I was
hoping to meet a new man." If Murphy A. Yes, because they were guilty of
is prosecuted under the Puritania law, prostitution.
she will likely:
B. Yes, because Chipmunks was
A. Not be convicted, because the clearly contrary to Puritania's
elements of the crime have not community standards.
been met.
C. No, because the prostitution
B. Be convicted, even though she could not be proven.
might claim her act was sym-
bolic speech.

CONSTITUTIONAL LAW 177


D. No, because there was no genital 72. The fictional state of South Kaliforn has
"exposure" as under the relevant pictured on its flag a western gunslinger.
Puritania statute. Because Duke finds not only guns, but
the whole gunslinger mentality patently
71. Darby is a young woman out of work in offensive, he calls a press conference
Puritania. She was an aerobics instruc- and publicly burns the South Kaliforn
tor at one of the city's health clubs; flag. In South Kaliforn, it is a misde-
but due to the recession the club has meanor to: "Purposely burn, destroy,
closed down. So, Darby takes out a mutilate, defile, desecrate or dirty the
vendor's license to sell hot dogs in one state or federal flag." Duke is pros-
of Puritania's open air malls. Unlike the ecuted and convicted in South Kaliforn.
other vendors, however, Darby wears a Will his federal appeal succeed?
thong bikini. She is arrested for violat-
ing the Puritania statute. Will Darby's A. Yes, because the crime was strict
First Amendment "symbolic speech" liability and, thus, overbroad.
argument succeed?
B. Yes, because the state infringed
A. Yes, because she was doing noth- his First Amendment rights.
ing unlawful.
C. No, because the flag he burned
B. Yes, because her sales approach was a state flag.
had artistic First Amendment
content. D. None of the above.

C. No, because there was no speech 73. The year is 2017, and you may assume
content to her acts or to the city's U.S. Constitutional law has remained
prosecution of her. the same as it is at present — with one
notable exception. The U.S. Constitu-
D. No, because she was guilty of the tion has been amended to place an
crime. outright and complete ban on the use of
alcohol, with the only exception being
for prescription drugs.

Father 0' Scalia, a Catholic Priest living


in Illinois, is prosecuted for using wine
during the Sacrament at his Church. If
he challenges the law based on First
Amendment grounds, he will:

178 CONSTITUTIONAL LAW


A. Succeed, because of the Free Questions 74 — 75 are based on the
Exercise Clause. following facts:

B. Succeed, because there is no Sick and tired of the poor quality of ap-
compelling state interest. pointees to the U.S. Supreme Court, both
houses of Congress pass a law whereby the
C. Succeed, because of the Freedom power to appoint U.S. Supreme Court judges
of Association Clause. is stripped from the President and resides
solely in the houses. The so-called "Supreme
D. Fail. Judiciary Act" is vetoed by the President; but
the veto is overturned.

74. What majority of Congress is necessary


to overturn a presidential veto?

A. Two-thirds of either house.

B. Two-thirds of the Senate.

C. Two-thirds of the House of Rep-


resentatives.

D. Two-thirds of both houses.

75. Assume for this question only, that the


President challenges Congress' power
to enact this act. Which of the following
propositions are correct?

I. Federal judicial selection


is solely a presidential
decision.

II. The Act is unconstitu-


tional.

A. I and II.

B. I only.

CONSTITUTIONAL LAW 179


C. II only. Questions 76 — 77 are based on the
following facts:
D. None of the above.
During a senate debate, U.S. Senator Tom
Semand argues that the scientific work pro-
duced by XYZ University "is not up to the
standards of grammar school — let alone fit
for a $150,000 federal grant".

76. If XYZ University sues Semand for


defamation, his best defense will be:

A. First Amendment Freedom of


Speech.

B. Article I Speech and Debate


Clause.

C. XYZ was a public entity.

D. Sovereign immunity.

77. Assume for this question only, that it


can be proven that Semand received a
$20,000 bribe from ABC University, the
competitor of XYZ University, for the
$150,000 grant. May Semand be sued
for taking a bribe in connection with the
federal grant application?

A. No, since there was sovereign


immunity.

B. No, because this kind of thing is


difficult to prove.

C. No, because of the Speech and


Debate Clause.

D. None of the above.

180 CONSTITUTIONAL LAW


0
Questions 78 — 79 are based on the B. Yes, if the crimes were commit-
following facts: ted prior to the date of President =
George's pardon.
0
Governor Pilson, governor of the fictional
state of Columbia, is indicted in Columbia C. Yes, if President George is still
for the embezzlement of state funds. United in office.
States President George, convinced that such
is a paltry political attempt to destroy one of D. Yes, unless the crime was infa-
his closest allies, pardons Pilson for "any mous.
and all crimes committed while in office,
up to and including those committed as of 80. The ambassador to the United States
this date." from the fictional foreign republic of
Sheisterdam, while touring the United
78. If the Columbia Attorney General chal- States, gets drunk while driving in the
lenges this pardon, she will: state of Oregon and is arrested by the
Oregon police. In fact, the ambassador
A. Fail, because such is one of the was not legally drunk and was unaware
enumerated presidential pow- of the Oregon prohibition of driving
ers. while intoxicated. The local district
attorney decides to prosecute. What is
B. Fail, because the President's the ambassador's best defense?
power is limited to pardon for
political crimes. A. Oregon does not have jurisdic-
tion.
C. Succeed, because the Presi-
dent's power is limited to federal B. He was not drunk.
crimes.
C. He was ignorant of the law.
D. None of the above.
D. Oregon does not have jurisdic-
79. Assume for this question only, that ex- tion over foreigners in their
Governor Pilson is federally indicted state.
two years after President George's
pardon for theft of government funds.
Will the President's pardon bar federal
prosecution?

A. Yes, if the crimes were commit-


ted while Pilson was in office.

CONSTITUTIONAL LAW 181



81. State Miseria Attorney General Ratlin Questions 83 — 85 are based on the

has a pattern of convening grand juries following facts:
to investigate his political rivals and ex-
Herbert and Jack are homosexual lovers.
tort "gifts" for his immediate relatives
They have been lovers for 10 years and
in violation of a federal statute on abuse
enjoy a monogamous relationship. Herbert
of office. Moss is one such individual
is of Asian descent and Jack is of African
so targeted for investigation. If Moss
sues the Attorney General of Miseria, descent. The fictional state of Madisonia
allows homosexual marriages, provided
his suit will:
that the would-be partners are of the same
race. Herbert and Jack apply for a marriage
A. Be dismissed, because of the
license, but such is denied. They sue under
doctrine of sovereign immu-
nity. the U.S. Constitution.

83. What is the state's best argument why


B. Be dismissed, because of the
Eleventh Amendment. their prohibition should be upheld?
A. There is no constitutional right
C. Be dismissed, because of the
Tenth Amendment. to engage in consensual homo-
sexuality.
D. None of the above.
B. There is no constitutional right
82. Independent Party presidential candi- to marry.
date Irwin Chummer is subpoenaed
C. There is no constitutional right
by Congress regarding an investiga-
in privacy.
tion into political corruption, and is
requested to furnish lists of all "party
affiliates." Chummer refuses to furnish D. Marriage regulations are within
a state's jurisdiction.
the lists, and is held in contempt of
Congress. Chummer challenges this.
What will be his best argument? 84. What is the defendants' best argu-
ment?
A. Separation of powers.
A. There is a constitutional right to
B. Independent State Grounds. privacy.

C. Irrelevance. B. There was a Due Process Clause


violation.
D. None of the above.
C. The state rule served no rational
basis.

182 CONSTITUTIONAL LAW


D. The state had no compelling II. The Post Office search
interest. was in violation of the
Fourth Amendment.
85. Under existing U.S. Supreme Court
precedent, who is likely to succeed on III.The First Amendment
federal appeal? protects the right to pos-
sess obscenity involving
A. The state, because there is no children in the home.
right to practice homosexuality.
A. I and II only.
B. The state, because there is no
right to homosexual privacy. B. I only.

C. The defendants, because there C. III only.


was a denial of Equal Protec-
tion. D. None of the above.

D. None of the above, because this


is an open issue.

86. Monte purchases a newspaper entitled


"Swinging Singles." Later that eve-
ning, in the privacy of his home, he
reads an ad describing a video. The ad
promises "an array of 13-year-old girls,
performing all imaginable sexual acts."
Monte sends for the video. The video
is lawfully intercepted by the Post Of-
fice, then delivered to Monte. A proper
warrant to search Monte's apartment
for the video is obtained. Monte is then
prosecuted in his state for the "knowing
possession of obscene materials involv-
ing minors." Which of the following
propositions are correct?

I. The warrant was over-


broad.

CONSTITUTIONAL LAW 183



O Questions 87 — 89 are based on the A. No, because of the Supremacy

following facts: Clause.

O The fictional County of Lemon, in the fic- B. No, because of the Privileges and
tional state of Reagonia, has been plagued Immunities Clause.
by a series of advertisements from certain
segments of the garment industry. The ads C. No, because the President is the
purport to offer the lowest prices. The ads Commander-in-Chief.
indicate that the prices are so low because
the goods are imported and "Not Made in D. Yes, because of the Tenth Amend-
America." Incensed by the ads, Lemon ment.
County makes it a misdemeanor to advertise
clothing in the state. 89. Assume for this question only, that
Lemon County has also banned all legal
87. Seeme Clothiers, of Lemon County, and medical advertising — because they
challenges the outright ban. Will they are "unprofessional and have decreased
succeed? the quality of services." How should
these regulations be analyzed if chal-
A. No, because such is part of the lenged?
state's police powers.
A. Vagueness Doctrine.
B. No, because there was a legiti-
mate state interest shown. B. Ripeness Doctrine.

C. Yes, because the ad was protected C. Commercial Speech Doctrine.


under the First Amendment.
D. Just Compensation Clause.
D. Yes, because of the Commerce
Clause.

88. Assume for this question only, that the


United States has made a treaty with
the foreign country which manufactures
Seeme's clothes, and the treaty allows
"free and unencumbered trade between
the two countries." Assume further that
the Lemon County law now prohibits
the sale of any clothing made outside
the United States. Is that revised Lemon
County ordinance lawful?

184 CONSTITUTIONAL LAW


90. The fictional state of Barrier requires C. Procedural due process concerns
all of its attorneys to pass a moral fit- the right to notice and hearing,
ness exam in order to be licensed. The while substantive due process
burden is on the applicant to show good concerns the interpretation of
moral character. Applicant Spendthrift, explicit and implicit liberties of
10 years earlier, had declared bank- individuals.
ruptcy. In the last 8 years, however, he
has established a fine credit rating and D. None of the above.
has held a steady job. Spendthrift, after
passing the bar, is denied a law license
by the Barrier Supreme Court, due
to his prior bankruptcy. What will be
Spendthrift's best argument on federal
appeal?

A. He is now morally fit.

B. Significant time has passed.

C. Barrier's ruling is contrary to


Congress's intent to allow bank-
rupt persons to start over.

D. None of the above.

91. Procedural due process differs from


substantive due process in that:

A. Procedural due process concerns


state regulation, while substan-
tive due process concerns federal
regulation.

B. There is no difference.

CONSTITUTIONAL LAW 185


Questions 92 — 94 are based on the II. Yes, if he was punished
following facts: because it was burned in
an unreasonable time,
Junior attends Jalonia Junior High School place and manner.
in the fictional city of Jalonia, a city in the
fictional state of Prisonia. Junior is a po- III. No, because there was no
litical activist. Having kept up on the latest hearing.
and most important United States Supreme
Court pronouncements, Junior realizes it is IV. No, because there is a
his constitutional right to burn a U.S. flag in right to burn the U.S. flag
political protest. Attempting to exercise this in political protest.
right, Junior burns the Jalonia Junior High
School's U.S. flag, in the main auditorium A. I and II only.
of the junior high school, in the middle of
the entering class's ceremony. B. III and IV only.

92. If Junior is summarily suspended for C. II only.


one month from Jalonia Junior High
School, his constitutional rights have D. IV only.
been:
94. Assume for this question only, that
A. Denied, because he was not Junior's parents take him out of the
given a prior hearing. Jalonia Junior High School, and enroll
him in the Bush Academy, a private
B. Denied, because flag-burning is school. Junior again burns a U.S. flag.
a right. What is Junior's best argument in ap-
pealing this suspension?
C. Not denied, because it was not a
specific political protest. A. Procedural due process.

D. Not denied, because it was like B. Substantive due process.


shouting "Fire!" in a crowded
theatre. C. First Amendment.

93. Can Junior be criminally punished for D. None of the above.


this act of flag-burning?

I. Yes, if he was punished


because it was not his
flag.

186 CONSTITUTIONAL LAW


95. Intent on restoring family values, the C. Yes, because there was no
state of Alabama has just enacted a rational basis.
statute whereby illegitimate children
cannot sue in tort for the wrongful death D. No, because there was a rational
of their parents. Is this a lawful state basis.
regulation?
97. The state of Hawaii legislature, still
A. Yes, because of the Tenth Amend- angered at the gross inequities placed
ment. on the many by the few old landown-
ers before the state made that conduct
B. Yes, because Alabama has a illegal, pronounces a list of the twenty
permissible interest in so legis- worst offenders. It also fines them
lating. $25,000 each and declares them guilty
of a "misdemeanor against the People
C. Yes, because Alabama has a ra- of the State of Hawaii." They are not
tional basis in so legislating. sentenced, however, to any time in jail.
Under which of the following consti-
D. None of the above. tutional clauses will these individuals
have a successful claim?
96. The fictional state of Columbia, pre-
cludes all individuals who are color I. Just Compensation.
blind and dyslexic from working as
technicians in the state's nuclear power II. Fifth Amendment Due
plant because of safety concerns. It Process.
has been shown that many of these
individuals are capable of performing III. The Bills of Attainder
the technical work required. Never- Clause.
theless, they are precluded. One such
individual, Hal Sims, sues based on a IV. The Ex Post Facto
denial of Fourteenth Amendment Equal Clause.
Protection. Will Hal's suit prevail?
A. I, II, III and IV.
A. Yes, if he could perform the re-
quired work. B. III and IV only.

B. Yes, because in this case there C. II only.


was no compelling state inter-
est. D. None of the above, since no jail
time was imposed.

CONSTITUTIONAL LAW 187


98. Fictional President George, after a Questions 99 — 100 are based on
highly difficult, but successful, re-elec- the following facts:
tion campaign, feeling duly grateful to
his Chief of Staff Snaker, bestows upon Rawson is sentenced to death at the capital-
him the title of "Royal Knight." The sentencing phase of his murder prosecution.
title has no significance whatsoever, Over objection of the defense, the jury is
but there is an official ceremony at the told that Rawson is a member of a white
White House. Ned Rader challenges supremacist group. Rawson is sentenced to
this gesture in federal court. Will Rader death and appeals the conviction based on
succeed? the prosecutor's allusion to his membership
in the white supremacist group.
A. No, because the title has no sig-
nificance. 99. What will Rawson's best constitutional
argument be?
B. Yes, because it is a title of Nobil-
ity. A. That he is being subjected to
Cruel and Unusual Punishment.
C. No, because it is analogous to a
Presidential Medal. B. That his Freedom of Association
Rights were abridged.
D. No, because Rader has no stand-
ing. C. That his Free Speech rights were
abridged.

D. None of the above.

100. If Rawson's white supremacist group


did not advocate disobedience to the
law, and if both Rawson and the victim
are white, will Rawson prevail?

A. No, because Victim Impact state-


ments are admissible.

B. No, because the error was harm-


less.

C. No, because it was relevant.

D. Yes, because it was irrelevant.

188 CONSTITUTIONAL LAW


101. A story is written about movie star
Birdie Jones, in the tabloid, the Nation-
al Perspirer. In the story, it is revealed
that Birdie is infected with genital
herpes. If Jones sues the newspaper in
tort for defamation, she will:

A. Prevail, because such is damag-


ing.

B. Prevail, if the story was know-


ingly false.

C. Prevail, since she has a right to


privacy.

D. Fail, because she does not have


a right to privacy.

ND OF QUESTIONS

CONSTITUTIONAL LAW 189


190 CONSTITUTIONAL LAW
CONSTITUTIONAL LAW 3. A is the correct answer. The test of Lem-
on v. Kurtzman, 403 U.S. 602 (1970)

ANSWERS is controlling. To not violate the Estab-


lishment Clause, a state practice must
have a secular purpose, a religiously
1. A is the correct answer. Junior's claim is neutral effect and must not entangle a
one of the Establishment of Religion in religion with the government. Under a
violation of First Amendment. B is in- similar fact pattern in Lee v. Weisman,
correct because it is not a Speech claim. 112 S.Ct. 2649 (1992), a benediction
It is true that First Amendment claims by a rabbi before a school graduation
against a state entity are made applica- was held violative of the Clause. B
ble against the state by the incorporation is incorrect because no facts indicate
of some of the Bill of Rights under the Quail was forbidden to practice his own
Fourteenth Amendment. However, the religion. Choice C requires a more dif-
true basis of his claim is the Religion ficult analysis. Although it reasonably
Clause of the First Amendment, so A is appears that all recognized religions are
better than C. D is not correct because being treated equally, non-recognized
A is correct. religions and atheism are not. More fun-
damentally, as Justice Kennedy, in Lee
2. Question 2 differs from question one v. Weisman, supra, indicated, what be-
in that Junior files his case in federal gins as a good faith practice in the area
court rather than state court. However, of religion, too often historically ends in
this difference makes no difference to persecution and forced indoctrination.
the question. The correct answer is A D is incorrect because the motives for
for the answer given in #1 above. a person suing are not relevant, the is-
sue is whether he has a good cause of
action.

CONSTITUTIONAL LAW 191


4. B is the correct answer because only 6. D is the correct answer, since only
I is incorrect because whenever a state bring a constitutionally-based claim, the
sponsors religious activity, the Estab- individual must have standing. Stand-
lishment Clause of the First Amend- ing requires that the individual's own
ment is implicated. (This Clause may rights were denied. See, e.g., Simon v.

A not require suspension of the activity,


but it is implicated.) Choice II is cor-
rect because significantly less First
Eastern Kentucky Welfare Rights Or-
ganization, 426 U.S. 26 (1976). Here,
since Don was, in fact, arrested under
O Amendment protection is given when this statute, he clearly has standing.
it is adults, not school children, who Thus, choice I is correct. Choice II
are subjected to the prayer/benediction. is correct because this controversy is
O See Marsh v. Chambers, 463 U.S. 783 "ripe." Ripeness requires that the chal-
(1983) (where Nebraska's practice of lenged statute is either being enforced,
allowing a prayer before legislative ses- has a real danger of imminently being
sion was allowed). Although a school enforced, or is dissuading individuals
board meeting is not identical to a leg- from pursuing their constitutionally
islative session, it is likely that Marsh protected liberties via its existence. See
controls. Choice III is correct because generally, Epperson v. Arkansas, 393
were the purpose of the school board's U.S. 97 (1968). Obviously, since Don
prayer to indoctrinate school commit- is actually being prosecuted under the
tee members, then there would be an statute, the controversy is ripe. Choice
Establishment Clause violation. III is incorrect for the same reason that
choice II is correct. Had Don not been
5. C is the correct answer. Don was at- arrested and had the statute been anti-
tempting to influence the marketplace quated and never enforced, there would
of ideas. Thus, the Free Speech Clause be a problem of mootness. But that was
is implicated. Since political parties are not at all the situation here. Choice IV
generally not considered to be religions, is incorrect because although aliens do
choice A, which cites to the Freedom of not enjoy all of the rights of citizens,
Religion Clause, is incorrect. Choice they do enjoy many, if not most of the
B is clearly incorrect because the Fifth constitutional freedoms in this country.
Amendment Due Process Clause is only See generally, Yick Wo v. Hopkins, 118
federally applicable-and this is a state U.S. 356 (1886).
case. D is incorrect for the same reason
that C is correct.

192 CONSTITUTIONAL LAW


7. A is the correct answer. Restrictions 9. C is the correct answer. Constitutional
on freedom of speech based on content claims must be ripe. As a general rule, a
are subjected to the most heightened statute which has been on the books for
degree of scrutiny. Because the law is some time, which is openly disobeyed
concerned with the message or content and which is not enforced indicates
of Don's speech, a high standard will be there is no real controversy. Poe v. Ull-
used. Since choices B and C use lesser man, 367 U.S. 497 (1961). A is incorrect
standards, they are incorrect. Since A is because, since the law is still on the
correct, obviously D is incorrect. books, the point being challenged has
not been legally settled — therefore, it
8. D is the correct answer. Although con- is, by definition, not "moot." B is incor-
stitutional claims based on the Freedom rect because the motive of the litigant
of Speech are strictly scrutinized, and, in bringing a law suit is, generally, of
thus, the state is required to show a de minimis importance. D is incorrect
compelling interest for the restriction because it misconstrues the meaning
to be in harmony with the Constitu- of the Tenth Amendment (Independent
tion, because of the particular danger State Grounds Clause). Although a state
of intimidation of voters at the polls, is permitted to expand the rights of its
this is one regulation which will satisfy citizens under its own constitution, it is
the difficult Strict Scrutiny test. Histori- not permitted to remove rights which
cally, intimidation and fraud at the polls are federally-guaranteed. Free speech
has been a real and recurrent evil. Thus, is federally mandated to the states by
Columbia's statute is constitutional. way of the Fourteenth Amendment Due
In Burson v. Freeman, 112 S.Ct. 1846 Process Clause.
(1992) a similar Tennessee statute (re-
quiring a 100 foot distance from the
polls) was upheld. Since the statute was
lawful, and the proper state burden was
a compelling interest, choices A, B and
C are incorrect.

CONSTITUTIONAL LAW 193


10. D is the correct answer. Although this 11. A is the correct answer. The Com-
question is technically one of Criminal merce Clause mandates that a state is
Procedure, it serves to show the broad not permitted to restrain the commerce
overlap between these two subjects. between itself and sister states. The
Under the Exclusionary Rule of the Clause essentially precludes economic
Fourth Amendment, if there was unlaw- protectionism. Since State X's rule fa-

A ful governmental activity and illegal


evidence is discovered as a result of
that activity, that evidence is generally
cially discriminated against State Y's
business, it is unconstitutional. The
language of the rule gave a "discount,"
O to be suppressed. See Wong Sun v. as opposed to tacking on a surcharge,
United States, 371 U.S. 471 (1963). In is of no moment to the substance of
the case at hand, it is stipulated that the the regulation. See Chemical Waste
O law is unconstitutional. Since the arrest Management Inc. v. Hunt, 112 S.Ct.
and follow-up search were the fruit of 2009 (1992). Since the Chemical Waste
that, its result (the marijuana) is to be case, supra, is directly on point, and
suppressed. [Please note: although the uses as its reason, the Commerce Clause
statute is, in law, constitutional, it is choices B, C and D are incorrect. D is
"fair game" for a question to make a also incorrect because state sovereignty
stipulation which is ultimately incor- falls to the requirements of the Com-
rect. Therefore, students should avoid merce Clause.
reading questions on an exam and then
using them to go back and change the 12. B is the correct answer. The FFL ordi-
answers on questions they have already nance is a clear example of a facially
answered.] A is incorrect for the same invalid ordinance in that it arbitrarily
reasons that D is correct. B is incorrect restricts the freedom of speech. Since
because there is no drug exception to the FFL, itself, was not outlawed, there
the warrant requirement. C is incorrect is no Freedom of Association claim.
because it disobeys the stipulation in Thus, A is incorrect. Since there are
the call of the question. no facts indicating the Simmee Board
treated other groups better, there is no
Equal Protection claim on these facts
and C is incorrect. Since it was speech
which was implicated, a more specific
claim would be phrased as "Speech,"
not "Due Process" so B is a better
choice than D.

194 CONSTITUTIONAL LAW


13. A is the correct answer because only 15. C is the correct answer. In order for
choice I is correct. Choice I is correct there to be a § 1983 claim, there must
because the Simmee Board and Mayor be a constitutional violation on the part
were restraining the public speech in of the city. It is not enough that the city
advance of its occurrence. This, by defi- committed a tort, choice A. Such tort
nition, is a prior restraint. Choice II is may give rise to a successful civil suit,
factually incorrect. Although a city may but not a § 1983 claim. See Collins v.
place reasonable time, place and man- Harker Heights, 112 S.Ct. 1061 (1992).
ner restrictions on public speech; this Choice B, although probably correct,
regulation is unlawful because it placed is not as good an answer as choice C.
too much power in the Board. See Although most crimes committed under
Forsyth County v. Nationalist Move- the color of governmental authority
ment, 112 S.Ct. 2395 (1992). Choice will also be violations of constitutional
III is obviously incorrect because a city rights, the true issue is whether there
government's regulation is considered was a constitutional violation — not
to be state/governmental action; it is the whether there was a crime. Since C is
official action of a government subdivi- correct, and since B is arguably correct,
sion. D is incorrect.

14. A is the correct answer because choices 16. D is the correct answer. A is incorrect
I and II are correct. Choice I is correct because mere negligence in the govern-
because although a city is allowed ing of a city does not in and of itself
to place time/place/manner limits on create a successful constitutional due
public speech, such must be narrowly process claim. Collins, supra answer
tailored. The Simmee Board, in its 15, indicated such (on similar facts). B
unchecked ability to set the cost, did is incorrect because it begs the ques-
not utilize such a narrow restriction. tion. For there to be a successful § 1983
Choice II is correct because one of the claim, there must be an underlying
fears underlying the First Amendment constitutional violation. Such is not
was governmental abuse of power used indicated here. C is factually incorrect .
to suppress speech. In Simmee, the The city appears to have been negligent.
Board had too much power. It could Sharon likely has a suit — but her suit
effectively drive out public speech by is not properly based on due process
setting an oppressively high licensing grounds. Since A, B and C are all incor-
fee. Choice III is incorrect because both rect; D (none of the above) is the correct
I and II are correct, B, C and D are not answer.
correct answers.

CONSTITUTIONAL LAW 195


17. B is the correct answer. The effect of 18. B is the correct answer. Duke was
the Perotia regulation was, in fact, eco- essentially expressing ideas. Thus,
nomic protectionism. Local waste-pro- the First Amendment (by way of the
ducers are afforded an advantage over Fourteenth Amendment Due Process
out-of-state waste-producers. Thus, the Clause) is most at issue — when it is
Commerce Clause prohibits such. See his expressions which are being cur-

A Fort Gratiot Sanitary Landfill v. Michi-


gan Department of Natural Resources,
112 S.Ct. 2019 (1992). Although choice
tailed by the state. Although choice A
is superficially appealing in that Duke
was expressing ideas about the Second
0 A offers a correct statement that a state Amendment, in no way were his gun
can prevent pollution, it is incorrect if rights then curtailed. Alternately stated,
cn the state violates the Constitution in so the ordinance regulated not only those
0 doing. C is factually incorrect because expressing ideas about gun rights, but
an economic advantage is given to in- also those expressing any ideas in the
state waste-producers. D is incorrect for airport setting. Thus, choice A is incor-
the same reason that B is correct. rect. Since Duke was not prevented
from associating with other believers
in the Second Amendment and since
he was free to organize Second Amend-
ment activities elsewhere, his Freedom
of Association rights were not abridged.
C is therefore incorrect. D is obviously
incorrect because in no way did these
facts indicate any restraint on interstate
commerce. If the student chose this
answer, he/she should remember that
previous questions on a multiple choice
exam do not influence the answers to
later questions !

196 CONSTITUTIONAL LAW


19. D is the correct answer. As set out 20. A is the correct answer. Commercial
in International Society for Krishna solicitation of funds in an airport can
Consciousness v. Lee, 112 S.Ct. 2701 be prohibited. Duke was asking for
(1992) [hereinafter, ISKC-I], airports money when arrested on July 4. The test
are not considered to be "public forums" to be used is whether the limitation is

A
for First Amendment Free Speech pur- merely reasonable. In ISKC-I, supra, it
poses. Airports are a new phenomenon was held similar regulation was reason-
and will not be given the status of, e.g., able. A compelling interest need not be
a public park. Since sidewalks outside shown by the state — since airports are
the airport are available for exercising not public forums. B is incorrect not
speech rights and due to the hurried only because it misses the key issue (the
nature of air travel, the most protec- solicitation of funds), but also because
tive level of expression was held to be we know of no evidence indicating
inappropriate. A is incorrect because the danger of small cactus plants. C
the mere fact that the public is allowed is incorrect because no facts indicate
in a place, does not make that place a the airport was privately-owned. D is
"public forum." For obvious example, incorrect for the same reasons that A is
the public is allowed within a courtroom correct.
to watch trials, but the courtroom is
not a public forum. B is incorrect for 21. A is the correct answer. On July 6 he
the same reasons that D is correct. C was again attempting to solicit funds
is incorrect because some areas which — this time by selling books. Since the
are privately-owned may become state county regulation was reasonable due
actors because of their relationship with to the nature of an airport (it is not a
the government. Once they are treated public forum), such can be prohibited.
as state actors and government property, See ISKC-I. B is incorrect because
they may then become public forums. if an individual is in a public forum,
Moreover, no facts indicated the Wayne although many will consider him/her
Airport to be privately-owned. to be a "nuisance," such activity is
nevertheless protected. C is incorrect
because no facts indicate the airport
was privately-owned. D is incorrect
because the First Amendment Free
Speech Clause is subject to reasonable
time, place and manner restrictions. It
is not an absolute.

CONSTITUTIONAL LAW 197


22. D is the correct answer. On July 8, 23. D is the correct answer. As held in
Duke was not soliciting funds. He was ISKC-I and ISKC-II, airports are not
giving literature away for free. Thus, public forums. Thus, a county can
since there was no commercial activ- restrict speech activity there if such
ity, there is a heightened First Amend- restriction is reasonable in light of the
ment protection. In short, although it purposes of the forum and not an at-

A is reasonable to restrict solicitation of


funds, it is not reasonable to restrict
disseminating literature at airports at
tempt to suppress the speaker's view-
point. The stringent compelling interest
test is to be used only in public forums.
O no fee. See Lee v. International Society A is therefore incorrect because the fact
for Krishna Consciousness, 112 S.Ct. that the First Amendment is implicated
cn 2709 (1992) [hereinafter, ISKC-II]. A does not require always utilizing the
O is incorrect because on July 8 he was compelling interest level of scrutiny. B
not soliciting funds. He was merely is incorrect because Duke was not ar-
giving away literature. Thus, the facts rested for a gun violation and because
distinguish this case from ISKC-I. B is the Second Amendment has not been
incorrect because if the U.S. Constitu- made applicable to the states. C is in-
tion enshrines a fundamental right, a correct for the same reasons that D is
state pattern of abrogating that right correct.
in no way shifts the inquiry to the
defendant regarding his assuming the 24. A is the correct answer. As indicated by
risk by continuing in his constitution- ISKC-I and ISKC-II, the airports are not
ally-protected activity. If such were the public forums but non-public forums,
law, there would be nothing left of the the reasonableness test is appropriate
Bill of Rights — provided a state had as long as the regulation is not an at-
a pattern of disallowing its clauses. C tempt to suppress the viewpoint of the
is incorrect because although a citizen speaker. B, C and D are incorrect for
may have other options in exercising the same reasons that A is correct.
her constitutional liberties, the state is
not permitted to use such as a defense
in curtailing those liberties.

198 CONSTITUTIONAL LAW


25. D is the correct answer because only 26. B is the correct answer. The Congres-
choice I is correct. It is stipulated that sional Act may infringe on the Tenth
Duke's rights were, in part, violated. Amendment, which mandates that the
The right violated occurred in the July federal government not take powers
8 arrest by Darby. Duke's distribution which were not expressly given to it,
of free literature on July 8 was First by the Constitution. It is arguable that
Amendment protected. A § 1983 suit in ordering the states to take title to
can be brought when there is such a their own waste and pay damages, the
violation of civil rights. Thus, the gov- federal government is usurping state
ernmental body responsible for the ordi- power. Although in choice A there may
nance can be liable. The Port Authority be some restraint on commerce, the re-
was that body, so choice I is correct. straint is by the federal government, so
Choices II and III are incorrect because the Commerce Clause is not implicated.
the arrests by Bart and Cary were for Although the Ninth Amendment might
commercial solicitation and, thus, were be relevant in that it protects against
valid under ISKC-I. Although Darby's federal encroachment of unenumerated
arrest was invalid, her (likely) good rights, the history of that Amendment
faith is a complete defense to a § 1983 indicates an unwillingness of the U.S.
claim, so choice IV is incorrect. Supreme Court to use it. B is incor-
rect. Choice D is viable, but not the
best alternative. It is difficult to see
how this could constitute a taking of
private property. Therefore, D is incor-
rect. Moreover, in New York v. United
States, 112 S.Ct. 2408 (1992) an issue
much like this was discussed. The High
Court chose to frame its analysis around
the Tenth Amendment. Therefore, since
there is a case directly on point, such is
controlling. The student should there-
fore note that success in Constitutional
Law depends as much on knowing the
key case holdings as it does on skill in
analysis.

CONSTITUTIONAL LAW 199


27. C is the correct answer. This is a difficult 28. B is the correct answer. The Island
question, which, again, may require Supreme Court should strike down the
knowledge of the case cited in answer more burdensome law of the city. A is
26. Our constitutional system sets up a incorrect because freedom of association
balance between the state and federal is not the reason Norris will prevail. C
governments. The federal government is incorrect because independant state

A can pre-empt state regulation which


is clearly contrary to federal interests.
grounds is a doctrine on federal court
power to review state decisions. Bob

.,,
...„.
But such jurisprudence falls short of
allowing the federal government to re-
Norris' action is being brought in state
court. The doctrine of independent state

.
,...,
quire the states to administer a federal grounds does not apply.
regulatory scheme. That would violate
the Tenth Amendment. In providing 29. D is the correct answer. Review of state
incentives to the states to dispose of court decisions by the U.S. Supreme
waste, the Tenth Amendment has not Court is generally discretionary. It is
been violated. Thus, clauses 1 and 2 by petition on writ of certiorari. The
of the Act pass constitutional muster. writ is granted when 4 of the 9 justices
Therefore, choices A and B are in- wish to hear the case. Cases may be
correct (because clauses 1 and 2 do accepted when there is confusion in
not violate the Tenth Amendment). constitutional law, when there is a split
However, in mandating the states to among the federal circuits or when the
follow a regulatory scheme (clause 3), justices desire to make a change in
Congress stepped over the fine line and the law. But, the point is, the decision
usurped states' rights. Therefore, C is whether or not to review is in the hands
correct and D is incorrect. With this of the judges of the High Court. Thus,
analysis, the legitimate federal interest choices A, B and C, which all indicate
in promoting waste disposal is allowed, the U.S. Supreme Court either must or
while the state government method is must not accept review, are incorrect.
not usurped.

200 CONSTITUTIONAL LAW


30. A is the correct answer. Under Illinois 32. D is the correct answer. Under Casey,
State Board of Education v. Socialist supra answer 31, the rigid trimester test
Worker's Party, 440 US 173 (1979), of Roe was abandoned. Under that test,
the U.S. Supreme Court indicated that the woman had a right to abortion in
the association rights implicit in the the First Trimester. Under Roe, there
First and Fourteenth Amendments are could be no pre-viability regulation,
infringed by ballot access rules. The since the state's interests were not
state must meet strict scrutiny to justify compelling enough at that time. Under
their ballot restrictions. B is incorrect Casey, however, a new "undue burden"
because it states the rational basis test. standard was created. Thus, pre-vi-
C is incorrect because this is not a fed- ability regulation is acceptable if such
eralism question, but an infringement does not place an undue burden on the
on fundamental federal constitutional constitutionally-protected privacy right
rights. Thus, the Tenth Amendment is of abortion. A one-day notice of alterna-
inapplicable. D is incorrect for the same tives requirement, although a burden,
reason that A is correct. is not an undue burden — and, thus, is
constitutional under Casey. A is incor-
31. A is the correct answer. Although in rect because the pre-viability regulation
Planned Parenthood of Southeastern aspect of Roe was abandoned. Although
Pennsylvania v. Casey, 112 S.Ct. 2791 B offers a correct statement of law,
(1992) [hereinafter Casey], the abor- abortion is not now... and even under
tion right was limited; such was not Roe, was not an absolute right. There
over-ruled. Under Roe v. Wade, 410 are and have been limitations. Choice C
U.S. 113 (1973), there was no right is correct. However, D is a legally more
to abortion when the fetus was viable, specific answer, and, thus, is the better
i.e., in the third trimester because the choice.
state's interest in protecting potential
life normally outweighed the mother's
right to terminate her pregnancy during
that time. Since this aspect of Roe has
remained intact, since Clause 1 did little
else than reiterate this and since the Cir-
cuit Court of Appeals must follow the
U.S. Supreme Court's pronouncements,
choice A is correct. Choices B and C are
incorrect because a first trimester fetus
has not been held to be a human. D is
incorrect for the same reasons that A is
correct.

CONSTITUTIONAL LAW 201


33. D is the correct answer. All three aspects 35. A is the correct answer. When it is a
of Clause 3 are constitutional under minor woman who wishes to have an
Casey, supra answer 31, in that none abortion, her decision may not be as
of them place an undue burden on the intelligently-made as that of an adult
woman seeking an abortion. The medi- female. Thus, under the Casey reason-
cal emergency aspect of Clause 3, in no ing, supra answer 31, this limitation is

A way limits the abortion right — thus, it


is certainly correct. The informed and
consent aspects of Clause 3, although
constitutional. Under the Casey analysis
as found by the Supreme Court, insu-
lation from others wishing to counsel
0 imposing a burden on the abortion right, against the abortion is not required.
...cc are little more than the requirement Thus, A is correct. Choices B, C and D
cn that the woman be told of other options are incorrect under the law of Casey,
0 prior. Thus, under the Casey test, they for the same reasons that A is correct.
are acceptable. Although choices A, B B and C are incorrect because the rigid
and C all include approval of one aspect trimester analysis of Roe has been aban-
of Clause 3, only choice D indicates all doned. D is incorrect because the issue
three aspects of the clause are correct. is no longer open.
Thus, A, B and C are incorrect and D
is correct. 36. D is the correct answer. Requiring abor-
tion clinics and doctors to keep records
34. D is the correct answer. It may seem indicating that they have complied with
that the requirement of notice to the the constitutional aspects of a state's
husband is no more burdensome than law is legitimate. But, a clinic/doctor
the prior clauses, but the U.S. Supreme cannot be compelled to keep a record
Court held otherwise. The reasoning is of something which they must not do.
predicated on the fact that the women The requirement of Clause 4 (husband
so required are beyond the age of ma- notification) places an undue burden
jority and that it is, thus, her decision on the woman, Casey supra answer 31,
— not her husband's. Such clause would thus, choice D is correct. Since Clauses
prove to be an undue burden when, 1, 2, 3 and 5 are constitutional; choices
for example, the husband and wife are A, B and C, which each require excis-
separated. Since it is the woman's inter- ing one or more of these, are logically
est which is paramount, this clause is incorrect.
unconstitutional. A is incorrect because
even notice to the husband is potentially
incendiary — and may be restrictive of
the abortion right. B is incorrect be-
cause the burden analysis focuses on the
woman — not her husband. C is incorrect
for the same reasons that D is correct.

202 CONSTITUTIONAL LAW


37. B is the correct answer. The Commerce 38. D is the correct answer. The Clin-
Clause is violated when a state imposes tonia Hate Crime Law violates the
a use tax on a mail order business which First Amendment Freedom of Speech
lacks a substantial nexus with the state. Clause (as applied to the states by the
Although selling goods in the state is Fourteenth Amendment Due Process
a sufficient minimum contact for Due Clause). It is "overbroad" in that it goes
Process Clause analysis, it will not per
se satisfy the protections required by
the Commerce Clause. In fact, in Quill
beyond the constitutional standard for
punishment in substantially all of its
applications. The law not only prohib-
A
Corp. v. North Dakota, 112 S.Ct. 1904 its so-called fighting words, but also
(1992), such a tax was held to violate prohibits the stating of views which,
the Commerce Clause. Since choice A though unpopular, are within the tra-
utilized the inappropriate Due Process ditional domain of First Amendment
test, it is incorrect. Since choices C and protection. For example, under the
D reached an incorrect conclusion (even Hate Crime Law as written, it would be
though the correct test was used), they criminal to give a speech denouncing
are incorrect. one race. Of course such is offensive
— but traditional and present analysis
protects such — leaving it to the free
market of ideas to prove it wrong. See
R.A.V. v. City of St. Paul, Minnesota,
112 S.Ct. 2538 (1992). Choice A is in-
correct because the fact that all of the
written elements of a crime are pres-
ent does not insulate the crime from
constitutional attack (or defense). B is
incorrect because although Skin's acts
were undoubtedly not constitutionally
protected, nevertheless, he must be
convicted under a legitimate, i.e., con-
stitutional crime. Thus, B is incorrect
for the same reasons that D is correct.
C is incorrect because although there
was no mens rea (mental state) in the
crime, this is not relevant to the Free
Speech question at issue. D is the better
choice.

CONSTITUTIONAL LAW 203


39. D is the correct answer. Although Skin 40. C is the correct answer, since both
committed a criminal assault, to be choices I and II are correct. This ques-
convicted he must be charged under tion does not call for a judgment wheth-
a constitutional crime. This crime er the statute is constitutional. Rather,
was facially unconstitutional in that it simply calls for statutory analysis.
it prohibited not only unprotected Since the facts indicate that Toggle

A speech and action, but it also prohib-


ited protected speech. For example, it
prohibited a public forum racist/sexist
was paid $7333 by Violence Press and
inferentially, that Violence Press did
not pay the Bronsonia Victim's Fund,
O speech. Though such is distasteful, the it is reasonable to conclude they vio-
U.S. Supreme Court has realized that lated §208.2. Thus, choice I is correct.
it is a greater danger to prohibit such These same facts reasonably indicate
0 than it is to allow it. Alternately stated, that Toggle kept this money — did not
the Hate Crime Law was overbroad. pay it to the Bronsonia Victim's Fund.
A is incorrect because a conviction Therefore, choice II is correct. Since
is unconstitutional if the crime is un- C alone includes choices I and II, it is
constitutional — regardless of whether correct.
the defendant was proven to commit it
and regardless of whether there is an- 41. D is the best answer. Bronsonia adopted
other, different, crime under which the a so-called "Son of Sam" law. The U.S.
defendant should have been charged. Supreme Court examined a similar law
B is incorrect. Although Skin's threat in Simon & Schuster Inc. v. Members
was a criminal assault — was "fighting of the New York State Crime Victim's
words" — the crime under which he was Board, 112 S.Ct. 501 (1991). In broad
charged is clearly unconstitutional. See statement, it was there held that such
R.A.V. v. St. Paul, supra answer 38. laws violate the First Amendment Free
Although C is arguably correct, some Speech and Press Clauses. Moreover,
crimes can be "strict liability" — often paying close attention to the facts of
the court will read in a mens rea — but Question #41, it appears that Junkins
more importantly, D is a better choice. was not even in violation of the statute!
Thus, C is incorrect. Junkins was neither the co-author nor
the lawyer. Therefore, choices A, B and
C are all obviously incorrect. Although
the reason why the prosecution would
fail (which is given in choice D) is
incorrect — or at the least incomplete
nevertheless, D is the best of the an-
swers given.

204 CONSTITUTIONAL LAW


42. B is the correct answer, since choices 44. B is the correct answer because only
I and II are correct. Choice I is correct choices I and III are correct. State-run
because, in order to limit First Amend- (public) schooling is considered to be
ment Free Speech and Press on the state-action; and, thus, as in Brown v.
basis of content, the regulation must be Board, there are constitutional ramifica-
narrowly tailored to further a compel- tions of such. In Brown, it was reasoned
ling state interest. Choice II is correct
because in placing a burden on those
who merely admitted to crimes (but
that separate public schools, based on
race, are inherently unequal. Thus,
choice I is correct. In discerning which
A
who were not prosecuted or convicted), aspect of the Fourteenth Amendment
§208.1 was overbroad. If First Amend- was violated, that court logically chose
ment freedoms are to be limited by the the Equal Protection Clause (since
law, the limitation must be narrowly similarly situated individuals were not
tailored. Choice III is false because the being treated in a like manner). Thus,
Bronsonia procedure is an after-the-fact choice III is correct. Since the court did
punishment of speech and therefore not rely on other aspects of the Four-
does not set up a prior resraint. teenth Amendment, choices II and IV
are incorrect.
43. C is the correct answer. Article I, §2 of
the U.S. Constitution provides certain 45. D is the correct answer. As recently
rudimentary requirements regarding held in United States v. Fordice, 112
the allotment to the states of Repre- S.Ct. 2727 (1992), the repercussions
sentatives (e.g., no fewer than one). of de jure segregation are in no way al-
However, constitutional jurisprudence leviated by an open admissions policy
has essentially left the matter in the (race-neutrality) that perpetuates past
hands of Congress. Since the early part discrimination. Thus, A is incorrect.
of this century, Congress has utilized Similarly, if the effects of segregation
an "equal apportionment" method. In still exist, then, as a matter of law, there
United States Department of Commerce is more that can be accomplished — so
v. Montana, 112 S.Ct. 1415 (1992), this B is incorrect. C is incorrect because
method was challenged. The High court the state-applicable Fourteenth Amend-
upheld it. A is incorrect because the ment specifically limits states' rights.
method of apportionment is essentially
within the domain of Congress. There-
fore, choices A and B, which make it
mandatory, are incorrect. C is correct
because Congress is within its legiti-
mate jurisdiction by using this method.
D is incorrect for the same reasons that
C is correct.

CONSTITUTIONAL LAW 205


46. B is the correct answer because choices 48. A is the correct answer. For those stu-
I, II and III are correct. Choice I is dents choosing an incorrect answer, it
correct because an individual cannot should be remembered that prior ques-
utilize court process unless he or she tions do not teach the law [i.e., Question
has a claim in which she is a party. 47 may have indicated incorrect law.]
Here, Daniel lacks sufficient nexus to As Yee, supra answer 46 indicated,
the issue. There are no facts that he is for there to be a governmental taking,
a mobile home park landlord, or that thus, implicating the Just Compensa-
he is trying to raise rents, or even that tion Clause of the Fifth and Fourteenth
he lives in Marblestone. Choice II is Amendments (via Due Process), there
correct because there is no justifiable must be a physical occupation/trespass/
argument at present (see previous an- invasion of the property or a regulatory
swer on standing). More fundamentally, taking that goes too far. This kind of
choice III is correct because there is rent control law does not implicate the
no present controversy, nor is Daniel a Clause. B is incorrect because the Just
party to anything. Choice IV is incor- Compensation Clause was selectively
rect because although Yee v. City of incorporated into the state-applicable
Escondido, 112 S.Ct. 1522 (1992) held Fourteenth Amendment Due Process
there is no constitutional taking in this Clause. C is incorrect because the case
situation, there is still a federal question is ready for adjudication. D is incorrect
to be decided. for the same reasons that A is correct.

47. A is the correct answer. An individual 49. D is the correct answer. As indicated in
needs more than a mere possibility of Richmond Newspapers. Inc. v. Virginia,
economic interest to have his case be 448 U.S. 555 (1980), the public and
heard. Norris' "case" is not ripe. For press have a First Amendment right
it to be ripe, he would have to buy the to have trial be open. Thus, open tri-
park and attempt to raise the rent. As als are not only for the benefit of the
Yee, supra answer 46 indicates, such parties — but are also for the benefit of
an ordinance is not a taking and, thus, society's perception of justice. Thus,
does not implicate the Just Compensa- party consent is essentially irrelevant
tion Clause. Therefore, choices B and and A is incorrect. Choices B and C are
C are incorrect. D is incorrect for the incorrect for the same reason that D is
same reasons that A is correct. correct.

206 CONSTITUTIONAL LAW


50. D is the correct answer. Richmond 53. C is the correct answer. Waller v. Geor-
Newspapers, supra answer 49, indicates gia, 467 U.S. 39 (1984), indicates that
that all trials must usually be open to the cameras in the courtroom is a matter of
press and public — regardless of party state decision. There is no constitutional
consent. Thus, D is correct and A, B and First Amendment Free Press right to
C are incorrect. televise court proceedings. Some states
do and some do not. Interestingly, the
51. C is the correct answer. In Gannett U.S. Supreme Court does not allow its
Co.. Inc. v. DePasquale, 443 U.S. 368 own proceedings to be televised. Noth-
(1979), it was held that in certain ex- ing in the Constitution mandates it, and
tenuating/prejudicial circumstances nothing prohibits it. Therefore, under
the press and public could be barred the Tenth Amendment (Independent
from pre-trial admissibility hearings State Grounds Clause), such is within
— provided that a transcript of the hear- the discretion of the states. For the same
ing was made available to the press and (above) reasons that C is correct, A, B
public soon thereafter. Thus, since there and D are incorrect.
is a diminished First Amendment right
in the fact-pattern of pre-trial, the press 54. B is the correct answer. Under Em-
and public do not have an absolute right ployment Division v. Smith, the Court
to be there. Therefore, A is incorrect. B stated that criminal statutes applied
is incorrect because, as a general rule, evenhandedly towards religion may
the press and public can be present at regulate conduct arguably the subject of
pretrial hearings. D is incorrect for the a Free Exercise claim. Thus, if Jessica's
same reasons that C is correct. requiring $400 dues per month is con-
strued to be a fee for her sexual encoun-
52. D is the correct answer. It matters not ters, then she may well be found guilty
that the trial concerns an alleged victim of prostitution. A is incorrect because
who is a minor. Richmond Newspapers, the Freedom of Religion Clause does
supra answer 49, draws no distinction. not protect all "conduct." C is incor-
Thus, post-trial availability of a tran- rect because consent is irrelevant to the
script does not cure the First Amend- crime of prostitution. D is incorrect for
ment flaw. Therefore, choices A and C two reasons. First, the U.S. Supreme
are incorrect. Similarly, since the free Court is loathe to adjudge the merits
press right is not only for the parties, but or lack thereof of a religion. Cf. United
for the public at large, minor consent States v. Ballard, 322 U.S. 78 (1944).
is essentially irrelevant and B is incor- Second, even if a religion is legitimate
rect. and traditional, nevertheless conduct of
its adherents can be regulated.

CONSTITUTIONAL LAW 207


55. A is the correct answer. Prostitution 57. B is the correct answer. Although the
is sex-for-hire. Obviously, if Jessica's First Amendment Freedom of the Press
church had other functions and activi- will not allow the incitement of another
ties and if the payment of money was to imminent lawless activity, most any-
not for sex with Jessica, she can argue thing short of that is protected. Bran-
that the gravamen of her religion was denburg v. Ohio, 395 U.S. 444 (1969).

A not sex-for-hire. These arguments may


not be accepted, but they are the best of
the offered answers. B is a weak answer
Writing a book on an unusual religion
and/or on unusual sex is certainly First
Amendment protected. A is arguably
0 — there can be a house of prostitution correct, but it is not the best answer,
even if there is only one prostitute. C simply because the book appears to be
cn might be correct, except that it is less available to the general public. It does
0 specific than A. Taken literally, C is a not appear to be the dogma of Jessica's
poor answer — because even in a brothel church. C is incorrect because the pen-
there are other activities besides sex umbra privacy right flowing from the
(e.g., presumably talking and sleep- whole Bill of Rights is not the basis for
ing). D is incorrect because religious her claim. She is not engaging in this
conduct is not always First Amendment behavior under this question, but writ-
protected, Smith supra answer 54. ing about it. D is incorrect because the
book has little to do with association
56. Answer B is correct. The Court in Law- and more to do with speech.
rence v. Texas (2003) stopped just short
of holding that there is a fundamental 58. A is the correct answer. In Griswold
right of privacy to engage in non-mari- v. Connecticut, 381 U.S. 479 (1965),
tal sexual activity for all persons. That it was held that making criminal the
case and Eisenstadt v. Baird, contained use of contraceptives between a hus-
much that casts great doubt on a state's band and wife was unconstitutional in
authority to criminalize consentual ex- that the marital right to privacy was
tra-marital sexual activity between two fundamental. Thus, what Jessica does
persons. Thus, answer A is not correct with her husband, in the bedroom, is
and answer B is closer to the sense of constitutionally protected. B is incorrect
Lawrence. Answers C and D are irrel- because it misreads the facts, Jessica's
evant. right to marry was not being questioned.
C is incorrect because religious conduct
is not always protected by the Free Ex-
ercise clause. Smith supra answer 54.
D is incorrect for the same reasons that
A is correct.

208 CONSTITUTIONAL LAW


59. D is the correct answer. In a long line 60. D is the correct answer. Although this
of cases, the U.S. Supreme Court has question poses a more difficult query
upheld the validity of the so-called than in Question 59, supra, the analysis
Sunday closing laws. Were the Court is the same. The Sunday closing laws
to have construed these laws as Estab- have been held not to be an establish-

A
lishing the Christian Sabbath, then, of ment of religion for the purpose of es-
course, they would have been suspect. tablishing the Christian Sabbath. Rather
However, they have been upheld on they are for creating a uniform day of
secular grounds. That is, they promote rest. The purpose is secular — not reli-
0
a uniform day of rest and repose in the gious. Thus, A is incorrect. Braunfeld v.
community. Such has been held to be a Braun, 366 U.S. 599 (1961); Gallagher
=11
valid state interest. McGowan v. Mary- v. Crown Kosher Market, 366 U.S. 67 0
land, 366 U.S. 420 (1961); Two Guys (1960). B is incorrect because the exer-
r-
v. McGinley, 366 U.S. 582 (1961). cise of Judaism, itself, was not impeded
A is incorrect because the law in no by the law. Some conduct motivated
way limits Rex Bookstore's offering by religion may have been made more
of specific books or their availability economically burdensome — but the law
to be open on other days of the week. was neutral on its face. C is incorrect
B is incorrect because the Commerce because the state, in fact, performed
Clause is irrelevant to intrastate regu- no taking. It may have made business
lation. It forbids states interfering with more difficult — but such does not rank
commerce between their sister states. as a taking. Therefore, since choices
C is incorrect because the Blue Laws A, B and C are incorrect, D is the best
have been construed as non-religious in answer.
content. Since A, B and C are incorrect;
D is the best choice.

CONSTITUTIONAL LAW 209


61. A is the correct answer. This question is 62. D is the correct answer. As indicated by
important in showing two points. First, Wooley, supra answer 61, Ted's appeal
in constitutional analysis, it is always will be successful on First Amendment
important to discern which clause is at grounds. A is incorrect because, even
issue. Second, the law is not always per- though he did deface public property,
fectly logical. Rather, the law is driven the First Amendment may still bar con-
by case decisions which are directly on viction for some crimes. B is incorrect
point. Thus, mastery of Constitutional because, although there is a constitu-
law requires knowing the holdings of tional distinction between belief and
the basic cases. A correct answer to conduct, not all conduct is automatical-
this question requires knowledge of the ly able to be regulated consistent with
important case of Wooley v. Maynard, the Free Exercise Clause. C is incorrect
430 U.S. 705 (1977). There, it was held because Freedom of Association is not
that a Jehovah's Witness had the right implicated by the problem. D is correct
to cover what he believed to be the for the same reasons that A, B and C are
sacrilegious motto of New Hampshire's incorrect.
license plate: "Live Free or Die".
Since the case precedent is couched
in terms of Free Speech and not Free
Exercise, A is the correct answer. Cur-
rently, Employment Division v. Smith
would make a free exercise claim much
weaker against a neutral state law of
general applicability, like the license
plate regulation. Since Wooley is con-
trolling, B is correct and A is incorrect.
Since no facts whatsoever indicate an
unlawful search or seizure, choice C
is obviously incorrect. Since no facts
indicate Ted's association rights were
in any way infringed, D is obviously
incorrect.

210 CONSTITUTIONAL LAW


63. C is the correct answer. Although none 64. B is the correct answer. Although there
of these answers state the correct rule are certainly some limits as to when a
of law with precision, C is the so-called parent can take a child out of a public
"best of the worst." In West Virginia school for due process Privacy reasons,
Board of Education v. Barnette, 319 when this is based on religious reasons

A
U.S. 624 (1943), it was held that requir- the U.S. Supreme Court has been more
ing a grade school student to salute the tolerant. Thus, in Wisconsin v. Yoder,
flag, when such impinged on his ability 406 U.S. 205 (1972), a parent was al-
not to be forced into supporting or ap- lowed to cease a child's education at 14
O
pearing to support the viewpoint of the even though the state required educa-
government violated his Free Speech tion until the age of 16. Since this case
rights. A is factually incorrect because is directly on point and corresponds to O
this act did infringe on his freedom the facts in this question, B is the correct
of religion and the Court should not answer. Note, Yoder was reconfirmed
be in the position of judging what is by the Court's opinion and Employment
truly contrary to a person's religious Division v. Smith, even after Smith
beliefs. B is incorrect because, in the changed Free Exercise jurisprudence.
case at hand, Jeremiah's beliefs are be- Although the child may "lose out," a
ing interfered with by the state. Thus, principle of our constitutional juris-
family values are being infringed. D is prudence is to not adjudge different
incorrect because adjudging the merits views on how religions conduct their
(or alleged lack thereof) of a religion is family affairs. Thus, A is incorrect. C
antithetical to the Free Exercise Clause. is partially correct and partially incor-
Cf. United States v. Ballard, 322 U.S. rect. The state does have a compelling
78 (1944). Although nothing in the interest in the education of minors and
Constitution explicitly protects "family this interest will allow interference of a
values," C is, nevertheless, the "best" parent in a minor's education but only
answer, since it reaches the correct up to a point. In Yoder, supra, that point
conclusion without badly misstating the was not met. So, essentially, C is legally
law of the Free Speech Clause. correct and factually incorrect. D is
clearly incorrect because public schools
have jurisdiction over their students.

CONSTITUTIONAL LAW 211


65. C is the correct answer. This issue was 66. A is the correct answer. The Free Exer-
directly addressed in Pierce v. Society cise Clause of the First Amendment has
of Sisters, 268 U.S. 510 (1925). Pro- been interpreted to prohibit "meddling"
vided that the private religious school by the state or federal governments in
teaches the basics (reading, writing, intra-church disputes (except those, of
science), the state cannot forbid such course, which regard crimes). Serbian
under Fourteen Amendment Due Pro- Orthodox Diocese v. Milijevich, 426
cess. The Supreme Court used Privacy U.S. 696 (1975). State interference
to decide Pierce (choice I). Choice II, would compromise the Free Exercise
Free Exercise, would also be correct of that religion. Therefore, A is the
here under Yoder. However, there is correct answer. Many religions are not
no claim that this is an establishment democratically run. Were the state to
clause violation, unless the state is try- impose its own notions of election on
ing to make students attend school in the religion, there is a very unrealistic
order to advance or inhibit religious argument that the state is choosing
belief. There are no facts on that point one form of religion over another and,
here. Thus, II is incorrect. Therefore, therefore, an Establishment Clause
C is correct and A, B and D are incor- violation. But B is not a better answer
rect. than A. Although there is a possible
case or controversy here, the Court has
refused due to Free Exercise questions
to have the federal courts hear these
types of cases. C is incorrect. D is also
incorrect.

212 CONSTITUTIONAL LAW


67. D is the correct answer. Although cer- 68. C is the correct answer. As indicated by
tain sexually explicit behavior is con- Barnes, supra answer 67, a community
sidered to be protected by the First and is permitted to prohibit topless dancing
Fourteenth Amendments, as "symbolic at public places by females. A is obvi-
speech"; nevertheless, the states are ously incorrect because were the law is
permitted great latitude in regulating overbroad, then the defendants would
public sexually explicit activity based succeed. B is incorrect because com-
on their own community standards munities are permitted to regulate pub-
— even if the activity is arguably artistic. lic nudity without offending the First
Recently, in Barnes v. Glen Theatre, Amendment. D is incorrect because
501 U.S. 540 (1991), the High Court although there was arguably an artistic
upheld the requirement that women not and, thus, "symbolic speech" aspect to
completely bare themselves in an adult the dancing, it has been held that requir-
club (breast "pasties" were a legitimate ing some clothing is inoffensive to the
requirement). Thus, since the Puritania First Amendment Free Speech Clause.
statute did little more than the regula- Barnes, supra answer 67.
tion in Barnes, it is not unconstitutional
on its face. A is incorrect because it 69. B is the correct answer. Although she
misunderstands the point that symbolic might claim that her actions were sym-
speech is given some First Amendment bolic speech, the question is not what
protection. B is incorrect because sexu- she intends as much as it is why the
ally explicit conduct can be regulated government is prosecuting her. If they
— just as Puritania has done. C is incor- were to prosecute her because of the
rect because the statute is not vague (a message of her conduct, she might have
person of ordinary intelligence would a symbolic speech claim. That is not the
know what was prohibited and the stat- case here. A is incorrect. The elements
ute does not give too much discretion of the crime have been committed. C
to the police). Since A, B and C are is incorrect. The fact that her acts were
incorrect, D is the correct choice. in public is relevant, but not the only
issue under the statute. D is incorrect
because her conduct was arguably lewd
and lascivious.

CONSTITUTIONAL LAW 213


70. D is the correct answer. A correct an- 71. C is the correct answer. This is a tricky
swer here requires interpreting the plain question. Clearly, Darby is not guilty of
meaning of the Puritania statute. Since Puritania's sexual nudity law, because
nudity means a lack of clothing, and she was not nude under the language
since exposure is used in the context of the statute. Thus, she has no need to
of the statute, it is apparent that the avail herself of the First Amendment

A statute does not regulate the wearing


of tight and shear clothing. Such is not
"exposure" under the Puritania statute.
defense. However, the call of the ques-
tion was whether a First Amendment
defense would prevail. The interroga-
O Therefore, D is correct. A is incorrect tory was not whether she was factually
because being guilty of a related crime guilty of the crime. Therefore, C is cor-
most obviously does not make one rect because there was no First Amend-
O guilty of another crime! B is incorrect ment protection to her non-artistic sales
because although Chipmunks was prob- approach. A is incorrect only because
ably quite contrary to Puritania's com- it is irrelevant. The query went not to
munity standards, that, in itself, cannot guilt — but rather to the applicability
be and was not made a crime without a of the First Amendment. B is incorrect
law on the books. C is incorrect because because no facts suggested Darby was
it is irrelevant. Prostitution was not the doing anything artistic while selling
crime which was charged. the hot dogs. D is incorrect not only
because it fails to address the call of
the question (the First Amendment), but
also because she is likely not factually
guilty (since she was not nude).

72. B is the correct answer. Flag-burning, as


political protest, be it of a state or a fed-
eral flag, is considered to be symbolic
speech, and is, thus, First Amendment
protected. United States v. Eichman,
496 U.S. 310 (1990). Therefore, B is
correct. A is incorrect for the same
reason that B is correct; and is also
incorrect because the crime was not
strict liability (it included the mens rea
of "purposely"). C and D are incorrect
for the same reason that B is correct.

214 CONSTITUTIONAL LAW


73. D is the correct answer. In Employment 75. C is the correct answer because only
Division of Oregon v. Smith, 494 U.S. choice II is correct. Pursuant to Article
872 (1990), the U.S. Supreme Court II, §2, the President — not Congress
indicated that it was permissible to — has the power of judicial appoint-
withhold employment benefits from ment. Therefore, the act in question is

A
a Native American who used the con- clearly contrary to the language of the
trolled substance of peyote as sacra- Constitution and is, thus, unconstitu-
ment in his Native American religion. tional. Cf. Buckley v. Valeo, 424 U.S.
His claim was, of course, that his First 1 (1976). For this reason, choice II is
0
Amendment Free Exercise of Religion correct. Choice I, however, is incorrect cn
rights were abrogated. However, the because the language of the Constitu-
Court held that even if a law works to tion (Article II, §7) indicates that such 0
the detriment of one religion, it does appointment must be with the advice
not violate the First Amendment unless and consent of the Senate. The power
that religion is singled out by the state of appointment is, therefore, not solely
as an object of persecution. In dictum, a presidential decision.
the Court also indicated that no compel-
ling interest need be shown — even if 76. B is the correct answer. Article I, §6,
the law works to the detriment of one clause 1 affords all U.S. Senators and
religion-provided the law is neutral on representatives immunity for their leg-
its face. Since the facts of question 73 islative functions. Since on the facts
are directly on point to the Smith case, given Semand was simply debating
Smith controls and the prosecution of whether or not to give a grant, he has
Father O'Scalia will succeed. A and B complete immunity. Gravel v. United
are incorrect for the same reason that States, 408 U.S. 606 (1972). A is not,
D is correct. C is incorrect because Fa- per se, incorrect. Rather, it is simply not
ther 0' Scalia was in no way impeded the best answer. The above-cited clause
from associating with members of his gives far better protection. C is incor-
church. rect for the same reason that B is cor-
rect; and also because no facts indicate
74. D is the correct answer. This is a simple XYZ University to be a public entity.
"black letter" question. Pursuant to D is incorrect for the same reason that
Article I, §7, both houses of Congress B is correct, and also because the facts
can overturn a presidential veto by two- indicate it was not the government, but
thirds vote. Because no U.S. Supreme rather Semand who was being sued.
Court has in any way modified the in-
terpretation of this clause, D is correct
and A, B and C are incorrect.

CONSTITUTIONAL LAW 215


77. D is the correct answer. The Speech and 79. B is the correct answer. The President's
Debate Clause (Article I, §6, clause 1) power to pardon is limited to pardon of
does not protect a legislator from ex- federal crimes. However, the pardon is
traordinary acts, such as taking a bribe. not prospective. Thus, the pardon does
United States v. Brewster, 408 U.S. 501 not cover crimes committed after the
(1972). A is incorrect since it was not effective date of the pardon. Therefore,
the government which was being sued. B is correct. Article II, §2. A is incorrect
B is inane because suits can be brought because Pilson may have committed
even when the allegations are difficult crimes while in office, but after the date
O to prove. C is incorrect since bribe of the pardon. C is incorrect because it
taking is beyond the protections of the is irrelevant whether the pardon-giving
Speech and Debate Clause. Since A, B president is still in office. D is incorrect
O and C are all incorrect; D (none of the because the pardon power is not limited
above) is the best answer. to non-infamous crimes.

78. C is the correct answer. The presidential 80. A is the correct answer. The U.S. Su-
power to pardon is broad. However, it preme Court has original and exclusive
does not include the right to pardon for jurisdiction over foreign ambassadors.
state crimes. The language of Article Article III, §2. Therefore, Oregon does
II, §2 is limited to crimes against the not have jurisdiction. Since it is a far
federal government. Since C states the better defense to prove that the pros-
limitation correctly, it is the correct ecuting court lacks jurisdiction than that
answer. A is constitutionally incorrect. an element of the crime was not met
The pardon power does not attach to (choice B) or that there is an available
state crimes. B is incorrect because affirmative defense (choice C), choice
the power to pardon extends beyond A is superior to choices B and C. Since
political crimes (which, in fact, may be a state, as a general rule, does have
a meaningless term). D is incorrect for jurisdiction over foreigners, choice D
the same reason that C is correct. is incorrect.

216 CONSTITUTIONAL LAW


81. D is the correct answer. This is another 82. C is the correct answer. Although the
question — so typical of the multistate Congressional power to subpoena
format where the answer which one is broad, it is not absolute. Thus, of
looks for is simply not there. Thus, one course, a subpoena cannot compel a
has to choose among a series of answers person to incriminate herself (as per
— none of which are perfect. There the Fifth Amendment self-incrimina-
is no constitutionally-conferred state tion clause). But, and more to point
sovereign immunity. Cf. Fitzpatrick v. here, the subpoena must be relevant. If
Bitzer, 427 U.S. 445 (1979). Thus, A the subpoena is not reasonably-related
O
is incorrect. B is incorrect because the to the investigation, it is beyond the
Eleventh Amendment grants immunity legitimate power of Congress. Gibson
to states for suits against a state in fed- v. Florida Legislative Committee, 372 O
eral court initiated by a citizen of the U.S. 539 (1963). A subpoena which had
state when the citizen seeks damages. required lists of "donors" or perhaps
But it does not grant immunity against lists of "other candidates" might have
suits for injunctive relief or against state been specific enough. However, this
officials in their individual capacity. subpoena was too broad, and, thus,
Fitzpatrick. C is completely incorrect in among other things (such as potentially
that the powers not given to the federal chilling First Amendment Freedom of
government are reserved to the states Association rights), it was irrelevant. A
under the Tenth Amendment. Since the is incorrect because there was neither
Constitution is silent on this point, this Executive nor Judicial function im-
choice is inapplicable. Since A, B and C paired by this subpoena. B is incorrect
are all incorrect, D is the best of a series because there was no state action. D is
of poor choices, since depending on the incorrect for the same reasons that C is
facts of Moss's suit, it may or may not correct.
be dismissed. But it can not be due to
the reasons stated in A, B or C. There-
fore, D is the most correct answer.

CONSTITUTIONAL LAW 217


83. D is the correct answer. Although the 84. D is the correct answer. A state may not
state is likely to lose this case and none be required to permit marriage between
of the answers are particularly good, D persons of the same sex, however once it
is the only one that is really possible has done so, it cannot prohibit marriage
here. A is incorrect because of Law- based on the race of the participants.
rence v. Texas (2003) which overruled Race-based regulations are accept-
Bowers v. Hardwick (1986). Thus, able only when there is a "compelling
state laws on homosexual sodomy are state interest." Race-based regulations
unconstitutional. B is incorrect because are inherently "suspect." On the facts
whether or not there is a constitutional given, it is unconstitutional for the state,
right to marry, that is not the issue on the one hand, to allow homosexual
— the racial classification is. Once the marriage, but on the other, to require
state permits gay people to marry, they such occur only between members of
cannot deny it on racial grounds. C is the same race. A is incorrect because
incorrect because the statement as it the Supreme Court has not held that the
stands is false. The Court has found in constitutional right to privacy includes
a number of cases that there is a right to homosexual marriage. B is incorrect
privacy. Although marriage regulations because the constitutional clause at
are not totally within the state's power, issue is not Due Process, but rather
they are traditionally there. Thus, D is Equal Protection (unequal treatment of
the most correct of these choices. those who are similarly situated). C is
incorrect because the correct analysis,
when race is at issue, is not whether the
state regulation has a rational basis, but,
rather, whether it is necessary to achieve
a compelling state interest.

218 CONSTITUTIONAL LAW


85. C is the correct answer. McLaughlin 87. C is the correct answer. Although
v. Florida, 379 U.S. 184 (1964) and speech with a profit motive is not
Loving v. Virginia, 388 U.S. 1 (1967), given the same degree of protection as
together stand for the proposition that political speech, it is nevertheless First
a state cannot make crimes or marriage Amendment protected. Provided the
depend on the race of the participants ad is truthful and not misleading, the
absent a compelling government in- state can limit the ad only if there is a
terest. Otherwise there is a denial of substantial interest which is narrowly
Equal Protection. Certainly a state is tailored to meet that interest. Virginia
not required to allow homosexual mar- State Board of Pharmacy v. Virginia
riage. However, once they allow it, they Citizens Consumer Council, 425 U.S.
cannot limit it to those of the same race 748 (1976). The outright clothing ad
— without violating the Equal Protection ban of Lemon County is certainly not
Clause. Therefore, the law is settled. narrowly-tailored; no facts indicate the
Just as in McLaughlin, supra, the state ad to be deceptive; and the state-inter-
was not required to allow sexual co- est is not shown. Therefore, the First
habitation — but once the state allowed Amendment offers protection. A is
it, the state cannot punish it differently incorrect because a state's police pow-
because of race. A and B are incorrect ers do not include the infringement of
because they miss the mark. There may First Amendment rights. B is incorrect
be a right to engage in private consen- because no legitimate state interest was
sual homosexual activity under the shown, and because the other prongs of
Due Process clause, Lawrence v. Texas the commercial speech doctrine were
(2003), but that is not relevant to this not met. D is incorrect because no facts
question. indicate a restriction of interstate com-
merce.
86. D is the correct answer because choices
I, II and III are all incorrect. Choice
I is incorrect because it misreads the
facts. The warrant was stipulated to be
"proper." Choice II is incorrect because
it also misreads the facts — the video
was "lawfully intercepted." Choice III
is incorrect because there is no right to
possess child pornography or obscenity
in the home. Osborne v. Ohio, 495 U.S.
103 (1990).

CONSTITUTIONAL LAW 219


88. A is the correct answer. The Supremacy 89. C is the correct answer. As Virginia
Clause of Article IV, §2 indicates that State Board of Pharmacy, supra answer
a ratified treaty is the supreme law of 87, and Bates v. State Bar Of Arizona,
the land and that a state cannot interfere 433 U.S. 350 (1977) indicate; ads by
with such. Nielson v. Johnson, 279 U.S. professionals (be they by pharmacists,
47 (1929). On the facts given, it appears lawyers, or, inferentially, doctors) are

A that Lemon County is directly impeding


the free trade between the foreign coun-
try and the U.S. Therefore, A is correct.
given significant First Amendment pro-
tection, as under the so-called "Com-
mercial Speech Doctrine." The statute
0 B is incorrect because no facts indicate may have some ambiguities in it, but it
that privileges or immunities of U.S. is not so vague that a person of aver-
citizens are being denied. C is incorrect age intelligence would not know how
O because it is so general an answer as to to conform his conduct to the law. If
be meaningless. Even the treaty power, you are a doctor or lawyer, you can not
just alluded to, requires Senate ratifica- advertise. Therefore, C is correct and
tion. D is incorrect because the Tenth A is incorrect. B is incorrect because,
Amendment only allows the states to on the facts, it does appear that certain
regulate in the areas of non-enumerated individuals in Lemon County are pres-
federal powers. Treaties, however, are ently affected. Since the controversy is
a federal power, supra. "ripe." B is incorrect. Since there was
no governmental taking, Just Compen-
sation Clause analysis is irrelevant and
D is therefore incorrect.

220 CONSTITUTIONAL LAW


90. C is the correct answer. Pursuant to 92. A is the correct answer. The procedural
Article I, §8, clause 4, Congress has aspect of due process requires that there
jurisdiction over the states regarding be notice and a hearing prior to the de-
the law of bankruptcy. Thus, since privation of property rights. Attendance
Congress has established bankruptcy at public school has been held to be a
laws (which allow individuals a new property right. Goss v. Lopez, 419 U.S.
beginning), a state cannot contravene 565 (1975). Therefore, there was a de-
those laws by forever penalizing an nial of due process in the case at hand
individual for a bankruptcy. Therefore, — because of the lack of a hearing. B is
0
C is correct. Choices A and B are also incorrect because although flag burning, cn
correct answers, but they are not as pre- in political protest, is a right — there are
cise as C. Moreover, state fact-findings certain limitations placed on that right. 0
are given great deference on federal ap- Obviously, it must be your own flag (or
peal. Thus, attacking Barrier's factual there is malicious mischief) and the flag
conclusions of a lack of moral fitness must be burned pursuant to reasonable
and insufficient "rehabilitation" are far time, place and manner restrictions. C
more difficult hurdles than attacking the is incorrect because the specificity of
law which they utilized to reach those a political protest is not relevant to the
conclusions. D is incorrect for all of the issue. D is incorrect because it is non-
above reasons. sense. The reference to shouting "Fire!"
in a crowded theatre is from Justice
91. C is the correct answer. Although there Holmes and he said that the government
is some variance in the use of these may punish sommeone for a FALSE
admittedly facially ambiguous terms, shout of "Fire!" in a crowded theatre,
the two aspects of due process are: 1) meaning that there were restrictions on
fair procedures before life, liberty or speech permitted to the government for
property is denied (procedural); and 2) the greater public good.
substantive rights without which society
would not be free (substantive). Since
choice C best approximates the above
definition, it is correct. The distinction
made in choice A has not been utilized
by the High Court; thus, it is incorrect.
Choices B and D are incorrect for the
same reason that choice C is correct.

CONSTITUTIONAL LAW 221


93. A is correct because only choices I and 95. D is the correct answer. Although not
II are true. If he was punished because offered as a choice, the applicable
the flag he burnt was not his own, that constitutional clause is Fourteenth
is not punishment for speech purposes Amendment Equal Protection. Ille-
and tested at a lower level. Similarly, if gitimacy has been deemed to require
the issue is time, place and manner re- intermediate scrutiny when a state

A strictions, they too are for non-speech-


related purposes and teested at a lower
standard. Statements III and IV are ac-
regulation adversely affects such status.
Moreover, Levy v. Louisiana, 391 U.S.
68 (1968), held a highly similar state
O curate as far as they go, but they do not statute to be unconstitutional on these
help resolve this problem, since those grounds. Choices A, B and C reach the
cn issues are not involved here. Choice III wrong conclusion; so each is obviously
O is incorrect because it is irrelevant. The incorrect. Moreover, A is incorrect be-
issue here is not suspension from school cause there are no Tenth Amendment
— it is criminal prosecution. Choice IV independent state grounds. The state-
is incorrect for the same reasons that applicable Fourteenth Amendment
choice II is correct. Since only letter A Equal Protection Clause is controlling.
includes choices I and II, it is the correct Choices B and C are incorrect, not only
answer. in reaching the wrong conclusion, but
also because they utilize the incorrect
94. D is the correct answer. Since the facts level of scrutiny as required by the
stipulated Junior's new school to be a Equal Protection Clause (which in this
private school, there is no state action; case requires mid-level scrutiny).
and, thus, the Fourteenth Amendment
Due Process Clause is inapplicable.
Therefore choices A, B and C are incor-
rect; and choice D is correct.

222 CONSTITUTIONAL LAW


96. D is the correct answer. Mental and 97. B is the correct answer since choices III
physical impairment is not a suspect and IV are correct. Choice I is incorrect
classification under Equal Protection because there was no governmental tak-
Clause analysis, and, thus, the state ing without compensation; the $25,000
regulation need only meet the rational is a fine, albeit an unconstitutional

A
basis test to be constitutional. City of one. If declared unconstitutional, the
Cleburne v. Cleburne Living Center, money need not be paid. Choice III is
473 U.S. 432 (1985). Although there correct because there was a legislative
is certainly a good argument that pronouncement meting out punish-
the non-hiring of Hal was wrong; it ment in the absence of a judicial trial,
is, nevertheless, unprotected by the contrary to Article I, § 10, clause 1 (the
Equal Protection Clause. Since it was Bill of Attainder Clause). Choice IV is
definitely implied that some individu- correct because this was punishment
als like Hal could not perform the work for a crime which did not exist at the
and since it is common knowledge that time of its alleged commission. Such is
nuclear plants are ultra-hazardous; it is prohibited by the Ex Post Facto Clause
quite clear that the state would meet (Article I, §10, clause 1). Choice II is
the minimal requirements of proving incorrect because the Fifth Amendment
a rational basis for their legislation. Due Process Clause is not applicable to
Therefore, D is the correct answer. A state citizens.
is incorrect because it ignores the ap-
plicable legal standard of analysis. B is
incorrect because it uses an incorrect
level of scrutiny (compelling interest).
C is incorrect because it misconstrues
the facts.

CONSTITUTIONAL LAW 223


98. D is the correct answer. Although, 99. B is the correct answer. Dawson v.
before Article I, §9, clause 8 is quite Delaware, 112 S.Ct. 1093 (1992), set
explicit: "No Title of Nobility shall be out a similar fact pattern. It was there
granted by the United States", since this held that such a reference violated the
is just what President George did, it is defendant's First Amendment Freedom
clearly unconstitutional. However, B is of Association rights. Association was
incorrect. The question asked whether at issue because the defendant was be-
Rader suit will succeed. Even if a law ing punished not for his criminal act, but
is unconstitutional, it can not be chal- rather for the company he kept. There-
lenged by someone without standing fore B is correct. A is incorrect because
to sue. Rader 's claim is only that the it is a now long-standing principle of
President has acted unconstitutionally. constitutional criminal procedure that
The Court has stated in Allen v. Wright capital punishment is not per se Cruel
that such a claim is non-justiciable. and Unusual. C is not the best answer
Thus, answer D is correct. Moreover, because no facts indicate statements
that it has no significance is arguable. or symbolic speech of the defendant
It could foreshadow the beginning of a was used against him. It is also not the
monarchy — and even given that such best answer, because such is not how
is insignificant, it is nevertheless un- the Court reasoned in Dawson, supra.
constitutional, but still irrelevant to the That is, there is High Court precedent
question. Therefore A is incorrect. C is directly on point. D is incorrect for the
incorrect because although it probably same reasons that B is correct.
is analogous to a Presidential Medal, it
is nonetheless expressly prohibited.

224 CONSTITUTIONAL LAW


100. D is the correct answer. Since it was 101. B is the correct answer. Under the
stipulated that both Rawson and the case of New York Times v. Sullivan,
victim were white and that Rawson's 376 U.S. 254 (1964), defamation suits
group advocated no related (or unre- are limited by the First Amendment
lated) crime, the statement of his mem- — particularly when the allegedly
bership is irrelevant. It infringed on defamatory remarks concern a public
iiiiitng LI uin-iinuing. IL vJ aS
highly prejudicial, without being at all also be made knowingly or in reckless
probative. Dawson, supra answer 99. disregard of their falsity. Since Jones
Therefore, D is correct. A is incorrect is a public figure, for there to be defa-
because it is totally irrelevant to the mation there must be knowledge or
given facts of this question. No facts recklessness (i.e., actual malice). Since
whatsoever indicated any prosecuto- only choice B goes to this requirement,
rial statements regarding impact to though incompletely stated, it is the
the victim or his family. B is incorrect best answer. A is incorrect because the
because no facts were given which press often damages a person — but that
would enable the student to discern is the side effect of having free speech.
whether there was overwhelming other C is incorrect because the right to pri-
evidence of guilt. Thus, although the vacy essentially is only a limitation on
error could have been harmless, such governmental conduct. D is incorrect
an analysis is extraneous to this ques- because it does not address any of the
tion's fact pattern. C is incorrect for elements of the New York Times v.
the same reasons that D is correct. Sullivan test.

END OF ANSWERS

CONSTITUTIONAL LAW 225


226 CONSTITUTIONAL LAW
CRIMINAL PROCEDURE - QUESTION BREAKDOWN

1. 4th Amendment - Expectation of 17. 5th Amendment - Coercive


Privacy Interrogation

2. Procedural Due Process 18. 8th Amendment - Harmless Error

3. 4th Amendment - Civil Action 19. 6th Amendment - Right to Counsel

4. 4th Amendment - Exception of 20. 6th Amendment - Right to Appeal


Consent
21. 4th Amendment - Plain View
5. 4th Amendment - Traffic Stop
22. 4th Amendment - Search
6. Search - Exception of Consent
23. 4th Amendment - Warrant
7. 4th Amendment - Exception Requirements
Automobile
24. 4th Amendment - Warrant
8. 5th Amendment Rights Requirements

9. 6th Amendment - Right to Counsel 25. 4th Amendment - Exclusionary Rule

10. Victim Impact Statements 26. Fruit of the Poisonous Tree

11. Cruel and Unusual Punishment 27. Exigent Circumstances

12. Cruel and Unusual Punishment 28. 4th Amendment - Search

13. 4th Amendment - Rights to Privacy 29. 4th Amendment - Search

14. 4th Amendment - Illegally Seized 30. 4th Amendment - Standing


Property
31. 4th Amendment - Standing
15. Cruel and Unusual Punishment
32. 4th Amendment - Administrative
16. 6th Amendment - Right to Counsel Exception

CRIMINAL PROCEDURE 229


33. 6th Amendment - Right to Counsel 52. 4th Amendment - Expectation of
Privacy
34. 6th Amendment - Right to Jury Trial
53. 4th Amendment - Search Warrant
35. 6th Amendment - Right to Counsel
54. 4th Amendment - Search
36. 5th Amendment - Double Jeopardy
55. 4th Amendment - Right to Privacy
37. 4th Amendment - Search
56. 4th Amendment Search - Reasonable
38. Exclusionary Rule - Inevitable Suspicion
Discovery
57. 4th Amendment Search - Reasonable
39. 4th Amendment - Detention of Others Suspicion

40. Warrant - Probable Cause 58. 4th Amendment Search - Expectation


of Privacy
41. Administrative Searches
59. 4th Amendment Search - Expectation
42. Harmless Error of Privacy

43. Miranda Rights 60. 4th Amendment - Warrant


Requirements
44. Out-of-Court Identification
61. Warrant Requirements
45. 6th Amendment - Right to Counsel
62. Warrant Requirements
46. 6th Amendment - Right to Counsel
63. Warrant Exceptions - Plain View
47. Competency of Defendants
64. 4th Amendment Search - Expectation
48. 6th Amendment - Confrontation of Privacy
Clause
65. 4th Amendment Search - Expectation
49. Cruel and Unusual Punishment of Privacy

50. Habeaus Corpus 66. 4th Amendment - Invasion of Privacy

51. 4th Amendment - Expectation of 67. 4th Amendment Search - Expectation


Privacy of Privacy

230 CRIMINAL PROCEDURE


68. 4th Amendment Search -Expectation 84. Warrant Execution
of Privacy
85. Warrant Execution
69. 4th Amendment Search -Expectation
of Privacy 86. 4th Amendment Search - Incident to
Arrest
70. 4th Amendment Search and Seizure
87. Search Incident to Arrest
71. 4th Amendment Search - Expectation
of Privacy 88. Plain View

72. 4th Amendment - Government Action 89. 4th Amendment Search

73. Warrant Exception - Consent 90. Public Safety

74. Expectation of Privacy - Hotel 91. 4th Amendment Search


Rooms
92. Reasonable Suspicion
75. Warrant Expectation - Consent
93. Seizure
76. 4th Amendment Search -
Probationers 94. 5th Amendment - Confession

77. 4th Amendment Regulatory Search 95. 5th Amendment - Interrogation

78. 4th Amendment Search - Illegal 96. Custodial Interrogation


Entry
97. Custodial Interrogation
79. 4th Amendment Search - Consent
98. Due Process
80. Concealment
99. Habeas Corpus
81. 4th Amendment Search - Voluntary
Consent 100. Pleas

82. Validity of Warrant

83. 4th Amendment Search - Mistaken


Identity

CRIMINAL PROCEDURE 231


232 CRIMINAL PROCEDURE
Q
CRIMINAL PROCEDURE Questions 2 — 3 are based on the
following facts:
QUESTIONS Marie is arrested on a charge of solicita-
tion to commit prostitution. Her arrest is
1. Mick is on a rough corner waiting for a performed by an undercover officer. She is
bus. Officer Stone sees Mick and since brought to jail. Three days later, still in jail,
he has nothing else to do, decides to she appears before a judge who refuses to let
investigate Mick. Stone walks up to her free on bail. Marie files a writ to the state
Mick and says: "Hold it right there, supreme court, demanding release.
mister." Mick takes flight and throws
away a marijuana joint which was 2. Her worst argument is:
on his person. Mick is prosecuted for
illegal marijuana possession. Mick's A. Her Eighth Amendment right to
attorney files a motion to suppress the bail was denied.
marijuana. This motion will:
B. She was denied due process.
I. Fail, because Mick has
no standing. C. There was impermissible delay
in appearing before a judge.
II. Fail, because Mick aban-
doned the property. D. She was entrapped.

III. Fail, because Mick was 3. Assuming that the state supreme court
not "seized." orders Marie be set free, she may have
a civil claim against the arresting of-
A. I only. ficer.

B. II and III only. A. True, provided he was in bad


faith.
C. I, II and III.
B. True, regardless of his bad
D. None of the above. faith.

C. False.

D. None of the above.

CRIMINAL PROCEDURE 233


Q
4. Since illegal drugs are often transported Questions 5 — 7 are based on the
by transients taking Blackhound buses, following facts:
Officer Grey decides to investigate the
Blackhound station in his precinct. He Acid is driving his car at 70 m.p.h. in a 55
obtains consent of the local bus station m.p.h. zone. He is pulled over by Officer
manager to enter buses and request if Quist. Quist asks Acid if he can search the
luggage can be searched. Grey enters a passenger compartment of Acid's car. Acid
bus at 11:00 p.m., before it is to leave, says, "Yes." Quist finds a closed bag in the
then announces the following to the back seat and in it is LSD (contraband). Acid
seven people in the bus: "Hi, my name says, "I didn't mean to allow you to get into
is Jack Grey. May I search each of that!" Quist then opens Acid's trunk and
your bags?" One by one, Grey walks to finds a closed suitcase. Quist opens it and
each passenger seat and asks for their finds $10,000 worth of LSD.
bag. Passenger Dawn hands over her
purse, which has cocaine in it. At trial 5. When Quist pulled Acid over, there
for Dawn's cocaine possession, is the was a:
cocaine admissible against Dawn?
A. Detention.
A. No, because there was no warrant
to search. B. De facto arrest.

B. No, because there was no exi- C. Illegal detention.


gency.
D. Legal traffic stop.
C. Yes, because of the automobile
exception. 6. Quist's opening of the closed bag in the
passenger compartment was:
D. Yes, because of the consent ex-
ception. A. Beyond the scope of the consent
given.

B. Within the scope of the consent


given.

C. A lawful search incident to ar-


rest.

D. A lawful search under the drug


exception to the warrant require-
ment.

234 CRIMINAL PROCEDURE


Q
-`1 0
73
7. Quist's search of the suitcase was: 9. Yolanda is represented by a retained Om
lawyer on a charge of manslaughter. PIE
A. Lawful under the auto excep- She is found guilty. She tells her at- oz
c >
tion. torney Rex, to appeal. Rex misses the 73
mi-
deadline. There is overwhelming evi-
B. Lawful under the search incident dence, however, that Yolanda is guilty.
to arrest exception. Yolanda seeks collateral relief. She
will:
C. Lawful under the exigency ex-
ception. A. Prevail, because Rex was inef-
fective.
D. Lawful under the drug excep-
tion. B. Fail, because there was no preju-
dice.
8. Nikki is charged with forgery and
requests a lawyer to represent her at C. Fail, because Rex was privately
trial. After the formal charging, she retained.
is released on bail. Officer Bix calls
Nikki on the phone and asks her about D. Fail, because collateral appeal is
an unrelated theft. Nikki admits to the not allowed.
theft. The admission is:
10. Prosecutor Chill, during the capital sen-
A. Admissible, because it was given tencing phase of Pip's trial, introduces
over the phone. evidence of the effect of Pip's killing
of Victim Vinny on Vinny's children.
B. Admissible, because it was vol- Such statement:
untary.
A. Violates due process.
C. Admissible, because her right to
counsel did not apply. B. Violates the cruel and unusual
punishment clause.
D. None of the above.
C. Violates the hearsay rule.

D. Is permissible.

CRIMINAL PROCEDURE 235


Q
Questions 11 —13 are based on the B. Her reasonable expectation of
following facts: privacy was abrogated.

Dixona is in jail serving three years for C. Her legitimate expectation of


grand theft. The jail is over-crowded and privacy was invaded.
as a result, Dixona is occasionally sexually
molested by other female prisoners. Dixona D. She was subjected to cruel and
files a writ of habeas corpus arguing cruel unusual punishment.
and unusual punishment.
13. Assume for this question only, that dur-
11. In order for the writ to succeed, the fol- ing a strip and body-cavity search by a
lowing must be proven: male guard, illegal contraband cocaine
is discovered. Dixona is prosecuted for
I. The sexual molestation such. Her worst argument is:
and over-crowding was
"cruel." A. Her Fourth Amendment rights
were violated.
II. The sexual molestation
and overcrowding was B. Her Due Process rights were
"unusual." violated.

III. The warden was "delib- C. Her Equal Protection rights were
erately indifferent" to violated.
Dixona's conditions.
D. Her Eighth Amendment rights
A. I and II only. were violated.

B. III only.

C. I, II and III.

D. None of the above.

12. Assume for this question only, that


Dixona is routinely stripped and body-
cavity searched by a male guard. Her
best argument via writ of habeas corpus
is:

A. Her Fourth Amendment rights


were denied.

236 CRIMINAL PROCEDURE


Q
73o
Questions 14 — 16 are based on the A. The trial was a farce and mock- -

°RI
following facts: ery.

Wilber Mercer is out of work and decides B. The judge was biased. cy
73
m
to make a "few bucks, quick." In order to
do such, he arranges to sell cocaine. He is C. Counsel was ineffective.
caught by the police with cocaine valued
at $300,000. Wilber has never before been D. The sentence was unconstitu-
convicted of a crime. At trial, Wilber takes tional.
the stand in his own defense and the co-
caine, which was illegally seized, is used
to impeach him. Wilber is convicted and
sentenced to life pursuant to the legislature's
guidelines. This was Wilber's first criminal
conviction.

14. The use of the cocaine to impeach is:

A. Harmless error.

B. Proper.

C. Improper.

D. Harmful error.

15. Wilbur's life-sentence was:

A. Cruel and Unusual Punishment.

B. Violative of the principle of len-


ity.

C. Proper.

D. Violative of Due Process.

16. On the facts given, Wilbur's best appel-


late argument is:

CRIMINAL PROCEDURE 237


Questions 17 — 20 are based on the A. Per se violates the right to coun-
following facts: sel.

Minko works for the police. He believes B. Per se violates the Equal Protec-
Russ to have raped Leigh. Minko has little tion Clause.
evidence. Frustrated, he finds Russ in a bar
and says, "If you don't admit you assaulted C. May violate the Equal Protection
Leigh, you may find yourself bleeding in a Clause.
dark alley very soon." Russ admits to the
rape. At trial, evidence of Russ' confes- D. May violate the right to coun-
sion is admitted without objection. Russ is sel.
convicted. Russ loses on direct appeal and
appeals collaterally. 20. In order to argue on collateral appeal
that admission of the confession was
17. Minko's interrogation of Russ can best erroneous, Russ must show:
be characterized as:
I. Cause and Prejudice.
A. Necessary under the public
safety exception. II. He did not rape Leigh.

B. Coercive. A. I only.

C. Voluntary. B. II only.

D. None of the above. C. I and II.

18. Introduction of Russ' confession at D. I or II.


trial:

A. Can be harmless error.

B. Is per se not harmless error.

C. Violates Equal Protection.

D. Is necessary under the confronta-


tion clause.

19. Counsel's failure to object to the admis-


sion of the confession:

238 CRIMINAL PROCEDURE



Q
Questions 21 — 25 are based on the A. Defective because it lacked a
signature.
following facts:
Officer J.G. West, while on routine patrol, B. Defective because it was based
takes a walk onto the property of Belinda. on illegally obtained evidence.
J.G. looks into Belinda's barn with his flash-
light and sees several machine guns. J.G. C. Not defective because warrants
then checks Belinda's dumpster. In it are do not need judicial approval.
expended hand grenades. Based on these two
pieces of evidence, J.G. obtains a warrant to D. Not defective because of the
search Belinda's barn, home, and office for good faith exception.
illegal ammunition. The warrant fails to in-
clude the magistrate's signature. Magistrate 24. If J.G. executes the warrant and finds
Louis had meant to sign it, but because of indicia of the illegal ammunition, such
having three martinis at lunch, he forgot. is:

21. J.G.'s initial discovery of the machine A. Admissible, if the warrant is


guns was lawful because: deemed valid.

A. Of the exigency exception. B. Admissible, if J.G. was in good


faith.
B. Of the good faith exception.
C. Admissible, if a statute so al-
C. Of the administrative excep- lows.
tion.
D. Inadmissible.
D. This was not a search.
25. Assume for this question only, that, be-
22. The discovery of the expended hand fore executing the warrant, J.G. notices
grenades was lawful because: there was no signature on the warrant
and his training indicates such to invali-
A. Of the exigency exception. date the warrant. In this instance, J.G. 's
discovery of the ammunition is:
B. Of the good faith exception.
A. Admissible, because of the good
C. Of the administrative excep- faith exception.
tion.
B. Admissible, because it inevitably
D. This was not a search. would have been discovered.

23. The warrant was:

CRIMINAL PROCEDURE 239


Q
C. Admissible, because there was Questions 27 — 31 are based on the
n independent source.
a following facts:

D. Inadmissible. Officer Slobog enters Sarah's house after


hearing screams. When he enters, Sarah says
"I was just fooling around with my boyfriend
26. Officer Browson tows Jill's car to the
Charles. Everything's okay." Sarah is flushed
local impoundment lot because of a
and wearing a "kinky" outfit, so Slobog be-
good faith belief that she was illegally
lieves her. On the way out, Slobog notices
parked. A routine inventory reveals
a brass case. His expertise indicates such
marijuana in the car. In fact, Jill was
cases can be used to hold cocaine. Slobog
legally parked. Jill is prosecuted for the
misdemeanor of marijuana possession. opens the case without consent and finds
cocaine in it.
Jill's motion to suppress will:

A. Succeed, because of the fruit of 27. Slobog's initial entry was:
the poisonous tree doctrine.
A. Lawful, because of his good
faith.
B. Fail, because this was a misde-
meanor.
B. Lawful, because of the exigency
exception.
C. Fail, because of the inventory
exception.
C. Lawful, under a doctrine of as-
sumption of the risk
D. Fail, because of the good faith
exception.
D. Not lawful.

28. Slobog's initial observation of the brass


case was:

A. Lawful, because of the exigency


exception.

B. Lawful, because of the doctrine


of assumption of the risk.

C. Lawful, because of the plain


view exception.

D. Lawful, because it was not a


search.

240 CRIMINAL PROCEDURE


Q
29. If Sarah is prosecuted for cocaine pos- A. Succeed, because he lacks stand-
session, she will likely be: ing.

A. Acquitted, because the cocaine B. Succeed, because he has "un-


was in her house. clean hands."

B. Acquitted, because the cocaine C. Fail, because he has standing.


was illegally discovered.
D. Fail, because of the LaFave doc-
C. Convicted, because of Slobog's trine.
good faith.
32. The City of Harzardville has enacted
D. Convicted, because the cocaine a policy whereby all field officers are
would inevitably have been dis- subjected to random drug testing. The
covered. City has made this policy because of the
widespread drug problem. If an officer
30. Assume for this question only, that is found to be using drugs, rehabilitation
the cocaine belongs to Wayne, Sarah's is offered unless other factors indicate
live-in boyfriend. Wayne will be: significant involvement with organized
crime. The policy is likely:
A. Convicted, because he lacks
standing. A. Lawful, under the drug exception
to the warrant requirement.
B. Convicted, because of Slobog's
good faith. B . Lawful, under the administrative
exception.
C. Acquitted, because he has stand-
ing. C. Lawful, under the exigency ex-
ception.
D. Acquitted, because of the LaFave
doctrine. D. Lawful, because examination of
breath, blood and urine are not
31. Assume for this question only, that the searches.
cocaine was Wayne's and that Wayne
was invited by Sarah to her apartment to
have sex for hire. Assume as well, that
this was Wayne's first visit to Sarah's
apartment. Wayne's prosecution for
cocaine possession will:

CRIMINAL PROCEDURE 241


Q
Questions 33 — 35 are based on the A. Violates the Double Jeopardy
following facts: Clause.

Kim is tried on a charge of prostitution. The B. Violates the Cruel and Unusual
potential jail term is 6 months. She requests Punishment Clause.
both counsel because she is indigent, and a
jury trial. Both requests are denied. She is C. Violates the Right to Counsel
tried and sentenced to one month in jail. Clause.

33. The judge's denial of counsel was: D. Is proper.

A. Not erroneous, because this was 36. Mary and Joe agree to commit a bank
a petty offense. robbery. Upon entering the First City
Bank, they draw their guns and flee
B. Not erroneous, because the of- with $7,000 of cash. Two days later,
fense was punishable by under they are captured. They are prosecuted
6 months jail. in the state for Conspiracy and Robbery.
Assuming that the prosecutor proves
C. Not erroneous, because Kim was each element of each crime beyond a
not indigent. reasonable doubt, will their convictions
of both crimes stand?
D. Erroneous.
A. Yes, because of the separate
34. The judge's denial of a jury trial was: sovereign limitation on Double
Jeopardy protection.
A. Plain error.
B. Yes, because Conspiracy and
B. Harmless error. Robbery are distinct offenses un-
der Double Jeopardy analysis.
C. Correct.
C. No, because each offense does
D. More facts are needed. not require proof of an additional
fact.
35. Assume for this question only, that two
years later Kim is picked up, prosecuted D. No, because Robbery is a lesser
on another charge of prostitution, given included offense of Conspiracy
counsel, and sentenced to one year in to commit Robbery.
jail as a recidivist. Such sentencing:

242 CRIMINAL PROCEDURE


Q
37. Officer Breeze shoots shoplifting 38. Officer Knuckles takes out his brass
suspect Wing, after Wing allegedly knuckles and beats Lou to a pulp. Final-
stole a set of Star Trek videos from the ly, Lou admits to having a half-pound
Videobliss Store. Breeze shot low and of marijuana. Unknown to Knuckles,
accurately. Thus, Wing's bullet injury to Officer Smart was on the verge of link-
his left leg healed completely. However, ing Lou to the drug trade in Smalltown.
the shot completely immobilized Wing Lou is prosecuted for Possession of a
who, prior to the shooting, was 15 yards Controlled Substance. Knuckles' testi-
away from Breeze. All the evidence in- mony and the half-pound of marijuana
dicates that Wing would have escaped, found by Knuckles are introduced into
but for Breeze's shooting. Wing is evidence. Lou's attorney makes all ap-
convicted of Petty Theft and Resisting propriate motions to suppress. These are
a Police Officer in Performance of his denied. Lou is convicted, and appeals
Lawful Duties. Wing appeals. Which within the statutory time limit. The
of the following propositions are cor- prosecution's best argument, regarding
rect? sustaining the conviction, is:

I. The conviction of Petty A. The Good Faith Exception.


Theft will be affirmed.
B. There was an independent
II. The conviction of Resist- source.
ing a Police Officer will
be affirmed. C. This was a state case.

III. The conviction of Pet- D. Inevitable discovery.


ty Theft will be over-
turned.

IV. The conviction of resist-


ing a Police Officer will
be overturned.

A. I and II only.

B. I only.

C. III and IV only.

D. I and IV only.

CRIMINAL PROCEDURE 243


Q
39. A search warrant is issued to search the 40. Officer Frome swears out an affidavit
home of Ebar. The warrant was based before a neutral judge. His information
on probable cause, properly sworn out is based on the confidential tip of a
before a neutral judge and was specific personally-known informant, that Rick
as to the address of Ebar. The warrant is illegally storing toxic waste in his
was executed at 1:00 PM on a Thursday barn at 123 Main Street. The warrant is
afternoon, the day after it was issued. issued. In executing the warrant, Frome
Ebar is detained for 45 minutes, while looks through all large containers in the
his house is searched for illegal am- barn. He finds not only toxic waste, but
munition and guns. At the time, Ebar's also several bales of marijuana. Which
two friends were also detained. After of the following propositions are cor-
the discovery of the illegal weaponry, rect?
Ebar is arrested. The officers also arrest
Ebar's two friends. After the arrest, the I. The search warrant must
two friends are found to be conceal- be based on probable
ing, on their persons, small amounts of cause.
crack cocaine. Which of the following
propositions is correct? II. The test for probable
cause is whether the in-
A. Ebar's detention was unlawful, formant was reliable and
thus the evidence will be sup- had a basis for his knowl-
pressed. edge.

B. The detention of Ebar's two III. The seizure of the mari-


friends was unlawful, thus the juana, though outside the
evidence against Ebar will be scope of the warrant, was
suppressed. probably lawful.

C. The detention of Ebar's two A. I only.


friends was unlawful, thus all
evidence against them will be B. I and II only.
suppressed.
C. I and III only.
D. None of the above.
D. II only.

244 CRIMINAL PROCEDURE


41. Pawnshops, in the city of Tomato, are 42. Over defense objection, an admission
closely regulated. The reason for the of the defendant is introduced. The
regulations are that, such are often used admission was taken after police inter-
to "fence" stolen property, that illegal rogation, in violation of Miranda .The
guns are often sold through pawnshops defendant is convicted. An appeal is
and that the city has received hundreds taken. What must the prosecution show,
of complaints regarding the shops. in order to preserve the conviction?
Among the city ordinances is the re-
quirement that, in order to do business, A. That there was no error.
the pawnshop must keep complete
records of all chattels received, from B. That the error was harmless, by
whom and also keep records of all chat- a preponderance.
tels sold, and to whom such were sold.
Moreover, in applying for a business C. That the error was harmless,
license, they also consent to inspection beyond a reasonable doubt.
of the above by the police.
D. That the error was harmless, by
The Dirtball, a Tomato City pawn- clear and convincing evidence.
shop, is inspected by Officer Lemon.
He finds several violations, including 43. Joe is placed under arrest. He is not
a weapon violation that subjects the told what crime he is arrested for. (The
owner of Dirtball to criminal liability. crime is extortion.) After being given
Will a motion to suppress this evidence the Miranda warnings, he says "I don't
succeed? want to talk." Two hours later, after the
routine booking, another officer interro-
A. No, because of the good faith gates Joe. Joe is first given Miranda. He
exception to the warrant require- waives the warnings, then starts talking.
ment. Ultimately, after 45 minutes, Joe makes
a highly incriminating statement. Is the
B. No, because of the administrative statement admissible, in a subsequent
exception. criminal trial, for extortion?

C. Yes, because a warrant was re- A. No, because extortion is not a


quired. drug offense.

D. Yes, because of Lemon's bad B. No, because the police initiated


faith. the subsequent interrogation.

C. No, because it was, per se, invol-


untary.

CRIMINAL PROCEDURE 245


D. None of the above. In order to help bolster their case, the
prosecution sends out a special inves-
44. Suzanne is arrested, based on probable tigator, to infiltrate Frank's circle of
cause, that she committed Assault. It friends, and thereby to hopefully learn
is stipulated that there was probable inculpatory facts regarding the rack-
cause. The arrest is made in public. eteering charge. The investigator learns
Assault is graded as a misdemeanor in several pieces of information, some
this jurisdiction. The alleged victim of from the active questioning of Frank,
the assault, within 5 minutes of same, is that are relevant to the racketeering
asked whether Suzanne is the perpetra- charge; and some of which are relevant
tor. She responds: "She is definitely the to other crimes of Frank (for which he
one!" Is this out-of-court identification has not been charged). Which of the
admissible? following propositions is the most ac-
curate?
A. Yes, because the Sixth Amend-
ment Right to Counsel had not A. The use of the investigator was
attached. unconstitutional.

B. No, because the Sixth Amend- B. No information found by the


ment Right to Counsel had at- investigator via dialogue with
tached. Frank is admissible.

C. No, because this was misleading C. Only that information discovered


and overly suggestive. by the investigator, regarding
crimes unrelated to the racketeer-
D. No, because Due Process was ing, is admissible.
denied.
D. All information discovered by
45. Frank is brought before a federal grand the investigator is admissible.
jury. He is formally charged with a rack-
eteering offense. He invokes his right to
counsel, by hiring a top-notch criminal
defense attorney. Frank is required to
post bail, and then is released.

246 CRIMINAL PROCEDURE


Q
46. Jennifer is charged with Petty Theft. 47. Hilton has gone on a wild killing spree,
Petty Theft, for first-time offenders, evidently initiated by the romantic
cannot exceed 6 months in jail. Jennifer rejection of a woman he loved. Hilton
is a first-time offender. She is 19 years has committed 5 gruesome and bloody
old and cannot afford counsel. Her re- Murders. He openly admits to the kill-
quest to have counsel is denied, on the ing and claims that it was "the devil's
grounds that she will likely not serve hand that led" him. Hilton's attorney,
any jail time even if convicted. In fact, pre-trial, moves that Hilton not be tried
she is convicted. No jail time is given. and be furnished with a psychiatric
Have Jennifer's constitutional rights examination. Under prevailing consti-
been denied? tutional rules, must the trial judge make
such an inquiry?
A. No, because the potential for jail
must be over 6 months, in order A. No, this claim should be heard
for the Right to Counsel to at- on appeal.
tach.
B. Yes, because insane defendants
B. Yes, because the conviction cannot be tried.
might potentially be used if she
is charged with another crime. C. No, because no facts indicate
disorientation or inability to
C. No, because no actual jail time participate in his own defense.
was imposed.
D. Yes, because incompetent defen-
D. Yes, because indigents have an dants cannot be tried.
absolute right to the appointment
of counsel at "critical stages", 48. Jill and Joyce are tried on a charge of
such as this. Conspiracy to Distribute Controlled
Substances. They are tried together.
Joyce takes the stand and argues that
Jill used duress in order to obtain her
participation. Jill and Joyce are repre-
sented by different defense attorneys.
Jill's attorney begins her cross-exami-
nation of Joyce. Joyce's attorney at once
objects, arguing that, if such proceeds,
Joyce's Self-incrimination rights will be
abrogated. How should the trial judge
rule?

CRIMINAL PROCEDURE 247


A. For Joyce, because she cannot 50. Karen is arrested on a charge of Murder.
be compelled to incriminate The arresting officer interrogates her,
herself. without giving the Miranda warnings.
Karen's attorney does not move to
B. For Jill, because she has the right suppress the admissions. Karen is ul-
to confront adverse witnesses. timately convicted. She files no appeal
to the state appellate and state supreme
C. A mistrial should be declared courts. Two years later, in jail, she files a
because two constitutional rights Habeas Corpus petition, to the state su-
are in direct opposition. preme court. It is summarily dismissed.
Karen then files a Habeas petition to the
D. None of the above. federal district court, arguing that her
Fifth Amendment, Fourteenth Amend-
49. The state of Columbia has, by legis- ment and Miranda rights were denied.
lative enactment, deemed all those Will this petition be successful?
convicted of capital crimes and duly
sentenced to death, to be executed by A. Yes, because there was plain er-
dismemberment. Defendant Horace ror.
is so sentenced. Which of the follow-
ing are relevant inquiries regarding B . Yes, because there was a stipu-
Horace's challenge to the method of lated violation of Miranda.
execution?
C. Yes, because her counsel was
I. Whether other states ineffective.
have similar statutes.
D. No, because there was inad-
II. Whether such is more equate cause for not moving to
painful than other meth- suppress the admissions at the
ods of execution. pre-trial, trial phase, or direct
appeal phase.
A. I only.
51. Ms. Apple, a teacher at Loon Bin Public
B. II only. High School, discovered a female stu-
dent, Aire Head, smoking in the girl's
C. I and II. bathroom. Since she was in violation of
school rules, Ms. Apple took Aire Head
D. Neither I nor II. to the Principal's office, where she met
with Mr. Locke, the vice-Principal, and
denied that she had been smoking.

248 CRIMINAL PROCEDURE


Locke led Aire Head into his private 52. Mr. Lifer, a prisoner in State Peniten-
office and demanded to see her purse. tiary, was sound asleep in his prison
He opened the purse, found cigarettes cell, dreaming of life on the outside,
inside and accused Aire Head of lying. before he killed his fiancé. His dream
While reaching into the purse, Locke was interrupted by Mr. Guard, a prison
noticed a small glass vile, containing a guard at "State Penn." Guard snuck
white powdery substance. Thinking it into Lifer's prison cell and proceeded
was likely cocaine, he further searched to search through Lifer's belongings.
the purse and found three more vials Guard woke Lifer up when he found
full of the same white substance, a glass three small balloons of heroin stuck un-
straw and a mirror. Aire Head admitted derneath Lifer's bed. Lifer denied any
to having a drug problem. knowledge of the heroin. Guard notified
the prison's warden. Lifer was sent to
Locke notified Aire Head's mother and solitary confinement for three weeks. At
turned the purse over to the police. The trial, Lifer, representing himself, files a
white powdery substance proves to be motion to suppress the heroin. Are the
cocaine. Aire Head was prosecuted for balloons of heroin admissible against
illegal possession of cocaine with the Lifer?
intent to sell. Aire Head's attorney files
a motion to suppress the cocaine and A. No, because Guard had no war-
cocaine paraphernalia. This motion will rant to conduct such a search.
fail because:
B. No, because there was no exigent
A. Aire Head has no standing. circumstances.

B. Locke's search did not violate the C. Yes, if Guard was acting on a tip
Fourth Amendment. from a reliable informant.

C. The search was conducted by a D. Yes, because Lifer is a prisoner
school official. in the State Penitentiary.

D. Locke's search, resulting in the


discovery of the evidence of
cocaine, was "reasonable."

CRIMINAL PROCEDURE 249


Q W
Questions 53 — 54 are based on the B. No, because the circumstances
Z
following facts: did not dictate that the U.S.
0 Marshal needed to secure a war-
E Mac, a driver for Fed-Excess, was arrested, rant.
without warrant, at his place of employment,
while waiting for his delivery truck to be C. Yes, because the U.S. Marshal
filled with packages. Other police officers acted without sanction of law.
were simultaneously searching Mac's home,
having been let in by a neighbor who knew D. Yes, because Mac made a sea-
where the key was kept. Inside, the officers sonal application for the return
found and confiscated various incriminating of his property.
papers and articles, which they afterwards
turned over to the United States Marshal. 54. Assuming the same facts in question 53,
should the government be permitted to
Later that evening, officers returned to use Mac's property at trial?
Mac's home with the Marshal, who thought
additional evidence could be seized. They A. Yes, because the U.S. Marshal
were let in by Mac's roommate, searched was merely trying to bring fur-
the premises once more and carried away ther proof to the aid of the gov-
incriminating letters and envelopes found in ernment and was acting under
a drawer. Mac was charged with use of the the color of his office.
mails for the purpose of transporting certain
coupons or tickets representing chances or B. Yes, because the circumstances
shares in a lottery or gift enterprise. Mac did not dictate that the U.S.
urged the court for the return of his property Marshal needed to secure a war-
on the grounds that the papers had been rant.
obtained without a search warrant and by
breaking into his home, in violation of the C. No, because to permit the use of
Fourth and Fifth Amendments. Mac's property upon trial would
result in prejudicial error.
53. Should the court grant Mac's request
for the return of his property? D. No, because the U.S. Marshal
acted without sanction of law.
A. No, because the U.S. Marshal
was merely trying to bring fur-
ther proof to the aid of the Gov-
ernment and was acting under
the color of his office.

250 CRIMINAL PROCEDURE


Q
55. Ms. Riskay was disturbed from her When the officers were inside, Riskay's
nightly TV watching by two Los An- attorney arrived, but the officers did not
geles police officers who knocked on allow him to enter the house, nor speak
her door and announced that they had to Riskay. Riskay demanded to see a
information "a large amount of child search warrant, none was produced
pornography" was in Riskay's home. and the officers continued to search
The officers knocked on the front door Riskay's residence. Irritated with Ris-
of the home and demanded entrance. kay's repeated pleading to see a search
Riskay telephoned her attorney, was warrant, one of the officers forcibly
advised not to let the officers inside handcuffed Riskay. The officers ended
without a search warrant, and refused up searching Riskay's entire residence,
to admit the officers. from bedrooms to bathrooms. Child
pornography was discovered during the
Four hours later, five additional of- search. At trial, the prosecution did not
ficers arrived on the scene, to join the produce a search warrant, nor was their
original officers who had subsequently failure to do so accounted for. Can the
undertook a surveillance of Riskay's child pornography found at Riskay's
house. The officers again sought en- residence be used at trial?
trance and, when Riskay did not come
immediately to the door, the door was A. Yes, because in a prosecution in
forcibly opened so the officers could a State court for a State crime the
gain entrance. Fourteenth Amendment does not
forbid the admission of evidence
obtained by an unreasonable
search and seizure.

B. Yes, because even if the officers'


search was unreasonable, they
were legitimately on the prem-
ises.

C. Yes, because child pornography


is inherently evil.

D. No, because the right to pri-


vacy is enforceable against the
States.

CRIMINAL PROCEDURE 251


Q
Questions 56 — 57 are based upon 57. Assume for the purpose of this question
the following facts: only, that Dinah Gloss' breath was not
excessively minty, nor was she stopped
Dinah Gloss, a high-fashion model, late by officer Crumpky for speeding, but
for a photo session, was speeding, travel- was instead stopped because the rear
ing 50 mph, in a residential district. Officer of her car was very low to the ground.
Crumpky, noticing the excessive speed, Suspecting that something illegal might
pulled Gloss over. When Gloss rolled down be in the trunk, Crumpky searched the
her window, Crumpky was overwhelmed car and found 12 cases of cigarettes not
by the smell of breath mints. Gloss was bearing California tax seals. The car and
asked to step out of the car and perform a cigarettes were seized and Gloss was
field sobriety test. Gloss passed the test, but arrested and charged with violation of
Crumpky felt Gloss was probably an alco- California law. Crumpky had neither a
holic who was able to perform most daily search nor an arrest warrant. Pursuant to
functions while intoxicated. In fact, he was a California statute, a petition for forfei-
positive that's why the smell of breath mints ture of the automobile was filed. Gloss,
was so strong. by timely objection, seeks dismissal
of the forfeiture petition, claiming the
Believing there was probably a bottle of al- evidence was illegally obtained. Will
cohol somewhere, Crumpky searched Gloss' the forfeiture petition be dismissed?
entire car. Under the driver's seat, he found
a baggy containing an 1/8 ounce of cocaine. A. Yes, because the seizure was
Gloss was arrested and taken immediately founded upon evidence illegally
to the station. Gloss' attorney files a motion obtained, since under the par-
to suppress the evidence. ticular circumstances Crumpky
acted without probable cause.
56. This motion will likely:
B. No, because the inventory search
A. Fail, because the overwhelm- exception allows Crumpky to act
ing smell of breath mints gave as he did.
Crumpky all the reason he need-
ed to search Gloss' car. C. No, because the exclusionary
rule does not apply to civil for-
B. Fail, because cocaine was feitures.
found.
D. No, because contraband, once
C. Fail, because of the inventory seized, cannot be returned to its
exception. owner.

D. Succeed.

252 CRIMINAL PROCEDURE


Q
Questions 58 — 62 are based on the C. No, because he only had license
following facts: to look for his golfball and noth-
ing else.
John Palmer, an off-duty police officer, was
playing golf one Sunday afternoon. His tee D. Yes, because no significant pri-
shot on the fourth hole was a bad slice and vacy expectation was invaded.
ended up out-of-bounds, in the backyard
of Mrs. Middo. Palmer climbed the hill to 59. The discovery of the hundreds of bag-
Middo's backyard and began looking for gies full of marijuana was unlawful
his golf ball, which was easy to do since because:
there was no fence to climb. While looking,
Palmer glanced into Middo's greenhouse A. No significant privacy expecta-
and saw several beautifully-manicured tion was invaded.
marijuana plants inside. He also noticed, on
a workbench inside the greenhouse, three B. Palmer was presented with an
ouzies. Suspicious, Palmer then proceeded exigent circumstance.
to go through two trash cans in the backyard.
In them, he found hundreds of baggies full C. Palmer acted in good faith.
of marijuana. Based on what he found in the
backyard, Palmer ended his golf game early, D. None of the above.
went to the nearby police station where he
worked and obtained a search warrant to 60. Since the warrant had no signature, it
search Middo's greenhouse and home for will be deemed:
marijuana and drug-dealing paraphernalia.
The warrant Palmer procured lacked the A. Valid, because the magistrate
magistrate's signature because the magis- inadvertently left his signature
trate inadvertently left his signature off the off.
form.
B. Valid, even if the signature was
58. Was Palmer's discovery of the green- purposefully left off, as long as
house marijuana and the three ouzies the magistrate had all of the facts
lawful? before him when preparing the
warrant.
A. No, because Palmer was off-duty
at the time. C. Valid.

B. No, because he had no probable D. Defective.


cause to enter Middo's back-
yard.

CRIMINAL PROCEDURE 253


Q
61. If the warrant is deemed invalid after 63. T.J. Booker ran a bookmaking operation
Officer Palmer executes it, anything out of his print shop, located on the third
found pursuant to that warrant will be: floor of an office building in downtown
Pennsylvania. An F.B.I. agent stood on
A. Inadmissible, if Palmer acted on a tall ladder and used binoculars to see
the warrant with the knowledge inside the print shop's windows. From
that it was invalid. this vantage point, the agent observed
gambling records. Did the F.B.I. agent
B. Inadmissible. engage in a search by climbing on top
of a ladder and using binoculars?
C. Admissible.
A. Yes, because he had to climb a
D. None of the above. ladder and use binoculars.

62. If Officer Palmer realizes the warrant B. Yes, because the window wasn't
is defective because it lacks a signature at street level.
and executes it despite this knowledge,
anything found pursuant to the warrant C. No, because the third floor win-
will be: dow to the print shop was open
to view.
A. Admissible.
D. Both A and B.
B. Admissible, only if the evidence
could have been found without
the issuance of a warrant.

C. Admissible, because Palmer's


findings while looking for his
golfball in Middo's backyard
were incriminating enough to
indicate that other contraband
would likely be discovered.

D. None of the above.

254 CRIMINAL PROCEDURE


Questions 64 — 66 are based on the 64. Since he built the huge barn with four
following facts: 30 foot walls, will the court find that
Johnny Potseed entertained a legitimate
Johnny Potseed built himself a huge barn expectation of privacy in the premises
so he could grow pot without any of his or area searched?
neighbors realizing what he was doing. The
barn had four walls, all of which were 30 feet A. Yes.
high. There were no windows in the walls
because Johnny didn't want anyone to be B. Yes, because an individual
able to see inside. Johnny didn't put a roof doesn't have to contemplate the
on the barn so the sun could shine down on possibility of an aircraft hovering
his wonderful marijuana buds, making them over their structure at an altitude
all the more potent. low enough to be able to see
inside the structure's rooftop.
Acting on a telephone tip from an anony-
mous pilot reporting the presence of mari- C. Yes, because the construction
juana growing within the four huge walls of the roof successfully denied
of the barn, two police officers flew over anyone adjacent to it, or paral-
the premises in a state police helicopter. At lel to it, a view of the goings on
an altitude of approximately 500 feet above inside of it.
the barn, the officers were unable to deci-
pher the exact kind of vegetation growing D. No, because the structure of the
in the barn. To get a better look, the officers roof made it like an open or un-
lowered their altitude to approximately 40 covered window.
feet over the barn. From this distance, the
officers, trained and experienced in the aerial
detection of marijuana, were able to tell that
Johnny was growing marijuana. The officers
hovered over the barn for approximately 5
minutes before leaving the area.

On the basis of the officers' observations, a


search warrant was issued. With warrant in
hand, the police forcibly entered the barn
and observed close to 100 large marijuana
plants, 18-20 feet tall.

CRIMINAL PROCEDURE 255


65. Assume for the purpose of this ques- 66. Assume that Johnny Potseed had both
tion only, that Johnny Potseed had a domestic and farm animals on his
roof on his huge barn, which he still property. When the police helicopters
used for his marijuana growing. During hovered 40 feet above the property,
a rainstorm, the roof leaked. Johnny the loud noises and vibrations caused
contracted X-Cell Roofing to install by the helicopter's proximity, caused
a new roof. X-Cell roofing acciden- Johnny's animals to go into a frenzy,
tally left a 20-square-foot portion of whereby they stampeded wildly around
the roof uncovered, though they told the property. During this frenzy, half of
Johnny, and believed so, that the roof the animals died from shock. In fact, the
was completely installed. Believing the vibrations were so strong that a glass
roof was installed, Johnny left town cabinet full of china crashed to the floor
for a few days of rest and relaxation. in the kitchen of Johnny's nearby home.
Upon arriving home, Johnny found his Johnny brings a cause of action against
marijuana crop being confiscated by the government for invasion of privacy.
the police. Johnny was placed under Will he prevail?
arrest and told that the marijuana had
been discovered in the same fashion it A. Yes.
had been discovered in question 64. At
trial, Johnny's lawyer made a motion to B. Yes, but only if the court decides
suppress the marijuana on the grounds the conduct of the officers in fly-
that his client's privacy interests had ing at 40 feet above the barn was
been violated. This motion will: hazardous to persons or property
on the surface.
A. Succeed, because Johnny did not
deprive himself of his privacy. C. No, because even though damage
to persons or property occurred,
B. Succeed, because Johnny con- the officers had a right to fly
structed a roof on the barn so that above Johnny's property.
no one could see inside.
D. No, because Johnny's marijuana
C. Fail, because the opening in the crop was clear and visible.
roof negated any expectation
of privacy from the eyes of the
world or of the police.

D. None of the above.

256 CRIMINAL PROCEDURE


Questions 67 — 68 are based on the During the search of Bob's house, large
following facts: quantities of hash were discovered along
with drug-dealing paraphernalia. Bob was
Bob Dealin put his garbage at the curb in arrested on felony narcotics charges. Bob's
front of his house on Sunday night because attorney made a motion to suppress the evi-
the garbage men pick up the neighborhood's dence, claiming the warrantless trash search
garbage on Monday morning. Bob put his violated the Fourth Amendment.
trash in black trash bags and tied them se-
curely shut. He used black trash bags and 67. The motion to suppress will:
tied them securely shut because he didn't
want anybody to see what was inside the A. Succeed, because the warrantless
bags nor be able to open them with ease. Bob search of Bob's trash violated the
didn't want anybody messing with his trash Fourth Amendment.
bags because Bob made his living dealing
hash to the neighboring colleges and much B. Succeed, because Bob had placed
of Bob's trash consisted of items indicative his trash in black trash bags,
of narcotic use and dealing. which no one could see through,
and tied the bags securely shut,
One of Bob's neighbors notified the police indicating his expectation of
that she observed on many occasions several privacy.
vehicles make brief stops at the house during
the late-night and early-morning hours. The C. Fail, because society would not
police, based on the neighbor's information, accept as reasonable Bob's claim
conducted a surveillance of Bob's home. to an expectation of privacy.
They, too, observed activity that indicated
potential drug dealing was taking place at D. Both A and B.
Bob's house.

One of the police officers asked the neigh-


borhood's regular trash collector to set aside
the bags from Bob's house without mixing
them with trash from any of the other resi-
dences. The trash collector did so and turned
the bags over to the officer, who promptly
looked through them. Items indicative of
narcotics use and drug dealing were found.
The officer was able to secure a search war-
rant to search Bob's home.

CRIMINAL PROCEDURE 257


68. Assume for the purpose of this ques- 69. Sigmund Frodd, experiencing grave fi-
tion only, that Bob, because he was nancial difficulties, decided it was time
getting ready to go out of town for a to cheat on his taxes. During 1994, he
couple days, had placed two opaque did many fraudulent things in an effort
trash bags, securely tied, just outside to get a refund from the government.
his front door, containing two eight- Frodd shredded all of the documents
ounce bricks of hash and other legal he used to mastermind his fraudulent
personal belongings. Bob was going activities because he didn't want any-
to take the bags with him on his trip. one to be able to figure out what he
A police officer, acting on a neighbor's had done. Frodd contemplated burning
tip that drug dealing was likely tak- the documents, but decided instead to
ing place at Bob's house, walked up merely put the shreddings in the trash
to Bob's front porch and confiscated because the documents had been virtu-
the two black trash bags. Based on the ally shredded beyond comprehension.
contents discovered therein, the officer
secured a search warrant for Bob's Frodd's tax return had been red-flagged
home and found quantities of hash and by the I.R.S. computer and two I.R.S.
drug-dealing paraphernalia inside. Bob agents had been sent to Frodd's house
files a motion to suppress the evidence to investigate. The agents searched
at trial. The motion would: through garbage bags outside Frodd's
house and found them full of shredded
A. Succeed, because the warrant- documents. The agents took the garbage
less search of Bob's black trash bags with them back to their headquar-
bags violated the Fourth Amend- ters and proceeded to painstakingly
ment. piece together the shredded documents.
Based upon the pieced-together docu-
B. Succeed, because the black trash ments, the I.R.S. was able to make a
bags were being used to store or case of tax fraud against Frodd. A mo-
transport personal possessions. tion to suppress the evidence would:

C. Fail. A. Succeed, because John shredded


the documents so no one could
D. Both A and B. read them.

B. Succeed, because the documents


in their shredded state were in no
way incriminating.

C. Fail.

258 CRIMINAL PROCEDURE


Q
D. Both A and B. A. No, because Prude, a private
party, opened the videos herself,
70. During their examination of a damaged revealing the pornographic video
package, employees of Federal Excess boxes.
observed a white powdery substance,
originally concealed within eight lay- B. No, because the video boxes
ers of brown paper and tape. A federal were obviously pornographic,
agent was summoned, who removed a given the pictures of sexual acts
trace amount of the white substance, on the outside of them.
subjected it to a test and determined that
it was cocaine. Did the federal agent's C. Yes, because the screening of
field test constitute an unlawful search the films by the agents was a
or seizure within the meaning of the governmental search.
Fourth Amendment?
D. Yes, because Prude did not watch
A. No. the videos.

B. Yes, because a protected posses- 72. John Dough, a postal worker, opened
sory interest was infringed. a suspicious-looking package with a
return address from Haight-Asbury
C. Yes, because the field test like- Street, in San Francisco. John was hop-
ly destroyed some of the evi- ing the package would contain some
dence. black-tar hash, given the return address
and the fact it was to be delivered to a
D. Both B and C. nearby fraternity house. John took the
package home with him and discovered
71. Ms. Prude mistakenly received a pack- 12 grams of blonde hash instead of
age containing apparently pornographic black-tar. A little disappointed, John
videos. Since there were people en- smoked some to get in a better mood.
gaged in sexual acts on the video boxes, Unfortunately, John became extremely
Prude turned the videos over to F.B.I. paranoid, realized what he had done
agents. The agents watched the videos was wrong and turned the hash over to
and were able to ascertain that they the F.B.I. the next morning. The agents
were, in fact, pornographic. The agents brought a prosecution action against
brought prosecution against the sender. the sender. Are John Dough's actions
Did the F.B.I. agents need a warrant to governed by the Fourth Amendment?
view the pornographic videos sent to
Prude by mistake? A. No, because John procured the
hash for his personal use.

CRIMINAL PROCEDURE 259


B. Yes, because John is a postal 74. Molly Made works for Weekend Inn, a
worker. chain of motels, where she is a part of
the room-cleaning staff. While cleaning
C. Yes, because John discovered room 237, Molly discovered a sizeable
the hash during the course and amount of cocaine on the night stand.
scope of his government employ- Molly quickly walked to the motel's
ment. donut shop where she told Officer
Winchell about the cocaine. Officer
D. Yes, because John's search and Winchell followed Molly to room 237
seizure was done to further a and Molly let Winchell inside the room.
governmental objective. Winchell further searched the room and
found more cocaine and cocaine para-
73. The police were looking for Tom Hoss, phernalia. Winchell waited in the room
a bookmaking kingpin. Two officers until the guest returned and arrested
drove to Tom's house, thinking he him for possession of cocaine. Was the
might be home, to ask him some ques- consent to search given by Molly Made
tions concerning the recent murder of a valid?
jockey. They knocked on the door and
Tom's butler answered the door, obvi- A. Yes, if it was after check-out time
ously intoxicated, reeking of martinis. for room 237.
"May we come in?" one of the officers
asked. "Yes, you may," the Butler said, B. Yes, because a hotel clerk or
with obviously-slurred speech. Did the maid has apparent authority to
officers obtain voluntary consent? consent to the search of a guest's
room.
A. No, if the officers knew that the
butler was drunk. C. No, because a third party cannot
give consent.
B. No, if the officers did not know
the butler was drunk. D. None of the above.

C. Yes.

D. Both A and B.

260 CRIMINAL PROCEDURE


Q
75. Mary Jane was a college student who Questions 76 — 77 are based on the
dealt drugs to make ends meet. One following facts:
night, while weighing her latest ship-
ment, there was a knock on Mary David Malcolm served a three-year sentence
Jane's door. "Who is it?" she asked. for burglary and upon his release from an
"The police! Open up!" Mary peered Idaho prison was placed on a five-year
through the peep-hole: it was the police. probation period. A condition of his proba-
"Sorry, but I can't let you in." "You'd tion was that he agree in advance to waive
better! We have a search warrant!" certain Fourth Amendment rights. Malcolm
Flustered, Mary let the police inside. attached his signature to the following:
Three of them bolted through the door
and thoroughly searched Mary's apart- "That probationer does hereby agree and
ment, finding two pounds of marijuana, consent to the search of his person, automo-
scales and other items indicating drug bile, real property, and any other property
use and drug dealing. at any time and at any place by any law
enforcement officer, peace officer, probation
During the search, Mary asked, "May officers, and does waive his constitutional
I see the search warrant, please?" The right to be free from such searches."
officers all started laughing. One of
them said, "We have no search war- Later, when officers obtained information
rant, you dumb broad!" Shocked, Mary that Malcolm was fencing stolen goods, they
screamed, "But that's the only reason I went to his home and searched it without a
let you in!" "Oh well," one of the of- warrant while he was out of town, and found
ficers shrugged. Did the officers obtain large amounts of stolen goods.
valid consent to enter and search Mary's
apartment? 76. The probation condition will be
found:
A. Yes, because the search incrimi-
nated Mary. A. Invalid, because an individual
cannot sign away his constitu-
B. Yes, because securing a search tional rights.
warrant is a mere formality
when the officers have probable B. Invalid, because there is implied
cause. coercion involved when making
a probationer consent in advance
C. Yes, if all three officers thought to surrender his constitutional
one of the other officers had a rights.
search warrant.

D. No.

CRIMINAL PROCEDURE 261


C. Invalid, because such a broadly Questions 78 — 81 are based on the
worded probation condition following facts:
could lead to invasions of pri-
vacy of innocent third parties. Officer Bust posed as a drug purchaser and
gained entry into the home of Tommy Dealer,
D. Valid. who ran a crack house in south Central L.A.
When inside, Bust witnessed several drug
77. Assume the same facts as question 76, sales take place. Tommy was arrested and
except the above clause is not a condi- charged with possession and sale of crack
tion of Malcolm's probation. Will the cocaine. Tommy's lawyer made a motion
police officer's search of Malcolm's to suppress the evidence because Bust had
home be upheld? gained entry by tricking Tommy.

A. No, because the police en- 78. This motion will:


tered Malcolm's home without
a search warrant. A. Succeed, because Bust misrep-
resented his identity.
B. No, because Malcolm was out of
town and could not consent to the B. Succeed, because Bust's entry
search. was not consensual, since he
tricked Tommy.
C. Yes, because, under the cir-
cumstances, it can be deemed a C. Fail, because Bust posed as a
"regulatory search." drug purchaser.

D. Both A and B. D. Fail, because Tommy allowed


Bust to enter his home.

79. Assume for the purpose of this ques-


tion only, that Bust gained entry into
Tommy's home in the manner he did
in question 78. Rather than witness
drug transactions taking place, Bust
took the opportunity while Tommy
was in the bathroom to look into closed
drawers and envelopes, finding cocaine
and cocaine paraphernalia. Tommy's
lawyer made a motion to suppress this
evidence. The motion will:

262 CRIMINAL PROCEDURE


Q
A. Fail, because Bust posed as a 81. Assume for the purpose of this ques-
drug purchaser. tion only, that Bust gained entry into
Tommy's home by telling Tommy he
B. Succeed, because Bust looked was looking for an escaped convict,
into closed drawers and enve- who broke out of the nearby State peni-
lopes while Tommy was in the tentiary. Actually, Bust was looking to
bathroom. bust Tommy. Once inside, Bust saw a
sizeable lump of cocaine, on a mirror,
C. Succeed, because Bust concealed on Tommy's coffee table. Tommy's
his status as a law enforcement lawyer made a motion to suppress the
officer. evidence because Bust used deceptive
practices to gain entry into Tommy's
D. Fail, because Tommy left the home. The motion will:
room, thus giving Bust implied
consent to search the closed A. Succeed, because Bust misrep-
drawers and envelopes. resented his purpose.

80. Assume for the purpose of this question B. Succeed, because Bust did not
only, that Officer Bust gained entry into represent himself as having an
Tommy's home by pretending to be illegal purpose.
a telephone technician. Once inside,
Bust witnessed drug transactions taking C. Fail, because Tommy's consent
place. Tommy's lawyer made a motion was voluntary.
to suppress the evidence. The motion
will: D. Fail.

A. Succeed, because Bust did not


represent himself as having an
illegal purpose.

B. Fail, because Bust posed as a


telephone technician.

C. Fail, because Bust did not make


any additional intrusions into the
premises of Tommy.

D. Fail.

CRIMINAL PROCEDURE 263


Q
82. Officers Starsky and Hooch had secured 83. Officer Jones was driving his patrol car
a warrant to search the person of For- down Sunset Blvd., looking for Tim
rest Dump and his place of residence Johnson, a suspect wearing blue jeans
known as "8709 Portapotty Place, fifth and a lime-green t-shirt, who just robbed
floor apartment." Starsky and Hooch a coin store. Jones spotted a man wear-
reasonably believed that there was ing blue jeans and a lime-green t-shirt,
only one fifth floor apartment at 8709 walking briskly down the boulevard.
Portapotty Place. In fact, there were two Jones sped up, got out of his patrol car
apartments located on the fifth floor, one and accosted the man who he thought
belonging to Dump and the other to a was Johnson. With gun drawn, Jones
man named Peter Walk. Before the of- instructed the suspect to put his hands
ficers realized there were two fifth floor over his head. "What's this all about?"
apartments, they had entered Walk's Johnson asked. "You know what it's
apartment, thinking it was Dump's, and all about," Jones replied. "Don't tell
found two kilos of hash on the dining me you don't know anything about the
room table, next to a drug scale. coin shop you just robbed, Mr. John-
son." "I'm not Johnson." "Yeah, right,"
Walk was subsequently arrested for replied Jones. Jones searched through
possession of narcotics with intent to the pockets of the suspect and found a
sell. Walk's attorney filed a motion to vile of cocaine and a metal straw, but
suppress the evidence because the of- didn't find any coins or any items sold
ficers had entered the premises without in a coin shop.
a search warrant. The motion will:
"Where'd you stash the coins, John-
A. Succeed, because the warrant son?" "I'm not Johnson. My name is
will be found defective. Howard Limbo. I'm on my way to catch
the bus." Jones allowed the suspect
B. Succeed, because the officers to produce his driver's license. The
searched the wrong residence. suspect was not lying, his name was
Howard Limbo. Nonetheless, Officer
C. Succeed, because the officers Jones handcuffed Limbo and drove him
should have investigated further to the coin shop, hopefully to be identi-
to determine if there was only fied by the coin shop's owner.
one fifth floor apartment.

D. Fail.

264 CRIMINAL PROCEDURE


Q
"No, that's not the guy," says the owner B. Fail, unless the warrant was ex-
of the coin shop. "Wearing similar ecuted in daytime hours.
clothes, but that's not him." Officer
Jones arrested Limbo for possession C. Fail, because the facts do not
of cocaine. Limbo's attorney made a indicate the warrant lacked the
motion to suppress the evidence. The necessary judicial/magistrate
motion will: signature or seal.

A. Fail. D. Succeed only if the warrant had


not expired.
B. Succeed, because Officer Jones
detained the wrong man.

C. Succeed, because Officer Jones


had no reason to search the pock-
ets of Howard Limbo.

D. Succeed, because the owner of


the coin shop proved Howard
Limbo's innocence.

84. Officer Thomas and Officer Hill secured


a warrant on June 1,1994, to search the
premises of Robert Biden. Innundated
with paper work and other law enforce-
ment matters, Thomas and Hill were
unable to act on the warrant until July
7,1994. Once they did execute the war-
rant, however, they were able to seize
the anticipated cocaine from Biden's
apartment. Biden was arrested for pos-
session of cocaine. Biden's lawyer filed
a motion to suppress the evidence. The
motion will:

A. Succeed, because the officers


acted on the warrant on July 7,
1994.

CRIMINAL PROCEDURE 265


Q
Questions 85 — 88 are based upon D. Fail, provided this was deemed

the following facts: a legitimate exigency.

Officers Kent, Camel and Kool had a warrant 86. Assume for the purpose of this question
for the arrest of J.J. Stonelli, a Mafia hitman, only, that Stonelli was home when the
with a known propensity for extreme vio- officers arrived and they were able to
lence. It was known that Stonelli stockpiled handcuff him without incident, in his
automatic weapons in his home, to protect living room. Knowing that Stonelli was
his large cache of heroin, cocaine and hash. "armed and dangerous," the officers
In fact, Stonelli was listed in police files as feared there could be armed and danger-
"armed and dangerous." ous associates of Stonelli elsewhere in
the house. The officers looked through
Officers Kent, Camel and Kool quietly ap- large mahogany cabinets in the living
proached Stonelli's home and busted down room as well as closets in an imme-
the front door, giving no notice or warning diately adjoining hallway, fearing at-
to Stonelli or anyone else inside the house. tackers could likely be present. During
They bolted through the house, found large their search, they found large amounts
amounts of drugs and illegal assault rifles, of cocaine. Stonelli's lawyer makes a
while Stonelli was in the shower with his pet motion to suppress the cocaine. The
poodle. Stonelli was arrested for possession motion will:
of narcotic and illegal assault rifles. At trial,
Stonelli's lawyer made a motion to suppress A. Succeed, because the officers did
the evidence. not have probable cause.

85. The motion will: B. Succeed, because the officers did


not have reasonable suspicion.
A. Succeed, because the officers
did not give notice of their au- C. Both A and B.
thority and purpose for entering
Stonelli's home. D. Fail.

B. Succeed, because the police un-


necessarily broke down the front
door to gain entry.

C. Succeed, because the actions of


the officers could have lead to
unnecessary violence and en-
dangered the lives of innocent
people.

266 CRIMINAL PROCEDURE


Q
11n
87. Assume the same facts as question 86, A. Succeed, because the officers
0
-

except the officers searched Stonelli's knew about the weapons, given
E
entire home, looking under every bed the fact they applied for a war- 02
cy
and opening all cabinets, cupboards and rant indicating weapons. xi
closets. During their extensive search,
the officers found large amounts of B. Succeed, because the officers
cocaine under the bed in the master hoped to find the weapons, given
bedroom. Stonelli's lawyer makes a the fact they applied for a war-
motion to suppress the evidence. The rant indicating weapons.
motion will:
C. Succeed, because the weapons
A. Succeed, because the officers were not discovered "inadver-
did not have probable cause to tently" for the plain view doc-
search the entire premises. trine to apply, given the fact the
officers applied for a warrant
B. Succeed, because the officers did indicating weapons.
not have reasonable suspicion to
search the entire premises. D. Fail, because it was a valid
search.
C. Fail, if a reasonably prudent of-
ficer would believe that a danger-
ous individual was harbored in
the house.

D. Fail, because the actions of the


officers exceeded what was law-
ful.

88. Assume for the purpose of this question


only, that the officers applied for a war-
rant to search Stonelli's house for weap-
ons and stolen property, but the warrant
as signed by the magistrate authorized
only to search for stolen property. The
officers executing the warrant did not
find any stolen property but found the
weapons in plain view. Stonelli's lawyer
made a motion to suppress the weapons.
The motion will:

CRIMINAL PROCEDURE 267


Questions 89 — 91 are based on the III. Reasonable.
following facts:
A. I and II only.
Three officers executed a valid warrant
to search for drugs in a neighborhood bar B. III only.
known for drug use. Once lawfully inside the
bar, the officers began searching customers C. I, II, and III.
of the bar for drugs. The officers searched
pockets, purses and wallets for any indicia D. II and III only.
of drugs.
91. Assume for the purpose of this ques-
89. The search of the customers was: tion only, that the officers temporar-
ily detained the customers of the bar
A. Lawful, because people who while they conducted their search of
frequent a bar known for drug the premises. Such an action would be
use are likely to have drugs on deemed:
their person.
I. Coercive.
B. Lawful, because the officers
searched pockets, purses and II. A violation of the Due
wallets. Process Clause.

C. Unlawful, because the officers III. Unconstitutional.


had no probable cause to arrest
the customers. A. I only.

D. Unlawful because the search B. II and III only.


warrant only allowed the officers
to search the bar, not the personal C. I, II and III.
effects of the customers.
D. None of the above.
90. Assume for the purpose of this ques-
tion only, that the officers frisked the
customers for weapons. Such an action
would be deemed:

I. A Fourth Amendment
right violation.

II. An invasion of privacy.

268 CRIMINAL PROCEDURE


Q
Questions 92 — 93 are based on the C. Yes, because the presence of
following facts: an expensive CD player in an
ill-furnished apartment in a
Police officers were sent to a downstairs poverty-stricken neighborhood
apartment where a man lay wounded from was a proper basis for reasonable
a gunshot wound. The officers determined suspicion.
that the bullet was shot from the upstairs
apartment as there was a hole in the ceiling D. None of the above.
of the downstairs apartment consistent with
a bullet hole. The officers lawfully entered 93. Assume for the purpose of this question
the upstairs apartment to search for the only, that one of the officers found and
shooter, weapons and other victims. When seized a box of pen guns in plain view.
inside the apartment, the officers found a Later, at the station, it was determined
rifle, a case of bullets as well as a discarded that these pen guns, which look exactly
shell. One of the officers also noticed an like ballpoint pens to the naked eye,
expensive CD player which didn't fit in with were in fact contraband (weapons). Was
the ill-furnished apartment or the poverty- the seizure of the pen guns lawful?
stricken neighborhood. Thinking it might be
stolen, he moved it away from the wall so I. Yes, because they were
he could read the serial number. He wrote discovered in plain
the serial numbers down and realized they view.
matched a CD player that had been stolen in
a recent armed robbery. The officer seized II. Yes, because they were
the CD player and the apartment's owner later determined to be
was charged with discharging a firearm in contraband.
city limits and possession of stolen goods.
In fact, discovery of the CD player led to III. No, because they were
the apartment owner's arrest for the recent later determined to be
armed robbery. contraband.

92. Was the officer's conduct concerning A. I


the CD player lawful?
B. I and II
A. Yes, because the officers lawfully
entered the apartment. C. III

B. Yes, because recording the se- D. None of the above.


rial numbers did not constitute a
seizure.

CRIMINAL PROCEDURE 269


Q
_,Lu
Qs D. Yes, because Johnny indicated
Z_ p
'"' LIJ
Questions 94 — 98 are based upon
the following facts: he would not make a written
2 (.)
..

Fe0 statement outside the presence


Oa Assume the following facts for the purpose of counsel.
of this question only: Johnny Boy ran a red
light late one night and was pulled over 95. Assume for the purpose of this ques-
by Officer 0' Hair. Johnny was asked to tion only, that nine police officers had
step out of the car because his pupils were lawfully entered Johnny Boy's house
dilated and he was talking extremely fast. at 3:00 a.m. and questioned him in his
O'Hair suspected cocaine use. Johnny failed own bedroom, without reading him the
a field sobriety test and was taken back to Miranda warnings. During this ques-
the police station. There, Johnny agreed to tioning session, Johnny made several
waive his Miranda rights and talk to Officer incriminating statements. Should the
O'Hair and another officer. Johnny, however, officers have read Johnny Boy his Mi-
insisted that nothing be put in writing and randa rights?
requested that all tape recorders be turned
off. Johnny also indicated that he would not A. No, because he was not being
make a written statement outside the pres- questioned at the police station.
ence of counsel. Johnny proceeded to make
incriminating statements. B. No, because a suspect cannot be
in custody in his own home.
94. Should Johnny's incriminating state-
ments be suppressed? C. Yes, because there were nine
officers in Johnny Boy's bed-
A. No, because the questioning was room.
consistent with the Fifth Amend-
ment. D. Yes, because a reasonable person
would have believed this was an
B. Yes, because when Johnny interrogation.
requested that none of his state-
ments be put in writing or tape- 96. Assume for the purpose of this ques-
recorded, the officers should tion only, that Johnny Boy accepted an
have realized that Johnny didn't invitation from a police officer to come
understand that his statements down to the police station for question-
could be used as evidence even if ing, but was not under arrest. Would
not written down or recorded. Johnny be deemed to be in custody?

C. Yes, because Johnny's waiver A. Yes, because the questioning


of his Miranda rights was con- took place at the police station.
ditioned.

270 CRIMINAL PROCEDURE


B. Yes, because Johnny was invited 98. Bob Youbanks, on a hot night in the
by a police officer. summer of 1994, engaged by himself
in a series of armed robberies in the
C. No, unless the invitation ap- state of Columbia. Youbanks, a home-
peared to be a kind of offer less entrepreneur, sold wedding dresses
that could not be reasonably at the local swap meet every weekend
refused. and, low on inventory, he decided to rob
five bridal supply shops. The victims,
D. No, because Johnny was not in each case, were the business owners
under arrest. who were forced to hand over three
designer wedding gowns at gun point.
97. Assume for the purpose of this ques- At two of the stores, Youbanks fired his
tion only, that Johnny Boy was pulled handgun, severely injuring four custom-
over by Officer 0' Hair under suspicion ers. In fact, one customer was purpose-
of drunk driving. Officer O'Hair pro- fully shot and killed by Youbanks.
ceeded to ask Johnny Boy a series of
questions, trying to ascertain whether Columbia state law provides that when
he had been drinking, how much he had a defendant pleads guilty, "the court
been drinking and when he had his last must cause the punishment to be de-
drink. Would Officer O'Hair's ques- termined by a jury" and may "cause
tioning of Johnny Boy be considered witnesses to be examined, to ascertain
custodial interrogation? the character of the offense."

A. Yes, because Officer O'Hair is a Before the matter came to trial, the
police officer. court determined that Youbanks was
indigent. He was provided with a public
B. Yes, because Officer O'Hair defender. A week later, at his arraign-
detained Johnny Boy. ment, Youbanks pleaded guilty to all six
offenses. Youbanks was found guilty of
C. No, because Johnny Boy was not all counts, based on credible eye-wit-
arrested. ness testimony, and sentenced to death
by electrocution.
D. No, but only if O'Hair knew
Johnny had been drinking. On automatic appeal, it was discovered,
as indicated in the record, that the judge
did not ask Youbanks any questions
concerning his plea, and Youbanks did
not in any way address the court. Can
Youbank's conviction be reversed?

CRIMINAL PROCEDURE 271


A. Yes, because he was denied due Later, at the plea hearing, Astro af-
process of law. firmed that he fully understood the
plea statement and the consequences
B. No, because he was found guilty of signing his name on the document.
based on credible eye-witness The judge accepted Astro 's guilty plea
testimony. and sentenced him according to the
state's recommendation: 20 years for
C. No, because he pleaded guilty. the murder and 5 years for the theft.
Astro was credited with the time he
D. No, because there cannot be had already served in prison. The judge
reversible error when the defen- advised Astro that he would be required
dant admits guilt. to serve at least one-third of the murder
sentence before he would be eligible
99. Billy Astro pleaded guilty to charges for parole.
of first-degree murder and theft of
property. Under Columbia state law, Almost three years later, Astro filed a
the murder charge carried a potential federal habeas corpus petition alleging
sentence of 15 to 50 years or life in that his guilty plea was involuntary
prison. Astro 's court-appointed attorney because his court-appointed attorney
successfully negotiated a plea bargain informed him, just as the judge did, that
with the state, where Astro would plead he would be eligible for parole after
guilty to both charges, if the State would serving one-third of the murder sen-
impose concurrent prison sentences of tence. In fact, because Astro had been
20 years for the murder and 5 years for convicted of a separate felony more
the theft. Astro signed a plea agreement than five years ago, he was classified
indicating that he understood the charg- as a "second offender," under Colum-
es against him and the consequences of bia state law, he would be required to
pleading guilty, and that his pleas had serve one-half of the murder sentence
not been induced by "force, fear, threat, before being eligible for parole. Astro
promise or intimidation." He affixed has asked the Columbia Court of Ap-
his signature below the last two lines peals to reduce his sentence to a term of
of the plea agreement, which read: "I years that would result in his becoming
fully understand this document. I fully eligible for parole in conformance with
understand what my rights are, and I his original plea-bargain expectations.
voluntarily plead guilty and realize the Will Astro be granted relief?
consequences of so doing."

272 CRIMINAL PROCEDURE


Q
A. No, if Astro did not allege in his 100. Robert Rotten was indicted on two
habeas petition that he would felony counts of bookmaking. After
have pleaded not guilty and negotiations, Rotten's attorney was
insisted on going to trial, had he able to persuade the district attorney to
been correctly informed about allow his client to plead to a lesser-in-
his parole eligibility date. cluded offense, which carried a maxi-
mum sentence of one year in prison,
B. Yes, because the court-appointed upon conviction. The district attorney
attorney's constitutionally inef- also agreed to make no recommenda-
fective performance affected the tions as to length of sentence. Rotten
outcome of the plea bargaining subsequently entered his plea of guilty
process. to the lesser offense. Rotten explained
to the sentencing judge that his plea
C. Yes, because Astro's guilty plea was voluntary and he understood
was not voluntary and intelli- all implications of pleading guilty.
gent. Rotten's plea was accepted and a date
for sentencing was set. Later, Rotten's
D. Yes, because Astro had a mis- attorney had discovered that evidence
taken belief as to when he would to be used against his client had been
become eligible for parole. illegally seized, and immediately
moved to withdraw the guilty plea.
The motion was denied.

The court then turned to consideration


of the sentence. At this proceeding,
a new and different prosecutor rec-
ommended that Rotten be given the
maximum one-year sentence. Rotten's
attorney immediately objected to this
because Rotten had been promised
that no sentencing recommendations
would be made. The sentencing judge
proceeded to impose the maximum
one-year sentence, claiming that the
prosecutor's recommendation had no
influence whatsoever on his decision.
Should Rotten be allowed to withdraw
his guilty plea?

CRIMINAL PROCEDURE 273


Q
A. Yes, because the earlier plea did
not irrevocably waive Rotten's
federal constitutional right to a
trial.

B. No, because the judge was not


influenced by the prosecutor's
recommendation.

C. No, because a plea cannot rest


on a promise or agreement of the
prosecutor.

D. Both B and C.

END OF QUESTIONS

274 CRIMINAL PROCEDURE


CRIMINAL PROCEDURE 3 . A is the correct answer. A civil suit
requires a violation of rights and bad
ANSWERS faith on the part of the officer. There
was a Fourth Amendment violation here
because Marie was held over 48 hours

A
1. C is the correct answer. Choices I and II before seeing a judge. See McLaughlin,
are correct because Mick has no reason- supra. However, since the officer may
able expectation of privacy in the area have had nothing to do with the delay,
searched, here a public place, since he and since good faith is a defense, there
abandoned the property. Had Mick been may not be damages. B, C and D are
seized, the answer would have been dif- incorrect for the same reasons that A is
ferent. That is, his abandonment of the correct.
marijuana was the "fruit" of the illegal
seizure. But, in California v. Hodari 4. D is the correct answer. An individual
D. (1991), it was held that there is no need not be told of her right to refuse
seizure unless there is actual restraint a search, provided there was no police
or a submission to police authority. coercion. The test for consent is simply
There was none here. Thus, C, which voluntariness under the totality of the
includes choice I, II and III is the correct circumstances. A similar fact-pattern
answer. occurred in Florida v. Bostick (1991),
and the consent theregiven was deemed
2. A is the best answer. Note: The call of valid. A is incorrect because often war-
this question is phrased in the negative. rants are not needed in order to search.
A is clearly worse because the Bail B is incorrect because exigency is not
Clause was never made state-appli- the only exception to the warrant re-
cable; thus, it is a very bad answer. B, quirement. C is incorrect because the
though amorphous, may be correct. Due auto exception requires probable cause
Process might offer Marie some protec- of contraband in the auto, and there was
tion. Since there was impermissible none here.
delay in appearing before a judge after
the warrantless arrest (48 hours is the 5. D is the best answer. It is lawful to stop a
maximum delay allowed — See County car when the driver is speeding. Choice
of Riverside v. McLaughlin (1991)), C A is correct because a traffic stop is a
is a correct statement, which makes it detention — but D is more specific and
incorrect under this call. Since Marie is, thus, a better answer. A traffic stop
may have been entrapped, choice D is is not considered to be an arrest, so B
not as bad as choice A (which is clearly is incorrect. See Berkemer v. McCarty
wrong). (1980). C is incorrect for the same rea-
sons that D is correct.

CRIMINAL PROCEDURE 275


6. B is the correct answer. Although it may 8. C is the correct answer. Unlike the Fifth
be argued that Acid's consent was not Amendment Miranda prophylactic
voluntary since he was stopped, consent right to counsel, the Sixth Amendment
has been broadly construed by the U.S. counsel right (which attaches at formal
Supreme Court. See United States v. charging) is offense specific. McNeil v.
Mendenhall (1984). Moreover, an argu- Wisconsin (1991). Since the theft was
ment that the consent was invalid was stipulated to be unrelated to the forg-
not an option provided in the answers. ery, the interrogation was permissible.
Thus, given that the consent was vol- A and B are incorrect because the fact
untary, the issue then becomes was the that the interrogation was not in-person
officer permitted to search into a closed and voluntary are not entirely relevant.
container. Florida v. Jimeno (1991) is D is incorrect for the same reason that
directly on point and indicates such to C is correct.
be permissible. A is incorrect for the
same reasons that B is correct. C is in- 9. A is the correct answer. When an at-
correct because a traffic stop is not an torney misses the filing deadline of an
arrest. D is incorrect because, although appeal of right, ineffective counsel is
it may appear otherwise, there is no presumed. Lozada v. Deeds (1991).
drug exception to the warrant require- Although Strickland ordinarily requires
ment. prejudice to the verdict, such is not
necessary in this one instance. Thus,
7. A is the correct answer. Once there is B is incorrect. Since all attorneys are
probable cause of contraband in a car, state-licensed, it is irrelevant that Rex
the whole car can be searched — includ- was privately retained. Thus, C is incor-
ing the search of closed containers. See rect. D is incorrect because collateral
California v. Acevedo (1991). Here, appeal is allowed on this issue. See
there was probable cause based on the Strickland v. Washington (1984).
drugs in the passenger compartment.
B is incorrect because the search inci- 10. D is the correct answer. The U.S. Su-
dent to arrest exception does not allow preme Court has ruled that "victim
search into the trunk of a car. See New impact" statements are permissible.
York v. Belton (1981). Since there was Payne v. Tennessee (1991). Thus, A, B
no immediate danger or hot pursuit, C and C are incorrect.
is incorrect. D is incorrect because there
is no recognized drug exception to the
warrant requirement.

276 CRIMINAL PROCEDURE


11. C is the correct answer because it 14. B is the correct answer. Although il-
includes choices I, II and III, which legally seized evidence cannot be used
are all necessary to satisfy an Eighth in the case-in-chief, such can be used
Amendment claim by a prisoner. See to impeach. United States v. Havens,
Wilson v. Seiter (1991). Choices I and (1980). Since the impeachment was
II require the treatment of a prisoner to proper, it necessarily follows that an-
be both cruel and unusual, following
the language of the Eighth Amend-
ment. (Thus, incidentally, mere over-
swers A, C and D are incorrect.

15. C is the correct answer. Provided that the


A
crowding, although cruel, may not be legislature so intended, a life-sentence
unusual.) Choice III requires intent on for a first-time drug offense does not
the part of the warden or the claim will violate the Cruel and Unusual Punish-
fail. ment Clause. See Harmelin v. Michigan
(1991). In this case, the Supreme Court
12. D is the best answer. Choice A, B and severely limited the traditional notion of
C all describe aspects of the Fourth proportionality. Thus, choice A is incor-
Amendment. However, habeas claims rect. B is incorrect because lenity is a
cannot include Fourth Amendment ar- term demanding ambiguous statutes to
guments; so these choices are clearly be construed favorably to the defense.
wrong. Stone v. Powell (1976). Al- Such is irrelevant to these facts. Choice
though the Eighth Amendment claim D is incorrect per the pronouncement of
will also be difficult, it has a chance of Harmelin, above.
success — so D is the best answer.
16. C is the best answer. On the facts given,
13. A is the correct answer. Prisoners enjoy one might ask why was Wilbur allowed
no Fourth Amendment rights, because to take the stand? Shouldn't he have
of the vastly reduced expectation of pri- been told of the danger of impeach-
vacy in prison and the need for safety. ment? Although the facts do not indi-
Bell v. Wolfish (1979). Although argu- cate that he was not told, nevertheless
ments of Due Process (choice B), Equal counsel's representation is suspect.
Protection (choice C), and Cruel and Since the standard of ineffectiveness
Unusual Punishment (choice D) may of counsel is not "farce and mockery,"
not succeed, they at least have such a A is incorrect. See Strickland v. Wash-
possibility. ington (1984). Since no facts indicate
the judge to be biased, B is incorrect.
Since, as indicated in the previous an-
swer, sentences can be quite long and
constitutional, D is incorrect.

CRIMINAL PROCEDURE 277


17. B is the correct answer. This question 20. C is the correct answer. If a contempora-
requires a fair reading of the facts. neous objection is not made (i.e., an ob-
Obviously Minko's threat of violence, jection contemporaneous to the alleged
"find yourself bleeding," makes the error), a successful collateral petition
interrogation coercive. Thus, C and D must show that there was good cause
are incorrect. Since there was no public for the failure (for example, conceal-
safety then at issue, the exception of- ment of evidence by the prosecution)
fered in choice A is irrelevant. Cf. New and prejudice to the verdict; and that
York v. Quarles (1984). the defendant is, in fact, innocent of the
crime based on a Constitutional error.
18. A is the correct answer. Under Arizona Only choice C offers this alternative
v. Fulminante (1991), the Supreme phrasing.
a. Court has held that even confessions
that are coerced can be subject to harm- 21. D is the correct answer. There is no
less error analysis. B is incorrect for the Fourth Amendment protection if there
same reasons that A is correct. Since was no "search." Viewing a barn is not
Russ is not a member of a suspect class, a search, since no significant privacy
and no facts indicated any other Equal expectation was invaded. United States
Protection issue, C is clearly wrong. D v. Dunn (1987). Since there was no
is incorrect because Russ' confession emergency, A is incorrect. Since no
has absolutely nothing to do with his warrant was obtained initially, the good
own rights of confrontation. faith exception (choice B) is incorrect.
Since J.G.'s "search" was not related to
19. D is the best answer. In order for there public safety, choice C is incorrect.
to be a violation of the right to effective
counsel, there must be attorney error 22. D is the best answer. Although the facts
and prejudice. Strickland v. Washing- do not indicate whether the dumpster
ton, (1984). On these facts, there was was outside the home, a fair reading
almost certainly attorney error (not of the facts so indicates. Thus, under
making an objection to the admission), California v. Greenwood (1988), there
but no facts indicate prejudice. Thus, D was no "search" for Fourth Amendment
is phrased correctly, and A is incorrect. purposes. Although this answer may not
Since no facts whatsoever indicate an necessarily be "correct," it is the best
issue of suspect class, choices B and C answer because choices A, B and C
are incorrect. are clearly wrong. A is wrong because
there was no emergency. B is wrong
because the good faith exception is
not applicable to warrantless searches.
Choice C is wrong because this was not
a public safety search.

278 CRIMINAL PROCEDURE


23. A is the correct answer. Warrants require 26. A is the correct answer. Under the Wong
judicial/magistrate approval and such is Sun rule, since the seizure of the car was
either by signature or seal. B is incor- unlawful, the evidence so discovered is
rect because J.G's discoveries were not to be suppressed. B is incorrect because
protected by the Fourth Amendment, the exclusionary rule is applicable to

A
since they were not "searches." C is misdemeanors. C is incorrect because
incorrect because warrants definitely the inventory exception requires lawful
need judicial approval. D is incorrect police possession of the property. D is
because, although the fruits of a search incorrect because the good faith excep-
performed in good faith reliance on a tion requires there to be a warrant.
warrant later deemed defective does not
require suppression of that evidence, 27. B is correct. An officer can enter a
the warrant, itself, is nevertheless still dwelling, if he has probable cause
defective. that there is an emergency. A scream
furnishes such a reasonable belief. A
24. B is the correct answer. Good faith reli- is incorrect because good faith alone,
ance on a warrant later deemed defec- without a warrant, is not a recognized
tive does not mandate suppression of exception. C is incorrect because in no
the evidence obtained. Massachusetts way did Sarah or her guest consent. D
v. Sheppard (1984). Although A is is incorrect for the same reason that B
legally correct, it misreads the facts. is correct. Cf. Welsh v. Wisconsin
This signatureless warrant was invalid.
Although C is legally correct, it is es- 28. D is the best answer. Naked eye sur-
sentially irrelevant since no statute was veillance, when the officer is lawfully
stated in the facts. D is incorrect for the positioned is not a search for Fourth
same reasons that B is correct. Amendment purposes. Cf. California
v. Ciraolo (1986). A is incorrect be-
25. D is the correct answer. In order to cause the observation was not related
utilize the good faith exception to the to the apparent exigency. B is incorrect
exclusionary rule, the officer must be because neither Sarah nor her guest
in objective good faith. Since both his consented/assumed the risk. C is in-
actual knowledge and his training in- correct because the brass case was not
dicate "bad faith," this exception is not "obviously incriminating." Cf. Arizona
available. Massachusetts v. Sheppard v. Hicks (1987).
(1984). Thus, A is incorrect. B is incor-
rect because no evidence whatsoever
indicates that the evidence would have
been discovered. C is incorrect because
there is no evidence whatsoever of any
independent source or sources.

CRIMINAL PROCEDURE 279


29. B is the correct answer. Although Slo- 32. B is the correct answer. The Skinner
bog was lawfully positioned, there was and Von Raab cases (1989) indicate be-
not probable cause that the brass case cause of the widespread drug problem
contained cocaine. Thus, the plain view and public safety, testing of railroad
exception is unavailable. Arizona v. employees associated with an accident
Hicks (1987). The evidence was illegal- and customs agents seeking promotion
ly discovered. A is incorrect because the to be lawful, under the administrative
mere fact that this was Sarah's "castle" exception. By logical analogy, police
does not invalidate the search and sei- officer testing is most probably accept-
zure provided there was a recognized able. A is incorrect because there is no
exception. C is incorrect because there drug exception to the warrant require-
is no general good faith exception if ment. C is incorrect because there was
there is no warrant obtained prior. D is no emergency. D is incorrect because
incorrect because no facts indicate the examination of bodily fluids is both a
evidence would have inevitably been search and a seizure. Skinner v. Railway
discovered. Labor Executives Association (1989).

30. C is the best answer. An invited over- 33. D is the correct answer. If any jail time is
night guest has standing to object to the to be served, counsel must be provided
search of his property in the premises. to the indigent. Scott v. Illinois (1979).
Minnesota v. Olson (1990). This answer Thus, A and B are legally incorrect. C
would have been better had it men- is incorrect because no facts indicated
tioned the illegality of the search, but Kim not to be indigent.
it is nevertheless the best of the listed
alternatives. A is incorrect for the same 34. C is the correct answer. The jury trial
reason that C is correct. B is incorrect right attaches only when the sentence
because there is no general warrantless is potentially over 6 months. Blanton v.
good-faith exception. D is incorrect North Las Vegas (1989). Thus, choices
because there is no LaFave doctrine. A, B and C are incorrect.

31. C is the best answer. An overnight guest


has standing to object to search of the
premises. Minnesota v. Olson (1990).
Had the answer included that the search
was illegal, it would have been better,
but this is the best alternative offered. A
is incorrect for the same reasons that C
is correct. B and D are incorrect because
there are no such doctrines.

280 CRIMINAL PROCEDURE


35. D is the correct answer. Although coun- 36. B is the correct answer. As a matter of
sel need not be given to the indigent if law, Conspiracy and its target offense
no jail is imposed, such a conviction can (on these facts, Robbery) are distinct of-
be used to enhance a later conviction. fenses, under Double Jeopardy analysis.
Nichols v. U.S. (1994). Since retrial for United States v. Felix (1992). A is ob-

A
the same crime is not at issue, double viously incorrect because the multiple
jeopardy (choice A) is irrelevant. Since crimes were not prosecuted in separate
prosecution and conviction for prosti- jurisdictions. Thus, the separate sov-
tution has never been considered to be ereign limitation on Double Jeopardy
cruel and unusual punishment, choice protection is inapplicable. C is incor-
B is obviously incorrect. Choice C is rect because Conspiracy requires proof
incorrect for the same reason that choice of the additional fact that there was an
D is correct. agreement; and Robbery requires proof
of an additional fact that the crime was
completed (not just agreed to). Thus,
utilizing the traditional Blockburger
analysis, these are separate offenses.
However, there is more to Double
Jeopardy protection than just this test.
Answer D correctly sets out the addi-
tional legal principle regarding Double
Jeopardy protection. A "lesser included
offense" is an offense in which all of the
written elements of one crime appear in
the other. Thus, on these facts, Robbery
is a lesser included offense of Conspir-
acy to commit Robbery. (Conspiracy
is defined here as the meeting of the
minds of two or more people with the
intent to commit Robbery.) D correctly
states this general rule of law, that lesser
included and greater inclusive offenses
are the same offense, in Double Jeop-
ardy analysis. But, the exception to this
rule is regarding Conspiracy — which
is the fact-pattern of this case. D is
therefore incorrect because it ignores
the Felix exception, supra.

CRIMINAL PROCEDURE 281


37. C is the correct answer, because only 38. D is the correct answer. The call of the
Choices III and IV are correct. Tennes- question queried as to what was the
see v. Garner (1985) held that a police prosecutor's best argument for allow-
officer can use deadly force on a fleeing ing in the evidence of Lou's marijuana
felon only when there is a present public possession. Since it was stipulated
danger. If deadly force is used when that another officer, Smart, was on the
there is not such a danger, the Fourth verge of independently discovering the
Amendment (as applicable to the states marijuana, the best argument is "inevi-
by the Fourteenth Amendment Due table discovery". This exception to the
Process Clause) mandates exclusion Exclusionary Rule allows introduction
of all evidence discovered (absent an of illegally seized evidence (here by
exception to the Exclusionary Rule). Knuckles' coercion), if the prosecution
On the facts given, Wing posed no shows, by a preponderance, that the
public danger. No facts indicated him evidence would inevitably have been
to be armed or in any way dangerous discovered. Nix v. Whiteside (1984).
to Officer Breeze or the public. Thus, Although the facts do not utilize the
there was a clear Fourth Amendment precise legal standard above, never-
violation, via this illegal seizure of theless, the stipulation indicates the
the person. Since Wing obviously has likely applicability of this exception.
standing (it was his own body which A is incorrect because the good faith
was seized) and since, on these facts, exception to the Exclusionary Rule
no exception to the Exclusionary Rule requires there to be a warrant. No facts
(such as inevitable discovery) appears indicated there to be a warrant in this
relevant, all evidence derived from situation. B is incorrect because, in
the illegal seizure must be suppressed. fact, there was no independent source.
Choice III is therefore correct because Officer Smart had not yet discovered
evidence of the Petty Theft was the the drugs. There was only a hypotheti-
product of the illegal seizure. Choice IV cal independent source. The name for
is correct because evidence of Resist- such is "inevitable discovery." Thus, B
ing a Police Officer is also the direct is incorrect for the same reasons that
product of the illegal seizure. Choices I D is correct. C is obviously incorrect
and II are incorrect for the same reasons because the Fourth Amendment and
that Choices III and IV are correct. its jurisprudence is fully applicable to
the states, by way of the Due Process
Clause of the Fourteenth Amendment.

282 CRIMINAL PROCEDURE


39. C is the correct answer. Although a 40. C is the correct answer because only
search warrant allows the detention of choices I and III are correct. Choice I
the lawful possessor of the premises to is obviously correct because the lan-
be searched, it does not allow the deten- guage of the Fourth Amendment, itself,
tion of those merely present at the situs requires probable cause, in order that a

A
of the search. Ybarra v. Illinois (1979). warrant be issued. Choice III is correct
Therefore, the detention of Ebar's two because items observed in "plain view",
friends was in violation of the Fourth while lawfully executing a warrant, can
Amendment, and evidence discovered be seized. This is a traditional exception
.t1 0
as a result, is to be suppressed. A is to the warrant requirement, and the facts O73
incorrect because the lawful possessor meet the requirements of this excep- 0
m
of premises to be searched can be de- tion (probable cause and object was cy z
tained. Such is an inherent power of a obviously incriminating). See generally rrl
search warrant. Michigan v. Summers Horton v. California (1990). Choice II
(1981). B is incorrect because, although is incorrect because the test for whether
the detention of Ebar's friends was there was probable cause, via a confi-
unlawful, such does not aid Ebar in dential informant's tip, is no longer the
any way. "Mere" police illegality, not two-pronged Aguilar-Spinelli test, but
directly related to evidence discovered is rather, simply, whether the totality of
against the defendant (here Ebar), in no the circumstances indicate there to be
way mandates suppression of evidence probable cause of crime, contraband
against the defendant. Fourth Amend- or evidence of crime. Illinois v. Gates
ment rights are personal. D is incorrect (1983).
for the same reasons that C is correct.

CRIMINAL PROCEDURE 283


41. B is the correct answer. Administrative 42. C is the correct answer. Although it was
searches are acceptable even without an stipulated in the facts that there was a
administrative ("area") warrant, when Miranda violation, the U.S. Supreme
such are based on a valid public policy, Court has indicated that most trial
there is a regulation allowing such and errors, provided that the prosecution
the area to be searched is considered to proves them to be harmless beyond a

A be a "closely regulated" business. The


facts of New York v. Burger (1987)
are directly analogous to the facts of
reasonable doubt, will not require the
setting aside of the finding of guilt.
Chapman v. California (1967). A is
this case. In Burger, junkyards were incorrect because it misreads the facts
considered closely regulated, there was an error was stipulated. Choices B and
a regulation allowing such, the search D are incorrect because they use the
was by a police officer and criminal wrong evidentiary standard.
prosecution resulted. It was held to be
no Fourth Amendment violation. Such 43. D is the correct answer because choices
was a valid administrative search. A is A, B and C are all incorrect. If an
incorrect because no warrant was is- arrestee merely invokes his right to
sued, under the given facts, and such silence, after being given the Miranda
is a requirement in order to utilize the warnings, then he can be reinterrogated.
good faith exception. Moreover, A is [There is the contrary answer, when the
also incorrect because the good-faith attorney right is invoked. Edwards v.
exception is to the Exclusionary Rule, Arizona (1981)] Michigan v. Mosley
it is not an exception to the warrant re- (1975). Answer A is obviously incorrect
quirement. C is incorrect for the same because, although the U.S. Supreme
reasons that B is correct. D is incorrect Court may appear to limit the protec-
for two reasons: First, no facts indicate tions in the Bill of Rights, particularly
Lemon to have acted in bad faith. Sec- in drug cases, this is certainly not the
ondly, the bad faith of the involved offi- stated law. B is incorrect for the same
cer is not relevant to the acceptability of reason that D is correct. C is incorrect
an administrative search. The elements for two reasons: First, there is no law
required of a proper administrative indicating factually voluntary admis-
search were set out above... and do not sions to be per se involuntary; secondly,
include good faith. a suspect not being told of the crimes for
which he is being interrogated, is of no
legal significance. Colorado v. Spring
(1987).

284 CRIMINAL PROCEDURE


44. A is the correct answer. Only when a 45. C is the correct answer. Once an in-
suspect has been formally charged with dividual is formally charged, here by
a crime (not just arrested), does the a grand jury, he possesses the Sixth
Sixth Amendment Right to Counsel at- Amendment Right to Counsel. Not
tach. If such attaches, an attorney must only must he be so informed (Patterson
be present during the post-charging, live v. Illinois (1988)), but if he invokes
identification procedure. If the attorney
has not been invited, the out-of-court
identification will be suppressed.
such, the police can neither directly
nor indirectly (via use of an infor-
mant) interrogate him on the charged
A
United States v. Wade (1967). Since crime. Massiah v. United States (1964).
there was no formal charging here, the However, unlike the Fifth Amend-
identification is likely admissible. B is ment Miranda protection, the Sixth
incorrect, for the same reasons that A is Amendment protection extends only to
correct. Choices C and D are incorrect interrogation regarding the indicted/ar-
because only extraordinarily unfair or raigned crime. It is "offense-specific".
"suggestive" visual identification pro- McNeil v. Wisconsin (1991). Choice
cedures violate Due Process. Although C incorporates the above principles,
a one-on-one identification (known as i.e., only the information, regarding
a "showup") is somewhat suggestive, the unindicted crimes, is admissible.
it is the extraordinarily rare case where A and B are partially incorrect in that
such is deemed to be a constitutional the information garnered regarding the
violation. On the facts of this case, the unindicted offenses is admissible. Hoffa
proximity of the identification with the v. United States (1966). D is incorrect
alleged crime, and the sureness of the for the same reasons that C is correct.
alleged victim, undoubtedly indicate the
identification to be admissible. Neil v. 46. C is the correct answer. The Sixth
Biggers (1972). Amendment Right to Counsel attaches
to indigents only when actual jail time
is imposed, or if the prosecution is for
a felony. Gideon v. Wainwright (1963).
Thus, if, as here, no jail is imposed,
there has been no constitutional viola-
tion. A is incorrect in that it confuses
the Jury Trial right, with the Counsel
right [which does attach when the jail
potential is over 6 months]. B is an
incorrect statement of law. Nichols v.
U.S. (1994). D is incorrect for the same
reason that C is correct.

CRIMINAL PROCEDURE 285


47. D is the correct answer. Pate v. Robin- 48. B is the correct answer. With few excep-
son (1966) indicated that trial judges, tions, the Sixth Amendment Confronta-
on their own motion, must insure that tion Clause, as applied to the states by
unless the defendant is mentally com- the Fourteenth Amendment Due Pro-
petent, he cannot be criminally tried. cess Clause, mandates that defendants
On the facts given, such an inquiry is be allowed to confront (cross-examine)
clearly warranted. Delusions indicate a adverse witnesses. Pointer v. Texas
substantial possibility of incompetence, (1965). Had Joyce not taken the stand,
i.e., the inability to aid in the defense, on her own volition, the answer would
and disorientation as to the facts of the have been different. A defendant waives
crime and reality (see Dusky v. United her Fifth and Fourteenth Amendment
States (1960)). Self-Incrimination protection by vol-
untarily taking the stand at the trial.
On the facts given, it was an unconsti- She cannot take the stand, testify on her
tutional violation of Fourteenth Amend- own behalf and then refuse to answer
ment Due Process to not require psychi- questions on cross-examination which
atric evaluation regarding competency are reasonably related, as here, to her
prior to the trial. A is incorrect for the testimony on direct. Brown v. United
identical reasons that D is correct. B States (1958). For these reasons and
is incorrect because the issue at hand case-holding, choice A is incorrect. C is
is not whether the defendant is Insane incorrect for similar reasons. Although
[because of mental disease he did not two constitutional rights do conflict here
know the wrongfulness of his actions]. (Confrontation and Self-Incrimination),
Insanity is an affirmative defense. It the Supreme Court has struck a reason-
has a different test than competency. able balance, via the above rulings. D
That is, a defendant may be sufficiently is incorrect for the same reasons that B
oriented and possess sufficient ability is correct.
to aid in his own defense and still not
know what he was doing was wrong.
Furthermore, Insanity queries: what
was the defendant's state of mind at the
time of the crime, itself. Competency
queries: what was the defendant's state
of mind at the time of trial. C is factually
incorrect because there are sufficient
facts indicating extreme mental disor-
der, and, thus, a real possibility of in-
competence. The inquiry must be made,
via pre-trial psychiatric examination.

286 CRIMINAL PROCEDURE


49. C is the correct answer because both 50. D is the correct answer. When a Habeas
choices I and II are relevant inquiries. Corpus petitioner does not object to the
The Eighth Amendment Cruel and admissibility of certain evidence at the
Unusual Punishment Clause, as applied appropriate time (usually this is pre-tri-
to the states by the Fourteenth Amend- al, but it could be at trial depending on
ment Due Process Clause, mandates the state's rules), then she is deemed
that methods of execution not be both
"cruel" and "unusual". See Robinson
v. California (1962) (which indicated
to have "procedurally defaulted" (by
failing to contemporaneously object). In
such instance, in order for this defaulted
A
this aspect of the Eighth Amendment claim to be heard on Habeas (collateral
to be state-applicable); and Weems v. appeal), she must show good cause
United States (1910) (which indicated why such was not brought up at the
that a violation of the Clause requires appropriate time, and also must show
that the punishment be both "cruel" prejudice to her case. Wainwright v.
and "unusual"). Choice I is therefore Sykes (1977). Since Miranda is and has
correct because it addresses the issue been settled law, Karen will be unable
of whether death by dismemberment is to show "cause" why she did not bring
an "unusual" practice. Choice II is cor- up this claim at the appropriate time. A
rect because it addresses the question is incorrect because, in not granting the
whether death by dismemberment is petition, the court is correctly following
unduly "cruel". A is not the best answer the above law. Such is not error at all. B
because it only includes Choice I. B is is incorrect because, even though there
not the best answer because it only in- was a Miranda violation, such must be
cludes Choice II. D is incorrect for the claimed at the appropriate time or is
same reasons that C is correct. generally forfeited. C is a correct state-
ment of fact. Karen's counsel was likely
ineffective. However, ineffectiveness of
counsel is its own Habeas claim. In fact,
it is the better claim here. Nevertheless,
the facts stipulated that such was not
the argument of her Habeas petition.
Therefore, choice C is irrelevant to the
actual claim which Karen made.

CRIMINAL PROCEDURE 287


51. D is the best answer. A is incorrect 53. C is the correct answer. Under the cir-
because Aire Head, a student, has no cumstances, the U.S. Marshal should
reasonable expectation of privacy, since have secured a search warrant upon
Mr. Locke, a school official, is merely sworn information, describing with
investigating whether a violation of a reasonable particularity the thing for
school infraction has occurred. D is which the search was to be made. By
correct because it is a better answer not doing so, the Marshal acted "with-
than B and C, since it specifically deals out sanction of law" in direct violation
with the concept of "reasonableness." of the constitutional prohibition action
In New Jersey v. T.L.O., 469 U.S. 325, of action. (See Weeks v. United States,
105 S.Ct. 733 (1985), it was held that 232 U.S. 383, 34 S.Ct. 341 (1914)). As
a school official's intrusion into the such, B is clearly wrong. A is wrong
privacy of a student was "reasonable" because law enforcement officials can-
since it was necessary to achieve the not further the government's case in
legitimate end of preserving order in direct violation of the constitution. D
the schools. Here, since Mr. Locke was is incorrect because a defendant is not
merely investigating whether a school necessarily entitled to the return of his
rule had been violated, his searching property merely because his application
of the purse for cigarettes would be for its return was seasonal.
deemed "reasonable."
54. D is the best answer. A and B are wrong
52. D is correct because A, B and C are for the reasons set forth in 53 above.
irrelevant. In Hudson v. Palmer, 468 Though C could be correct, D is more
U.S. 517 (1984), the Supreme Court specific as to the reason why the gov-
concluded that the requirements of ernment should not be permitted to use
prison security make it unreasonable Mac's property at trial.
for prisoners to expect protection for
their privacy. 55. D is the best answer. In Mapp v. Ohio,
367 U.S. 643, 81 S.Ct. 1684 (1961),
it was held that "the right to privacy
embodied in the Fourth Amendment
is enforceable against the States, and
that the right to be secure against rule
invasions of privacy by state officers
is, therefore, constitutional in origin."
A, B and C are wrong because they are
incorrect statements of law.

288 CRIMINAL PROCEDURE


56. D is the correct answer. Absent other 59. D is the best answer. A is incorrect
facts, the mere smell of breath mints because the trash cans were not on the
does not mean the driver is intoxicated. curb, but on Middo's home property,
For that reason, A is incorrect. B is therefore he does have a privacy ex-
incorrect because it is irrelevant what pectation. Cf. California v. Greenwood

A
was found in the face of an unlawful (1988). B is wrong because the facts do
search. C is incorrect because there are not indicate an emergency. C is wrong
no facts which give rise to this excep- because the good faith exception does
tion. Moreover, the inventory exception not apply to warrantless seearches (and
requires lawful police possession of the is also wrong because relying on a
property. clearly defective warrant is not reason-
able good faith). D is therefore the best
57. A is the correct answer. Crumpky had choice.
no probable cause to search Gloss' car.
The rear of the car could have been low 60. D is the correct answer because all
because she was transporting bags of warrants require judicial/magistrate ap-
fertilizer home to do gardening work. proval indicated by either signature or
A low rear-end does not necessarily seal. As such, A, B and C are wrong.
speak of illegal activity. B is wrong for
the same reasons C in question 56 is 61. A is the best answer because a warrant
wrong. C is an incorrect statement of secured in bad faith which is subse-
law. (See One 1958 Plymouth Sedan quently deemed invalid will keep the
v. Commonwealth of Pennsylvania, evidence found pursuant to the warrant
380 U.S. 693, 85 S.Ct 1246 (1965)). from coming in. (See Massachusetts v.
D is irrelevant because the car is not Sheppard (1984)). B is also correct but
contraband. is not as fact specific as A. C and D are
wrong because A is correct.
58. D is the correct answer. Viewing a
greenhouse is likely not a search as con- 62. D is the correct answer because A, B
templated by the Fourth Amendment. and C are wrong. A is wrong because
No privacy expectation was invaded. the facts indicate that Palmer was act-
(See United States v. Dunn (1987)). A is ing in bad faith. B is wrong because
incorrect; off-duty police officers may there are no facts to indicate that other
engage in law enforcement activity if evidence would have been discovered.
the situation so warrants. B and C are C is wrong because a warrant, under the
nonsense. circumstances articulated in the answer,
is required despite the incriminating
nature of the evidence already discov-
ered.

CRIMINAL PROCEDURE 289


63. C is the correct answer because "it [is] 66. B is the correct answer and is a better
incumbent on the suspect to preserve answer than A because it is more fact
his privacy from visual observation." specific than A. C is wrong because
(See Commonwealth v. Hernley, 216 helicopters are permitted to fly at any al-
Pa.Superior Ct. 177, 263 A.2d 904 titude only if the operation is conducted
(1970), cert. denied, 401 U.S. 914 without hazard to persons or property
(1971)). As such, A, B and D are incor- on the surface. (See Commonwealth v.
rect. Johnson, supra). D is wrong because it
is irrelevant that the marijuana was clear
64. D is the correct answer because there and visible when the search was unrea-
"can be no expectation of privacy when sonable and violative of constitutional
one leaves his windows open or uncov- requirements prohibiting unreasonable
ered so that any passerby might peer searches and seizures, since hazard to
into one's dwelling with impunity, and persons or property on the surface oc-
the same result should apply regard- curred.
less of where the window or opening
is placed in the dwelling." (See Com- 67. C is the correct answer because Bob
monwealth v. Pennsylvania, 525 Pa. exposed his garbage to the public suf-
250, 579 A.2d 1288 (1990), where the ficiently to defeat his claim to Fourth
expectation of privacy did not attach Amendment protection. (See California
to a translucent rooftop). As such, A, B v. Greenwood, 486 U.S. 35, 108 S.Ct.
and C are wrong. 1625 (1988)). A is wrong because the
Fourth Amendment does not prohibit
65. C is the correct answer because failure the warrantless search and seizure of
of a defendant, or those with him, or garbage left for collection outside the
those employed by him, to close his curtilage of a home. B is wrong for the
premises to the world negates defen- same reason as A, even though Bob's
dant's argument that he was deprived trash bags were opaque and securely
of his privacy. (See Commonwealth v. tied. D is wrong because A and B are
Johnson, 247 Pa.Superior Ct. 208, 372 wrong.
A.2d 11 (1977)). As such, A, B and D
are wrong.

290 CRIMINAL PROCEDURE


68. D is the correct answer. As articulated 70. A is the correct answer. "A chemical
in California v. Greenwood, supra, the test that merely discloses whether or
(black) plastic bags involved here are not a particular substance is cocaine
protected from warrantless searches does not compromise any legitimate
when used to store or transport personal interest in privacy." (See United States

A
possessions. Since there are no facts v. Jacobsen, 466 U.S. 109, 104 S.Ct.
indicating that the officer had reason to 1652 (1984)). B is wrong because the
believe that the specific bags on Bob's federal agent did not infringe any con-
porch contained anything illegal inside stitutionally protected privacy interest
them, the officer, thus, had no right that had not already been frustrated as
to search through them. As such, this the result of private conduct (Federal
warrantless search and seizure would Excess employees). C is wrong because
be prohibited by the Fourth Amend- a field test can destroy some of the
ment. For the foregoing reasons, C is evidence, not all of it, and still render
wrong. the search and seizure valid. The facts
do not indicate all of the evidence was
69. C is the correct answer. Frodd "is in destroyed. As such, the seizure was not
no better position than the citizen who unreasonable. D is wrong because both
merely tears up a document by hand and B and C are wrong.
discards the pieces on the sidewalk."
(See United States v. Scott, 975 F.2d 71. D is the best answer and is better than
927 (1st Cir.1992)). Thus, A, B and D C because it characterizes the actions of
are wrong because the Fourth Amend- the agents as having gone beyond the
ment does not prohibit the warrantless scope of the private search, e.g., Ms.
search and seizure of garbage left for Prude's mere opening of the package
collection outside one's home. versus the viewing of the videos inside
the package. (See Walter v. United
States, 447 U.S. 649 (1980)). As such,
A and B are wrong.

72. A is the correct answer. By procuring


the hash for his personal use, John was
not furthering a governmental objective
and, thus, his actions were not governed
by the Fourth Amendment. (See United
States v. Smith, 810 F.2d 996 (10th Cir.
1987)). As such, D is wrong and B and
C are irrelevant.

CRIMINAL PROCEDURE 291


73. Though inexplicit, C is the best answer. 75. D is the correct answer. Mary only
The central queries with the Fourth gave consent to the officers because
Amendment ... and its consent excep- she believed they had the authority
tion to the warrant requirement, is who to enter her home. Moreover, even if
has standing to consent and whether she hadn't given consent, she most
the police acted in an objectively rea- likely believed the officers would have
sonable manner. Certainly a butler has entered her home anyway under the
apparent authority to consent. Illiinois power of the warrant. (See Bumper v.
v. Rodriguez (1990). Thus choice B North Carolina, 391 U.S. 543 (1968)).
is incorrect—the butler has apparent As such, A, B and C are wrong. While
authority to allow entry. Additionally, it might appear C could be correct under
the focus is on whether the police over- the good- faith exception, that excep-
reached, i.e., used intimidation. See tion only applies when an officer(s) has
Florida v. Bostick (1991). Since the physical possession of a warrant later
facts indicate they did not, the consent deemed defective. There are no facts
is voluntary and valid. Thus, although here to indicate, either in the fact pat-
arguable, choice A, which emphasizes tern or answer choice C, that an actual
the subjective mental state of the butler, warrant existed.
is not supported by case law. D is incor-
rect for the same reasons that A and B 76. D is the best answer as such probation
are incorrect. conditions are held to be valid in some
jurisdictions. (See State v. Gawron, 736
74. A is the best answer. Hotel management P.2d 1295 (Idaho 1987). C is a poten-
or staff is entitled to retake possession tially true statement; however, there
of a room after check out time because, is no case law taking this view. A and
at this time, the hotel guest no longer B are wrong because the fact pattern
has standing. (See Stoner v. California, involves a probationer.
376 U.S. 483 (1964)). While B is a true
statement of law, A is the better answer 77. C is the correct answer. Without invok-
for the reasons discussed above. C is ing any concept of consent, the search
not necessarily true because there are of a probationer by an officer can be
times when a third party can give con- upheld as a "regulatory search" justi-
sent. D is wrong because A is the better fied by the special needs of effective
answer. probation supervision. (See Griffin v.
Wisconsin, 483 U.S. 868, 107 S.Ct.
3164 (1987)). A and B are wrong be-
cause Malcolm is a probationer. D is
wrong because A and B are wrong.

292 CRIMINAL PROCEDURE


78. C is the correct answer. Many cases 81. C is the correct answer. The court in
have held that there is no illegal entry Bumper v. North Carolina, 391 U.S.
when a police officer poses as a drug 543 (1968), "upheld the voluntariness
purchaser to gain consensual entry to of consent despite the deceptive prac-
the premises of a drug dealer. (See tices by government agents." In short,

A
Lewis v. United States, 385 U.S. 206 officers can disclose their identity but
(1966)). As such, answers A and B are misrepresent their purpose. (See also
wrong. D is wrong because C is more State v. Johnson, 253 Kan. 356, 856
fact specific as to why Tommy allowed P.2d 134 (1993)). Here, because Bust's
behavior was not threatening or coer-
730
Bust to enter his home.
cive, and Tommy was willing to let Bust OE
79. B is the correct answer. Officer Bust's
additional intrusions into the drawers
enter, and Bust did not exceed the scope
of the consent, the evidence would be
cy
mr—
and envelopes while Tommy was in admissible. As such, A and B are wrong.
the bathroom could not be justified on D is a correct answer, but C is better
the basis of consent, since Tommy only because it sets forth the reasons why
consented to let Officer Bust inside his the motion will fail.
home, not his drawers and envelopes.
A is wrong because of the additional 82. D is the correct answer. The warrant will
intrusions made by Officer Bust. C is be found valid because it was based on
wrong for the reasons articulated in the reasonable belief that there was only
answer 78. D is nonsense. one apartment on the fifth floor, and
the search pursuant to that warrant was
80. A is the correct answer. Under Lewis lawful because the officers reasonably
v. United States, supra, concealment believed they were searching Dump's
of one's status as a law enforcement apartment when they discovered the
officer does not invalidate the consent, two kilos of hash and the drug scale.
at least in the case of a transaction oc- (See Maryland v. Garrison, 480 U.S.
curring between the "officer" and the 79 (1987)). As such, A, B and C are
defendant that is understood by both wrong.
parties to be illegal. The result would
likely be different if, as here, the police
officer gained entry by pretending to
be a telephone technician or gas-meter
reader. In such a scenario, there is no
understanding that an illegal transac-
tion is about to take place since the
defendant believes the officer to be a
telephone technician and not a person
there to buy drugs. As such, B, C and
D are wrong.

CRIMINAL PROCEDURE 293


83. A is the correct answer. Hill v. Califor- 85. D is the correct answer. If an officer
nia, 401 U.S. 797 (1971), upheld the knows that a suspect(s) is armed and
lawfulness of an arrest that turned out dangerous, the officer may enter with-
to be a case of mistaken identity. Since out giving advance notice of her author-
Officer Jones' had a reasonable belief ity and purpose. Since Stonelli was a
that Howard Limbo was the suspect known Mafia hitman listed as "armed
at the time the search was made, his and dangerous," the actions of Kent,
actions will be deemed lawful, even Camel and Kool were proper. As such,
though his reasonable belief turned out A, B and C are wrong.
to be mistaken. As such, B is wrong.
C is wrong because Officer Jones had 86. D is the correct answer. Officers are
sufficient reason to search the pockets allowed to "look in closets and other
of Howard Limbo, since Limbo was spaces immediately adjoining the place
wearing almost exactly what the suspect of arrest from which an attack could im-
was wearing. D is wrong because it is mediately be launched." (See Maryland
irrelevant. v. Buie, 494 U.S. 325 (1990)). Since
Stonelli was known to the officers
84. D is the best answer. In some jurisdic- to be "armed and dangerous," it was
tions, unreasonable delay in acting reasonable for the officers to believe
upon a warrant may make the arrest that one of Stonelli's associates could
unlawful. Since approximately 5 weeks indeed be armed and dangerous. As
had passed before Thomas and Hill ex- such, the officer's search, limited to the
ecuted the warrant, it could be possible living room and an adjoining hallway,
that the information the warrant was was valid. A and B are wrong because,
based on had become stale. Typically, under such circumstances, the arresting
warrants have to be acted upon within officers need neither probable cause nor
10 days, thus A is incorrect. B is incor- reasonable suspicion. (See Maryland v.
rect because we are not told if the facts Buie, supra). Thus, C is wrong.
are taking place in a state that requires
that a search warrant be executed in
daytime hours. C is wrong because un-
der the explanation for answer choice
A, a warrant can still be deemed invalid,
even if technically sound.

294 CRIMINAL PROCEDURE


87. C is the correct answer. A search 91. D is the correct answer. Such a deten-
could occur if "a reasonably prudent tion would likely prevent flight in case
officer believe[d] that the area to be grounds for arrest are found in the
swept harbors an individual posing a search, minimize the risk of harm to
danger to those on the arrest scene." the officers, and facilitate the orderly

A
(See Maryland v. Buie, supra). Such a completion of the search. (See Michi-
search would be supported and found gan v. Summers, 452 U.S. 692 (1981)).
reasonable because Stonelli is deemed A is wrong because there are no facts
by police files to be "armed and danger- indicating coercive behavior by the of-
ous." As such, A and B are wrong. D ficers, especially since the detainment
is wrong because such a search, under was temporary. B and C are wrong
the circumstances, would be deemed because D is correct.
lawful.
92. D is the best answer. The officer likely
88. D is the correct answer. The motion will had a basis for reasonable suspicion
fail because the search would not be but not probable cause. In the absence
invalidated merely because the officers of probable cause, the officer was not
knew about the weapons and hoped to entitled to inspect the CD player. Rea-
find them during the search. (See Hor- sonable suspicion is not enough. (See
ton v. California, 496 U.S. 128 (1990)). Arizona v. Hicks, 480 U.S. 321 (1987)).
As such, A and B are wrong. C is wrong A is wrong because, even though the
because evidence does not need to be officers lawfully entered the apartment,
discovered "inadvertently" for the plain they were not entitled to inspect the CD
view doctrine to apply. (See Horton v. player. B, though a true statement, is
California, supra). not as good an answer as D because the
issue is whether an unlawful search of
89. C is the correct answer. The facts do the CD player occurred. C is wrong as
not indicate the officers had probable discussed in the explanation of answer
cause to arrest any of the customers. D.
(See Ybarra v. Illinois, 444 U.S. 85
(1979)). As such, A and B are wrong. 93. C is the correct answer, under United
D is nonsense. States v. Beal, 810 F.2d 574 (6th Cir.
1987), which held that the seizure of
90. A is the correct answer. Such an action pen guns in plain view was unlawful be-
would likely be deemed unreasonable cause it was not immediately apparent
to protect the safety of the officers. (See that these items were contraband. A is
Ybarra v. Illinois, supra). The mere pro- wrong because it contradicts the correct
pinquity to crime does not allow search answer. B is wrong because answer B
of all of those near. As such, B, C and is part of answer A. D is wrong because
D are wrong. C is correct.

CRIMINAL PROCEDURE 295


94. A is the correct answer. "The fact that 97. C is the correct answer. An officer can
[the officers] took the opportunity pro- investigate suspicious circumstances
vided by [Johnny Boy] to obtain an oral without triggering the need to give
confession is quite consistent with the Miranda warnings. Such roadside ques-
Fifth Amendment." See Connecticut v. tioning would likely be deemed a Terry
Barrett, 479 U.S. 523 (1987)). As such, stop. (See Berkemer v. Mc Carty, 468

A B, C and D are wrong and only serve as


distracters.
U.S. 420 (1984)). A is wrong because
questioning by a police officer is not
always custodial interrogation. B is
95. C is the correct answer. Nine officers wrong because, as discussed in the ex-
in a suspect's bedroom would likely be planation for answer C, Johnny can be
found to create a potential for compul- detained for suspicion of drunk driving.
sion equivalent to interrogation at the D is wrong because O'Hair only needs
police station. (See Orozco v. Texas, a good-faith belief Johnny was drunk.
394 U.S. 324 (1969)). A is wrong be-
cause individuals can be in custody 98. A is the correct answer. The record
outside the police station. B is wrong indicates that Youbanks was not asked
because a suspect can be in custody in any questions by the judge concerning
his own home. D is wrong because C his guilty plea, nor did Youbanks ad-
is more fact specific in that it explains dress the court. As such, "the record
why a reasonable person would believe does not disclose that the defendant
this was an interrogation. voluntarily and understandingly entered
his pleas of guilty." In other words, the
96. C is the correct answer. If the invitation record indicates that the judge accepted
was the kind that could not be refused, Youbanks' guilty plea without any af-
coercion would likely be found present. firmative showing that it was intelligent
If so, then a finding of custody is likely. and voluntary, since the judge did not
A is wrong because an individual isn't question Youbanks in regard to his plea.
necessarily in custody just because they Thus, his conviction can be reversed.
are at the police station. B is wrong (See Boykin v. Alabama, 395 U.S.
because it would depend on what kind 238, 89 S.Ct. 1709 (1969)). B is wrong
of invitation was extended to Johnny because reversible error can occur even
Boy. D is wrong because an individual if a defendant's conviction is based on
doesn't have to be under arrest to be in credible eye-witness testimony. C and
custody. D are wrong for the reasons discussed
in the explanation of answer A.

296 CRIMINAL PROCEDURE


99. A is the correct answer. Astro, the 100. A is the correct answer because "when
defendant, must demonstrate in his a plea rests in any significant degree on
habeas petition that there was a rea- a promise or agreement of the prosecu-
sonable probability that, but for the tor, so that it can be said to be part of
court-appointed attorney's errors, he the inducement or consideration, such

A
would not have pleaded guilty and promise must be fulfilled." Thus, even
would have insisted on going to trial. though a new and different prosecutor
Thus, if this was not alleged, no relief broke the government's promise, the
will be granted. (See Hill v. Lockhart, prosecutor's office has the burden of
474 U.S. 52, 106 S.Ct. 366 (1985)). B "letting the left hand know what the
is wrong for the reasons articulated in right hand is doing." (See Santobello
the explanation of answer choice A. v. New York, 404 U.S. 257, 92 S.Ct.
C is wrong because the United States 495 (1971)). As such, B, C and D are
Supreme Court has never held that the wrong.
United States Constitution requires the
State to furnish a defendant with infor-
mation about parole eligibility in order END OF ANSWERS
for the defendant's plea of guilty to be
voluntary. (See Fed. Rule Crim. Proc.
11(c); Advisory Committee's Notes on
1974 Amendment to Fed. Rule Crim.
Proc. 11, 18 U.S.C.App., p. 22). D is
wrong because Astro's mistaken belief
will have no bearing on whether he is
granted relief absent a showing that he
would have acted otherwise had he had
the correct information at his disposal.
(See explanation to answer A).

CRIMINAL PROCEDURE 297


298 CRIMINAL PROCEDURE
EVIDENCE - QUESTION BREAKDOWN

1. Foundation - Admissibility of 16. Character Evidence


Photographs
17. Lay Opinion Testimony
2. Character Evidence
18. Character Evidence - Civil Cases
3. Present State of Mind
19. Relevancy - Impeachment
4. Bias
20. Impeachment - Collateral Issue
5. Legal Relevancy
21. Hearsay Exception - Excited
6. Character Evidence Utterance

7. Hearsay/Exception - Present Sense 22. Admission


Impression
23. Hearsay Exception - Physical
8. Impeachment - Prior Inconsistent Condition
Statements
24. Admissibility - Settlement Offers
9. Subsequent Remedial Measures
25. Present Recollection Refreshed
10. Payment of - Medical and Similar
Expenses 26. Character Evidence - Defamation

11. Privileges - Physician/Patient 27. Marital Communication Privilege

12. Authentication 28. Impeachment - Extrinsic Evidence

13. Privileges - Attorney/Client 29. Admission

14. Privileges - Attorney/Client 30. Character Evidence

15. Hearsay Exception - Present Sense 31. Privilege - Attorney/Client


Impression

EVIDENCE 301
32. Expert Testimony 50. Impeachment - Prior Inconsistent

33. Privileges 51. Impeachment - Prior Inconsistent


Statement
34. Subsequent Remedial Measures
52. Admissibility of Medical Expenses
35. Competency to Testify
53. Hearsay Exception - Physical
36. Settlement or Compromise Offers Condition

37. Subsequent Remedial Measures 54. Observations

38. Vicarious Admission 55. Privilege - Attorney/Client

39. Party Admission 56. Impeachment

40. Judicial Notice 57. Character Evidence

41. Specific Instance of Conduct - 58. Character Evidence


Extrinsic
59. Hearsay Rule
42. Self-Incrimination
60. Authentication
43. Impeachment
61. Personal Knowledge
44. Hearsay
62. Expert Testimony
45. Admission
63. Opinion Testimony
46. Hearsay Exception - Business
Records 64. Subsequent Remedial Measures

47. Hearsay 65. Impeachment

48. Adopted Admissions 66. Impeachment - Specific Instances of


Conduct
49. Hearsay Exception - Present Sense
Impression 67. Lay Opinion

302 EVIDENCE
68. Privileges - Attorney/Client 85. Hearsay Exception - Present Sense
Impression
69. Privileges - Attorney/Client
86. Hearsay Exception - Present Sense
70. Best Evidence Rule - Collateral Impression
Matter
87. Logical Relevancy
71. Impeachment - Extrinsic Evidence
88. Impeachment - Collateral Matters
72. Character Evidence - Specific Acts
89. Non-Hearsay
73. Impeachment - Conviction of a
Crime 90. Not-Hearsay - Command and
Questions
74. Settlement Offers
91. Not-Hearsay - Machine or Animals
75. Privileges - Attorney/Client
92. Hearsay
76. Subsequent Remedial Measures
93. Impeachment - Extrinsic Evidence
77. Payment of Medical and Similar
Expenses 94. Hearsay

78. Character Evidence - Prior Bad Acts 95. Character - Wrongful Death Action

79. Character Evidence 96. Character - Specific Instances of


Conduct
80. Expert Testimony
97. Best Evidence
81. Character Evidence
98. Impeachment
82. Privilege - Attorney/Client
99. Hearsay
83. Character Evidence
100. Authentication - Opinion Testimony
84. Character Evidence - Civil Cases
101. Non-Hearsay

EVIDENCE 303
102. Hearsay Exception Physical 106. Privilege — Marital Communication
Condition
107. Privilege — Attorney/Client
103. Character Evidence
108. Hearsay
104. Payment of Medical and Similar
Expenses 109. Hearsay

105. Privilege — Husband/Wife 110. Habit Evidence

304 EVIDENCE
m
EVIDENCE QUESTIONS Questions 2 - 5 are based on the
following facts. am
1. Petunia sued Daisy for injuries that she Mistress was tried for the October 31st
suffered when Daisy's bicycle collided murder of Wife.
with hers. The issue is whether Daisy
was riding her bicycle on the correct 2. Mistress calls Frieda as her first witness
side of the roadway. Petunia wants to to testify to Mistress' reputation in the
introduce a photograph that shows that community as a "peaceful woman." The
Daisy was riding her bicycle on the Court will rule the testimony:
wrong side of the road. Daisy objects
to the introduction of the photograph. A. Admissible, as it tends to prove
The Court will rule that the photograph that Mistress is believable.
is:
B. Admissible, as it tends to prove
A. Admissible, if there is testimony Mistress is innocent.
offered showing the photograph
to be an accurate representation C. Inadmissible, because Mistress
of the scene of the accident. has not yet testified.

B. Admissible, because Daisy can D. Inadmissible, because reputa-


cross-examine Petunia as to the tion evidence is inadmissible to
accuracy of the photograph. prove one's character.

C. Inadmissible, unless the photog- 3. Mistress calls Westley to testify that


rapher testifies to the accuracy of on October 30, Mistress said she was
the photograph. leaving that day to visit relatives in
another state. The Court will rule the
D. Inadmissible, unless the photo- testimony:
graph was taken by an investiga-
tive agent at the accident scene. A. Inadmissible, as hearsay not fall-
ing within any exception.

B. Inadmissible, because it is irrel-


evant.

C. Admissible, as non-hearsay.

D. Admissible, because it is a decla-


ration of Mistress' present state
of mind.

EVIDENCE 305
4. Mistress calls Wanda to testify to 6. Joe was robbed by a man carrying an
her alibi. On cross-examination, the unusual psychedelic-painted gun. Sam
prosecutor asks, "Aren't you Mistress' was arrested and charged with armed
Aunt?" The Court will rule the ques- robbery. At trial, the prosecution seeks
tion: to offer evidence that Sam robbed Pete
with the same gun. The Court will rule
A. Improper, because the question the evidence:
goes beyond the scope of direct
examination. A. Inadmissible, because Sam's
good character is at issue.
B. Improper, because the question
is irrelevant. B. Inadmissible, because this evi-
dence will unduly prejudice
C. Proper, as it tends to prove Sam.
bias.
C. Admissible, because it shows
D. Proper, because a relative cannot that Sam is capable of commit-
testify as to another relative's ting armed robbery.
reputation.
D. Admissible, as it tends to iden-
5. Mistress called Willy to testify as to her tify Sam as the man who robbed
alibi. On cross-examination the pros- Joe.
ecutor asks Willy, "Isn't it true that you
were on a jury that acquitted Mistress of
a criminal charge?" The Court sustains
Mistress' objection to the question. The
best reason is:

A. The question goes beyond the


scope of direct examination.

B. The probative value is out-


weighed by the prejudicial ef-
fect.

C. It is a leading question.

D. Prior jury service renders Willy


incompetent as a witness.

306 EVIDENCE
Q
m
Questions 7 - 10 are based on the 8. Joe's Bar and Grill calls Pete as a wit-
following facts. ness, expecting him to testify that he 5
m
was sober when he left. To their sur- z
Paul sues Joe's Bar and Grille for personal prise, Pete testified that he may have C)
m
injuries he received in an automobile acci- had a little too much to drink that night.
dent caused by Pete. Pete had been a patron Joe's Bar now wants to confront Pete
of Joe's Bar and Grille. Paul claims that Pete with a statement he made at his depo-
was permitted to drink too much liquor at the sition that he was sober when leaving
bar before the accident. that night. Paul objects. Which of the
following is the most likely ruling by
7. Wilma, a patron of the bar, testified that the court regarding Pete's statement in
Pete was drunk on the night of the acci- his deposition?
dent. Wilma testified that she remarked
to another patron, "Pete is so drunk, he A. It is inadmissible, because one
can hardly sit on his barstool." Wilma's cannot impeach his own wit-
testimony is: ness.

A. Admissible, as an excited utter- B. It is inadmissible, because it is


ance. hearsay.

B. Admissible, as a present sense C. It may be used to refresh Pete's


impression. memory.

C. Admissible, as a prior consistent D. It is admissible to impeach Pete


statement. and as evidence that Pete was
sober.
D. Inadmissible, because Wanda
had no expertise in determining 9. Paul then offers evidence that, after the
whether one is drunk. accident, the owner of Joe's Bar and
Grille put into effect a rule limiting all
patrons to 2 drinks per hour. The Court
will rule that this evidence is:

A. Admissible, to show that Joe's


Bar was negligent on previous
occasions.

B. Admissible, to show that Joe's


Bar knew precautionary mea-
sures were needed.

EVIDENCE 307
C. Admissible, because subsequent Questions 11 -12 are based on the
measures are binding on Joe's following facts.
Bar and Grille.
iu
Mario and Angelo, young hoodlums, agreed
D. Inadmissible, because subse- to "wack" a local competitor who was
quent remedial measures are muscling in on their boss' turf. If they were
not admissible to prove culpable successful, they would be inducted as "blood
conduct. members" into the mafia family. Mario and
Angelo successfully completed the "hit" on
10. Paul offers evidence that the owner of Monday, then Mario was to pick up their fee
Joe's Bar and Grille came to see him in from the boss himself. Mario was to meet
the hospital after the accident, and of- Angelo at a nearby purveyor of fine Italian
fered to pay all of his medical expenses. food, Pizza Madre, to split-up the money
Owner then stated, "That's the least I on Wednesday. On Tuesday, Angelo was
can do after letting Pete leave the bar so injured when he fell from the scaffolding
drunk." The statement made by owner, at his job as a painter. When he went to St.
in regard to Pete's drunkenness, is: Peters Hospital for examination, he was told
that he would have to stay at the hospital
A. Admissible, as an admission for a week because his arm and both legs
in connection with an offer to were broken. Angelo refused and said, "I
compromise. have to leave because I have to meet Mario
on Wednesday." Mario and Angelo were
B. Admissible, as a party admission charged with conspiracy.
by the owner that Pete was drunk
when he left the bar. 11. The prosecution offers the doctor's tes-
timony of Angelo's statement into evi-
C. Inadmissible, as hearsay not dence. Angelo objects. What result?
within any exception.
A. Inadmissible, because of the
D. Inadmissible, as an admission physician-patient privilege.
made in connection with an offer
to pay medical expenses. B Admissible, only if the admitting
nurse heard the statement.

C. Admissible, because it is not pro-


tected by the physician-patient
privilege.

D. Admissible, because the state-


ment Angelo made was not made
in confidence.

308 EVIDENCE
Q
m
12. The prosecuting attorney wants to B. Inadmissible, because of the at-
introduce a letter that Angelo wrote to torney-client privilege. a
Mario before the "hit." Angelo wrote: I'
"Mario, we should have done it sooner, C. Admissible, because others were o
m
I've always wanted to be part of a fam- present at the conference other
ily." It was signed, "Angelo." than Duce and Attorney.

In order for the letter to be admissible D. Admissible, because Ace is an


the prosecution must: adverse party.

A. Provide the Court with the origi- 14. Poultry sues Duck Co. for injuries sus-
nal letter. tained when Poultry's truck collided
with Duck's truck. Duck's manager
B. Show evidence of the authentic- prepared a report of the accident, at the
ity of the letter. request of the company's attorney. Dur-
ing discovery, Poultry demanded that
C. Show why the letter is relevant. the report be produced. Will the Court
rule for the production of the report?
D. Have an expert witness testify
that Angelo wrote the letter. A. No, because it is a privileged
communication.
13. Ace, an employee of Duce Trucking
("Duce"), gets into an auto collision B. No, because the report contains
with Card. Card brings suit against hearsay.
Duce and Ace, as joint defendants.
Duce and Ace consult an attorney about C. No, because the report is self-
the suit. Attorney for Duce and Ace serving.
calls his investigator, Sneak, into the
conference. Sneak makes notes as to D. Yes, because business reports are
the discussion of what happened. not privileged.

Card calls Ace to testify to admissions


made by Duce in the conference. Duce
objects. The Court should rule that
Ace's testimony is:

A. Inadmissible, because the best


evidence is Sneak's notes.

EVIDENCE 309
Q
("3 15. Pat sued David for damage to his A. Inadmissible, because it is not
z
W home. The damage resulted from a relevant.
n chemical explosion from an experiment
5
W David was conducting in the garage. B. Inadmissible, because character
A relevant fact in Pat's lawsuit is the cannot be proved by instances of
magnitude of the explosion. On direct misconduct.
examination, Pat was asked if he re-
membered the explosion. He stated, "I C. Admissible, to show his intent to
recall my daughter, Ella, running out defraud.
from her bedroom and screaming that
the bedroom windows had just shat- D. Admissible, to show Deft's char-
tered." Ella is 14-years-old. acter for honesty.

David's attorney objects. The Court will 17. Farmer John sues Jolly Rancher for
rule Pat's testimony: damages done to Farmer's crops by
Rancher's cow. Farmer offers testimony
A. Admissible, even if Ella is avail- that he looked up Rancher's telephone
able to testify. number in the telephone book. Farmer
proceeded to call that number and a
B. Inadmissible, because Ella is voice answered saying, "This is Jolly
available to testify. Rancher." Farmer then asked, "Was
it your cow that trampled my sweet
C. Inadmissible, as hearsay not peas?" The voice replied, "Yes." The
within any exception. Court should rule the testimony:

D. Inadmissible, because Ella is a A. Admissible, because Rancher


child. identified himself as the speaker,
coupled with the accuracy of the
16. Deft is on trial for fraud. He is charged telephone book, and the phone
with selling "phony" leases to a condo- transmission system, furnishes
minium timeshare. In his opening state- sufficient authentication.
ment, Deft's attorney states that Deft
didn't know the leases were phony. At B. Admissible, because judicial
trial, the prosecution seeks to introduce notice may be taken of telephone
evidence that Deft had, on five other books.
occasions, set up phony condo deals.
Deft's attorney objects. The Court will C. Inadmissible, unless Farmer
rule that the evidence is: can prove he is familiar with
Rancher's voice.

310 EVIDENCE
Q
D. Inadmissible, unless Rancher Questions 19 - 20 are based on the 1-1

has been given the opportunity following facts. 5


m
to admit or deny whether the z
conversation took place or not. Porter sues David for personal injuries re- o
m
sulting from a car accident. Porter alleges
18. Prissy sues Evil for injuries received in the collision was caused by David's running
a motorcycle accident. Prissy alleges a red light. Porter calls Witt to testify that
that Evil was speeding and driving reck- David's car, in which he was riding, ran
lessly, and performing dangerous stunts the red light. Witt, however, testified that
on the public highway. Prissy calls Ed David's car did not run the red light.
to testify that Evil has the reputation in
the community of being a "dare devil" 19. Porter then calls Valor to testify that
rider and frequently performs danger- David's car did run the red light. The
ous stunts on the public highways. The Court should rule that Valor's testimony
Court will rule Ed's testimony: is:

A. Admissible, because it shows A. Admissible, because Valor's


Evil is a negligent motorcycle testimony is relevant to material
rider. issues.

B. Admissible, because it is habit B. Admissible, because Porter was


evidence. surprised by Witt's testimony.

C. Inadmissible to show negli- C. Inadmissible, because Porter is


gence. bound by Witt's testimony.

D. Inadmissible, because Evil has D. Inadmissible, because Porter


not offered testimony of his good cannot impeach his own wit-
character. ness.

20. On cross-examination Valor is asked


by David's attorney if he was "high"
at the time he witnessed the collision.
Valor replied, "No, I have never touched
alcohol or drugs in my life." David's
attorney now seeks to introduce testi-
mony of Tattle that Valor had been high
from a joint he smoked at a party 18
months previous. The Court will rule
Tattle's testimony:

EVIDENCE 311
Q
bj A. Inadmissible, because a witness Questions 21 - 25 are based on the
z
W cannot be impeached by specific following fact pattern.
ci acts.
5
W Driver hit and injured Jogger while Jogger
B. Inadmissible, because the ques- was jogging across a marked crosswalk.
tion of whether Valor was high There were several witnesses at the scene of
or not is a collateral matter. the accident. Jogger alleges that Driver was
under the influence when he struck him.
C. Admissible, to show Valor is not
truthful. 21. Jogger's counsel calls Polly who was
a passenger in Driver's car along with
D. Admissible, to impeach Valor as her sister Dolly. Polly testifies that
to his lack of memory. just before the accident Dolly yelled
out, "Be careful, there is a man in the
crosswalk." The Court will rule this
testimony:

A. Inadmissible, because the state-


ment preceded the accident.

B. Inadmissible, because Dolly is


available as a witness.

C. Inadmissible, as hearsay not


within any exception.

D. Admissible, as an excited utter-


ance.

22. Jogger's counsel next calls Policeman


who testifies, "Polly, the passenger,
said, 'We hit Jogger while he was in
the crosswalk'. Driver was present but
remained silent." The Court will rule
this testimony:

A. Admissible, because Polly's


statement is imputed to Driver.

B. Admissible, as a party admis-


sion.

312 EVIDENCE
m
C. Inadmissible, because Driver C. Admissible, as a party admis-
probably was just experiencing sion. 5
m
shock.
D. Admissible, but only if the judge C)
D. Inadmissible, unless Driver has gives an instruction that the
already testified. statement only goes to liability,
not damages.
23. Jogger's counsel desires to introduce
Jogger's testimony that the day after the 25. On the day after the accident, Polly
accident Jogger said, "My hip must be passenger wrote a letter to Uncle Solly
fractured, it is killing me." The Court describing the details of the accident.
will rule the testimony: At trial, Polly cannot remember some
details of the accident. Jogger's coun-
A. Inadmissible, because it is self- sel wants to show Polly her letter over
serving. the objection of Driver's attorney. The
Court should:
B. Inadmissible, as hearsay not
within any exception. A. Allow the letter under past recol-
lection recorded.
C. Admissible, as a statement de-
scribing the declarant's physical B. Allow the letter under present
condition. recollection refreshed.

D. Admissible, to prove Jogger's C. Disallow the letter because it


hip was fractured. was written one day after the
accident.
24. Counsel for Jogger seeks to introduce
evidence that after the accident, Driver D. Disallow the letter because the
offered $3,000 to Jogger to settle the letter is self-serving.
claim. The Court will rule this evi-
dence: 26. Candy sues Bill for defamation. Candy
alleges that Bill has told all her friends
A. Inadmissible, because of public and employer that she once was a "call
policy. girl." At trial, Bill calls Officer Smith
to testify that 8 years ago Candy was
B. Inadmissible, because it is not arrested for prostitution. The Court will
relevant to the question of dam- rule that Smith's testimony is:
ages.
A. Admissible, because truth is a
defense.

EVIDENCE 313
Q
(L.I. 51 B. Admissible, because character Questions 28 - 29 are based on the
Z itself is at issue. following facts.
LLI
o
5
W C. Inadmissible, as hearsay. Push sues Shove for injuries Push received
in an auto accident. Push alleges that Shove
D. Inadmissible, because the preju- was exceeding the speed limit and was weav-
dicial effect outweighs any pro- ing over the center line.
bative value.
28. Walt is called as an eyewitness by Push.
27. Felicia, a divorcee who despises her Walt testifies, on direct examination,
ex-husband, Stan, reads in the paper that Shove was wearing a red jacket at
that he is being tried for fraudulent the time of the accident. Shove's coun-
stock transactions. Felicia remembers sel calls Wally to testify that Shove's
that while she was married to Stan, he jacket was green. Wally's testimony
had explained the scheme to her. Felicia is:
wants to testify for the prosecution. Stan
objects. Will the Court allow Felicia to A. Admissible, as tending to prove
testify? a material fact.

A. Yes, because once the marriage is B. Admissible, as bearing on Walt's


dissolved the privilege no longer truthfulness.
exists.
C. Inadmissible, because Walt's ca-
B. Yes, because Stan committed a pacity to observe is not at issue.
crime.
D. None of the above.
C. No, because it was a confidential
communication made during 29. Shove testifies that he was only driv-
marriage. ing 30 m.p.h. Counsel for Push does
not cross-examine Shove as to speed.
D. No, because Felicia is only doing Instead, he calls Officer to testify that
it out of spite for Stan. Shove stated to Officer that he was
driving 40 m.p.h.. The Court will find
Officer's testimony:

A. Inadmissible, because it is hear-


say not within any exception.

B. Inadmissible, because no foun-


dation was laid.

314 EVIDENCE
C. Admissible, as a prior inconsis- B. An objection by Lolly's attorney,
tent statement. on the ground of doctor-patient
privilege.
D. Admissible, as an admission.
C. A finding by the trial judge that
30. Hilda is on trial for the murder of Betty. the patient did not actually re-
Hilda does not plan on taking the stand. ceive treatment.
However, Hilda calls Lionel to testify
that Hilda has a reputation for non- D. The assertion of a privilege by
violence. Will the Court allow Lionel's Lolly's attorney, present at trial
testimony? as a spectator at Lolly's request,
and allowed by the trial judge.
A. Yes, as an offer of evidence to
show Hilda's good character.

B. No, because character evidence


is not admissible to support a
defendant's claim she is innocent
of the crime.

C. No, because Hilda has not yet


taken the stand.

D. None of the above.

31. Don Perry, a physician, called as a


witness by the defendant in the case of
Jones v. Smith, was asked to testify to
statements made by Carmen Lolly, his
patient, for the purpose of obtaining
treatment from Dr. Perry. Which of the
following is the best basis for excluding
evidence of Lolly's statement?

A. An objection by Dr. Perry as-


serting Lolly's privilege against
disclosure of confidential com-
munications made by Lolly to
Dr. Perry, for purposes of treat-
ment.

EVIDENCE 315
Q
Questions 32 - 33 are based on the D. Can rely on the judge, as an
following facts: expert in law, to advise the jury
whether there was a breach.
Lawyers Abel and Baker are the members
of the law partnership of Abel & Baker, in a 33. In addition to proving that Abel and
small town that only has one other lawyer. Baker were negligent, Client must pres-
Abel and Baker do primarily personal injury ent evidence that she:
work. Client was severely and permanently
injured in an automobile collision when A. Would have, but for her lawyers'
struck by an automobile driven by Driver. negligence, recovered against
Client employed the Law Offices of Abel & Driver.
Baker to represent her. At the time Client em-
ployed the firm, the statute of limitations had B. Had a good faith claim against
six weeks to run on her claim. The complaint Driver.
was never prepared, nor filed. Abel and
Baker each thought that the other had filed C. Was severely and permanently
the complaint. The statute of limitations on injured, when struck by Drivers'
Claimant's claim has run against Driver. automobile.

Client has filed suit against Abel and Baker D. Did not contribute to her own
for negligence. This case is on trial with a injuries.
jury.
34. Paul sued Hotel 6 for injuries he sus-
32. In order to establish a breach of standard tained in a fall in the hotel lobby. The
of care owed to her by Abel and Baker, lobby floor was covered with tile. The
Client: evidence was that the lobby floor had
been waxed approximately an hour
A. Must have a legal expert from before Paul slipped on it and, although
the same locality testify that de- the wax had dried, there appeared to be
fendant's conduct was a breach. excessive dried wax caked on several
tiles. Hotel 6 denied Paul's claim that
B. Must have a legal expert from it was negligent. Hotel 6 offered proof
the same state testify that defen- that the week before Paul's fall, at least
dants' conduct was a breach. 11,000 people had walked across the
lobby floor without incident. The trial
C. Can rely on the application of the judge should rule the evidence:
jurors' common knowledge as to
whether there was a breach. A. Admissible, because it tends to
prove that Paul did not use rea-
sonable care.

316 EVIDENCE
m
B. Admissible, because it tends to C. Yes, because a mentally-ill per-
prove that Hotel 6 was careful son is disqualified from testify- m
in maintaining the floors. ing.
0
C. Admissible, because it proves D. Yes, unless Zack was insane.
that no dangerous condition ex-
isted. 36. Cam sued Dan for injuries suffered by
Cam when their automobiles collided.
D. Inadmissible, because it does At trial, Cam offers into evidence a
not bear on the issue of Hotel properly authenticated letter from Dan
6's exercise of due care on this that says, "Your claim seems too high,
occasion. but because I might be found at fault,
I'm prepared to offer you half of what
35. Defendant was on trial for murder. The you ask." The letter is:
only eyewitness to the crime was Zack,
whose testimony was largely uncor- A. Admissible, as an admission.
roborated by other evidence. Zack was
called by the prosecution to testify to B. Admissible, as a statement
what he saw on the night in question. against Dan's pecuniary inter-
Defendant's counsel objects to Zack est.
testifying on the ground that Zack
was incompetent to testify by reason C. Inadmissible, as hearsay.
of a long history of mental illness.
Defendant's counsel offered documen- D. Inadmissible, because Dan's
tary evidence to the court that Zack had statement was made in an effort
recently been in a mental institution, to settle a claim.
and moved to have the court order Zack
to submit to a psychiatric examination.
The court denied the motion. Did the
court make an error in permitting Zack
to testify?

A. No, because a person who has


a mental illness or defect is not
competent to testify .

B. No, because Zack was the only


witness to the crime.

EVIDENCE 317
Questions 37 - 39 are based on the A. Jamie first proves that Sarah was
following facts: an employee of Powell Corpo-
ration at the time she wrote her
Jamie and two passengers, Buddy and statement, and that the statement
Charles, were injured when their car was concerned a matter within the
struck by a truck owned by Powell Corpo- scope of her employment.
ration. The truck was driven by Powell's
employee, Adam. Sarah, also a Powell em- B. Jamie produces independent
ployee, was riding in the truck. The issues in evidence that Adam was not
Jamie v. Powell Corp. include the negligence wearing his glasses.
of Adam in driving too fast, and in failing to
yield the right-of-way. C. Sarah is shown to be beyond the
court's subpoena power.
37. Jamie's counsel offers evidence that,
shortly after the accident, Powell Cor- D. The statement was made under
poration put a speed governor on the oath in an affidavit form.
truck involved in the accident. The
evidence is: 39. Powell's counsel seeks to have Officer
testify that while he was investigating
A. Admissible, as an admission. the accident Jamie told him , "This was
my fault." The evidence is:
B. Admissible, as a declaration
against interest. A. Admissible, since it is exempted
from the hearsay rule.
C. Inadmissible, as a subsequent
remedial repair. B. Admissible, as an exception to
the hearsay rule.
D. Inadmissible, because it would
lead to the inference that Powell C. Admissible, as a declaration
Corporation was at fault. against interest.

38. Jamie's counsel seeks to introduce Sar- D. Inadmissible, since it is hearsay


ah's written statement that Adam had not within any exception.
left his glasses (which he is required to
wear while operating a motor vehicle)
at the restaurant where they had lunch
just before the accident. The statement
is admissible against Powell Corpora-
tion, if:

318 EVIDENCE

m
40. Birdie was arrested and charged with C. Inadmissible, because it is im-
illegally driving a stolen vehicle across peachment of a collateral mat- F7
m
the state line. Evidence was presented ter.
that he drove the car from New York to
Chicago. The judge took judicial notice D. Inadmissible, because it is ex-
of the fact that it was impossible to drive trinsic evidence of a specific
from New York to Chicago without instance of misconduct.
crossing the state line. The judge's tak-
ing of judicial notice: 42. At the trial of Bill for a murder that
occurred in Newport, the prosecution
A. Satisfied the prosecutor's burden called Wally, who testified that he saw
of production on that point. Bill kill the victim. Bill believed that
Wally was 200 miles away in San Di-
B. Shifted the burden to Birdie, to ego, engaged in a narcotics deal with
present evidence in rebuttal. George, a mutual acquaintance, on the
day in question. On cross-examination
C. Shifted the burden on Birdie, to by Bill, Wally was asked whether he
prove that he did not drive from had, in fact, been in San Diego purchas-
New York to Chicago. ing narcotics on that date. Wally refused
to answer the question on self-incrimi-
D. Conclusively proved the point, nation grounds.
that to drive from New York
to Chicago, state lines were The judge ordered Wally to answer the
crossed. question, or his testimony would be
stricken from the record. The order to
41. In a negligence action, Abel testified testify or have the testimony stricken
against Dave. Dave then called Willy can best be supported on the basis
to testify that Abel once perpetrated a that:
hoax on the Church of Saint Francis.
Willy's testimony is: A. Wally had not been charged with
any crime and, thus, could claim
A. Admissible, because a hoax in- no privilege.
volves untruthfulness.
B. Wally's proper invocation of the
B. Admissible, provided that the privilege prevented adequate

hoax resulted in conviction of cross-examination.
Abel.

EVIDENCE 319
Q
W
.) C. The public interest in allowing 44. In a suit attacking the validity of a trust
z
W an accused to defend himself executed ten years ago, Plaintiff alleges
a outweighs the interest of a non- mental incompetency of Jack, the set-
5
W party witness in the privilege. tlor, and offers into evidence a properly
authenticated affidavit of Henry, Jack's
D. The trial record does not estab- brother. The affidavit stated that Henry
lish that Wally's answer could be had observed Jack closely over a period
incriminating. of a month, that Jack had engaged in
instances of unusual behavior (which
43. Dick is being prosecuted for burglary. were described), and that Jack's appear-
At trial, Dick testifies on his own be- ance had changed from being neat, alert
half, denying that he had committed and aware, to disordered and absent-
the burglary. On cross-examination, minded. The judge should rule Henry's
the prosecution asks Dick whether he affidavit:
had been convicted ten years ago for
burglary. The question of the earlier A. Inadmissible, as opinion.
burglary conviction is:
B. Inadmissible, as hearsay, not
A. Proper, if the court determined within any exception.
that its probative value out-
weighed its prejudicial effect. C. Admissible, since the declaration
was the best evidence that the
B. Proper, because the prosecutor statement was made.
is entitled to make such an in-
quiry. D. Admissible, as an official state-
ment.
C. Improper, because it is character
evidence. 45. Allen and Neal were arrested for armed
robbery. They were taken to the police
D. Improper, because the conviction station and placed in an interrogation
is ten years old, and the defense room. After the police officer gave
must have been given notice of them their Miranda warning, Allen
its use prior to trial. said, "Look, Neal planned the whole
thing, we robbed the place, and I was
dumb enough to follow along." Neal
said nothing. Neal was then taken to
another room, and a full confession was
obtained from Allen.

320 EVIDENCE
m
If Neal is brought to trial for armed A. Admit the invoice, only because
robbery, the fact that Neal failed to ob- it is a record of regularly con- m
ject to Allen's statement and remained ducted business activity.
silent, is:
0
B. Admit the invoice, only because
A. Admissible, as an admission. it is recorded recollection.

B. Admissible, because a state- C. Admit the invoice, because it


ment of a participant in a crime would qualify as a business
is admissible against another record and as a record of recol-
participant. lection.

C. Inadmissible, because it is hear- D. Exclude the invoice, as hearsay


say. not within any exception.

D. Inadmissible, because, under the 47. James, while crossing the street at the
circumstances, there was no duty intersection of Nutwood and State Col-
for Neal to respond. lege, was hit by a car driven by Ginger.
James sued Ginger for his injuries.
46. A case of whiskey was stolen from
a Johnny Walker truck when it was At trial, James called Officer Sparky
parked outside the Western Saloon. At to testify that, ten minutes after the
trial, to prove that the whiskey was on accident, a bus driver stopped and said
the truck, Loaden, the Johnny Walker to him, "Officer, a few minutes ago I
shipping manager, was called by the saw a hit and run accident at Nutwood
prosecution to testify. Loaden testified and State College involving a red truck
that he did not have first-hand knowl- and a pedestrian. I followed the truck
edge that the whiskey was aboard the to McPlace for Ribs, on Third Avenue."
truck. Thereupon, the prosecution asked Sparky's testimony should be:
Loaden, "Did you receive a shipping
invoice listing the contents on that A. Admissible, as a present sense
truck for that particular day?" Loaden impression.
replied, "Yes, I did receive from the
shipping clerk an invoice listing the B. Admissible, as a statement of
merchandise, as customary." Loaden perception.
then produced the invoice. If the pros-
ecuting attorney offers the invoice into C. Inadmissible, as hearsay not
evidence, the trial judge should: within any exception.

EVIDENCE 321
D. Inadmissible, because it is irrel- Questions 48-50 are based on the
z evant. following facts:
a
Raymond, a Hallmark salesman, was a guest
at the Holiday Hotel in Arizona. After check-
ing into the hotel, he went to the elevator on
the lobby level floor to proceed to his room.
When the elevator doors opened, he stepped
into the elevator shaft, unaware that the
elevator car itself had not descended to the
lobby. Raymond fell 10 feet to the elevator
shaft floor. The elevator actually was out of
service because it was being repaired that
day. Raymond sues the hotel, claiming he
suffers from permanent injuries as a result
of the incident.

At trial, Raymond proposed to testify that


he called out after the incident, "I have just
fallen while getting into the lobby elevator
and my knee really hurts." The only response
was from Jean Paul, the concierge, who said,
"Hold on, I'll get some help." Jean Paul ob-
served Raymond later that day swimming in
the hotel pool.

48. Raymond's proposed testimony re-


garding his statement to the concierge
should be:

A. Admitted, as a statement of pres-


ent physical condition.

B. Admitted, as evidence of liability


against the hotel.

C. Excluded, because the statement


was not made to a treating doc-
tor.

322 EVIDENCE
Q
1
D. Excluded, because Raymond A. Admissible, because the state- ., "
is not qualified as an expert to ment contradicts Raymond's 5
m
determine his own medical con- claim that he had no prior in- z
dition. jury. 0
m

49. Assume for this question, that Raymond B. Admissible, because the state-
intends to offer testimony that he never ment relates to Raymond's medi-
complained of knee pain prior to his cal condition which is in issue.
fall. Raymond's proposed testimony re-
garding the absence of prior complaints C. Inadmissible, because the state-
of knee pain is: ment is protected from disclosure
by the physician-patient privi-
A. Admissible, because the state- lege.
ment is reliable.
D. None of the above.
B. Admissible, because the state-
ment relates to a condition per-
ceivable by a lay person.

C. Admissible, because Raymond


may give an opinion as to his
own diagnosis and required treat-
ment.

D. None of the above.

50. Assume for this question, that Raymond


seeks to exclude certain statements
he made to his general physician, Dr.
Cure, 3 years before the incident. On
that occasion, Raymond told Dr. Cure,
"I awoke this morning with severe knee
pain, after I helped a college buddy with
moving yesterday." The hotel seeks to
admit Raymond's prior statement to Dr.
Cure over Raymond's objection. The
statement is:

EVIDENCE 323
Q
Questions 51-52 are based on the A. Sustain the objection, because
following facts: the police report is the best evi-
dence of Don's statement.
Paul sues Don for personal injuries that
he suffered when he was struck by Don's B. Sustain the objection, in the
bicycle. Don's bicycle hit Paul as he was interest of humanitarian consid-
walking across South Crest Plaza's parking erations.
lot. Immediately after the accident, Don ran
over to Paul and said, "I know I was doing C. Overrule the objection, because
a wheelie, but you were looking in your Paul's attorney opened to the
shopping bag instead of where you were door to the admission of the
walking. But, if you're hurt, I'll pay your remainder of Don's statement to
medical bills." Paul pertaining to the cause of
the accident.
51. At trial, Paul calls as his first witness,
Stan, who was an eyewitness to the ac- D. Overrule the objection, because
cident. Stan proposes to testify that he statements should be offered in
heard Don tell Paul, "I know I was do- their entirety.
ing a wheelie." Don's attorney objects.
If Stan's testimony is admitted, it will 53. Larry was charged with vehicular bat-
most likely be admitted because the tery after driving through a red light
proffered evidence is: and crashing his Cadillac into Marie's
Lotus. Marie suffered massive internal
A. Admissible, as an opinion. injuries and lapsed into a coma until
several hours after she reached the
B. Admissible, as a party admis- hospital. Sharon, Marie's best friend,
sion. was a passenger in her car. Sharon mi-
raculously received only a few minor
C. Admissible, as a present sense bruises. After the collision, Sharon
impression. stayed with Marie, trying to comfort
her until the ambulance arrived.
D. Admissible, as a declaration
against interest. At trial, Sharon is called to the stand by
the district attorney, who asks Sharon if
52. Suppose that on cross-examination Marie said anything to her before being
Don's attorney asks Stan if he heard taken to the hospital. Sharon answers
Don's entire statement. Stan responds affirmatively, and testifies that Marie
affirmatively. Don's attorney now seeks was conscious for a few brief moments
to have Stan testify to Don's complete after the accident when she had stated,
statement. Paul's attorney objects. The "He never stopped for that red light."
trial judge should:

324 EVIDENCE
m
Upon objection by Larry's attorney, the 55. Fred and Wilma Strong had been mar-
court should rule the testimony con- ried for thirty years. Late one Sunday 57
1
m
cerning Marie's statement to Sharon: afternoon, while on his way home from
the pub, Fred hit a young child with his
A. Inadmissible, because Marie's car and drove off. The young child died
statement is hearsay. and, after months of police investiga-
tion, Fred was charged with vehicular
B. Inadmissible, because Marie's manslaughter. Thereafter, Fred called
statement lacks trustworthiness. Attorney to set up a meeting, in order
to retain him for the pending trial.
C. Admissible, as a present sense
impression by Marie. At the consultation with Attorney, Fred
was accompanied by Wilma. During the
D. None of the above. conference, Secretary took notes of the
meeting. A month later, Secretary quit
54. A tall man with red hair robbed Bob's her job. Fred decided not to hire Attor-
liquor store. Later, Pete was arrested ney, but to act in pro per instead. Dur-
and charged with the armed robbery. ing Fred's trial, the prosecution called
At Pete's trial, several eyewitnesses Secretary to testify to what was said at
testified they had seen a tall red-haired the initial conference. The proposed
man pull out a gun and rob Mr. Bob, testimony is:
the store's owner. Pete appeared at trial
with a shaved head. The prosecution A. Admissible, because Fred did not
calls Deputy Fife to testify that Pete hire Attorney.
had red hair when he was first brought
to jail. Pete's attorney objects. The trial B. Admissible, because Secretary's
judge should rule Fife's testimony is: presence at the conference de-
stroyed the privilege.
A. Admissible, as a prior identifica-
tion. C. Inadmissible, because of the at-
torney-client privilege.
B. Admissible, for the limited pur-
pose of clarifying the discrep- D. Inadmissible, as hearsay, not
ancy in the witness' testimony. within any exception.

C. Inadmissible, as hearsay not


within any exception.

D. Inadmissible, because it is opin-


ion.

EVIDENCE 325
56. In a battery action against Katz by Questions 57-58 are based on the
Doug, Doug's attorney called Mel to following facts:
testify that the incident occurred on
December 23. Although Mel was not One morning, Don telephoned his best
questioned about a deposition he made friend, Troy, and asked him if he could
before trial, Mel had previously testi- borrow his car. Don explained to Troy that
fied at his deposition that the incident he had an audit scheduled with the Internal
occurred on December 14. After Mel's Revenue Service, which he had to attend and
trial testimony, Katz's attorney offers he needed his car to get there. Don promised
Mel's deposition into evidence. The Troy that he would return the car late that
trial judge should rule the deposition: afternoon. Troy agreed to allow Don to bor-
row his car. While driving Troy's car, Don
A. Admissible, as substantive evi- was involved in an automobile accident
dence the incident occurred on with Bud. Bud has filed suit against Troy
December 14. for negligence.

B. Inadmissible, because Mel was 57. In his case in chief, Bud calls Vic to
available to testify at the trial. testify to three incidents of careless
driving on Don's part. The trial judge
C. Inadmissible, as hearsay not should rule the testimony:
within any exception.
A. Admissible, as circumstantial
D. Inadmissible, because there were evidence that Don was negligent
no facts showing that Mel was on that occasion.
intentionally untruthful.
B. Admissible, since Vic had per-
sonal knowledge of Don's poor
driving.

C. Admissible against Troy, as evi-


dence of Don's lack of fitness.

D. Inadmissible, as hearsay not


within any exception.

58. In his defense, Troy offered testimony


of his wife, Winnie, that Don had a
reputation as a careful driver. Winnie's
testimony is:

326 EVIDENCE
Q
A. Admissible, as evidence of D. It was not hearsay, because Vir- 1. "
habit. ginia had firsthand knowledge of 5
m
the events as they transpired. z
C)
B. Admissible, as present sense m
impression. 60. Polly sues Marie, an artist, and Ben,
an art gallery owner, for libel. Polly's
C. Admissible, because it is proper complaint alleges that Marie painted a
character evidence. picture portraying a figure recognized
as Polly engaging in sexual conduct,
D. Inadmissible, because of bias. and the painting was published by
allowing the painting to be publicly
59. Sally is charged with murder for shoot- displayed. Further, Ben also published
ing Brian with her gun. Sally testified the libelous painting by hanging the
at trial that she honestly believed that painting for public viewing in his art
Brian had already died from a heart gallery and where it was viewed by
attack before he was shot. In rebuttal, the people visiting his gallery. At trial,
the district attorney called Virginia to Polly offers testimony of Witness that
testify that, just before she saw Sally he was present in Ben's gallery and ob-
shoot Brian, she heard Brian shout, "I'm served Ben hanging the picture. Witness
going to die." further offers to testify as to the scene
portrayed in the picture and that the
Sally's attorney objects to Virginia's picture was signed with Marie's name
proposed testimony on grounds that in the lower right-hand corner. Upon
it is inadmissible hearsay. If the trial proper objection the court should rule
court judge overrules the objection, the Witness' testimony:
reason will most likely be that:
A. Inadmissible, because Witness'
A. Although hearsay, Brian's state- testimony is not the best evi-
ment was made in the belief of dence.
impending death.
B. Inadmissible, because Witness'
B. Although hearsay, Brian's state- description of the picture, as
ment related to his present physi- against Ben, is insufficiently
cal condition. authenticated.

C. It was not hearsay, because Bri- C. Admissible, because Witness


an's statement is admissible to properly authenticated the paint-
show "notice or knowledge" by ing by identifying the artist who
Sally that Brian was still alive. signed the painting as Marie.

EVIDENCE 327
D. Admissible, because Witness has Questions 61 - 62 are based on the
ersonal knowledge of the facts
p following facts:
in which he is testifying to.
5
U.1
Winona sues Bob for the wrongful death of
her husband, Henry. At trial, Winona offers
the testimony of Henry's business partner,
Paul, who states that the average net profit
from the business of each partner over the
preceding five years was $100,000. Fur-
ther, evidence is presented showing that the
partnership books are in Paul's possession
and that Paul is willing to allow for their
inspection.

61. The testimony of Paul is:

A. Admissible, since the informa-


tion is relevant and the books are
available for inspection.

B. Admissible, since the facts


sought prove the earnings of the
partnership and Paul had per-
sonal knowledge of those facts.

C. Inadmissible, as hearsay not fall-


ing within any exception.

D. Inadmissible, because Paul's tes-


timony is not the best evidence.

328 EVIDENCE
Q
62. Assume that Paul's testimony is inad- 63. Greg dies, leaving a will by which ''
missible. Winona now offers the testi- he bequeaths his entire estate to his a
m
mony of a properly-qualified expert,
Alvin, a certified public accountant,
friend, Bobby. Peter, Greg's only heir
at law, brings a suit to contest the will om
who states that he had reviewed the on grounds of incapacity. At the trial,
books of the partnership covering the Bobby calls Dr. Jan and Mrs. Alice as
preceding five years and, that in his witnesses, both of whom offer to testify
opinion, the books reflect an average affirmatively when asked whether Greg
annual profit for that period allocable to knew enough about "the nature and
Henry of $100,000. Upon objection, the extent of his property, and the natural
court should rule Alvin's testimony: objects of his bounty" at the time of
Greg's execution of the will. Other
A. Admissible, because Alvin has evidence has shown: (1) that Mrs. Alice
personal knowledge of the earn- is Greg's former housekeeper, who had
ings, and is a qualified expert. seen him daily for over 25 years, until
the date of Greg's death; and (2) that Dr.
B. Inadmissible, because of the best Jan is a psychiatrist, stipulated by the
evidence rule. parties to be qualified, who had occa-
sion to examine Greg the week before
C. Inadmissible, because Alvin does Greg executed his will. Upon objection,
not have personal knowledge of the court should:
the facts to which he is testify-
ing. A. Exclude the testimony of both
witnesses, because both are at-
D. Inadmissible, as hearsay not fall- tempting to testify to their opin-
ing within any exception. ions on an ultimate issue in the
case.

B. Admit Dr. Jan's testimony, and


exclude Mrs. Alice's testimony.

C. Admit Mrs. Alice's testimony,


and exclude Dr. Jan's testimo-
ny.

D. Admit the testimony of both wit-


nesses.

EVIDENCE 329
Q
61 64. Baby sued Dan for damages, for injuries B. Proper impeachment on cross-
Z that Baby incurred when a badly rotted
W examination, even though ex-
a limb fell from a tree in front of Dan's trinsic proof will not be allowed
11' home and hit Baby. Dan claimed that if the question is answered in the
the tree was on city property; thus, was negative.
not liable to Baby for his injuries. At
trial, Baby offers testimony that a week C. Improper impeachment.
after the accident, Dan had the tree cut
down. The evidence is: D. Improper, because specific in-
stances of misconduct may not
A. Inadmissible, since such is be used to impeach.
against public policy so as to
further safety precautions. 66. Terry and Larry were indicted for con-
spiracy to sell an illegal substance. Dan,
B. Inadmissible, since it is irrel- an undercover agent, was the key wit-
evant. ness. Terry and Larry claim entrapment
as their defense.
C. Admissible, to show that the tree
was on Dan's property. At trial, Dan testifies for the prosecution
that he was present at a meeting with
D. Admissible to show that the tree the defendants at the Grand Hotel on
was in rotten condition. July 7, at which Terry and Larry agreed
to sell two kilos of cocaine to Dan for
65. Nancy was prosecuted for the murder $150,000.
of Merry. At trial, Anna testified against
Nancy. On cross-examination, Nancy's Thereafter, the defense calls as a wit-
attorney asked Anna the following ness Tom, an old college roommate
question, "Isn't it true that charges of Dan's, to testify that Dan once
against you as an accomplice in Merry's purchased marijuana. Tom's proposed
murder are being dropped in exchange testimony is:
for your testimony against Nancy?" The
question is: A. Admissible, because the pur-
chase of marijuana is probative
A. Proper impeachment, because it of Dan's untruthfulness.
shows Anna has a self-interest in
testifying against Nancy. B. Admissible, provided that the
purchase of the marijuana re-
sulted in a criminal conviction.

330 EVIDENCE
m
C. Inadmissible, because it is extrin- 68. Denny was driving his Toyota truck
sic evidence. when he struck Stan, a 6-year-old boy, m
who was crossing the street on the way
D. Inadmissible, as improper lay to school. Stan was seriously injured. C)
opinion in regard to whether or
not the substance previously pur- Stan visited attorney, Jacoby, accom-
chased by Dan was marijuana. panied by his father, Tom, in regard to
bringing a case against Denny for the
67. Victim was injured in an automobile injuries Stan sustained. Tom did not
accident and rushed to the emergency retain Jacoby to represent his son in
room at Hospital. Victim subsequently the lawsuit. Instead, he hired another
brought an action against Hospital for lawyer, Parker, to handle the case.
malpractice, claiming that Hospital
delayed in giving her prompt medical At trial, Denny's attorney calls Jacoby
attention, which resulted in a delay in to testify what Stan had said to him in
her recovery period. regard to his physical condition dur-
ing the consultation that he (Jacoby)
At trial, Nurse proposes to testify that had with Stan and his father. Jacoby's
when Victim was brought into the testimony is:
emergency room she was unconscious.
Victim's attorney objects. The trial A. Admissible, because no attorney-
judge should: client relationship existed.

A. Sustain the objection, because B. Admissible, because the element


Nurse is not a qualified expert. of confidentiality is not satisfied
when a third person is present
B. Sustain the objection, because with the attorney and client.
it goes to an ultimate issue of
fact. C. Inadmissible, because it was
a statement of physical condi-
C. Overrule the objection, because tion.
Jogger can corroborate her testi-
mony. D. Inadmissible, because of the at-
torney-client privilege.
D. Overrule the objection, because
Nurse had observed Victim when
she was brought into the hospi-
tal.

EVIDENCE 331
Q
("3 69. Cam is being sued for breach of em- 70. In a suit between Pamela and Darren,
z ployment contract by Paul. Paul hired arising out of an automobile accident,
I I
L Cam to obtain a patent on all his inven- Benny, a bystander, was called to the
5
w tions within 30 days of their finalization stand by Pamela's counsel, to testify
stage. The day of the finalization of the that Darren had driven through a red
Zit Zap Gun is in question. Paul claims light, which caused the collision. On
that the finalization date was May 1 cross-examination, Benny is asked if
1992. Cam claims that it was June 1 he has ever been convicted for false
1992. pretenses, which Benny denies. The
prosecutor offers a certified copy of
At trial, Cam is called to testify. His the criminal conviction into evidence,
counsel asks him if he is certain about upon Benny's denial. Pamela's counsel
the date, June 1, 1992, on which the Zit objects. The court should:
Zap Gun was finalized. Cam replies,
"Yes, it was June 1, because I remember A. Overrule the objection, because
reading a story in that day's newspa- it bears on the witness' credibil-
per about the 8.0 earthquake in L.A., ity.
which destroyed most of downtown."
Paul's counsel objects to the reference B. Overrule the objection, because
of the newspaper and makes a motion it is evidence that Benny is not
to strike. The judge should: telling the truth.

A. Grant the motion, on the grounds C. Sustain the objection, because it


that the best evidence rule re- is irrelevant.
quires production of the news-
paper. D. Sustain the objection, because
the witness' character is not in
B. Grant the motion, because the issue.
reference to the newspaper story
does not fit within any exception 71. Myron was arrested and charged with
to the hearsay rule. the rape of Patty. He claims he did not
rape Patty because she had consented to
C. Deny the motion, on the grounds his advances. At trial, Myron is prepared
that the court can take judicial to testify that he had engaged in sexual
notice of the local newspaper. intercourse with the victim on many
occasions during the previous month
D. Deny the motion, because a before the rape allegedly occurred with
witness may refer to a collateral the victim's consent. Myron's testimony
document without providing the is:
document.

332 EVIDENCE
Q
A. Admissible, as evidence of spe- 73. Lovejoy sued Eric for injuries suffered
cific acts to show consent. by him when Eric collided his motor-
cycle into Lovejoy. At trial, Lovejoy
B. Admissible, as evidence of offers into evidence a properly authen-
habit. ticated letter from Eric which states,
"Your claim of damage seems too high,
C. Inadmissible character evi- but because I have no insurance, and I
dence. was at fault, I'm prepared to pay the
amount in which you have asked." The
D. Inadmissible, because reputa- letter is:
tion or opinion evidence of past
sexual behavior of the victim is A. Admissible, under public policy
not permitted. reasons.

72. On January 3 1993, Dick was prosecuted B. Admissible, because it is a party


for burglary. At trial, Dick testified on admission.
his own behalf, denying that he commit-
ted the burglary. On cross-examination, C. Inadmissible, because Eric's
the prosecution asked Dick if in 1980 he statement is opinion.
had been convicted of robbery. Dick's
attorney objects. The court should: D. Inadmissible, because the state-
ment was made during an effort
A. Overrule the objection, because to settle a claim.
the prosecutor may test the cred-
ibility of the witness. 74. Bonnie was sued by Brent for the sale
of her shares in the Acme Corporation.
B. Overrule the objection, because Bonnie consulted her friend Steve, an
the testimony is immaterial. attorney, regarding the sale of her 80%
interest of the stock of a closely held
C. Sustain the objection, because corporation. She explained the reason
the crime is more than 10 years for the sale to Steve, and disclosed the
old. financial condition of the corporation.
Steve determined that the transaction
D. Sustain the objection, because was too complex and in violation of a
the adverse party was not given shareholder agreement. Steve decided
written notice. to decline the case. Bonnie then hired
Carrie to represent her.

EVIDENCE 333
At trial, Brent calls Steve to testify to 75. Mr. Roper owns a house at 24211
z the contents of the conversation be- Lemon Street. He rents the house to
tween Steve and Bonnie. Bonnie's Jack. The lease states that Jack will
5 counsel objects. The court should: maintain the house and yard in its pres-
ent condition. While walking home
A. Sustain the objection, because of one day, Chrissie was hit on the head
the attorney-client relationship. when a tree limb fell from Mr. Roper's
property, seriously injuring her. It is
B. Sustain the objection, because agreed between the parties that the tree
the conversation is not relevant fell because of dry rot. Mr. Roper hired
to the sale of the stock. Larry the next day to have the whole
tree removed from the property. Chris-
C. Overrule the objection, because sie now brings suit against Mr. Roper
Steve was not retained by Bon- in Superior Court. At trial, Mr. Roper
nie. testifies that he rented out the house to
Jack, who he claims was responsible for
D. Overrule the objection, because the maintenance of the yard, under the
the attorney-client privilege did terms of the lease. Chrissie introduces
not attach. into evidence that Mr. Roper had Larry
remove the tree from the property the
next day. The evidence is:

A. Admissible, because the evi-


dence is not being used to prove
negligence or culpable conduct.

B. Admissible, because Mr. Roper


was at fault.

C. Inadmissible, because Mr. Rop-


er's acts were subsequent reme-
dial measures.

D. Inadmissible, because the pro-


bative value substantially out-
weighed its prejudicial effect.

334 EVIDENCE
Q
76. Norm was employed at BeerCo as a 77. Botch was named as a criminal defen-
quality control tester of the "flip-top" dant for allegedly killing the cashier at
openers on 12-ounce beverage cans a local convenience store. One of his
produced by his employer for their defenses is that he is innocent by reason
number one product, Cheers beer. One of a mental disease or defect such that
day while at work, Norm was injured he could not appreciate the wrongful-
when one of the "flip-tops" fell unno- ness of his acts. The defense presented
ticed into the can he was testing, only expert testimony through Dr. Wright
to be swallowed by Norm on his next who was properly qualified as an expert.
sip of beer. Norm dies as a result. Dr. Wright testified that Botch suffered
a mental disease or defect at the time
Vera, Norm's wife, brings a wrong- of his acts, as to the characteristics of
ful death action against BeerCo. At that mental disease or defect, and his
trial, Vera's counsel seeks to introduce diagnosis of Botch. The defense then
BeerCo's offer to pay funeral expenses. offered Dr. Wright's opinion that Botch
BeerCo objects. The court should: did not know his acts were wrong when
committed, over objection of the district
A. Sustain the objection, since of- attorney. How will the court rule on the
fering to pay funeral expenses prosecution's objection?
is not admissible to prove liabil-
ity. A. Overruled, because Dr. Wright
was properly qualified as an
B. Sustain the objection, since the expert before testifying.
offer is hearsay.
B. Overruled, if Dr. Wright's expert
C. Overrule the objection, since opinion that Botch suffered from
Beer Co's statement is a party a mental disease or defect was
admission. uncontroverted.

D. Overrule the objection, since C. Sustained, because an expert


Beer Co's statement is a declara- cannot testify on whether the
tion against their interest. defendant had the mental condi-
tion constituting an element of a
defense to the crime charged.

D. Sustained, because Dr. Wright's


testimony was biased in favor of
Botch.

EVIDENCE 335
Questions 78 - 80 are based on the 79. Yvette's attorney called to the stand
following facts: Lou, a cashier at Sparkle Jewelry Store,
to testify as follows: two hours before
Yvette leaves Nordstrom without paying for she was charged with shoplifting at
a blouse. She is stopped after exiting and is Nordstrom's, Yvette had shopped at
charged with shoplifting. At trial, the pros- Sparkle Jewelry Store and, after paying
ecution introduces into evidence that Yvette for a $2,000 diamond ring she had pur-
exited from the department store without chased, walked out of the store leaving
paying for the blouse, which was found on the diamond ring on a store counter. The
her baby stroller. Yvette testifies that she prosecution objects to the testimony of
walked out of the store with the blouse hang- Stephanie. The court should:
ing on the outside of her stroller and that she
forgot that she had placed it there while she A. Sustain the objection, because
looked for a matching skirt. the testimony pertains to an im-
permissible character trait.

78. Yvette's attorney calls Stephanie to


B. Sustain the objection, because it
testify that Yvette forgot her baby son at
is improper opinion testimony.
Nordstrom the last time she went shop-
ping, six months ago. The prosecution
C. Overrule the objection, because
objects to the testimony of Stephanie.
Lou has first hand knowledge of
The court should:
Yvette's conduct in the store on
the occasion in question.
A. Sustain the objection, because
the testimony pertains to an im-
D. Overrule the objection, because
permissible character trait.
Lou's testimony tends to dis-
prove Yvette's intent to com-
B. Sustain the objection, because it
mit the crime of shoplifting at
is improper opinion testimony.
Nordstrom's.

C. Overrule the objection, because


80. Yvette also calls Burt, a psychologist, to
Stephanie has personal knowl-
testify that he had treated Yvette and, in
edge of Yvette's character.
his opinion, Yvette was suffering from
a psychological condition which im-
D. Overrule the objection, because
paired her memory and caused her to be
it tends to disprove Yvette's in-
forgetful. The prosecution objects to the
tent to commit the crime.
testimony of Burt. The court should:

336 EVIDENCE
m
A. Sustain the objection, because D. Overrule the objection, because
the testimony pertains to an im- once character evidence is ad- m
permissible character trait. mitted into evidence, the door
is opened for further character
B. Sustain the objection, because it evidence.
is improper opinion testimony.
82. Mark is charged with murder. He pleads
C. Overrule the objection, if Burt is not guilty. At the request of Mark's
a qualified expert. lawyer, the trial judge appoints Doctors
Art and Cathy, psychiatrists, to examine
D. Overrule the objection, because Mark and advise Mark's lawyer as to
Burt's testimony is consistent whether Mark should rely upon the de-
with a finding of innocence. fense of mental defect. At trial, Mark's
defense is diminished capacity. Mark's
81. Deft is charged with the sale of heroin. lawyer calls Dr. Art, who gives an
He denies making the alleged sale. Deft opinion in support of Mark's defense.
calls Fred, his friend, who testifies that Mark's lawyer rests her case. In rebut-
for six years he has known Deft socially tal, the prosecution calls Dr. Cathy, to
and has lived in the same neighborhood. give an opinion contrary to that of Dr.
Deft then asks Fred whether Deft has Art. Mark's lawyer objects. The court
a character trait for non-violence. The should:
prosecution objects. The court should:
A. Sustain the objection, because of
A. Sustain the objection, because the attorney-client privilege.
the testimony is impermissible
character evidence. B. Sustain the objection, because
of the psychiatrist-patient privi-
B. Sustain the objection, because lege.
the evidence of the character trait
of non-violence is irrelevant to C. Overrule the objection, because
the issue of selling narcotics. as Mark's expert medical provid-
er, she is not permitted to testify
C. Overrule the objection, because adverse to Mark's interest.
the testimony is permissible
character evidence. D. Overrule the objection, because
Dr. Cathy was never listed as
the prosecution's expert prior to
trial.

EVIDENCE 337
Q
6" 83. Deft is charged with the murder of Rob- C. Not allow the testimony, be-
z
W ert. As part of the prosecutions's case cause it is improper character
ci in chief, the prosecutor offers a certi- evidence.
5
W fied copy of Deft's prior conviction for
murdering the President. Deft objects. D. Not allow the testimony, because
The evidence is: Dave has not testified.

A. Admissible, because it tends to 85. Lucy sued Fred for injuries suffered by
prove Deft's plan. her when Fred collided his blue truck
into her MG. At trial, Lucy calls Ricky
B. Admissible, because it is a mate- to the stand. Ricky testifies that he was
rial fact. driving the MG when the accident hap-
pened. He further testified that when he
C. Inadmissible, because it is im- pulled onto the tollway, Lucy said to
proper character evidence. him, "Watch out for that blue truck - it
looks like the driver has been drinking,
D. Inadmissible, because its pro- and he can't seem to keep control over
bative value is substantially his car." Fred's counsel objects. The
outweighed by its prejudicial testimony is:
effect.
A. Admissible, for public policy
84. Polly sues Dave for damages arising reasons.
out of an automobile accident. Polly
claims that Dave was negligent. At trial, B. Admissible, as a present sense
Polly calls Tim to the stand. Tim testi- impression.
fies that he is a member of the church
which Dave attends, and that Dave C. Inadmissible, as hearsay.
has a reputation for being a "careless"
driver. Dave's attorney objects to the D. Inadmissible, as improper lay
testimony. The trial court should: opinion.

A. Allow the testimony, because it


is trustworthy.

B. Allow the testimony, because it


is relevant to the issue of whether
Dave was negligent.

338 EVIDENCE
m
86. While driving within the scope of At trial, the prosecution offers the tape
employment for Petro Corp., Axelrod into evidence. The tape shows Tina
r„ no
C 1,17 Aar 71-t,a 1 rn NA- a uhrd nhcF•ruPri ancwerina nnectinnc hv nnlire nfficerc
while standing on her front lawn. After an unsteady manner. Defense counsel
quickly dialing for an ambulance from objects to the introduction of the tape.
her mobile phone, Maybel walked The court should:
over to the injured Thelma, and stated,
"It's too bad that truck never slowed A. Admit the tape, because its pro-
down.” bative value is not substantially
outweighed by the danger of
In an action by Thelma against Petro unfair prejudice.
Corp., evidence of Maybel's statement
is: B. Admit the tape, because it is a
party admission.
A. Inadmissible, because it was
not authorized by Axelrod's C. Not admit the tape, because it is
employer. hearsay not within any excep-
tion.
B. Inadmissible, because Maybel
was not under a state of excite- D. Not admit the tape, because it is
ment when the statement was extrinsic evidence of a specific
made. instance of misconduct.

C. Admissible, because Petro Corp.


ratified Axelrod's conduct when
it hired Axelrod as an employ-
ee.

D. Admissible, as a present sense


impression.

87. While riding her bike home from


school, Susan was struck by a car driven
by Tina. Tina was driving home from
a Wazoo mixer. Tina was arrested at
the scene for driving while under the
influence. When Tina was booked at
the police station, she was videotaped
in her intoxicated state.

EVIDENCE 339
Q
Questions 88 - 89 are based on the 89. Betty testified in her own behalf that she
following facts: was going 25 m.p.h. On cross-exami-
nation, Dino Sawyer (Fred's attorney),
While driving home from work, Betty Grav- did not question Betty in regard to her
el ran her rock-mobile into Fred Stone. Fred speed. Dino called Officer BamBam to
was seriously injured. Fred sued Betty for the testify that in his investigation of the
injuries he received in the auto accident. Fred accident, Betty told him that she was
claims that Betty was negligent because she traveling 40 m.p.h. Officer BamBam's
was driving over the speed limit. testimony is:

At trial, Fred calls Wilma Granite to testify. A. Admissible, as a declaration


Wilma saw the accident from the street cor- against interest.
ner and testifies that Betty was wearing a
pink polka dot jumpsuit at the time of the ac- B. Admissible, because it is not
cident. Betty calls Barney Pumice to testify hearsay.
that Betty was wearing a green dress.
C. Inadmissible, because it is hear-
88. Barney's testimony is: say not within any exception.

A. Admissible, because it is relevant D. Inadmissible, because it lacks


to the issue of who was negli- foundation.
gent.
90. While driving home from work, George
B. Admissible, because it tends to stopped at Harry's Bar for a drink. After
test the credibility of the wit- having a few drinks, George asked Joe,
ness. the bartender, for another. Joe, worried
about George's intoxicated state, told
C. Inadmissible, because it is ex- George that he had already had enough
trinsic evidence on a collateral to drink. George then became very an-
matter. gry, and said to Joe, "Get me a drink or
else." Joe again refused.
D. Inadmissible, because it is irrel-
evant.

340 EVIDENCE
Angered by Joe's response, George A. Not allow the testimony, because
jumped off his bar stool and hit Joe. the evidentiary foundation was
Joe now brings suit against George to insufficient to establish all ele-
recover for his injuries. At trial, Joe's ments of a speeding violation.
attorney put Joe on the stand. Joe's pro-
posed testimony is that George stated to B. Not allow the testimony, because
him, "Get me a drink or else." George's it is hearsay.
attorney objects to the testimony. The
court should: C. Allow the testimony, because
radar readings are an exception
A. Overrule the objection, because to the hearsay rule.
it is not hearsay.
D. Allow the testimony, because it
B. Overrule the objection, because is not hearsay.
it is probative of the declarant's
state of mind. 92. Tami sues Jim for fraud, alleging that
a Rolls Royce that Jim sold to her was
C. Sustain the objection, because it really a Bentley. In order to prove the
is hearsay not within any excep- Rolls Royce was genuine, Jim intro-
tion. duces the bill of sale he received at a
car auction when he purchased the car.
D. Sustain the objection, because Tami's counsel objects. The bill of sale
George could not form the intent is:
to commit a battery while intoxi-
cated. A. Admissible, because documents
of title are conclusive of rightful
91. While out on patrol, Officer Speedtrapp ownership.
pulled over Burt in his new black Trans
Am for speeding. He issued Burt a B. Admissible, because it is a party
speeding ticket. After laying a founda- admission.
tion for admitting the radar reading,
Speedtrapp testifies that he was stopped C. Inadmissible, because it is hear-
at the corner of 5th and Golden and say not within any exception.
pointed his radar gun at Burt. The
gun rated Burt's speed to be 45 m.p.h. D. Inadmissible, because it is not
Burt's attorney objects to the testimony. the best evidence.
The trial court should:

EVIDENCE 341
Q
93. Mo Tivv is arrested for burglary. At 94. John is charged with battery on Ponch,
z trial, Ali By testifies that Mo Tivv was a police officer, growing out of his
watching television at Ali's home at the arrest by Ponch. John's defense is self-
time of the burglary. On cross-exami- defense, against excessive force used by
nation of Ali, the prosecutor asks him, Ponch. Through discovery proceedings,
"Don't you certify art reproductions John secures the police department's
ac antirme ari °inn] c in order to sell disciplinary records relating to Ponch,
store?" The prosecutor was informed of citizens Tom, Dick, and Harry, stating
this by Vinny, who is Ali's employee. that Ponch used excessive force on
Certifying art reproductions as originals each of them within six months prior to
is punishable as a misdemeanor. If Mo John's incident with Ponch. John offers
Tivv's attorney objects, the objection the records in evidence. The prosecu-
will be: tion objects. The court should rule the
disciplinary reports are:
A. Sustained, because it relates to a
collateral matter. A. Not allowed, because they are
hearsay.
B. Sustained, unless Ali has been
convicted for certifying repro- B. Allowed, because it is proper
duction art works as originals. character evidence under the
"victim's exception."
C. Overruled within the discretion
of the court, because certifying C. Allowed, because of the business
reproductions as originals bears record exception.
on Ali's truthfulness.
D. Allowed, because the report is
D. Overruled within the discretion permissible to prove this char-
of the court, because certifying acter trait of Ponch.
reproductions of art works as
originals is punishable as a mis-
demeanor.

342 EVIDENCE
m
95. Laverne, the widow of Squiggy, sues The prosecution then calls Peggy to
Shirley in a wrongful death action, aris- testify. Peggy testifies that six weeks
m
ing out of Squiggy's death in 1990. On earlier, when she was with a girl friend,
the issue of damages, Laverne testifies Al and another male picked them up 0
that she and Squiggy had a happy and and offered to take them to a party,
affectionate marital relationship during but instead drove them to a secluded
their entire marriage. In defense, Shir- area, where Al and his friend forced the
ley seeks to introduce evidence of the women to submit to sexual intercourse.
fact that from 1981 to 1983, Squiggy The women escaped when the men were
separated from Laverne and lived with momentarily distracted. Al objects to
another woman. Laverne objects. The Peggy's testimony. The Judge should:
court should:
A. Overrule the objection, because

A. Sustain the objection, since the it is permissible character evi-
evidence is self-serving. dence.

B. Sustain the objection, because of B. Overrule the objection, because


the dead man's statute. the testimony is relevant to Al's
state of mind.
C. Overrule the objection, because
it is proper character evidence. C. Sustain the objection, because it
is impermissible character evi-
D. Overrule the objection, because dence.
the evidence is relevant.
D. Sustain the objection, because it
96. Al is charged with the rape of Kelly, a is too prejudicial.
17-year-old girl. At trial, the prosecu-
tion seeks to establish that Al and an-
other man picked up Kelly and offered
her a ride to a party. A few hours later,
after allegedly being picked up by Al,
Kelly's body was found in a ditch.

EVIDENCE 343
Q
97. Alex was fired from his job for drunken- 98. X is prosecuted for assault with a deadly
ness and incompetency. His supervisor weapon upon A. A testifies that X struck
had warned Alex about his conduct, but him with an iron pipe without cause.
was fed up when Alex failed to show up The prosecution then calls B to the
for work for three straight days. Alex stand, who testifies that he has known
retaliates by suing for back wages, and lived next door to A for ten years.
claiming that his employment contract The prosecution then asks B: "What
provided wages at below the mini- is the reputation as to whether A is an
mum wage and set forth a "waiver" of honest and a truthful man?" X objects.
overtime wages at any rate higher than The court should:
the regular hourly rate. His employer,
Andre Ristorante, disputes Alex's al- A. Sustain the objection, because
legations as to the contractual terms. opinion evidence of a witness'
Alex intends to testify as to the alleged character is never allowed to at-
wage rates set forth in his employment tack or support the credibility of
contract with his employer. Andre's at- the witness.
torney objects. The court should:
B. Sustain the objection, because
A. Admit the testimony, because A's character for truthfulness has
Alex has personal knowledge of not yet been attacked.
the contents of the contract.
C. Overrule the objection, because
B. Admit the testimony, because it it is relevant.
is a material issue in the action.
D. Overrule the objection, because
C. Not admit the testimony, because it is a collateral matter.
it is hearsay not within any ex-
ception. 99. Doug and Jon are arrested for robbing a
jewelry store. Jon makes a plea with the
D. Not admit the testimony, be- district attorney to turn state's evidence
cause the contract is the best against Patty for an unrelated crime.
evidence.
Doug is now on trial for the robbery.
William, Doug's roommate, is the sole
defense witness. Doug's counsel calls
William to the stand, to testify that
Doug told him (William) that Jon alone
robbed the jewelry store. The prosecu-
tion objects to William's testimony. The
testimony is:

344 EVIDENCE
m
A. Admissible, as a co-conspirator Questions 100 - 101 are based on
party admission. the following facts: m

B. Admissible, because it is direct Abel sues Cain for damages for personal C)
proof that Doug did not commit injuries arising out of a rear-end automobile
the robbery. accident. While testifying, Abel produces
a typewritten letter that states, "My foot
C. Inadmissible, as hearsay not slipped off the brake. Don't worry, I will see
within any recognized excep- that all your damages are paid." The letter
tion. is signed "Cain", but contains no return ad-
dress. Abel is asked by his counsel if he is
D. Inadmissible, because William's familiar with Cain's signature. Abel answers,
testimony is biased. "No, but I received the letter in the mail two
weeks after the accident." Abel offers the
letter into evidence.

100. Cain objects on the grounds of authen-


tication. The letter is:

A. Admissible, because the contents


of the letter lay the foundation
for authentication.

B. Admissible, because it is direct


proof that Cain was at fault.

C. Inadmissible, because Abel is not


familiar with Cain's signature.

D. Inadmissible, because the letter


is self-serving.

101. Suppose that Abel overcomes the au-


thenticity objection. Cain then objects
on the grounds of hearsay: The letter
is:

A. Admissible, because it is not-


hearsay.

EVIDENCE 345
Q
B. Admissible, because it is direct 103. Mark was charged with armed robbery
proof that Cain did cause the ac- of a 7-11 convenience store. At trial,
cident. Mark took the stand and denied that he
was the person who robbed the 7-11.
C. Inadmissible, as hearsay not The prosecution seeks to enter into
within any recognized excep- evidence that Mark had robbed two gas
objects to this evidence. The evidence
D. Admissible, as hearsay that falls is:
within a recognized exception.
A. Admissible, to prove Mark did
102. After being admitted to the Cinema rob the 7-11 store.
movie theater, Paul slipped on a loose
piece of carpeting in the theater lobby. B. Admissible, to prove a pertinent
Paul sustained injuries to his back. At trait of Mark's character.
trial, Cinema calls Wendy to testify
that a week before the accident Paul C. Inadmissible, because character
told her, "I cannot finish our golf game evidence may not be used to
because of pains in my back." Paul prove Mark acted in conformity
objects to the testimony. The court therewith.
should rule the testimony:
D. Inadmissible, because character
A. Admissible. evidence must be proven by
reputation or opinion evidence.
B. Admissible, as a present sense
impression. 104. While driving home from school,
Punkey hit Brandon as he was crossing
C. Inadmissible, as hearsay. the intersection. Brandon sued Punkey
for the personal injuries he sustained.
D. Inadmissible, as an improper lay At trial, Brandon testifies that imme-
opinion. diately after the accident, Punkey got
out of her car, raced over to Brandon,
and said, "Don't worry, I'll pay all
your medical bills." Punkey objects to
Brandon's testimony. The testimony
is:

A. Admissible, because it is an ad-


mission of liability.

346 EVIDENCE
Q
B. Admissible, because it is an ex- Questions 105 - 106 are based on 1-1

cited utterance. the following facts: 5


m
z
C. Inadmissible, because it is hear- Mike was a famous musician. While play- 0
m
say not within any recognized ing in a night club one night, he met Teena.
exception. The couple fell in love, were married, and
started a musical career together. However,
D. Inadmissible, because it is an Mike has a serious drug problem, and as a
rsff.v to bau mr.rlioal bilk result, he frequently becomes enraged when
when he was unable to come up with lyrics
to a new song, he "blind-sided" Thomas with
his fist. Thomas was seriously injured, but
never knew what hit him. Teena was the only
witness to the incident. The prosecutor files
battery charges against Mike and wants to
have Teena testify at trial as the only witness
to the incident. Teena objects because of her
fears it will ruin their careers.

105. Must Teena testify at trial?

A. Yes, because Teena is qualified


to testify as to her perceptions.

B. Yes, because she had personal


knowledge of the incident.

C. No, because she would be testify-


ing against Mike.

D. Yes, because Mike's conduct is


not considered a marital com-
munication.

EVIDENCE 347
106. Assume for this question, that Mike 107. Holmes has been a private detective
has been arrested for cocaine traffick- for the past 30 years and is well known
ing. After Mike's arrest, he privately for his ability to solve difficult crimes.
spoke to Teena, saying that he was Holmes has been unable to capture one
guilty, but that he would "beat the rap" suspected criminal during his career.
and they would make many years of The criminal, Moriarty, is a master of
music together. Teena was fed up with disguise, who began wearing disguises
Mike's ways and obtained a marital to engage in criminal activity as the
dissolution. She has now been called result of a large disfiguring scar on
to testify at Mike's trial concerning his left cheek that was caused when
Mike's statement to her. Upon ob- he once narrowly escaped capture by
jection by Mike's attorney, the court Holmes.
should:
Intent on turning the tables and put-
A. Exclude the evidence, because ting Holmes in jail, Moriarty poses
the statement was a confidential as a criminal defense attorney, after
communication. he finds out that Holmes is in trouble
with the law. Holmes is stopped for a
B. Admit the evidence, because the traffic violation and is found to have
testimonial privilege terminates cocaine in his possession. When he
upon dissolution of marriage. visits Moriarty's "law" office, Holmes
admits to Moriarty that he possessed the
C. Admit the evidence, because the cocaine for his personal use on the date
marital privilege is inapplicable in question. It is unknown to Holmes
to admissions. that Moriarty is not an attorney.

D. Admit the evidence, because After the visit, Holmes does not hire
Teena will testify voluntarily. Moriarty to represent him. Armed with
Holmes' confession, Moriarty calls the
district attorney and "cuts a deal" in
exchange for evidence of Holmes' guilt.
The prosecutor seeks to have Moriarty
testify at Holmes' drug possession trial.
Holmes' attorney objects. The testi-
mony of Holmes' confession is:

A. Inadmissible, because Moriarty


obtained the evidence through
clandestine means.

348 EVIDENCE
Q
B. Inadmissible, because of the at- 109. Bob was charged with the murder of ""
I

torney-client privilege. his wife, Valerie. At trial, the prosecu- 5


m
tion calls Terry to the stand. Terry was z
C. Admissible, as a statement the maid for the couple. Terry's pro- 0
m
against interest. posed testimony is that Valerie came
to her late one night and said, "Bob
D. Admissible, because Moriarty has poisoned me." Valerie died shortly
was merely acting as an attor- thereafter. Bob's counsel objects to the
ney. introduction of Valerie's statement to
Terry. The trial court should:
108. Railroad's train was involved in a se-
rious collision with Peter. As a result A. Permit the testimony, because it
of that collision, Eddie, the train's en- was a dying declaration.
gineer, was slightly injured and Peter
died. Peter's heirs bring a wrongful B. Permit the testimony, because it
death suit against Railroad. is relevant to Valerie's state of
mind.
At trial, Railroad seeks to introduce
Railroad's accident report filed by Ed- C. Exclude the testimony, because
die at the time of the accident. Eddie it is hearsay.
is no longer an employee of Railroad,
and has moved to an unknown loca- D. Exclude the testimony, because
tion in Mexico. Peter's heirs object to of the dead man's statute.
introduction of the accident report. The
report is: 110. While shopping at Vons, Bert slipped
and fell on a piece of candy on the floor
A. Admissible, because it is a busi- of an aisle. Bert sustained serious back
ness record. injuries and now brings suit against
Vons for his injuries.
B. Admissible, because Eddie is
unavailable to testify.

C. Inadmissible, because it is hear-


say.

D. Inadmissible, because it is not


the best evidence.

EVIDENCE 349
Q
At trial, Vons' counsel, Billy Davilla,
calls Mr. Clean, the store janitor, to the
stand. Mr. Clean testifies that he regu-
larly sweeps the floor every half hour
and that he never leaves anything on the
floor. He further testifies that if there
had been anything on the floor when
he swept, including a piece of candy,
he would have seen it. Bert's attorney
objects to Mr. Clean's testimony. The
court should:

A. Admit the testimony, because it is


relevant.

B. Admit the testimony, because it is


proper habit evidence.

C. Exclude the testimony, as specula-


tive.

D. Exclude the testimony, as preju-


dicial.

END OF QUESTIONS

350 EVIDENCE
EVIDENCE ANSWERS 3. D is the correct answer. A declarant's
statement as to her present state of mind
is admissible, if it tends to establish the
1. A is the correct answer. If a proper foun- declarant's state of mind or intent to
dation is laid, the photograph is admis- do something in some future time. A is
sible into evidence. This requires that incorrect because the statement is hear-
there be someone who can testify that say, but is admissible under the present
the photograph is a true and accurate mental state exception. B is incorrect
representation of the accident scene. because the defendant's mental state
Since anyone who was present at the is certainly probative, i.e., relevant - it
scene when the photograph was taken may establish her alibi. C is incorrect
can testify to the accuracy of the photo- because the statement is hearsay. It is
graph, the photographer does not have an out-of-court statement going to the
to be present. B is incorrect because the truth of the matter asserted, i.e., it is
burden is on Petunia, not Daisy, to lay a direct assertion of Mistress' state of
the foundation showing that the photo- mind — her intent to visit her relatives
graph is a true and accurate representa- — offered to prove that Mistress had
tion of the accident scene. C is incorrect that intent to show that Mistress acted
because the photographer does not have on her intent (i.e., that she, in fact, vis-
to be present. D is incorrect because the ited her relatives in another state.) This
photo could be taken by anyone; it does is the "Hillmon" use of the state of mind
not have to be an investigative agent. exception to the hearsay rule.

2. B is the correct answer. Mistress may in- 4. C is the correct answer. Questions which
troduce evidence of her good character point to possible prejudice or bias are
to show that she is not the type of person always relevant. A is incorrect, since,
likely to murder Wife. A is incorrect as a matter of law, bias and prejudice
because Mistress has not taken the stand do not go beyond the scope of direct
and put her credibility into issue. C is examination. B is incorrect because
incorrect because, at this point, Mistress the credibility of a testifying witness is
is not trying to prove her character for always a consequential (i.e., provable)
truthfulness. D is legally incorrect be- proposition. D is incorrect because it is
cause reputation is a permissible way an incorrect statement of law.
to prove character.

EVIDENCE 351
5. B is the correct answer. Although there 6. D is the correct answer. The general rule
is nothing wrong with the prosecutor's is that other crimes or wrongs are not
question, the prejudicial effect on the admissible to prove a person's character
jury may be great. Therefore, the judge (i.e., propensity to act a certain way)
has the discretion to decide that the to show action that the person acted
probative value is substantially out- in conformity with his character (i.e.,
weighed by the potential prejudice. A propensity). However, other crimes,
is incorrect because a question tending wrongs or acts are admissible for a
to prove bias is never beyond the scope non-character purpose, i.e., to show
of direct examination. C is incorrect identity (by modus operandi). That is,
because leading questions are permitted other crimes are admissible only if, as
on cross-examination. D is incorrect, as here, the hand-gun used by Sam in the
it is an incorrect statement of law. prior robbery against Pete is so unique
that it tends to establish the identity of
Sam as the one who robbed Joe. Here,
Sam's use of a psychedlic-painted gun
to rob Pete is a "signature" crime. The
psychedelic-painted gun is unique
enough to be relevant, in a non-char-
acter way, to establish Sam's modus
operandi. A is incorrect because, in this
fact pattern, Sam's character is not "in
issue". B is incorrect because, although
the evidence may be prejudicial, the risk
of causing unfair prejudice to Sam (by
showing his propensity to rob) does not
substantially outweigh the evidence's
strong probative value (to show Sam's
identity through M.O.). C is incorrect
because mere possession of the gun
would not show Sam is capable of
armed robbery.

352 EVIDENCE
7. B is the correct answer. Under the 8. D is the correct answer. The F.R.E. al-
Federal Rules of Evidence [hereinaf- low a prior inconsistent statement to
ter F.R.E.1, Rule 803(1), there is an be used for impeachment purposes as
exception to the hearsay rule for pres- well as substantive evidence of the fact
ent sense impressions. Since Wilma in question, if the statement was made
made a statement about a condition under oath, at a proceeding (including
she observed while she was observing a deposition), and the declarant is avail-
it, this would qualify the statement as a able to testify concerning the statement.
present-sense impression. A is incorrect A is incorrect because, under the F.R.E.,
because the excited utterance exception one may impeach his own witness. B is
requires that the statement was made incorrect because a prior inconsistent
under the stress of an exciting event. statement, under oath, is not hearsay.
This is not the case, under these facts. C is incorrect because it is not the best
C is incorrect because a prior consistent answer. Although it can be used to
statement is used to rebut a charge of refresh Pete's memory, the better use
recent fabrication. Wilma's credibility here would be to impeach and can also
as a witness has not been attacked (i.e., be used as substantive evidence. There-
impeached) as a recent fabrication. D is fore, D is the better answer under the
incorrect since the state of drunkenness facts.
is the type of subject a lay person may
give an opinion on. 9. D is the correct answer. F.R.E. 407 ex-
cludes evidence of subsequent remedial
measures. The rationale falls under
public policy; if such evidence could
be used to establish the prior existence
of a dangerous condition or practice,
the person liable would be reluctant
to take corrective action. A, B and C
are incorrect for the reasons stated in
answer D.

EVIDENCE 353
10. B is the correct answer. Under F.R.E. 12. B is the best answer. Under F.R.E.
409, evidence that a party has offered 901, a letter must be authenticated be-
to pay medical expenses is not admis- fore it will be admitted into evidence.
sible to prove liability. However, any F.R.E. 901(a) states, "the requirement
admission of fact made in connection of authentication or identification as a
therewith is admissible. A is incorrect condition precedent to admissibility is
because, under an offer to compromise satisfied by evidence sufficient to sup-
accompanying the admission, it would port a finding that the matter in ques-
most likely be inadmissible. C is in- tion is what the proponent claims." A
correct because an admission made by is an incorrect statement of law - there
a party is an exception to the hearsay is not an absolute rule that the Court
rule. D is incorrect under F.R.E. 409, receive the original document. C is an
admissions of fact accompanying the incorrect answer, as the issue here is
offer are admissible. authentication. D is incorrect because,
under the F.R.E., the letter need not be
11. C is the correct answer. For the physi- authenticated by an expert.
cian-patient privilege to attach, the
information obtained from the patient 13. B is the correct answer. Attorney-cli-
must be necessary to enable the physi- ent communications remain privileged
cian to treat the patient. Here, Angelo's even when made in the presence of joint
statements to the doctor do not relate to clients or others, if the others are "rea-
treatment and, thus, are not protected sonably necessary" to the consultation.
by the privilege. A is incorrect for the A is incorrect because the testimony
reasons stated in correct answer C. B is not offered to prove the contents of
is an incorrect statement of law. Confi- Sneak's notes - thus, the best evidence
dentiality, which is an essential element rule is inapplicable. C is incorrect be-
to the formation of the privilege, would cause the other persons present were
not be impaired by the fact that the ad- reasonably necessary to the consulta-
mitting nurse overheard Angelo's com- tion. Thus, the privilege attaches. D is
munication to the doctor. D is incorrect incorrect because Ace and Owner are
because, whether Angelo intended this joint clients, and the attorney-client
statement to be confidential or not privilege would attach.
would not, the privilege did not "form"
for the reasons stated in answer A.

354 EVIDENCE
14. A is the correct answer. A business re- 16. C is the correct answer. Under F.R.E.
port prepared as a communication from 404(b), evidence of other crimes,
client to attorney is privileged. B and C wrongs, or acts is not admissible to
are incorrect because even though a re- prove the character of a person in order
port contains hearsay or is self-serving, to show that he acted in conformity
it does not prevent its discovery by the therewith. However, it may be admis-
opponent. D is incorrect for the reasons sible for other purposes, such as proof
supporting answer A. of intent, motive, preparation, opportu-
nity, knowledge, plan, scheme or iden-
15. A is the correct answer. F.R.E. 803(1) tity. Under these facts, this evidence
holds that a present sense impression may be used to show Deft's intent as
and excited utterance are not excluded well as common scheme or plan. A is
by the hearsay rule, even though the incorrect, as it is an incorrect statement
declarant is available as a witness. A of law. This evidence is very relevant
present sense impression is a statement to the charges against Deft. Both B and
describing or explaining an event or D are incorrect since they miss the key
condition made while the declarant legal issue of utilizing the evidence to
was perceiving the event or condition, show intent.
or immediately thereafter. Thus, Ella's
statement would qualify. B is incor- 17. A is the correct answer. Because there
rect because the declarant, Ella, does was self-identification and a phone
not have to be unavailable. C is incor- book, the voice would be considered
rect because although the statement is properly identified. B is incorrect be-
hearsay, it falls within the present sense cause the accuracy of a telephone book
impression exception. D is an incorrect is not an easily verifiable fact. C is in-
statement of law. Although Ella is not a correct because Farmer does not have
legal adult (18 years of age), she is still to be familiar with Rancher's voice for
able to perceive and communicate. authentication. D is an incorrect state-
ment of law.

EVIDENCE 355
18. C is the correct answer. The general 20. B is the correct answer. The question as
rule is that in a civil case, evidence of to whether Valor was high 18 months
character to prove probable conduct is prior to the accident is collateral to the
inadmissible. A is incorrect because of issue of whether David ran the red light.
the reasoning stated in correct answer A witness may not be impeached on a
C. B is incorrect because reputation matter not directly relevant to the issues
testimony is not "habit" evidence. Also, in the case. A is incorrect because the
being a "dare devil" rider is too general F.R.E. allow for the impeachment by
to be classified as "habit". Habit is a specific acts of conduct. C is incorrect
specific and unvarying response to a because Valor's use of marijuana is not
specific situation. D is incorrect because logically relevant to show that Valor has
this answer states the rule for criminal an untruthful character. D is incorrect
cases, not civil. because Sam's use of marijuana 18
months prior to the event to which he
19. A is the correct answer. Porter's testi- testified in court is not logically relevant
mony is relevant in two ways: first, it to show an impaired memory of that
is substantive evidence that tends to es- event.
tablish a consequential proposition, i.e.,
that David, in fact, ran the red light and, 21. D is the correct answer. Although the
second, it impeaches Witt's testimony statement is hearsay, F.R.E. 803(2)
by contradicting him (i.e., by showing allows a statement relating to a star-
that Witt's testimony — that David's car tling event or condition made while
did not run the red light — was false). B the declarant was under the stress of
is not correct because the mere fact that excitement caused by the event or
Porter was surprised does not give him condition. This is known as the excited
the right to impeach. C and D are both utterance exception. A is incorrect be-
incorrect. Under F.R.E. 607, any wit- cause although the statement preceded
ness may be impeached by any party. the accident, it was still made relating
to a startling event, allowing for the
F.R.E. exception. B is incorrect because
unavailability is not a requirement. C is
incorrect because although it is hearsay,
it falls within an exception.

356 EVIDENCE
22. B is the correct answer. Here, Driver's 25. B is the correct answer. Under the doc-
failure to deny Polly's statement may trine of present recollection refreshed, a
constitute an admission by silence. A witness may be shown anything which
is an incorrect statement of law. C is may reasonably refresh the recollection.
incorrect because no facts support it. A is incorrect because past recollection
D is incorrect because no foundation recorded introduces the contents of a
is required for an admission. writing into evidence and requires the
laying of a foundation. C is incorrect
23. C is the correct answer. Although the because the time lapse factor here is
statement is hearsay, it is admissible not enough to prevent use of the letter.
under the F.R.E., which allow for the D is incorrect because evidence may
statement of the declarant describing not be excluded merely because it is
his own physical sensation or condition. self-serving.
A is incorrect because self-serving evi-
dence may be admissible. B is incorrect 26. B is the correct answer. Whenever a
because it falls within the F.R.E. 803(3) party's character is at issue under the
exception for physical condition. D is pleading, character evidence is fully
incorrect because only an expert could admissible. In this case, character is
testify to whether the hip was fractured an ultimate fact in dispute, and it must
or not. be proved by competent evidence. Al-
though A is legally correct, it is not the
24. A is the correct answer. Settlement of- best answer because the more relevant
fers, although they may be relevant, are inquiry is whether evidence of character
inadmissible. This policy is to encour- is available in the case-in-chief. C is
age settlement negotiations. B, C and incorrect because the statement is not
D are incorrect for the reasons stated hearsay. Officer Smith is testifying di-
in answer A. rectly. D is incorrect because character
is directly at issue in a defamation case,
i.e., such is highly probative.

EVIDENCE 357
27. C is the correct answer. A communica- 29. D is the correct answer. An admission
tion made during marriage is privileged, is an out-of-court statement made by a
even after the marriage has ended. party which is offered at trial against
Either spouse may assert the privilege, that party by the opposing party. A
to bar the other from testifying. A is is incorrect because an admission is
incorrect for the same reasons that C not inadmissible hearsay, under the
is correct. B is incorrect because the F.R.E. B is incorrect as no foundation
marital privilege may be asserted in is required. C is incorrect because an
both civil and criminal proceedings. D admission is admissible whether or not
is incorrect because "spite" has noth- it discredits prior testimony.
ing to do with whether Felicia will be
allowed to testify or not. 30. A is the correct answer. The accused in
a criminal case may always offer evi-
28. D is the correct answer. A is incorrect dence of her good character to show that
because the color of the jacket is not of she was unlikely to commit the crime
consequence to the action. B is incor- charged. B is an incorrect statement of
rect because Wally's testimony is not law. C is incorrect because Hilda does
relevant to show Wally's untruthful not have to take the stand for Lionel's
character. Wally's is relevant to contra- testimony to be admissible. D is incor-
dict Wally on the color of Shove's jacket rect for the reasons A is the correct
which tends to undermine Wally's cred- answer.
ibility as a witness by casting doubt
on how attentive he was in observing 31. A is the correct answer. The doctor-pa-
the event in question. However, the tient privilege protects any communica-
color of Shove's jacket is a collateral tion that was intended to be confidential
matter because its only relevance is to and was made for the purpose of medi-
contradict Wally. Extrinsic evidence is cal diagnosis and treatment. This privi-
inadmissible to impeach a witness on a lege protects any information obtained
collateral matter. C is incorrect because by verbal statements, examination,
Walt's capacity to observe is a relevant and any diagnosis and treatment. The
issue. patient is the holder of the privilege, but
if the patient is not present at trial, the
Doctor is ethically obligated to assert
the privilege on the patient's behalf.
The facts state that Dr. Perry was called
in to testify as a defense witness in the
case Jones v. Smith, and since Loll is
not present, Dr. Perry must assert the
privilege. B, C and D are incorrect for
the reasons stated in answer A.

358 EVIDENCE
32. C is the correct answer. As a general 34. D is the best answer. Since the facts
rule, expert testimony is usually im- state that the hallway had been waxed
proper when the jurors are competent approximately one hour before the ac-
to draw their own conclusions on the cident, there was no substantial identity
issue. Negligence for failing to file of material circumstances between the
within the statute of limitations in this freshly waxed condition of the floor,
example is a conclusion which jurors and the floor which was used by 11,000
are competent to draw from the facts other people without incident over the
presented at trial. A and B are incorrect prior one-week period. A and B are
for the reason stated in correct answer incorrect for the reasons stated in the
C. D is an incorrect statement of law correct answer D. C is incorrect be-
because the judge is not allowed, in cause evidence of the absence of prior
a jury case, to resolve the issue of the accidents to show the nonexistence of
existence of negligence in the action a dangerous condition or causation is
before her. subject to the "substantial similarity"
requirement. Under the stated facts,
33. A is the correct answer. Client must the hallway floor had just been waxed
prove that "but for" the lawyers' neg- an hour before Paul walked on it. Willy
ligence she would have recovered. B, did not lay a foundation showing that
C and D are incorrect for the reasons the floor was in substantially similar
stated in answer A. condition (i.e., excessively waxed) the
previous week when the 11,000 people
walked on it. Because the substantial
similarity requirement was not met, D
is the best answer.

35. A is the correct answer. There is no rule


which disqualifies a person with mental
illness or defect from testifying. The
test of competency is not insanity, but
only whether a witness can appreci-
ate the obligation to tell the truth and
have sufficient ability to perceive and
communicate perceptions. B, C and D
are incorrect for the reasons stated in
answer A.

EVIDENCE 359
36. D is the correct answer. F.R.E. 408 pro- 40. A is the correct answer. In a criminal
vides that any evidence that one of the case the court shall instruct the jury
parties offered to settle or compromise that it may, but is not required to, ac-
the claim against another is inadmis- cept as conclusive any fact judicially
sible to show liability. A, B and C are noticed. By the judge taking judicial
incorrect for the reasons stated in the notice, such merely allowed the jury to
answer D. accept the fact as true, or not, that Birdie
drove across state lines (an element in
37. C is the correct answer. The placing the crime charged). B is an incorrect
of a speed governor on the truck is a statement of law. C is incorrect for the
subsequent remedial repair. F.R.E. 407 reasons stated in correct answer A. D
prohibits evidence of subsequent repairs is incorrect because this is a criminal
to prove negligence, culpable conduct, case.
or product defect because public policy
favors making such repairs. A, B and D 41. D is the correct answer because Abel's
are incorrect for the reasons stated in committing the hoax on the church is a
answer C. specific act of misconduct which may
not be proven by extrinsic evidence.
38. A is the correct answer. A statement Therefore, his purported testimony is
made by a party's agent or employee inadmissible pursuant to F.R.E. 608. A,
concerning a matter within the scope B, and C are incorrect for the reasons
of his agency or employment may be stated in answer D.
offered against the party. Under the
F.R.E., no authority to speak is required. 42. B is the correct answer. If a witness'
C is incorrect because an admission testimony concerning a crime could
does not require proof of unavailability lead to a criminal conviction, the privi-
to be admissible. B and D are incorrect lege against self-incrimination can be
for the reasons stated in answer A. claimed. However, in a criminal case,
after direct examination of a witness,
39. A is the correct answer. Jamie's state- the direct testimony can be stricken
ment to Officer is a party admission. if the witness invokes the privilege so
Under the F.R.E., an admission is ex- that a defendant is not denied his con-
empted from the hearsay rule. B, C and stitutional right to confrontation. A is
D are incorrect for the reasons stated in incorrect because there need not be a
answer A. charge pending, in order to invoke the
privilege. C and D are incorrect for the
identical reasons stated in answer B.

360 EVIDENCE
43. A is the correct answer. Under F.R.E. 46. A is the correct answer because Loaden
609, evidence of being convicted of a received the invoice from the shipping
crime shall be admitted, if the crime was clerk as part of his regular-conducted
punishable by death or imprisonment business responsibilities. His testimony
in excess of one year, if the accused is merely authenticated the invoice and is
the witness, and if its probative value sufficient to lay a foundation in order
outweighs the prejudicial effect to the to allow the shipping invoice into evi-
defendant. B is an incorrect statement of dence. B is incorrect because Loaden
law. C is incorrect because the question had no personal knowledge of the
deals with impeachment. D is incorrect merchandise listed upon the invoice.
because F.R.E. 609 provides notice C is incorrect for the reasons stated in
must be given for crimes more than ten answer A. D is incorrect because the
years old. Here, the crime took place invoice qualifies as a business record,
exactly ten years ago; thus, requiring an exception to the hearsay rule.
no notice.
47. C is the correct answer. Hearsay is an
44. B is the correct answer. The affidavit out-of-court statement offered to prove
is being offered in evidence to prove the truth of the matter asserted. Here,
Jack's incompetency (i.e., offered for the statement is offered to prove that
the truth), and is an out-of-court state- Ginger hit James. A is incorrect because
ment by the declarant, Henry. The af- a present sense impression is a state-
fidavit is inadmissible hearsay, unless ment describing or explaining an event
there is an exception. Based upon the or condition made at the time of the
facts, there is no exception that will ap- event or immediately thereafter. A ten-
ply. A is incorrect because lay opinion minute time lapse between the accident
as to common perceptions is permitted. and the statement made, allows time for
C and D are incorrect for the reasons fabrication. Thus, the statement is not
stated in answer B. trustworthy and does not qualify as a
present sense impression. B is incorrect
45. D is the correct answer because a for the reason stated in answer C. D is
criminal defendant, after being read his an incorrect statement of law.
Miranda rights, has a right to remain
silent. Although an adoptive admission
by silence can generally be used against
a party who manifested his adoption or
belief in its truth by conduct or silence,
here it cannot, since Neal is exercising a
constitutional right. A, B and C are in-
correct for the reasons stated in answer
D.

EVIDENCE 361
48. A is the best answer. Pursuant to F.R.E. 49. D is the best answer by a process of
803(3) (and California Evidence Code elimination. There is no applicable
1250), a person's statement concerning rule that renders Raymond's testimony
his then-existing physical condition is inadmissible. "Absence of complaint"
admissible when the declarant's physi- to prove that a condition did not exist
cal condition at a specified time is at is not hearsay. A is incorrect because
issue and the statement relates to the there are no facts to indicate Raymond's
time his condition is at issue. Although statement is reliable. Raymond, as a
Raymond is claiming permanent in- plaintiff-witness, is biased. Further-
juries resulting from the incident, he more, A is not based on any applicable
was observed swimming in the hotel legal principle. B is incorrect because
pool later in the same day. Because Raymond did not testify to his opinion,
these facts raise an issue of whether but simply that, as a matter of fact, he
Raymond suffered injuries as a result never complained of knee pain before
of the incident, his statement of physi- his fall. C is incorrect for the same rea-
cal complaint to Jean Paul immediately son that B is incorrect.
after his fall is admissible as evidence
that he was, in fact, injured. B is incor-
rect because it does not provide a suf-
ficient basis for admitting Raymond's
statement. C is incorrect because there
is no requirement that the declarant's
statement be made to a treating doctor.
D is incorrect because Raymond, as a
lay person, is permitted to describe his
own condition concerning knee pain.

362 EVIDENCE
50. B is the correct answer. Although 51. B is the best answer. F.R.E. 409 provides
Raymond's statement to Dr. Cure is that evidence of furnishing, offering or
hearsay, if offered to prove the truth promising to pay medical, hospital or
of Raymond's assertion, i.e., that he similar expenses for an injury is not ad-
suffered knee pain after helping a col- missible to prove liability for the injury.
lege buddy move, the statement is ad- However, an admission is admissible
missible hearsay under F.R.E. 803(4). when made in connection with an offer
Under this rule, a statement made for or promise to pay medical or hospital
the purpose of medical diagnosis or expenses, although the offer or promise
treatment and describing medical his- is inadmissible. A, C and D are incorrect
tory, or past or present symptoms, pain for the reasons stated in answer B.
or sensations, or the inception or gen-
eral character of the cause or external
source thereof insofar as reasonably
pertinent to diagnosis or treatment, is
admissible. A is incorrect because it
does not address the hearsay issue. C is
incorrect because Raymond's statement
to his physician is not covered by the
physician-patient privilege. The physi-
cian-patient privilege, under California
Evidence Code 994, grants a privilege
to a patient to refuse to disclose, and to
prevent his physician from disclosing,
any confidential information obtained
by his physician while attending the
patient. However, under the "patient-
litigant exception," the privilege cannot
be claimed in a personal injury lawsuit
where the patient relies on a physical
condition as an element of his claim
or defense. D is incorrect because B is
correct.

EVIDENCE 363
52. C is the best answer. Stan only testi- 53. C is the best answer. Under F.R.E.
fied to part of Don's statement to Paul, 803(1), a declarant's statement describ-
leaving out the remainder of the state- ing or explaining an event or condition
ment pertaining to the accident which made while the declarant was perceiving
diminishes the force of Don's apparent the event or condition, or immediately
admission of fault. On cross-examina- thereafter, is an exception to the hear-
tion, Don's attorney seeks to elicit from say rule. Marie's statement concerning
Stan the remainder of Don's statement Larry's failure to stop for the red light
pertaining to the cause of the accident, described the incident. Because Marie
in order to give the trier of fact a more was conscious for only a few moments
accurate understanding of what Don after the accident, her statement was
said to Paul about the cause of the ac- necessarily made immediately after the
cident. A judge might rule that Paul has accident occurred. Therefore, Marie's
"opened the door" to the remainder of statement is a present sense impression.
Don's statement to Paul. A is incorrect A, B and D are incorrect for the reasons
because it assumes facts not contained explaining answer C.
in the fact pattern (i.e., that there was
a police report) and, furthermore, the 54. B is the best answer. Deputy Fife is
best evidence rule is inapplicable here merely testifying in regard to his ob-
because Stan's testimony is not being servations of Pete's appearance when
offered to prove the contents of a writ- he was arrested. Fife's testimony was
ing (Stan testified to what he overheard not admissible as a prior identification
Don say to Paul). B is incorrect because because admissibility on such grounds
Don's attorney is not offering Don's of- requires the witness to make a statement
fer to pay Paul's medical bills in order of identification prior to trial which did
to prove Don's fault. Under F.R.E. 409, not occur in these facts. A, C and D are
evidence of offering to pay medical ex- incorrect for the reasons stated in the
penses is inadmissible only when used answer B.
to prove liability for the injury. D is
not the best answer because it states an
incorrect legal principle. By process of
elimination, C is the best answer.

364 EVIDENCE
55. C is the correct answer. The attorney- 57. C is the correct answer. Under F.R.E.
client privilege protects those com- 405, character or trait of character of a
munications which the client either person, if an essential element of a claim
expressly made confidential or would or essential to the case of the defense,
reasonably be assumed, under the may be proven by specific instances of
circumstances, to be confidential. In conduct. Bud sued Troy for negligently
the case where a lawyer has a clerk or allowing Don to use his car ("negligent
secretary present, the privilege is still in entrustment").Don's propensity to drive
effect, even if the client does not retain carelessly is an element of the cause
that attorney. A and B are incorrect for of action for negligent entrustment. A,
the reasons stated in answer C. D is an B and D are incorrect for the reasons
incorrect statement. stated in answer C.

56. A is the correct answer. Under the F.R.E. 58. C is the correct answer. Under F.R.E.
and California Evidence rules, a prior 405(a), "in all cases in which evidence
inconsistent statement in a deposition of character is admissible (including
may be used as substantive evidence. where character is an element of a claim
Comment to F.R.E. 613(b) says that, or defense), proof may be made by tes-
although foundation must be laid for timony as to reputation or by testimony
extrinsic proof of a prior inconsistent in the form of an opinion." A is incor-
statement, the witness does not have to rect because habit refers to a routine,
be shown the prior inconsistent state- activity, or response frequently repeated
ment before introducing the intrinsic over a protracted period of time. B is
evidence, as long as the witness, at incorrect because reputation evidence
some time during trial, has the op- of character, although hearsay to prove
portunity to explain. B is incorrect the character trait, is admissible under
because unavailability to testify is not F.R.E. 803(21) which creates a hearsay
a requirement under the rule stated in exception for reputation as to character.
A. C is incorrect for the reasons stated D is incorrect because Don's reputa-
for A. D is incorrect because intentional tion as a careful driver is not relevant
untruthfulness is not a requirement for to prove bias.
admissibility of deposition testimony
under the rule stated in A.

EVIDENCE 365
59. C is the best answer. The statement is 62. A is the correct answer. Where facts
not hearsay because it is being used to are attempted to be shown through
circumstantially show Sally's knowl- written records, the best evidence rule
edge that Brian was in fact still alive generally applies because the contents
when he was shot. A and B are incorrect of the writing are then being proved.
because the statement was not hearsay However, a qualified expert may testify
- it was not being used to prove whether as to summaries of complex records,
Brian "was going to die." D is incorrect even though the records themselves are
because firsthand knowledge is not the not introduced. The only requirement
test as to whether a particular statement attached to the admissibility of Alvin's
is hearsay or not. testimony, is that the original books be
available for inspection by the adver-
60. A is the best answer. The Best Evidence sary. Here, the books are available for
Rule requires the painting, which can be inspection. B, C and D are incorrect for
easily produced, to be introduced into the reasons stated in answer A.
evidence to show whether the scene
portrayed is libelous or not, a key is- 63. D is the correct answer. F.R.E. 704(a)
sue in the case. B is incorrect because abolished the "ultimate issue" objection
Witness testified that she observed Ben that once prevented lay and expert wit-
hanging the painting she described. C nesses from giving testimony phrased
is incorrect because there are no facts in terms of the ultimate facts that the
indicating that Witness is competent to jury must determine. Modernly, opinion
testify whether or not it was Marie's testimony is admissible if it would be
signature on the painting. D is incorrect helpful to the jury. See F.R.E. 701 (lay
for the reasons stated for answer A. opinion) and 702 (expert opinion).With
respect to this question, while lay or
61. B is the correct answer because the tes- expert opinion as to the legal conclu-
timony of Paul proves the earnings of sion that the decedent had the " capacity
the partnership in which he has personal to make a will" is improper, because
knowledge, and is relevant. A is incor- unhelpful, lay and expert opinion as
rect because it is immaterial whether to the factual basis of such capacity is
the books can be inspected to decide admissible as an aid to the jury because
whether Paul's testimony is permissible it would assist the jury in determin-
or not. C is incorrect because Paul is tes- ing the legal issue of the decedent's
tifying to facts in which he has personal "testamentary capacity." An expert is
knowledge, not to the contents of the expected to bring to bear principles of
books. D is incorrect because Paul is not his or her specialty when testifying as
testifying to the contents of a writing. to an ultimate issue of fact. A, B and C
are incorrect for the reasons stated in
answer D.

366 EVIDENCE
64. C is the correct answer. F.R.E. 407 67. D is the correct answer. A lay witness
permits admissibility of evidence of may testify in the form of opinions
subsequent remedial measures when and inferences, which are rationally
offered to prove ownership and control. based on the perception of the witness
A, B and D are incorrect for the reasons and helpful to a clear understanding of
stated in answer C. either his testimony or a fact in issue.
A, B and C are incorrect for the reasons
65. A is the correct answer. In a criminal stated in answer D.
case, self-interest may be shown when
the witness testifies for the state and 68. D is the correct answer. A confidential
it is shown that a criminal indictment communication between an attorney
has been forgiven or lessened for her and his client pertaining to their rela-
testimony. B, C and D are incorrect for tionship is privileged from disclosure.
the reasons stated in answer A. Any communications made in the
course of preliminary discussions with a
66. C is the correct answer. This question view of employing the lawyer are privi-
is testing the student's understanding leged, even though the employment is
of impeachment. F.R.E. 608(b) permits not accepted. The presence of Stan's
cross-examination into specific in- father did not remove the element of
stances of misconduct, which have not "confidentiality." Since Stan is 6 years
resulted in a criminal conviction, if such old, his father's presence was necessary
misconduct is pertinent to truthfulness for the attorney to render legal service
or veracity. However, this rule prohibits to Stan. Thus, the presence of A, B and
the use of extrinsic evidence to establish C are incorrect for the reasons stated in
a prior bad act. Defense counsel cannot answer D.
use Tom's testimony for 2 reasons: (1)
because it is extrinsic evidence and (2) 69. D is the correct answer. The best evi-
because Tom's testimony regarding the dence rule provides, where a writing
purchase of marijuana is not pertinent contains terms which are material, the
of truthfulness or veracity. A and B original writing must be produced,
are incorrect for the reasons stated for unless reasons are shown as to why
answer C. D is an incorrect statement it is unavailable or unless the writing
of law regarding competency to testify pertains to a collateral matter. Here, the
by a lay person. reference to the newspaper is collateral
to the issue of the finalization date. A,
B and C are incorrect for the reasons
stated in answer D.

EVIDENCE 367
70. A is the correct answer. Under F.R.E. 72. D is the correct answer. Under F.R.E.
609 evidence of a public record of a 609, evidence that an accused was
criminal conviction involving dishon- convicted of a crime can be used to at-
esty or false statement is admissible for tack the credibility of the accused who
the purpose of attacking the credibility testifies as a witness, if the crime was
of a witness. B is incorrect because the punishable by death or imprisonment
reference to the newspaper story is not in excess of one year and the probative
being offered to prove that there was, in value of the conviction outweighs its
fact, an earthquake in L.A., but merely prejudicial effect, or the crime involves
to fix the date, in Cam's mind, on which "dishonesty or false statement" (wheth-
the Zit Zap Gun was finalized. C and D er a felony or a misdemeanor). A crimi-
are incorrect for the reasons stated in nal conviction over 10 years old may be
answer A. admitted within the court's discretion,
if it is found that the probative value
71. A is the correct answer. F.R.E. 412 substantially outweighs the prejudicial
permits evidence of specific instances effect, and the adverse party was given
of past sexual behavior of the victim written notice that the conviction was
with the accused regarding the issue to be used. Since no written notice was
of the victim's consent. B is incorrect given to the defense before trial, even if
because it is an incorrect statement of the court finds that the probative value
law. C is incorrect because F.R.E. 412 substantially outweighs the prejudicial
will permit this type of testimony as effect, the objection must be sustained.
circumstantial evidence that defendant A, B, and C are incorrect for the reasons
was on intimate sexual terms with the stated for answer D.
victim. D is incorrect because A is the
correct answer. 73. D is the correct answer. Under F.R.E.
408, evidence that one party offered to
settle or compromise the claim against
another is inadmissible to show liabil-
ity, unless it is offered to prove bias, or
prejudice, or to negate the contentions
of undue delay, or to obstruct a criminal
conviction. Here, the settlement offer
had an admission within it, but any
statements made by the parties when
trying to settle are inadmissible. Note
that the F.R.E. do not permit sever-
ance of admissions of liability from
compromise discussions. A, B and C
are incorrect for the reasons stated in
answer D.

368 EVIDENCE
74. A is the correct answer. A confidential 77. C is the correct answer. Under F.R.E.
communication between an attorney 704, testimony in the form of an expert
and her client pertaining to their rela- opinion is not objectionable because it
tionship is privileged from disclosure. embraces the ultimate issue or issues to
Bonnie's communication to Steve, be decided by the trier of fact. However,
while made with a view of employing pursuant to F.R.E. 704(b), an expert
Steve, is privileged, even though the who testifies in regard to a criminal
employment is not accepted. B, C and defendant's mental state or condition
D are incorrect for the reasons stated in may not testify as to whether the defen-
answer A. dant did or did not have the mental state
or condition constituting an element
75. A is the correct answer. Under F.R.E. of the crime charged or of a defense
407, evidence of subsequent repairs or thereto. Thus, the jury is permitted to
other precautionary measures made fol- draw its own conclusions on the issue.
lowing an accident are inadmissible to A is incorrect even if an expert is prop-
prove negligence or culpable conduct. erly qualified to testify on matters of
However, such measures are admis- mental state or condition. B is incorrect
sible to show ownership, control, or because the evidence is improper, even
feasibility of precautionary measures, if it were uncontroverted. D is incorrect
if controverted, and are also admissible because C is the better answer, and there
for purposes of impeachment. Since is no evidence of bias presented in the
Mr. Roper denied responsibility, but facts.
went ahead and hired Larry to remove
the tree, the evidence is admissible to
show his control in making the repairs.
B, C and D are incorrect for the reason
stated in answer A.

76. A is the correct answer. Pursuant to


F.R.E. 409, evidence of furnishing,
offering or promising to pay medical,
hospital, or similar expenses occasioned
by an injury is not admissible to prove
liability for the injury. Funeral expenses
are considered to be "similar expenses"
by the court, under this rule. B, C and
D are incorrect for the reason stated in
answer A.

EVIDENCE 369
78. A is the best answer if the evidence of 79. D is the correct answer. This question is
prior forgetfulness is offered to show intended to further clarify the answer to
that Yvette tends to be forgetful. The Question 78. In contrast to Question 78,
testimony of Stephanie is specific in- the incident of forgetfulness in Question
stance evidence of Yvette's propensity 79 occurred only two hours before the
to be forgetful to prove that Yvette was incident at Nordstrom's (where Yvette
forgetful on the specific occasion in was charged with shoplifting) and is
question (i.e., that she forgot she had more closely related to the incident at
placed the blouse on the outside of her Nordstrom's, involving store merchan-
stroller). While it is true that evidence dise. Yvette's act of leaving behind a
showing Yvette forgot that she put the diamond ring for which she had, in fact,
blouse on the outside of the stroller paid, tends to support her story that she
tends to show that Yvette was mentally similarly forgot that she had placed the
incapable of forming an intent to steal blouse on the baby carriage as she left
the blouse, the evidence does so by Nordstrom's.
showing that Yvette acted forgetfully
(walking out of the store without her 80. C is the correct answer. F.R.E. 702
baby) on a prior occasion. B is incorrect provides that a witness qualified as an
because Stephanie is not testifying as expert by knowledge, skill, experience
to her opinion, but instead is testifying or training may testify on matters of
concerning facts of a prior circum- scientific, technical or other specialized
stance. Although C may be factually knowledge thereto, in the form of an
correct, it is not the best answer. D is opinion, if it will assist the trier of fact to
the second best answer because it could understand the evidence or to determine
be argued that Yvette lacked the mental a fact in issue. Here, Burt's testimony
capacity to form the requisite intent to will assist the trier of fact to determine
steal the blouse (forgetting she had it whether or not Yvette intended to take
when she left the store) because she had the blouse without paying for it. A and
the same mental state leaving the same B are incorrect for the reasons stated
store on a prior occasion (forgetting her for answer C. D is incorrect because it
baby). This reasoning is weak because, is an incorrect statement of law.
when offered to prove intent or lack
of intent, the prior acts generally must 81. B is the correct answer. F.R.E. 404(a)
involve similar conduct committed requires that evidence of a character
under similar circumstances. Leaving a trait must be relevant to the issue being
store with a blouse, without paying for tried. Here, non-violence is irrelevant
it, and leaving the same store without to the issue of innocence of the crime
one's baby, six months previously, are of selling narcotics. A, C and D are in-
arguably very different circumstances. correct for the reasons stated in answer
B.

370 EVIDENCE
82. A is the correct answer. Where a doctor 85. B is the correct answer. F.R.E. 803(1)
examines a party to enable the client to provides an exception to the hearsay
communicate his or her condition to the rule, where a statement describing or
attorney, the attorney-client privilege explaining an event or condition is
applies. Note that where an attorney made while the declarant is perceiving
employs a physician to examine the an event or condition, or immediately
client, the physician's report may not thereafter. Here, Lucy's statement is
be privileged under the physician-pa- admissible because she made it while
tient privilege because no treatment is observing Fred's driving manner and
contemplated. Therefore, Dr. Cathy's her statement describes Fred's driving
opinions, which are based on com- manner. A is an incorrect statement.
munications from Mark, are protected C and D are incorrect for the reasons
from disclosure, because the doctor stated in answer B.
was an agent of Mark's lawyer when
the doctor examined Mark. B, C and D 86. B is the best answer. Pursuant to F.R.E.
are incorrect for the reasons stated in 803(1), a statement made while perceiv-
answer A. ing an event or condition, or immedi-
ately thereafter, and which describes
83. C is the correct answer. F.R.E. 404(b) or explains that event or condition, is
provides that character evidence, to admissible. However, the comment to
prove that the accused is a bad person or F.R.E. 803(1) states that a "slight" time
had the propensity to commit the crime lapse is permissible. Although Maybel
for which he is charged, is inadmis- made the statement a brief period of
sible. A is incorrect because there are time after the incident, her dialing for
not enough facts in the hypothetical to the ambulance followed by walking
evidence a plan. B and D are incorrect across the street suggests more than
for the reasons stated in answer C. just a "slight" period of time, and may
indicate deliberate thought. A, C, and
84. C is the correct answer. Under F.R.E. D are incorrect for the reasons stated
404, evidence of a party's character is in answer B.
generally inadmissible in civil cases,
unless it is an element of a claim, charge
or defense. Evidence of a person's
character or a trait of character is not
admissible for the purpose of proving
that he acted in conformity therewith,
on a particular occasion. A, B and D are
incorrect for the reason stated in answer
C.

EVIDENCE 371
87. A is the correct answer. Under F.R.E. 89. B is the correct answer. Under the
401, evidence having any tendency to F.R.E., an out-of-court statement made
make the existence of any fact that is of by a party and offered at trial against
consequence to the determination of the that party is an admission by a party-
action more probable or less probable opponent which is exempt from the
is generally admissible. Relevant evi- hearsay rule. Thus, it is not-hearsay. A is
dence is admissible, unless its probative incorrect because Betty is not unavail-
value is substantially outweighed by the able to testify. C and D are incorrect for
danger of unfair prejudice. Under these the reasons stated in answer B.
facts, the video tape is relevant to show
Tina's intoxication, and although the 90. A is the correct answer. Statements of
tape is prejudicial, its probative value commands and questions have no as-
substantially outweighs its prejudice sertive content; therefore, they cannot
to Tina. Thus, it is admissible. B is in- be hearsay. B, C and D are incorrect for
correct because the F.R.E. follows the the reason stated for answer A.
Wigmore theory, which views Tina's
non-assertive conduct as non-hearsay. 91. D is the correct answer. Any statement
C and D are incorrect for the reasons made by a machine or animal is not
stated in answer A. hearsay. The rationale is that state-
ments from machines and animals are
88. C is the correct answer. Extrinsic trustworthy. A, B and C are incorrect
evidence is inadmissible to impeach a for the reasons stated in answer D.
witness on a collateral matter. Wilma's
testimony of what Betty was wearing on 92. C is the correct answer. Under F.R.E.
the day of the incident is extrinsic evi- 801, hearsay is an out-of-court state-
dence of a collateral matter because its ment offered to prove the truth of the
sole relevance is to contradict Wilma's matter asserted. Here, the bill of sale
testimony regarding Betty's dress. Wil- is being introduced to prove the truth
ma's testimony does not tend to prove of Jim's representation that the car is a
or disprove what caused the accident Rolls Royce. Thus, it is hearsay and is
nor is it relevant to impeach Wilma by inadmissible. A, B and D are incorrect
an impeachment method which allows for the reasons stated in answer C.
extrinsic evidence (e.g., to show the
witness' impaired capacity to perceive
or remember). A, B and D are incorrect
for the reasons answer C is correct.

372 EVIDENCE
93. C is the correct answer. Under FRE 608 95. C is the correct answer. Under F.R.E.
(b) "specific instances of the conduct 405(b), where character or a trait of
of a witness, for the purpose of attack- character of a person is an essential
ing or supporting his credibility, other element of a claim, charge, or defense,
than conviction of a crime as provided proof may be made by reputation, opin-
in rule 609, may not be proved by ex- ion, or specific instances of conduct. In
trinsic evidence. They may, however, in a wrongful death action, the surviving
of truthfulness or untruthfulness, be I lie survivor is enuueu to prove 111(11 LUG
inquired into on cross-examination of decedent had a character trait for being
the witness (1) concerning his character loving and affectionate to demonstrate
for truthfulness or untruthfulness, or (2) the extent of the loss of companionship.
concerning the character for truthful- Therefore, the decedent's character trait
ness or untruthfulness of another wit- becomes a disputed issue. Shirley is
ness as to which character the witness entitled to rebut Laverne's testimony
being cross-examined has testified." by evidence that Squiggy was separated
The court, therefore, has the discretion from Laverne for over a two-year pe-
to permit the question to be answered riod, since the decedent's character is
to impeach Ali By because the question in issue. A and B are incorrect for the
inquires into Ali's character for truthful- reasons stated in answer C. D is incor-
ness or untruthfulness. A, B and D are rect because C is the better answer.
therefore incorrect.
96. A is the correct answer. In a criminal
94. A is the correct answer. Under F.R.E. case in which the defendant is accused
801, an out-of-court statement offered of sexual assault, evidence of a prior
to prove the truth of the matter asserted rape is admissible for any relevant pur-
is hearsay and is not allowed in court, pose. (Also, a prior rape by defendant
unless there is an exception. Under in a rape case does fit within the 1994
these facts, there is no exception to Violent Crime Control Act.) B is incor-
allow the disciplinary reports in. B is rect because it is an incorrect statement
incorrect because, in federal court, spe- of law. C is incorrect because, as stated
cific instance evidence of character is above, evidence of a prior rape by de-
inadmissible under the "victim's excep- fendant is not character evidence. D is
tion" (i.e., to prove Ponch's propensity incorrect because it is not prejudicial.
to use excessive force to show that
Ponch used excessive force against the
defendant, John). C is incorrect because
a written complaint by a citizen is not a
business record. D is incorrect for the
reasons stated above.

EVIDENCE 373
97. D is the correct answer. Under F.R.E. 100. A is the best answer. F.R.E. 901(b)(4)
1002, the original writing must be of- suggests that the contents of the letter
fered into evidence, to prove the con- (which Abel testifies was mailed to
tents of that writing. Here, Alex seeks him 2 weeks after the accident) satis-
to prove the terms of the contract that fies the requirement of authentication.
were not complied with. Alex must This means that the letter will be ad-
produce the contract, since the terms mitted so that the jury can determine
of the contract are in dispute. A, B and whether, based upon the contents, they
C are incorrect for the reasons stated in believe the letter was written by Cain.
answer D. The contents of the letter is probably
sufficient circumstantial evidence to
98. B is the correct answer. Under F.R.E. let this issue of conditional relevance
608(a), evidence offered to support the go to the jury. B, C and D are incorrect
truthfulness of a witness is inadmissi- for the reasons stated for answer A.
ble, until evidence has been admitted to
attack the witness' credibility. Here, the 101. A is the correct answer. Under F.R.E.
prosecution seeks to support A's cred- 801(d)(2), an admission by a party-
ibility as a witness by B's testimony, opponent is a statement which is
before X has offered any evidence to not-hearsay. B is incorrect because it
impeach A. A, C and D are incorrect is an incorrect statement of law. C is
for the reasons stated in answer B. incorrect because, under the F.R.E., a
party admission is not-hearsay, rather
99. C is the correct answer. Under F.R.E. than an exception to the hearsay rule.
801, hearsay evidence is an out-of-court D is incorrect for the same reasons that
statement offered to prove the truth of A is correct.
the matter asserted. Here, William's
testimony is being offered to prove the 102. A is the best answer. Under F.R.E.
matter asserted (i.e., that Jon committed 803(3), a declaration of a then-exist-
the robbery). A, B and D are incorrect ing physical condition is an exception
for the reasons stated in answer C. to the hearsay rule. The declaration
made by Paul, indicating his present
existing bodily condition, is therefore
admissible as an exception. B, C and
D are incorrect for the same reasons
stated in answer A.

374 EVIDENCE
103. C is the correct answer. Pursuant to 106. A is the correct answer. The marital
F.R.E. 404(a), character evidence is in- communication privilege excludes
admissible by the prosecution to prove evidence of confidential communica-
conduct by defendant in conformity tions made between spouses while
therewith on a particular occasion, they were married from admissibility
unless one of the exceptions applies at trial. Since both Mike and Teena are
(e.g., defendant "opens the door" un- holders of the privilege, even if Teena
der the defendant's exception - F.R.E. wanted to testify, Mike can prevent
404 (a)(1)). A and B are incorrect for her from disclosing the confidential
the reasons stated for answer C. D is statement. If the statement was made
incorrect because even if character during marriage, a subsequent annul-
evidence in the form of reputation or ment or dissolution does not terminate
opinion were offered, it is not the type the privilege. B, C and D are incorrect
of evidence that makes the evidence for the reasons stated in answer A.
inadmissible, but that defendant has
not "opened the door" by offering 107. B is the correct answer. Although Mo-
evidence of his good character. riarty is not an attorney, the evidence
is inadmissible where the client has
104. D is the correct answer. Pursuant to a reasonable belief that he is seeking
F.R.E. 409, evidence of offering or advice from an attorney at the time of
promising to pay medical expenses his communication. A, C and D are in-
occasioned by an injury is not admis- correct for the reason stated in answer
sible to prove liability. A, B and D B.
are incorrect for the reasons stated in
answer D. 108. C is the correct answer. Under F.R.E.
801, any out-of-court statement of-
105. C is the correct answer. Under Tram- fered to prove the truth of the matter
mel v. U.S., 445 U.S. 40 (1980), the asserted is inadmissible hearsay, unless
witness-spouse may choose whether or the hearsay objection can be overcome.
not to testify against the other spouse, A is incorrect because the report was
whether or not the other spouse con- prepared for litigation, not to record
sents. Thus, Teena may properly refuse regular conduct of the business. Also,
to testify against Mike at his battery the source of the information, Eddie,
trial. A, B and C are incorrect for the indicates a "lack of trustworthiness,"
reasons stated in answer C. since Eddie could potentially be sued,
and therefore has an interest in being
self-serving and making the report
favorable to the Railroad. A, B and
D are incorrect for the same reasons
stated in answer C.

EVIDENCE 375
109. C is the correct answer. Under Shepard
v. U.S., 290 U.S. 96 (1933), an out-
of-court statement asserting the past
conduct of another person is not
admissible because it is hearsay, and
does not fall under the state of mind
exception ("memory" or "belief' of
the declarant does not qualify as the
declarant's "state of mind"). A is in-
correct because the statement is not
a dying declaration. B is incorrect
because the statement is being used to
prove the past conduct of a third party,
not the declarant's state of mind. D is
an incorrect statement of law.

110. B is the correct answer. Under F.R.E.


406, evidence of a person's habit or
routine practice is relevant to prove
conduct in conformity therewith.
Mr. Clean's testimony is relevant to
rebut Bert's allegations that Vons was
negligent in permitting the candy to
be present on the floor. A, C and D
are incorrect for the reasons stated in
answer B.

376 EVIDENCE
PROPERTY - QUESTION BREAKDOWN

1. Future Interest 19. License

2. Assignment 20. Covenants - Running With The Land

3. Rule Against Perpetuities 21. Implied Reciprocal Negative


Easement
4. Grantor - Grantee Index
22. Breach Of Condition
5. Fee Simple Absolute
23. Deeds
6. Adverse Possession
24. Rule Against Perpetuities
7. Delivery Of Deed
25. Tenancy In Common
8. Defeasible - Removal Rights
26. Vested Remainder Subject To
9. Statute Of Frauds Divestment

10. Covenants 27. Equitable Conversion

11. Covenants To Pay Money 28. Equitable Conversion

12. Life Tenant - Mortgage Principal 29. Deeds - Legal Description

13. Lease/Subsequent Conveyance 30. Deeds - Acreage Discrepancy

14. Retaliatory Eviction 31. Lateral And Subjacent Support

15. Tenancy By The Entirety 32. Contingent Remainders

16. Easement By Prescription 33. Express Easement

17. Joint Tenancy 34. Negative Easements

18. Statute Of Frauds 35. Deeds - Grantor Retaining Possession

PROPERTY 379
36. Deed - Grantors Intent 56. Covenants - Benefit Running With
The Land
37. Remainders - Duty Owed To Future
Interest 57. Covenants - Writing Requirement

38. Warranty Deed 58. Covenants - Privity

39. Tenancy By The Entirety 59. Subjacent Support

40. Recording Act - Race - Race Notice 60. Executory Interest

41. Recording Act - Bona Fide Purchase 61. Tenants In Common

42. Executory Interest 62. Tenants In Common - Co-Tenant


Rights
43. Recording Act - Notice Statute
63. Rule Against Perpetuities
44. Recording Act - Chain Of Title
64. Vested Remainder Subject To Open
45. Recording Act - Donee
65. Adverse Possession
46. Recording Act - Race Statute
66. Rule In Dumpor's Case
47. Assignment Of Lease
67. Covenants
48. Rule Against Perpetuities
68. Privity Of Contract / Privity Of Estate
49. Fixtures
69. Adverse Possession
50. Ameliorative Waste
70. Covenant Of Seisin
51. Ameliorative Waste
71. Title Insurance
52. Rule Against Perpetuities
72. Warranty Deed
53. Executory Interest
73. Quit Claim Deed - Warranties
54. Profits A Prendre
74. Adverse Possession
55. Covenants - Running With The Land

380 PROPERTY
75. Adverse Possession - Statute of 95. Easements - By Implication
Limitations
96. Running Of The Burden And Benefit
76. Riparian Rights Covenants

77. Statute Of Frauds 97. Covenants - Horizontal Privity

78. Dower Rights 98. Equitable Servitudes - Specific


Enforcement
79. Dower Rights
99. Equitable Servitudes
80. Statute Of Frauds - Exception
100. Tenancy At Sufferance
81. Reversion
101. Mortgage/Deed Of Trust
82. Life Estate
102. Installment Contract
83. Fee Estate - Fee Tail
103. Mortgage Liability
84. Reversions
104. Mortgage Transferability
85. Riparian Rights
105. Mortgage Liability
86. Zoning
106. Mortgage - Assumption
87. Easements
107. Mortgage - Assumption
88. Express Easements
108. Mortgage - Foreclosure
89. License
109. Mortgage - Foreclosure
90. Fee Simple Absolute
110. Mortgage - Foreclosure
91. Possibility Of Reverter
111. Mortgage - Foreclosure
92. Tenancy For Years
112. Mortgage - Lenders Interest
93. Landlord Duties
113. Mortgage - Proceed Distribution
94. Easements

PROPERTY 381
114. Mortgage — Recording Act 120. Mortgage — Recording Act

115. Mortgage — Recording Act 121. Mortgage — Transferability

116. Mortgage — Covenants 122. Mortgage — Joint Tenancy

117. Mortgage — Recording Act 123. Mortgage — Transferability

118. Mortgage — Priority Of Title 124. Mortgage — Interest Extinguished

119. Mortgage — Estoppel By Deed 125. Mortgage — Interest Extinguished

382 PROPERTY
PROPERTY QUESTIONS C. Lose, because the language of the
deed created only a contractual
obligation and did not create
1. Juan Antonio conveyed his house and a reversionary interest in Juan
land to Carlos by a validly executed and Antonio.
delivered warranty deed which stated:
D. Win, because the language of the
"To have and to hold the above- deed created a fee simple subject
described tract of land in fee to condition subsequent, which
simple absolute, subject to the leaves a power of termination,
understanding that said grantee or right of entry, in the grantor.
shall construct and maintain a fa-
cility for the preservation of wild 2. Terry and Harry leased a house from
horses on said property." Larry, the landlord. The lease was in
proper form and contained the follow-
Carlos did, in fact, construct and main- ing language:
tain a facility for the preservation of
wild horses on the property which he "Any assignment, subletting or
operated for ten years after the grant transfer of any rights under this
from Juan Antonio. However, at the lease without the express written
end of ten years, Carlos converted the consent of Larry, the landlord,
property to a dude ranch. All parties shall be null and void and is pro-
agree that a dude ranch is not a facility hibited."
for the preservation of wild horses.
Despite this clause, after Terry and Har-
Juan Antonio seeks a court declara- ry moved into the house, Harry verbally
tion that the change in the property's invited Gary to share the house with
use means that the land and house has Terry and himself. Gary agreed to pay
reverted to him. In this lawsuit, Juan his portion of the rent to Larry who did
Antonio will: not object to this arrangement. Terry,
however, objected to Gary's sharing
A. Lose, because an equitable the dwelling, even if Gary paid a fair
charge is enforceable only in share of the rent.
equity.

B. Win, because the language of the


deed created a fee simple deter-
minable, which leaves a possibil-
ity of reverter in the grantor.

PROPERTY 383
As soon as Gary moved into the house, "To my surviving widow for life,
0
Terry sued Harry, Gary and Larry for a and then to such of my children
0 declaratory judgment, that Harry was who shall reach 30 years of age;
CE
without rights to assign to Gary. Harry however, if any such child should
answered that he and Terry were tenants die under the age of 30 and such
in common for the estate of a term of child shall be survived by a child
years, and that he, Harry, had the right or children, that child or children
to assign a part interest in his undivided shall step up and take his or her
one-half interest in the leasehold. The parents' share being that share
court should award judgment in favor which would have gone to that
of: parent had he or she attained the
age of 30 years."
A. Terry, because a co-tenant has no
right to assign any part or all of In 1970, at the time of the will, Oscar
a leasehold without the consent was married to Wendy and he and
of all relevant parties. Wendy had two children, Arnold and
Beulah. Beulah had one child, Carlotta.
B. Terry, because the lease contained In 1980, Wendy died and Oscar remar-
a "no assignment" clause. ried a woman named, Winfred. In 1981,
Winfred and Oscar had a son named,
C. Harry, because Terry is not the Delbert. In 1985, Oscar died survived
beneficiary of the "no assign- by Winfred, Arnold, Beulah, Carlotta
ment" clause in the lease. and Delbert.

D. Harry, because Terry's claim is In a state which applies the common


essentially a restraint on alien- law Rule Against Perpetuities unmodi-
ation and is disfavored. fied by statute, the result of applying
the Rule to the interests under the will
3. In 1970, Oscar executed his will in is that:
which he left Ochreacre:
A. The remainder to Oscar's chil-
dren and grandchildren is void
because Oscar could have mar-
ried a person not yet in being
at the time Oscar's will was
executed.

B. The remainder to the children is


valid and the substitute gift to the
grandchildren is also valid.

384 PROPERTY
Q
C. The remainder to Arnold and D. Either Orene or Nancy, depend-
Beulah is valid, but the remain- ing on whether Nancy's deed is
der to Carlotta is void as is any deemed to be recorded within
gift to Carlotta's children. Orene's chain of title.

D. The remainder to the children is


valid, but the substitute gift to
the grandchildren is void because
Oscar could have married a per-
son not yet in being at the time
Oscar's will was executed.

4. In 1990, Andrea was the owner of


Pinkacre. However, in 1990, Mary gave
a warranty deed to Pinkacre to Nancy.
Nancy promptly recorded that deed. In
1991, Andrea conveyed Pinkacre by
warranty deed to Mary who promptly
recorded her deed. In 1992, Mary gave
a warranty deed to Pinkacre to Orene
who promptly recorded her deed. Orene
paid fair market value for Pinkacre and
was unaware of any prior conveyances
of Pinkacre. Both Orene and Nancy
claim title to Pinkacre. The court should
award judgment to:

A. Nancy, because her deed is prior


in time to Orene's.

B. Orene, because she was a pur-


chaser for value without notice
of Nancy's claim.

C. Either Orene or Nancy, depend-


ing on whether Pinkacre is lo-
cated in a state which applies
estoppel by deed.

PROPERTY 385
C. Xavier as first taker of the fod-
Questions 5 — 6 are based on the
LLI
der.
0-
following facts:
0
ce In 1970, Orville owned Purpleacre in fee D. The person who then held title
simple absolute. Unbeknownst to Orville, in to Purpleacre in fee simple ab-
1971, Xavier entered Purpleacre under color solute.
of title and began to use a square of land on
the eastern side of Purpleacre to cut fodder 6. After Orville's 1993 conveyance to
for food for Xavier's sheep. After 14 years Norris, title to Purpleacre was in:
of possession of Purpleacre, Xavier gave
possession of Purpleacre to Yves. Xavier A. Xavier.
also sold Yves all his sheep and purported
to transfer to Yves, Xavier's interests in B. Orville.
the fodder and fodder area by means of a
document which was adequate to transfer C. Yves.
personal property (the sheep), but inadequate
to transfer real property. D. Norris.

In 1986, Xavier leased Purpleacre from 7. While her daughter, Darla, was away at
Orville for a term of five years. At the end school, Marie executed and promptly
of the five year term, Xavier remained on recorded a deed in the proper form to
Purpleacre for two more years before he Marie's property, Yellowacre, to Darla.
quit Purpleacre. In 1993, Orville conveyed When Marie told Dada what she (Ma-
Purpleacre by quitclaim deed to Norris. rie) had done, Darla said, "Mother, I
Purpleacre is located in a state where the don't want Yellowacre. You take that
statute of limitations for adverse possession deed right back. It's yours." Before
is ten years. any other relevant events, Darla died
leaving her entire estate to Butch.
5. After Orville's 1993 conveyance to Marie then sued Butch to quiet title in
Norris, title to the fodder area was in: Yellowacre. If Marie wins, it will be
because:
A. Yves, as the purchaser of the
fodder area under the 1985 docu- A. The presumption of delivery
ment selling Xavier's sheep to arising from recordation is in-
Yves. valid unless the grantee has
knowledge of the deed at the
B. The person who owned fodder time of recording.
rights as a necessary incident to
the fodder.

386 PROPERTY
Q
B. There was no effective ac- 9. John and Joan, an unmarried couple,
ceptance of the delivery of the owned Goldacre as tenants in common.
deed. After five years, they orally agreed that
there was no need for them to make
C. Darla's renunciation of Yellow- wills since the survivor of either of
acre was a constructive convey- them would own Goldacre solely and
ance to Marie. outright per this agreement. Three years
later, John died intestate. One day later,
D. The court will impose a construc- Joan died, also intestate. John's sole heir
tive trust to carry out Darla's is his father, Dave; Joan's sole heir is
intent. her aunt, Sophie. Dave claims an inter-
est in Goldacre; Sophie claims all of
8. Abe conveyed Silveracres to "Ben and Goldacre. Goldacre is located in a state
his heirs, but if Ben dies survived by with the Statute of Frauds, but no other
children who attain the age of 30, then applicable statute. That state does not
to Carl and his heirs." Once Ben began recognize common law marriages.
possessing Silveracres, he began to
cut the valuable timber on Silveracres In the Dave v. Sophie litigation regard-
without giving Carl any notice of his ing the ownership of Goldacre, the court
actions. Ben is married to Darlene but should find that:
has no children. If Carl sues for an ac-
counting for the value of the cut timber A. Dave and Sophie each own an
and for an injunction to prevent future undivided interest in Goldacre
logging, and the court gives judgment because the deaths of John and
against Carl and in favor of Ben, it will Joan were essentially simultane-
be because: ous.

A. Carl has no interest in Silver- B. Dave and Sophie each own an


acres. undivided interest in Goldacre
because the Statute of Frauds
B. The right to log and remove tim- applies.
ber is an incident to possession.
C. Sophie owns Goldacres solely
C. The right to log and remove tim- because John and Joan did not
ber is an incident to a fee. make wills in detrimental reli-
ance on the oral agreement.
D. There is no showing Ben acted
in bad faith.

PROPERTY 387
D. Sophie owns Goldacre solely be- Questions 10 —11 are based on the
cause she is entitled to equitable following facts:
reformation of the title to reflect
the oral agreement. Orson is the owner of a 1,000 acre tract of
land called Greyacres. Orson wishes to de-
velop Greyacres into a planned residential
community surrounding a man-made lake
and marina. He needs to ensure that pro-
spective purchasers of lots in Greyacres are
guaranteed the following 3 things:

1) Apart from the lake and marina,


no non-residential use will be
made of Greyacres.

2) Residents of lots in Greyacres


will have clear access to and use
of the marina and lake which ac-
cess and use will be capable of
being transferred to subsequent
purchasers of those lots.

3) Residents of lots in Greyacres


must be obligated to pay a pro-
rated share for the maintenance
of the lake and marina regardless
of their actual use of the facili-
ties.

10. In the context of the entire scheme of de-


velopment, which of the following will
offer the best method of implementing
Orson's requirement that residents of
Greyacres pay money to maintain the
lake and marina?

A. An easement.

B. A covenant.

C. A mortgage.

388 PROPERTY
Q
D. A personal contractual obligation 12. Azureacre is owned by Amanda as a life
by each purchaser. tenant and Bailey as remainderman in
fee simple. Amanda and Bailey were
11. Which of the following is the greatest conveyed Azureacre subject to a mort-
difficulty that Orson will face in estab- gage of $100,000, payable in $10,000
lishing his scheme on Greyacres? installments of the principal over ten
years at ten percent interest per year
A. The scheme, if effective, renders payable with each installment of the
title unmarketable. principal. Azureacre produces $50,000
in net income per year after paying all
B. One or more of the essential re- expenses except the mortgage. Amanda
quirements for development of and Bailey each dispute who should pay
Greyacres constitute a restraint the mortgage principal. If Bailey seeks
on alienation. legal advice as to his obligations for
the mortgage principal of Azureacre,
C. There is a judicial reluctance to he should be told:
recognize an affirmative burden
to pay money over an indefinite A. He could compel Amanda to pay
period as an obligation which the mortgage principal because
attaches to land and passes to the income from Azureacre is
future purchasers of land. more than adequate to meet this
obligation.
D. Judicial enforcement of these
requirements constitutes state ac- B. Amanda could compel Bailey
tion which would raise substan- to pay his share of the principal
tial questions as to the constitu- because the discharge of the
tionality of such requirements mortgage enhances his remain-
under the 14th Amendment. der.

C. He could not be held person-


ally liable for the mortgage, but
Bailey's remainder could be lost
if the mortgage is not paid.

D. Bailey's only protection lies in


instituting an action for partition
to compel the sale of Amanda's
life estate to obtain the value of
his remainder.

PROPERTY 389
13. Otto owned Violetacre, a house and lot. C. Beatrice, because Otto's permis-
ce
Aaron and Beatrice occupied Violetacre sion to occupy pre-dated Otto's
0
0 rent-free with Otto's oral permission. conveyance.
ce Both Aaron and Beatrice were 21-year-
old actors looking for acting work in D. Beatrice, because Otto is Bea-
show business. Otto conveyed, properly trice's landlord, not Aaron.
executed and recorded a warranty deed
to Violetacre as follows: 14. In May, Tina moved into and began
paying a monthly rental of $200 for
"To Aaron and his heirs, upon an apartment in a ten-unit apartment
condition that he obtain an act- building owned by Lulu. The hallways
ing role in a Hollywood movie and stairwells of the building needed
by the time he reaches the age repair. Lulu refused to undertake those
of 35. However, if he does not do repairs, even after Tina requested she
so, then Violetacre is to pass to do so. Therefore, in late August, Tina
my daughter, Della, for life, and met with the other tenants and discussed
then to Della's two children, Ed with them the tenants' legal options
and Fred." against Lulu for failure to make repairs.
When Tina paid September's rent, Lulu
At the time of the conveyance, neither notified Tina that beginning in October,
Aaron nor Beatrice had ever acted in a Tina's rent would be $400 per month.
Hollywood movie. One month after the When Tina protested that all of the other
conveyance to Aaron, Aaron informed tenants were paying $200 per month,
Beatrice that he, Aaron, was now her Lulu gave Tina the statutorily-required
landlord and expected a fair rental 30-day notice to vacate the apartment.
each month, payable to him. Beatrice If Tina prevails in a lawsuit contesting
refused. Violetacre is located in a ju- the termination of her tenancy, the most
risdiction with no statute applicable to likely reason for her victory is that:
this situation. In a properly filed action
for ejectment of Beatrice by Aaron, the A. Her payment of monthly rental
court should award judgment for: implies the existence of a peri-
odic tenancy.
A. Aaron, because Otto's convey-
ance created a fee simple sub- B. The rent increase from $200
ject to executory limitation in to $400 is unconscionable and
Aaron. will shock the conscience of the
court.
B. Aaron, because Otto's convey-
ance terminated Beatrice's ten-
ancy.

390 PROPERTY
Q
C. The fact that Tina would have 16. 011ie owned Blackacre, a one-acre tract
to pay double the rental rate of land, in fee simple absolute. How- 0
m
charged to other tenants violates ever, Paul was in adverse possession
the implied agreement that rent- of Blackacre. During Paul's adverse
als for similar apartments be possession, 011ie gave Agnes oral per-
comparable. mission to use the south forty feet along
Blackacre as a road to reach Whiteacre,
D. The doctrine of retaliatory evic- a tract of Agnes' land. During all times
tion is part of the law of this relevant to this question, Agnes regu-
jurisdiction. larly used Blackacre as a road to reach
Whiteacre from the public highway.
15. Indiogoacres was conveyed to Adolph
and Bette by a deed which created a A few years later, Paul quit possession
co-tenancy with equal rights of pos- of Blackacre without gaining title by
session, equal shares in interest, and adverse possession. Agnes continued
a gift of survivorship. Adolph con- to use the south forty feet along Black-
veyed to Cal his "undivided one-half acre from the time of her original use
interest in Indiogoacres" to Cal. After for a sufficiently long period of time to
Adolph's death, Bette sues Cal claim- gain an easement by prescription. All
ing Indiogoacres is her sole property. of this use was accomplished without
Cal claims he is owner of an undivided further communication between Agnes
one-half of Indiogoacres. The court and 011ie. 011ie then blocked the road
should award judgment for: on the south forty feet along Blackacre
and has refused to permit Agnes' further
A. Cal, if Bette had actual or con- use. Agnes then sued to determine her
structive knowledge of Adolph's continued use of the road. In that law-
deed to Cal. suit, Agnes should:

B. Cal, since he and Bette own A. Win, because her use was ad-
Indiogoacres as tenants in com- verse to Paul and once adversely
mon. begun, it continued in that man-
ner until some affirmative show-
C. Bette, because she is the sole ing of a change.
owner of Indiogoacres.
B. Win, because Agnes made no
D. Bette, but only if the original ten- attempt to renew 011ie's permis-
ancy created in Adolph and Bette sion after Paul quit possession of
was a tenancy by the entirety. Blackacre.

PROPERTY 391
C. Lose, because her use was per- Questions 17 —18 are based on the
LLI missive. following facts:
0-
0
ce D. Lose, because there is no evi- Two sisters, Alice and Betty, contributed
dence that she continued to one-half of the purchase price of Yellow-
adversely use Blackacre for the acres, a 50-acre tract of land. They held Yel-
required period after Paul quit lowacres together as joint tenants. Five years
Blackacre. after the original purchase, Alice proposed
that the two sisters develop Yellowacres
as a retail shopping center. Betty did not
want to do so. She orally agreed with Alice
that Alice could alone develop the western
one-half of Yellowacres as she wished and
that Betty would do whatever she wanted
with the eastern one-half of Yellowacres.
Alice developed her retail shopping center
on the western 25 acres and Betty gave oral
permission for the Sierra Club to use the
eastern one-half of Yellowacres as a wildlife
sanctuary. Alice died, leaving her entire es-
tate, by will, to her son, Sam. The will also
named Betty executrix of Alice's estate, but
she refused to serve.

17. In an appropriate lawsuit to determine


Sam and Betty's respective interests in
Yellowacres, if Sam is adjudicated to
be the owner of the western 25 acres of
Yellowacres, the most likely reason for
the judgment will be that:

A. Betty's actions during Alice's life


has estopped her from asserting
title to the western one-half of
Yellowacres.

B. The joint tenancy was terminated


by the oral agreement of Alice
and Betty at the time it was
made.

392 PROPERTY
"CP
C. Betty had a fiduciary obligation 19. Baker and Selwyn entered into a valid,
to Sam by reason of her be- enforceable written contract in which 0
ing named executrix of Alice's Baker agreed to buy and Selwyn agreed m

will. to sell Orangeacre, Selwyn's residence.


The contract contained a provision al-
D. The close blood relationship be- lowing Selwyn the right to remain in
tween Alice and Betty removes residence of Orangeacre for 30 days
the necessity to comply with the before relinquishing possession. The
Statute of Frauds. closing took place as scheduled; title
passed to Baker and Selwyn remained
18. In an appropriate lawsuit to determine in possession. A week later, however,
Sam and Betty's respective interests in the house Selwyn was scheduled to
Yellowacres, if Betty is adjudicated to move into was destroyed. At the end
be the sole owner of Yellowacres, the of the 30-day period, Selwyn refused
most likely reason for the judgment will to leave Orangeacre. Instead, Selwyn
be that: tendered to Baker a monthly rental pay-
ment in excess of the fair rental value
A. The record title of the joint ten- of Orangeacre. Baker refused the rent
ancy to Yellowacres can only and sued to gain immediate possession
be changed by a duly recorded of Orangeacre. There is no law of this
instrument. jurisdiction dealing with this situation
and the contract is similarly silent. The
B. The Statute of Frauds prevents landlord-tenant law of this jurisdiction
the proof of Betty and Alice's requires a 30-day notice by the landlord
oral agreement. before he can evict a tenant. Baker did
not give the 30-day statutory notice.
C. Alice could not unilaterally sever Baker's best legal argument in support
the joint tenancy. of his action for immediate possession
is that Selwyn is a:
D. Betty's nomination as executrix
of Alice's estate does not prevent A. Licensee.
her from asserting her claim
against Sam. B. Tenant at sufferance.

C. Tenant from month to month.

D. Trespasser ab initio.

PROPERTY 393
Q
Questions 20 — 21 are based on the C. Della, because she is a remote
following facts: grantee — not a direct grantee of
Oliver Wendell.
Oliver Wendell, the owner of a 100-acre tract
of land, prepared and properly recorded a D. Della, because she did not re-
subdivision plan called the Oliver Wendell ceive a deed containing a restric-
Homes. The plan contained 40 two-acre lots tive covenant.
and a 20-acre tract in the southeast corner.
Oliver Wendell sold 20 of the lots to indi- 21. Assume for this question only, that
vidual purchasers. Those deeds all referred Carol has announced her intention to
to the recorded plan and contained the build a multifamily condominium com-
following clause, "No multi-family dwell- plex on the 20-acre parcel and that Abby
ings shall be erected within Oliver Wendell has sued to prevent its construction. If
Homes." Carol wins, it will be because:

Abby purchased one of these original lots A. Carol's proposed use will allow
from Oliver Wendell. Eight years later, Oli- a sufficient diversity in economic
ver Wendell sold the remaining land (20 lots classes in the vicinity to satisfy
and 20 acre parcel) to Beth by a deed which the requirements of the Equal
referred to the plan and contained the same Protection clause of the Constitu-
clause prohibiting multi-family dwellings. tion.
Beth then resold the 20 lots to individual
purchasers and the 20 acre parcel to Carol. B. Restriction in deeds are to be
None of Beth's deeds referred to the plan, construed in favor of free alien-
nor contained the multi-family dwelling ation of land and against the
restriction. grantor.

20. Assume for this question only, that C. There has been an insufficient
Della, a purchaser from Beth, has begun showing that the common de-
construction of a multi-family apart- velopment scheme applied to the
ment building on her lot and that Abby entire subdivision.
sues to prevent Della's apartment build-
ing. Who will prevail in this lawsuit? D. Restrictive equitable servitudes
bind only those in privity with
A. Abby, because the restrictive the original covenanting par-
covenant in the deed runs with ties.
the land.

B. Abby, because the apartment


building will lower Abby's land
value.

394 PROPERTY
Q
22. In 1960 Juan, the owner of a fee simple B. The judge should grant the in-
absolute in Greenacres, conveyed junction, but not award damages
Greenacres to the Oak Ridge School. since Juan and Juanita are not
The operative words of conveyance parties to the lawsuit.
were "to the Oak Ridge School for the
life of my daughter, Juanita, and then at C. The judge should award dam-
that time to all of my grandchildren and ages, but not the injunction.
their heirs in equal shares; provided,
however, that the Oak Ridge School D. The judge should grant the in-
shall use Greenacres for school purpos- junction and impound the dam-
es only." After the conveyance, the Oak ages to be awarded.
Ridge School began holding classes on
Greenacres and has continued to do so 23. Maria owned three condos in Condo
for all relevant times. Heaven, numbers 6, 7 and 21. The price
and fair market value of condos 6 and 7
In 1980, Oak Ridge School granted was $50,000, and number 21, $75,000.
Shelly a right to mine and remove Nellie was interested in purchasing a
minerals from the northwest portion condo in Condo Heaven but couldn't
of Greenacres. Shelly began to remove decide between #6 or #7. Therefore,
minerals under this grant. Nellie gave Maria $50,000 and Ma-
ria prepared and executed a deed to
All three of the present grandchildren Nellie for a condo in Condo Heaven,
of Juan, as well as a guardian ad litem, complete in every way except that the
appointed to represent Juan's unborn unit number was left blank. Maria told
grandchildren, have sued the Oak Ridge Nellie to fill in unit #6 or #7 as she
School and Shelly for damages and wished and then Nellie should record
injunction for the mining operations the deed. The next day, Nellie went to
on Greenacres. There is no applicable Condo Heaven, filled in unit #21 in the
statute. Which of the following best deed and recorded the deed. In an action
describes the probable outcome of the by Maria to rescind the transaction, if
litigation? Maria loses, the most likely reason for
that judgment is that:
A. Plaintiffs should prevail, since
the Oak Ridge School's interest A. The necessity for certainty in
terminated the instant Shelly land title records controls.
began mining operations on
Greenacres. B. The agency implied to complete
the deed cannot be restricted by
the oral understanding.

PROPERTY 395
C. The recordation of the deed In an action to determine the property
LLI precludes any questioning of the rights in Beigeacres, it was alleged that
0-
0 deed in its recorded form. the gifts to Harold's grandchildren were
ce void and that the interests in Beigeacres,
CL
D. Maria's casual business practices after Dolly's children's life estates
caused her loss. passed to Harold's children, passed
absolutely under the residuary clause of
24. In 1980, Harold willed Beigeacres as the will. Assuming a properly instituted
follows: action with all parties represented, the
judge should decide that:
"To my daughter, Dolly, and at
her death to those of Dolly's chil- A. The attempted gifts to the grand-
dren who survive her, provided, children are void as restraints on
however, that no such child of alienation.
Dolly shall have the power to con-
vey, sell or mortgage such child's B. The attempted gifts to the grand-
interest prior to attaining the age children are void under the Rule
of 30 years; and if any such child Against Perpetuities.
of Dolly shall not comply with
this provision then that child's C. The attempted gifts to the grand-
interest shall determine and pass children are valid except for the
to the remaining children of Dolly provisions against sale, convey-
alive at that time, share and share ance or mortgage.
alike."
D. The attempted gifts to the grand-
Harold's 1980 will devised Whiteacres children are entirely valid and
to his son, Solomon, in an identical will be enforced as written.
manner. The residuary clause in the
1980 will gave the residual estate to
Harold's two children, Dolly and Solo-
mon, equally. In 1990, Harold died sur-
vived by Dolly and Solomon and their
two children each. Since 1990, Dolly
has had one additional child.

396 PROPERTY
'CP
25. Thelma Jones and Louis Boggs lived 26. In 1991, Wilma conveyed Redacres as
together for ten years. They never for- follows: 0
mally married, although they always m
referred to each other as husband and "To my friend Fred for life and
wife, and Thelma always identified then to Fred's two children, Bar-
herself as Thelma Boggs or Mrs. Louis ney and Betty, and their heirs,
Boggs. During this time, they decided provided that should any of them
to purchase a home. The deed was in move to California, then that
proper form and identified the grantees child's interest to pass to the Red
as "Thelma and Louis Boggs, as hus- Cross."
band and wife, and their heirs forever,
as tenants by the entirety." Thelma In 1991, Fred was alive and had two
provided a down payment and she and children, Barney and Betty. In an appro-
Louis gave a mortgage for the balance. priate action to determine the interests
The mortgage was signed by both in Redacres, the gift to Fred's children
Thelma and Louis, as husband and wife. shall be held to be:
Thelma made payments on the loan for
one year, until she abandoned Louis and A. An executory interest.
the house and moved to another state.
Louis made one further mortgage pay- B. A vested remainder subject to
ment after Thelma's departure and then complete divestment.
sued for partition. The suit for partition
should be: C. An indefeasible vested remain-
der.
A. Denied, since no single tenant by
the entirety has a right to sue for D. A contingent remainder.
partition.

B. Denied, since Thelma has sole


title to the property.

C. Granted, because the tenancy by


the entirety that was created by
the deed was destroyed when
Thelma abandoned Louis.

D. Granted, because the deed never


created a tenancy by the en-
tirety.

PROPERTY 397
Questions 27 — 28 are based on the 28. Assume for this question only, that
following facts: Carla dies before the real estate clos-
ing and her will gives her real property
Bernie is the owner of Blueacres, a tract of to Ricky and her personal property to
land. In 1990, he contracted to sell Blueacres Paula. Assume further, that the contract
to Carla. Bernie and Carla executed an agree- is otherwise fully performed. Which
ment for Carla to purchase Blueacres from statement is most correct?
Bernie for $50,000 cash. Assume Blueacres
is located in a jurisdiction which recognizes A. Ricky has the right to specifically
the principle of equitable estoppel, but has perform the contract.
no statute specifically dealing with these
problems. B. Bernie has the right to can-
cel the contract and refuse the
27. Assume for this question only, that $50,000.
Bernie dies before the real estate clos-
ing and his will leaves his real property C. Since the land sale contract
to Rhoda and his personal property to could have provided for the con-
Peter. Assume further that the contract tingency of death of one of the
is otherwise fully performed. Which parties but did not do so, death
statement is correct? terminates the contract.

A. Rhoda is entitled to the $50,000 D. The death of the buyer renders


on closing since equitable con- title to Blueacres unmarketable.
version is inapplicable here.

B. Peter is entitled to the $50,000 on


closing since equitable conver-
sion applies here.

C. Since the land sale contract


could have provided for the con-
tingency of death of one of the
parties but did not do so, death
terminates the contract.

D. The death of the seller renders


title to Blueacres unmarketable.

398 PROPERTY
Q
Questions 29 — 30 are based on the C. Is enforceable, because no ambi-
following facts: guity appears on the face of the
deed.
In 1975, Wanda gave a deed to her boyfriend,
Bill, containing the covenant of general war- D. Is sufficient, if Bill paid consid-
ranty, quiet enjoyment and right to convey. eration.
That deed contained legal descriptions of
land as follows: 30. The legal description of 1313 Mocking-
bird Lane is:
(a) "All of my land and house
known commonly as 1313 Mock- A. Insufficient, because it contains
ingbird Lane, Addamsville, United no description by metes and
States, the same comprising 3/4 of bounds.
an acre."
B. Insufficient, because the acreage
(b) "All that part of my ranch, be- given is inaccurate.
ing a square with 600 foot sides,
the northeast corner of which is C. Insufficient, because a grantor
the west line of my neighbor, Ru- cannot convey more than she
fus Choate." owns.

On the date of the deed, Wanda handed the D. Sufficient, because the discrep-
document to Bill who immediately gave ancy in area is not fatal.
it back to Wanda for safekeeping. Wanda
took the deed to her bank and put it in her 31. Ethel was the fee simple owner of Weed-
safe-deposit box. The deed was not re- acres (a vacant tract of land). Weedacres
corded. As it turns out, the property at 1313 was located adjacent to Hedgeacres,
Mockingbird Lane was, in fact, only 5/8 of a tract of land with a house and barn
an acre but contained a house, garage and located on it and owned in fee simple
tool shed. Wanda owned no other land on by Fred. In 1990, Ethel began digging a
Mockingbird Lane. foundation for a building on Weedacres
to be located near the property line with
29. The legal description of the portion of Hedgeacres. Ethel's digging caused the
Wanda's ranch given to Bill: barn and land on Hedgeacres to collapse
due to earth movement. In a lawsuit by
A. Is insufficient, because of vague- Fred against Ethel for damages to his
ness. barn and land, due to Ethel's excava-
tion, the prevailing party should be:
B. Is enforceable, if the deed con-
tinued the covenant of seisin.

PROPERTY 399
A. Fred, because he has an absolute Questions 33 — 34 are based on the
right to lateral support of his following facts:
land.
Quintin was the owner of Sandacres (a large
B. Fred, unless the weight of the tract of undeveloped land) in fee simple
barn contributed to the damage. absolute. In 1970, he subdivided Sandacres
into 3 parcels, #1-3. Quintin conveyed lot
C. Ethel, but only if she was negli- #1 to Rhoda, lot #2 (the middle lot) to Susan
gent in her excavation. and lot #3, he retained as vacant land for
speculation. Rhoda and Susan built houses
D. Ethel, unless her excavation and swimming pools on their lots. The land
would have caused the same around Sandacres was undeveloped, unim-
damage to unimproved land, or proved and unzoned. After their houses were
she was negligent. built, Rhoda and Susan agreed to build and
jointly construct a sewer line running from
32. Francisco owned Farmacres (a tract of the street in front of their houses along the
land) in fee simple absolute. He con- common property line to the rear of their
veyed it "to Gilbert for life, and then respective lots and connecting into each
to the children of Helen in fee simple." house. The sewer line was so constructed
At the time of the conveyance, both that one half of the line was located on each
Gilbert and Helen were alive; neither lot. Rhoda and Susan exchanged sewer ease-
had any children. What interest, if any, ment grants by which each of them granted
was created in Helen's unborn children to the other, her heirs and assigns, the right
at the time of the conveyance? to use and maintain the common sewer line.
Both documents were properly recorded.
A. A vested remainder, subject to
partial divestment. In 1990, the area surrounding Sandacres was
more developed and the city government
B. A contingent remainder. decided to install and maintain a new sewer
line running along the rear of the three lots.
C. An executory interest. That new line removed the need for the com-
mon line constructed by Rhoda and Susan.
D. None. Rhoda informed Susan that she considered
the new sewer line to be a replacement for
the old common line, that she intended to
discontinue use of the common line and that
she expected Susan to do the same. In 1990,
Quintin decided to develop lot #3 into a
high-rise condominium building, containing
20 stories and 200 units. If developed in that
way, Quintin's building would completely

400 PROPERTY
Q
shade Susan's swimming pool, for all but A. Susan, if, and only if, she was
forty minutes a day. This change would misled by Quintin's failure to
substantially affect the fair market value of complain when she was building
Susan's lot. her pool.

33. If Susan sues Rhoda to enjoin Rhoda B. Susan, because Quintin's build-
from interfering with Susan's contin- ing would be an unreasonable
ued use of the common sewer line, the obstruction with her natural right
prevailing party will be: to an easement for light, air or
view.
A. Rhoda, because the termination
of the necessity for the sewer C. Quintin, because Susan has no
easement terminated the ease- natural right to an easement for
ment. air, light or view.

B. Rhoda, because the continued use D. Quintin, if, and only if, he can
of the common easement after a show that his intention to build
material change in circumstances in this manner was known to
would diminish the fair market Susan before or at the time she
value of the lots, without a cor- received her conveyance from
responding and commensurate the common grantor, Quintin.
increase in value.

C. Susan, because an incorporeal


hereditament lies in grant and
cannot be terminated without a
writing.

D. Susan, because the termination


of the necessity for the ease-
ment which had been created by
express grant does not affect the
rights in the easement.

34. If Susan sues Quintin to enjoin him from


erecting the high-rise condominium in
such a way to shade Susan's swimming
pool, the prevailing party will be:

PROPERTY 401
Questions 35 — 36 are based on the 36. In a lawsuit by Victoria against Wilma
following facts: to establish that Victoria has title to
Plantacres, if Victoria loses, it will be
Victoria was the fee simple owner of Planta- because:
cres (a vacant tract of land). On January 1,
1990, she went to the house of her 30-year- A. Parol evidence is inadmissible to
old daughter, Wilma. In her daughter's prove an oral condition rebutting
presence, Victoria executed a warranty deed valid delivery.
to Plantacres and said, "Here is a deed to
Plantacres which I am giving you as a gift." B. As between the parties to a trans-
Wilma thanked her mother, and Victoria action, recording is not required
took the deed and subsequently placed it in to transfer title after a valid de-
her (Victoria's) safe deposit box. In 1993, livery.
Victoria received an offer from Xenaphon to
purchase Plantacres for several times the cur- C. Wilma has proven that Victoria
rent market value. The recording act in this intended to part with Plantacres
jurisdiction states that unless a conveyance irrevocably.
or other transfer of land is duly recorded,
every deed or other conveyance of an interest D. This is a donative escrow, which
in land is void as to a subsequent purchaser creates consideration for the
for value in good faith who records. transfer through the relationship
of blood or marriage.
35. In a lawsuit by Victoria against Wilma
to establish that Victoria has title to
Plantacres, if Wilma loses it will be
because:

A. The events of January 1, 1990


were a conditional delivery to a
grantee without an escrow.

B. Wilma is a donee and not a bona


fide purchaser under the record-
ing act.

C. The events of January 1, 1990


were insufficient to constitute a
valid delivery.

D. Wilma did not record her deed as


required by the recording act.

402 PROPERTY
Q
37. Ursula conveyed Vineacres (a house 38. Yvette conveys Bambooacres (a tract
and tract of land) "to Will for life, re- of land) to Zerlina by warranty deed as
mainder to Xerxes and his heirs, subject a gift for Zerlina's birthday. One year
to a first mortgage in favor of First Fed- later, Yvette conveys the same property,
eralist Savings and Loan for $100,000." Bambooacres, to Abelia for $100,000.
The mortgage had an unpaid balance of Abelia is without constructive or actual
$100,000, payable in monthly install- knowledge of the prior deed to Zerlina.
ments of $1,000 plus interest at 8% Abelia records her deed to Bamboo-
on the balance. The next payment of acres first. Abelia receives the property
principal and interest is due May 1. The pursuant to the recording act in this
reasonable rental value of the property jurisdiction. If Zerlina sues Yvette for
is greater than the sum necessary to having lost the property to Abelia, the
meet all current charges. Will is cur- outcome of Zerlina's lawsuit against
rently living on the property. Assume Yvette will depend on:
that the common law rules governing
contributions between life tenants and A. Whether Yvette's warranty deed
remaindermen apply in this state. How to Zerlina was invalid ab initio.
should the burden for repayment of the
first mortgage to First Federalist Sav- B. Whether deeds without cov-
ings and Loan be allocated between enants are sufficient to transfer
Will and Xerxes? title to real property.

A. Will must pay both the principal C. Whether the jurisdiction views
and interest. title covenants as personal or
running with the land.
B. Xerxes must pay both principal
and interest. D. Whether Zerlina is a bona fide
purchaser.
C. Will must pay the principal and
Xerxes must pay the interest.

D. Xerxes must pay the principal


and Will must pay the interest.

PROPERTY 403
39. Benjy owned Bushacres (a tract of land) Questions 40 — 41 are based on the
in fee simple absolute. He conveyed following facts:
0 Bushacres in 1954 "to Carla and Dan, as
ce In 1978, Fiona owned Palmacres (a tract of
a_ wife and husband." In this jurisdiction,
Benjy's conveyance would be sufficient land) in fee simple absolute. In that year, she
to create a tenancy by the entirety. executed and delivered to Gus a quitclaim
Thereafter, Carla conveyed by a deed deed which purported to convey to Gus all
in valid form and delivery "my undi- Fiona's interest and title in Palmacres. Gus
vided one-half interest in Bushacres" accepted the deed and placed it in his bank
to Esther. Carla died one year later. If safe deposit box. In satisfaction of a prior
Esther sues Dan to quiet title in himself debt she owed to Hal, Fiona executed and
and herself as co-owners of Bushacres, delivered a warranty deed to Hal also pur-
she will: porting to convey full fee title to Palmacres
in full release of her obligation to him. Hal
A. Not prevail, unless Carla and promptly recorded his deed. Two years
Dan were not legally married later, Gus recorded his deed. At no time has
according to the laws in this there been evidence of any occupation of
jurisdiction during the relevant Palmacres. Assume that provisions of the
times in this problem. recording statute in this jurisdiction require
that a junior claimant be in good faith and
B. Not prevail, because he is the have provided value in order to claim protec-
sole owner of Bushacres. tion of the recording act.

C. Prevail, if, and only if, Esther had 40. The recording act in this jurisdiction
no actual or constructive notice may be:
of the deed to Dan and Carla.
I. Race
D. Prevail, since Esther and Dan
own Bushacres as tenants in II. Race-Notice
common.
III. Notice

A. I or II.

B. II or III.

C. I or III.

D. None of the above.

404 PROPERTY
Q
41. Which is the most accurate statement A. Valid contingent remainder.
regarding the conflicting claims to
Palmacres of Gus and Hal? B. Invalid contingent remainder.

A. Gus cannot prevail, since he C. Valid executory interest.


received a quitclaim deed which
automatically puts him on notice D. Invalid executory interest.
and, thus, not in good faith.

B. Gus' failure to record until after


Hal, equitably estops Gus from
asserting title against Hal (but
not against Fiona).

C. The outcome will turn on wheth-


er Hal paid value within the
meaning of the recording act.

D. None of the above.

42. Quincy was the owner of Shrubacres


(a tract of land) in fee simple absolute.
In 1950, he conveyed the property "to
Randy and her heirs as long as the prop-
erty is used for residential purposes, but
if it is ever used for non-residential pur-
poses prior to 1970, then to UNICEF, a
charity." In 1964, Quincy died leaving
a valid will which devised all Quincy's
real property to her niece, Selma. The
will had no residuary clause and Quincy
was survived by Selma and Quincy's
son Ted, her sole heir. Assume that the
common law Rule Against Perpetuities
applies and that all future interests are
alienable, devisable, and descendible
in the same manner as possessory in-
terests in land. In 1951, the interest in
Shrubacres held by UNICEF could have
best been described as a (an):

PROPERTY 405
Q
›-
Questions 43 — 46 are based on the B. Max, because a mortgagee can-
LLJ following facts: not be deemed to have paid
0-
0 value within the meaning of the
ce Lana owned Pineacres (a tract of vacant statute.
CL
land) in fee simple absolute. The recording
act for this jurisdiction states that unless a C. Nancy, because she recorded
conveyance or other transfer of land is duly first.
recorded, every deed or other conveyance of
an interest in land is void as to a subsequent D. Nancy, because she lent money
purchaser for value in good faith. In 1970, without notice of Max's interest
Lana sold Pineacres in fee simple for fair in Pineacres.
market value to Max by general warranty
deed. Max recorded his deed in 1985. In 44. Assume for this question only, that in
1975, Lana signed a mortgage to Pineacres 1995 Max sues Pedro claiming that
in favor of Nancy in return for money lent Max owns the property and that Pedro
by Nancy to Lana. Nancy was without ac- does not. If Pedro prevails, it will be
tual notice of the prior deed to Max. She because:
recorded her mortgage immediately. In 1980,
Lana gratuitously gave her then-boyfriend, A. As between two warranty deeds,
Oliver, a fee simple, general warranty deed the subsequent one controls.
to Pineacres. Oliver promptly recorded
his deed. In 1990, Oliver conveyed a fee B. Oliver, Pedro's grantor, had no
simple to Pineacres at full fair market value notice of Max's interest in Pine-
by general warranty deed to Pedro. Pedro acres.
had no actual knowledge of any of the prior
transactions in Pineacres. Pedro promptly C. Max's prior recorded deed is
recorded his deed. outside Pedro's chain of title.

43. Assume for this question only, that in D. Oliver, Pedro's grantor, recorded
1995 Max sues Nancy claiming that before Max.
Max owns the property and that Nancy
does not. The prevailing party owning 45. Assume for this question only, that in
Pineacres will be: 1989 Max sued Oliver claiming that
Max owned Pineacres and that Oliver
A. Max, because a mortgagee can- did not. The prevailing party owning
not be a subsequent purchaser Pineacres should have been:
within the meaning of the stat-
ute. A. Max, because Oliver was not a
bona fide purchaser for value.

406 PROPERTY
Q0
B. Max, because he was a purchaser 47. Irene leased her house and surround-
-

XI
for full market value and had no ing yard to Juan for fifteen years. The 0
13
actual or constructive notice of lease contained a clause prohibiting m
-I
the deed to Oliver. Juan from assigning his interest in the -<
lease. Kris has approached Juan about
C. Oliver, because Oliver had no subletting the house and yard to him.
notice of Max's interest in Pine- May Juan do so?
acres.
A. Yes, because restraints on alien-
D. Oliver, because Oliver's deed ation are invalid.
was the first recorded.
B. Yes, because a lease term pro-
46. Assume for this question only, that in hibiting assignments is strictly
1970 Lana did not sell Pineacres to construed.
Max; she gave it to him free of charge.
Assume also that Max did not receive C. No, because the term "assign-
a general warranty deed, but received ment" includes "subleasing" and
a quitclaim. Assume further, that in vice versa.
1989 Max sued Oliver claiming that
Max owned Pineacres and that Oliver D. No, because a tenant can never
did not. The prevailing party owning assign or sublease without the
Pineacres should have been: landlord's permission, since to
do so would be to impair the
A. Oliver, because Oliver's deed landlord's security interest in the
was the first recorded. property.

B. Oliver, because Max was not a 48. Ignatz owned Grassacres (a tract of
purchaser for value under the land) in fee simple absolute. He con-
statute. veyed it "to Jenny, her heirs and assigns
while the property is used for ranch
C. Oliver, because Max received purposes; then to Kelly and his heirs
a quitclaim deed which should forever. Assume the common law Rule
have put him on constructive Against Perpetuities applies to this con-
notice. veyance. Kelly's interest in Grassacres
as a result of this conveyance is best
D. Max, because his was the first described as:
conveyance from Lana.
A. A possibility of reverter.

PROPERTY 407
Q
B. A right of entry (or power of Questions 49 — 51 are based on the
LLI termination). following facts:
O
CL C. An executory interest. Terah leased a single family residence from
Utley for a term of four years. Without
D. Nothing. Utley's knowledge, Terah, a skilled carpen-
ter, installed three beautiful, hand-crafted
floor-to-ceiling oak bookcases, which he se-
curely bolted to the wall next to the fireplace.
He also replaced the old wooden steps lead-
ing to the front door with a new, larger and
more efficient set of stone steps and he added
a banister. At the end of his lease, Terah re-
moved the bookcases, leaving several deep
holes and other damage to the wall.

49. In an action for damages to the house


brought by Utley against Terah for the
removal of the bookcases, the prevail-
ing party will be:

A. Terah, but only if the bookcases


were used in his trade or busi-
ness.

B. Terah, but only if the jurisdiction


follows the modern trend in the
law of fixtures.

C. Terah, because the bookcases are


considered personal property.

D. Utley, because the bookcases are


considered real property.

50. In an action by Terah against Utley to


recover the cost of replacing the old
front door steps, Terah will:

408 PROPERTY
Q
A. Win, because he has created an D. Utley, if the jurisdiction pre-
improvement to Utley's freehold serves the common law doctrine
interest in the property. of ameliorative waste.

B. Win, but only if he added value


of the improvement will last be-
yond the term of Terah's lease.

C. Lose, if the jurisdiction preserves


the common law doctrine of per-
missive waste.

D. Lose, if the jurisdiction preserves


the common law doctrine of
ameliorative waste.

51. Assume for this question only, that


Terah does not remove the bookcases
when his leasehold ends, but leaves
them intact next to the fireplace. Con-
sequently, there are no holes or other
damage to the wall. In an action for
damages brought by Utley against
Terah regarding the bookcases, the
prevailing party will be:

A. Terah, because he has created an


improvement to Utley's freehold
interest in the property.

B. Terah, but only if the added value


of the improvement will last be-
yond the term of Terah's lease.

C. Utley, if the jurisdiction pre-


serves the common law doctrine
of permissive waste.

PROPERTY 409
Q
Questions 52 — 53 are based on the D. Denied, because Marcy has a
following facts: valid interest in Grassacres.

In 1968, Laura owned Grassacres (a tract 53. In 1969, UNICEF's interest in Gras-
of land) in fee simple absolute. She con- sacres could have best been described
veyed the property "to Marcy and her heirs as:
as long as the property is used for resi-
dential purposes, but if it is ever used for A. A valid contingent remainder.
non-residential purposes, then to UNICEF,
a charity." In 1974 Laura died, leaving a B. An invalid contingent remain-
valid will which devised all of Laura's real der.
property to her niece, Nella. The will had no
residuary clause and Laura was survived by C. A valid executory interest.
Nella and Laura's son, Oscar, her sole heir.
Assume that the common law Rule Against D. An invalid executory interest.
Perpetuities applies and that all future inter-
ests are alienable, devisable and descendible 54. Yetta was the fee simple owner of
in the same manner as possessory interests Wheatacres, a tract of land planted with
in land. a yearly, seasonal wheat crop. Yetta
reached agreement with her neighbor,
52. In 1980, Marcy and Oscar contracted Zedidiah, that in exchange for an an-
with Paul to sell Grassacres to Paul nual payment of $1000, after Yetta had
in fee simple absolute. After a title harvested her wheat crop, Zedidiah had
examination, Paul refused to perform the privilege to take and use the leftover
the contract because he claimed Marcy wheat from Yetta's fields to feed his
and Oscar could not convey good title animals. That arrangement was formal-
in fee simple absolute. In a lawsuit for ized in a written document, duly signed
specific performance by Marcy and Os- and recorded. By its terms, Zedidiah's
car against Paul, specific performance privilege was exclusive against all
will be: others except Yetta, who reserved the
right to use Wheatacres for any purpose
A. Granted, because Marcy and whatsoever, including harvesting all the
Oscar together own Grassacres wheat from her fields. Two years later,
in fee simple absolute. the state condemned Wheatacres by
eminent domain for use as a municipal
B. Granted, because Marcy alone golf course. In an action by Zedidiah
owns Grassacres in fee simple against the state for a portion of the
absolute. condemnation award for Wheatacres,
Zedidiah will:
C. Denied, because UNICEF has a
valid interest in Grassacres.

410 PROPERTY
Q
A. Win, because he has a profit a Questions 55 — 58 are based on the
prendre, which is a property right following facts:
protected by the due process
clause of the federal Constitu- Adam is the fee simple owner of Oakacres
tion. (a tract of land) located next to the Elmacres,
a tract of land which he also owned in fee
B. Win, because he has a license, simple. In 1980, Adam deeded Elmacres to
which is a property right pro- Boris in return for a promise in that same
tected by the due process clause document that Boris would not construct
of the federal Constitution. anything other than a single-family dwelling
on Elmacres. In 1985, Adam sold Oakacres
C. Lose, because he has a profit a to Carole. In 1990, Boris sold Elmacres to
prendre, which is not a prop- Della. With the promise in force, Oakacres
erty right protected by the due is worth $100,000. If Elmacres is used for
process clause of the federal anything other than a single family dwelling,
Constitution. Oakacres would be worth $60,000.

D. Lose, because he has a license, 55. Assume for this question only, that in
which is not a property right pro- 1984, Boris built a multiple-family
tected by the due process clause apartment house on Elmacres. In a
of the federal Constitution. lawsuit for $40,000 (damages to com-
pensate Adam for the fact that Boris did
not honor the 1980 agreement), Adam
should:

A. Win, because Boris broke the


1980 promise.

B. Win, but only if the burden of the


covenant touches and concerns
land.

C. Win, but only if the benefit of the


covenant touches and concerns
land.

D. Win, but only if the benefit and


the burden of the covenant touch
and concern land.

PROPERTY 411
Q
56. Assume for this question only, that in C. Lose, because Adam and Boris
1988, Boris built a multiple-family are not in horizontal privity with
apartment house on Elmacres. In a each other.
lawsuit for $40,000 (damages to com-
pensate Carole for the fact that Boris did D. Lose, but only if Adam seeks
not honor the 1980 agreement), Carole damages and not an injunction.
should:
58. Assume for this question only, that the
A. Lose, even though Boris broke 1980 promise by Boris to Adam was
the 1980 promise. not made in the Adam-Boris deed, but
made in a written, notarized document
B. Win, but only if the burden of the executed, delivered and recorded one
covenant runs with the land. month later for which Adam paid Boris
$1,000. Assume further that in 1991,
C. Win, but only if the benefit of the Della built a multiple-family apartment
covenant runs with the land. house on Elmacres. In a lawsuit for
$40,000 (damages to compensate Car-
D. Win, but only if the benefit and ole for the fact that Della did not honor
the burden of the covenant run the 1980 agreement), Carole should:
with the land.
A. Win, because Della broke the
57. Assume for this question only, that the 1980 promise.
1980 promise by Boris to Adam was
not made in the Adam-Boris deed, but B. Win, but only if the benefit and
made in a written, notarized document the burden of the covenant touch
executed, delivered and recorded one and concern land.
month later for which Adam paid Boris
$1,000. Assume further that in 1984, C. Lose, because Adam and Boris
Boris built a multiple-family apartment are not in horizontal privity with
house on Elmacres. In a lawsuit for each other.
$40,000 (damages to compensate Adam
for the fact that Boris did not honor the D. Lose, but only if the surround-
1980 agreement), Adam should: ing properties were zoned for
multi-family dwellings so as to
A. Win, because Boris broke the make Della's restriction an eco-
1980 promise. nomically inefficient use of the
land.
B. Win, but only if the benefit and
the burden of the covenant touch
and concern land.

412 PROPERTY
Q
1:I
59. Ginny is the fee simple owner of A. Valid, because Ellen's son's
Strawacres (a tract of land). In 1980, interest is vested subject to com- 0
'CP
Ginny built a house on Strawacres and plete divestment. m
began occupying the property. In 1992,
because she had never had a place to B. Valid, because the Rule Against
park her car before, Ginny built a garage Perpetuities does not apply to
on Strawacres. In 1991, she conveyed Ellen's son's interest
the mineral rights to the land beneath
Strawacres to Horace, who immediately C. Valid, because Ellen's son's in-
began mining the minerals. Horace terest will vest, if at all, within a
enlarged his mining operations under life in being at the creation of the
Strawacres, in 1993, until his exca- interest.
vation caused subsidence damage to
Ginny's house and garage. In a lawsuit D. Invalid.
by Ginny against Horace for subsidence
damage to her house and garage on
Strawacres, if Ginny cannot prove that
Horace negligently excavated beneath
Strawacres, she will recover:

A. Nothing.

B. Damages for the house, but not


for the garage.

C. Damages for the garage, but not


for the house.

D. Damages for the house and ga-


rage.

60. Chad owned Treeacres (a tract of land)


in fee simple absolute. He conveyed it
"to Della, her heirs and assigns; pro-
vided that if my sister, Ellen, should
ever have a son, then to that son, his
heirs and assigns." The limitation to
Ellen's "son, his heirs and assigns" is:

PROPERTY 413
Questions 61— 62 are based on the C. Not owe 1/2 of the property taxes
following facts: paid by Carl, but receive an offset
for the fair market rental value of
In 1980, Albertine conveyed Blackacre, Carl's use of the property.
which she owned in fee simple absolute,
"to Benny and Carl, jointly and forever." D. Not owe 1/2 of the property taxes
In 1990, Benny died, willing Blackacre to paid by Carl, and not receive an
Della. Carl, believing himself to be the sole offset for the fair market rental
owner of Blackacre, lived on Blackacre and value of Carl's use of the prop-
paid all property taxes. erty.

61. In 1990, after Benny's death, the title 63. Ellen was the owner of Whiteacre, a
to the property is held: tract of land which she owned in fee
simple absolute. In 1950, Ellen con-
A. By Carl only. veyed Whiteacre as follows: "to Fran-
cisco for life, and then to Francisco's
B. By Della only. children for life, and then to Francisco's
grandchildren." In 1950, Francisco was
C. By Carl and Della as joint ten- 40-years-old and had two living chil-
ants. dren, Guadalupe and Hortensia, but no
grandchildren. In 1952, another daugh-
D. By Carl and Della as tenants in ter of Francisco, Isabella was born, but
common. Isabella only lived until 1982. Which of
Francisco's grandchildren may receive
62. Assume for this question only, that an interest under Ellen's 1950 convey-
Carl and Della are tenants in common. ance?
In 1993, if Della sues Carl for an ac-
counting as a result of that action, Della A. The children, if any, of Guada-
will: lupe, Hortensia and Isabella,
since the "all or nothing" rule
A. Owe 1/2 of the property taxes applies.
paid by Carl, but receive an
offset for the fair market rental B. The children, if any, of Guada-
value of Carl's use of the prop- lupe and Hortensia, but not Isa-
erty. bella, since Isabella was born af-
ter the date of the conveyance.
B. Owe 1/2 of the property taxes
paid by Carl, but not receive an
offset for the fair market rental
value of Carl's use of the prop-
erty.

414 PROPERTY
C. The children, if any, of Isabella 65. Larry is the fee simple absolute owner
only, since in a per stirpes dis- of Greenacre, a tract of land with a 0
-0
m
tribution, each child will receive house located in the state of Columbia.
-4
his or her deceased parent's In 1980, Larry was 18 and lived with
share. his parents in the state of Marbury,
100 miles away from Greenacre. In
D. None, since the "all or nothing" 1980, Maxella moved into the house
rule applies. on Greenacre, planted a vegetable
garden, paid all taxes and assessments
64. In 1960, Joe was the owner of Blueacre, on Greenacre, and received mail and
a tract of land which he owned in fee telephone service at that address. In
simple absolute. In that year he died, 1982, Larry suffered a traumatic injury,
willing Blueacre "to Kermit for life, became insane and was committed to a
then to Kermit's children for life, and psychiatric hospital. The age of major-
then to Kermit's heirs." In 1960, Ker- ity in Columbia is 21. Assuming that
mit had one child. Kermit's interest in Columbia follows the common law of
Blueacre, after this devise, is a: real property, when, if ever, does Max-
ella gain title to Greenacre?
A. Life estate.
A. The year 2000.
B. Fee simple absolute.
B. The year 2003.
C. Life estate and a vested remain-
der. C. Twenty years after Larry regains
his sanity.
D. None of the above.
D. Never.

PROPERTY 415
Q
>.
i-
ce Questions 66 — 68 are based on the C. The Rule in Dumpor's case re-
w following facts: quires the landlord to accept this
CL
0 assignment.
ce Laurette leased to Ted 400 square feet of of-
a
fice space and four parking spaces in a park- D. The Rule in Spencer's case re-
ing garage to be constructed next door to the quires the landlord to accept this
leased office building. That lease contained assignment.
a covenant by Ted to pay Laurette $200 per
month for a term of three years, and another 67. Assume for this question only, that To-
covenant to paint and maintain the parking masso pays rent to Laurette who accepts
spaces in conformance with city codes. The it, but that Tomasso refuses to maintain
lease also prohibited assignments and/or the parking spaces according to the city
subleases without the prior permission of code. If Laurette sues Tomasso for dam-
the landlord. Six months after Ted signed ages under the lease provision regarding
the lease, the parking garage was completed the maintenance of the parking spaces
and Ted began the rental term and moved and Laurette loses, it will most likely
into the office. One year into his lease, Ted be because:
transferred his interest in the office and
parking spaces to Tina. Laurette wrote a A. A covenant concerning a thing
letter to Tina, stating that she (Laurette) ac- not yet in existence must ex-
cepted Tina as an assignee under the lease. pressly mention assignees in
After nine months, however, Tina moved order to bind them.
out and transferred her interest in the office
and parking spaces to Tomasso, who moved B. There is no horizontal privity for
into the property. the covenant.

66. Assume for this question only, that C. The benefit of the covenant does
Laurette refused to accept Tomasso as not increase or enhance the value
a tenant and accepted no money from of the land or the landlord's es-
him. If Tomasso sues Laurette to force tate therein.
her to accept him as a tenant, Tomasso's
best argument would be that: D. The burden of the covenant does
not curtail the use of the land or
A. Lease covenants are strictly con- diminish the value of the tenant's
strued against the landlord. estate therein.

B. A covenant prohibiting assign-


ments does not prohibit sub-
leases, and vice versa.

416 PROPERTY
Q
68. Assume for this question only, that C. Nevill, because intermittent use
Laurette accepted Tomasso as an as- is consistent with the nature of
signee, but that Tomasso never moved the property.
in. Assume further, that neither Tina nor
Tomasso ever paid rent. At the end of D. Nevill, if intermittent use is
the lease term, Laurette may collect: consistent with the nature of the
property.
A. $3000 from Tomasso, under
privity of contract.

B. $3000 from Tomasso, under


privity of estate.

C. $4800 from Tina, under privity


of contract.

D. $1800 from Tina, under privity


of estate.

69. In 1990, Nevill is the fee simple ab-


solute owner of Yellowacre, a tract of
land with a house located in the state of
Franklin. In 1990, Olive moved into the
house on Yellowacre and lived in that
house intermittently for twenty years,
without Nevill's permission. Assuming
that Franklin follows the common law
of real property, after the twenty year
period, who holds title to Yellowacre?

A. Olive, because intermittent use


is consistent with the nature of
the property.

B. Olive, if intermittent use is


consistent with the nature of the
property.

PROPERTY 417
Questions 70 — 73 are based on the A. $20,000.
ce
0
following facts:
0 B. $25,000.
ce Owen was the fee simple absolute owner of
Redacres, a twenty acre tract of land. Owen C. $40,000.
contracted to sell Redacres to Albert in fee
simple absolute, for $40,000, by warranty D. $50,000.
deed. After the close of escrow and delivery
of the warranty deed to Albert in fee simple 72. Assume for this question only, that
absolute, Albert discovered that Owen was Albert remained unaware of Owen's
not the sole owner of Redacre; he was a ten- tenancy in common interest in Redacres
ant in common with Lee. The value of that with Lee. Albert then sold Redacres by
land as a sole interest was $40,000; the value warranty deed for $50,000 to Bea. After
of that land as an undivided 1/2, as tenant in Bea paid the money to Albert and ac-
common, was $15,000. cepted the deed, Lee sued Bea to quiet
title in himself as a tenant in common
70. Assume for this question only, that in Redacres with Bea. If Bea then sues
Albert sues Owen in a timely manner Albert for breach of one of the cov-
for breach of the covenant of seisin. If enants for title in the Albert-Bea deed,
Albert prevails, his damages will be: Bea will recover:

A. $15,000. A. $20,000.

B. $20,000. B. $25,000.

C. $25,000. C. $40,000.

D. $40,000. D. $50,000.

71. Assume for this question only, that Al- 73. Assume the same facts as in question
bert purchased a title insurance policy 72, except that the deed from Albert to
for $50,000 on the property, but that the Bea was a quitclaim deed. If Bea sues
title insurance company did not disclose Owen for breach of one of the title
to Albert the fact that Owen only owned covenants, her damages will be:
a tenancy in common interest in Red-
acres. That fact was discovered by the A. $0, she received a quitclaim
title insurance company during its title deed.
search. If Albert sues the title insurance
company and prevails, his damages will B. $20,000.
be:

418 PROPERTY
C. $25,000. D. Never.
0
'CP
D. $40,000. 76. Wilbur owns a one-acre tract of land m
in the state of Jefferson. That land was
74. In 1950, Paulette, the fee simple owner located five miles away from the near-
of Purpleacres, conveyed Purpleacres, a est source of fresh water, the Jefferson
house and farm in the state of Madison, River. If Wilbur wants to transport
"to Quintin for life, then to Rula in fee water from the Jefferson River to use
simple." Quintin never moved onto to irrigate his crops, which system of
Purpleacres. Instead, in 1951, Sara did water rights for the state of Jefferson
so and used the property as her own. In will best allow him to do so?
1966, Quintin died. Rula never moved
to Purpleacres, and Sara continued to I. Natural Flow Riparian
live there. Assuming that the state of Rights.
Madison follows the common law of
real property, when, if ever, does Sara II. Reasonable Use Riparian
become the owner of Purpleacres? Rights.

A. The year 1969. III. Prior Appropriation


Rights.
B. The year 1971.
A. I and II only.
C. The year 1986.
B. II only.
D. Never.
C. III only.
75. Assume the same facts as in question
74, except that Sara moved onto Pur- D. None of the above.
pleacres in 1949. Assuming that the
state of Madison follows the common
law of real property, when, if ever,
does Sara become the owner of Pur-
pleacres?

A. The year 1969.

B. The year 1971.

C. The year 1986.

PROPERTY 419
77. Sam orally agreed to sell Orangeacre to Questions 78 — 79 are based on the
Beth for $100,000. That agreement was following facts:
reduced to a writing, which complied
with all the formalities for the statute of In 1930, Lee, the fee simple absolute owner,
frauds. It stated that Sam would transfer conveyed Violetacres "to Harry for life, then
by marketable title via a warranty deed to Lee's heirs." In 1940, Mark, the fee simple
a fee simple absolute in Orangeacre to absolute owner, conveyed Indigoacres "to
Beth on July 1. Sam contracted in writ- Harry as long as California requires attorneys
ing for Theresa to sign that Sam-Beth to pass the California Bar Examination." In
agreement as Sam's agent. Beth orally 1950, Nell, the fee simple absolute owner,
arranged to have Carlos sign that same conveyed Purpleacres "to Harry forever." In
agreement for Beth as her agent. The 1960, Harry married Wendy. In 1970, Harry
agreement described above was signed died; willing all his real property to Oscar.
by "Theresa as agent for Sam, the seller Violetacres, Indigoacres and Purpleacres are
"and by Carlos, "as agent for Beth, the all located in the state of Adams, and Adams
buyer." Beth later refused to complete retains the law pertaining to dower.
the purchase of Orangeacre. If Sam
sues Beth, and Beth pleads the statute 78. If Wendy sues to claim her dower
of frauds as a defense, Sam will: interest in Harry's property, in which
property(s) will she receive an inter-
A. Win, because the land sales con- est?
tract was in writing.
I. Violetacres.
B. Win, because the Sam-Theresa
contract was in writing. II. Indigoacres.

C. Lose, because the Beth-Carlos III. Purpleacres.


contract was not in writing.
A. I, II and III.
D. Lose, because the land sales con-
tract was not personally signed B. I and II only.
by the parties to be charged.
C. II and III only.

D. Neither I, II nor III.

79. If Wendy sues to receive her dower


interest in Harry's real property, the
interest she will receive is:

420 PROPERTY
Q
A. A fee simple absolute in all B. Succeed, because if the price
seised lands. agreed upon is a fair and equi- 0
table one, the seller is estopped m
B. A life estate in all seised lands. from pleading the statute of
frauds as a defense and that term
C. A 1/2 interest in all seised may be implied into the written
lands. memorandum.

D. A 1/3 interest in all seised C. Fail, because Bella has not of-
lands. fered evidence that $100,000
is the fair market value of Grey-
80. Bella orally agreed to purchase, and Sal- acre.
ly agreed to sell Greyacre for $100,000.
The parties reduced the agreement to a D. Fail, because price is an essential
writing which contained all of the ele- element of a written memoran-
ments required by the statute of frauds, dum necessary to satisfy the
except that there was no mention of the statute of frauds.
$100,000 purchase price (upon which
Sally and Bella agreed.) Sally has re-
fused to consummate the transaction
and deliver a deed to Greyacre. In an
action for specific performance by Bella
against Sally, Sally has claimed the
statute of frauds as a aefense. If Bella
offers into evidence the above described
written agreement, and also credible
evidence that the parties discussed and
agreed upon the $100,000 purchase
price, Bella's lawsuit for specific per-
formance will:

A. Succeed, because the law will


imply that the reasonable market
value of Greyacre was discussed
and agreed upon by the contract-
ing parties — even if the fair mar-
ket value was not the purchase
price orally agreed upon.

PROPERTY 421
Q
Questions 81 — 84 are based on the A. A life estate.
following facts:
B. A fee simple absolute.
In 1950, Lee, a fee simple owner of Scarleta-
cre, conveyed Scarletacre "to Agnes and the C. A remainder after Bruno's life
heirs of her body." In 1965 Agnes conveyed estate.
the property " to Bruno for life." In 1980,
Agnes died, survived by Bruno, Carlotta (the D. A fee tail.
child of Agnes) and Lee.
84. In 1980, after Agnes' death, what is the
81. In 1950, after the conveyance of Scarle- interest that Lee has in Scarletacre?
tacre to Agnes, what is the interest that
Lee has in that property? A. A possibility of reverter.

A. A possibility of reverter. B. A right of entry.

B. A right of entry. C. A reversion.

C. A reversion. D. None of the above.

D. None of the above. 85. Yvette owns a tract of land located in the
state of Monroe, adjacent to the Monroe
82. In 1965, after Agnes' conveyance to River. Yvette's land is downstream from
Bruno, what is the interest that Bruno the land owned by Zack. Both Yvette
has in Scarletacre? and Zack have water rights protected
by the laws of Monroe. Assume that
A. A life estate for the life of Agnes Yvette is receiving the same amount of
or Bruno, whomever dies first. water for her needs as she always had,
but that Zack has increased his use of
B. A life estate for the life of Agnes the water, thereby reducing the flow of
or Bruno, whomever dies last. water past Yvette's land. If Yvette seeks
to enjoin Zack's additional diversion
C. A life estate for the life of Agnes of water for his use, she will prevail if
only. Monroe follows:

D. A life estate for the life of Bruno A. Natural flow riparian rights
only. theory only.

83. In 1980, after Agnes' death, what is the B. Reasonable use riparian rights
interest that Carlotta has in Scarleta- theory only.
cre?

422 PROPERTY
Q
C. Either natural flow riparian rights Questions 87 — 89 are based on the
theory or reasonable use riparian following facts:
rights theory.
In 1960, Juan was the fee simple owner of
D. None of the above. Taupeacres, a tract of land with a house on
it. In 1961, the telephone company requested
86. The state of Jackson has a zoning or- and received a written agreement from Juan
dinance which classifies property uses giving the company the right to install and
into four types: single-family residen- maintain telephone poles and wires across
tial; multi-family residential; com- Taupeacres. The telephone company did
mercial; and industrial. Single-family in fact install poles and wires across Tau-
residential is considered the highest use peacres.
of a property, next multi-family, then
commercial, and industrial is the lowest 87. If in 1972 Juan sues to remove the
use. Jackson's zoning ordinances are telephone wires from Taupeacres and
cumulative. Which uses are permitted he loses, it will probably be because:
in the multi-family residential zone?
A. The telephone company has a
I. Single-family residen- license.
tial.
B. The telephone company has an
II. Multi-family residen- easement.
tial.
C. The telephone company has
III. Commercial. gained title to Taupeacres by
prescription.
IV. Industrial.
D. The telephone company has an
A. I and IV only. easement by necessity, since
phone service is necessary in
B. II only. today's society.

C. I and II only. 88. Assume for this question only, that in


1975 the telephone company permitted
D. II, III and IV only. a cable television company to place
its cables on the telephone company's
poles which crossed Taupeacres. If Juan
sues to prevent the cable company's use
of his land, Juan's best argument will
be:

PROPERTY 423
Q
A. The telephone company has a Questions 90 — 91 are based on the
non-exclusive easement. following facts:

B. The telephone company has an In 1960, Chad, the fee simple owner of
exclusive easement. Magentaacres, a tract of land, conveyed that
property "to the school board to be used for
C. The telephone company has an school purposes only." The school board
easement in gross. accepted the gift and constructed a school
on the property in 1961. The school board
D. The telephone company has an continued to use the property as a school
easement by prescription. until 1993, when due to a population shift
in the community, the school board decided
89. Assume for this question only, that the to level the school building and construct a
1961 agreement between Juan and the gas station/mini-market on Magentaacres.
telephone company was oral instead of All parties concede that the property has not
written. If in 1972 Juan sues the tele- been used for school purposes since 1993.
phone company to force the company
to remove its poles and wires from 90. In 1994, the school board seeks to quiet
Taupeacres and Juan loses, it will most title to Magentaacres in itself. A court
likely be because: will most likely quiet title to the prop-
erty in:
A. The telephone company has a
permanent easement. A. The school board, in fee simple
absolute.
B. The telephone company has a
revocable license. B. The school board, but only until
Chad retakes the property.
C. The telephone company has a
prescriptive easement. C. Chad, in fee simple absolute.

D. The telephone company has an D. Chad, but only if it is not eco-


irrevocable license. nomically efficient to continue to
use Magentaacres as a school.

91. For which interest in land should Chad's


lawyer argue, in order to retain for Chad
the greatest possible rights in Magenta-
acres, as a result of this conveyance?

A. A right of entry.

424 PROPERTY
Q
B. A power of termination. Questions 92 — 93 are based on the
following facts:
C. A possibility of reverter.
Louisa leased an apartment to Tilly for
D. A reversion. twenty-four months, beginning January 1,
1991. The lease was in writing and provided
for an annual rental of $4,800, payable in 12
monthly installments. On January 1, 1991,
Louisa handed the keys to the apartment to
Tilly. However, the previous tenant, Tom,
was still in possession of the apartment.

92. The type of tenancy created in Tilly by


the lease agreement was a:

A. Periodic tenancy, whose term is


month-to-month because of the
monthly rental installments.

B. Periodic tenancy, whose term is


year-to-year because the lease
states an annual rental term of
$4,800.

C. Tenancy at will, which is turned


into a periodic tenancy by the
acceptance of rent.

D. Tenancy for years, because a


definite ending date is provided
in the lease.

93. If Tilly sues Louisa claiming a violation


of the landlord's duties towards a ten-
ant because of the continued presence
of Tom in the apartment, Tilly, under
modern law, should:

A. Win, because Tilly is entitled to


actual possession of the estate
promised by Louisa.

PROPERTY 425
B. Win, because Louisa's lease Questions 94 — 95 are based on the
LLI
CL
with Tilly is an election by the following facts:
0 landlord to treat Tom's tenancy
at sufferance as a trespass. In 1960, Bo owned twenty acres with a house
on the south 1/2 of the property. Bo cleared
C. Lose, because Tilly, as the owner a dirt road from the house to the only public
of the non-freehold estate, is road which ran along the north frontage of
entitled to possession of the the property. In 1970, Bo gave, by warranty
apartment and therefore is the deed, the north 1/2 of the property to Celia
only one entitled to sue Tom in and kept for himself the south 1/2 with the
ejectment. house. Bo continued to use the dirt road to
reach the public highway.
D. Lose, because Louisa has fully
delivered the legal right to pos- 94. In 1989, if Bo is able to use the dirt
session to Tilly. road access Celia's property over her
objections, it is because he has an:

I. Easement by Implica-
tion/Quasi-Easement.

II. Easement by Necessity.

III. Easement by Prescrip-


tion.

A. I and II only.

B. II and III only.

C. I and III only.

D. I, II and III.

95. Assume for this question only, that in


1988 the county put in a new road along
the southern frontage of Bo's property.
If Celia seeks to prevent Bo from using
the dirt road across Celia's property,
Bo's strongest argument in opposition
will be that he (Bo) has:

426 PROPERTY
A. An easement by prescription. Questions 96 — 99 are based on the
following facts: 0
"CP
B. An easement by implication/
Quasi-easement. Minny and Nora are owners of adjoining
land. In 1970, Minny promises Nora that
C. An easement by necessity. Minny will maintain her land forever, as a
single-family residence, only. That promise
D. A license. was made in writing and was recorded with
the county recorders office. In 1975, Nora
conveyed her property to Peter, who did not
have actual knowledge of the promise.

96. Assume for this question only, that in


1980 Minny develops her land as a
multiple-family apartment complex.
If Peter sues Minny for damages for
breaching the 1970 Minny-Nora prom-
ise, Peter will:

A. Win, because the covenant runs


with the land.

B. Lose, because there is no vertical


privity.

C. Lose, because there is no hori-


zontal privity.

D. Lose, because the covenant does


not touch and concern land.

97. Assume for this question only, that in


1976 Minny conveys her property to
Otto, who does not have actual knowl-
edge of the promise. In 1980, Otto
develops his land as a multiple-family
apartment complex. If Peter sues Otto
for damages for breaching the 1970
Minny-Nora promise, Peter will:

PROPERTY 427
A. Win, because the covenant runs 99. Assume for this question only, that the
with the land. 1970 promise was not in writing and
therefore was not recorded. Assume
B. Lose, because there is no vertical further, that in 1976 Minny conveys
privity. her property to Otto, who does not have
actual knowledge of the promise. In
C. Lose, because there is no hori- 1980, Otto wishes to develop his land as
zontal privity. a multiple-family apartment complex. If
Peter sues Otto to specifically enforce
D. Lose, because the covenant does the 1970 Minny-Nora promise against
not touch and concern land. Otto, Peter will:

98. Assume for this question only, that A. Win, but only if Minny and Nora
in 1976 Minny conveys her prop- had an interest in the same land
erty to Otto, who does not have actual at the same time in 1970.
knowledge of the promise. In 1980,
Otto wishes to develop his land as a B. Win, but only if the character
multiple-family apartment complex. If of the land was such to make
Peter sues Otto to specifically enforce Otto aware of the single-family
the 1970 Minny-Nora promise against restriction.
Otto, Peter will:
C. Lose, because the 1970 promise
A. Win, but only if Minny and Nora was not in writing.
had an interest in the same land
at the same time in 1970. D. Lose, because there was no com-
mon plan or scheme for all of the
B. Lose, because the promise does lots.
not touch and concern land.

C. Lose, because there is no vertical


privity.

D. Lose, but only if the notice was


insufficient.

428 PROPERTY
Q
100. Larry leased Brownacres to Tilly for D. A deed of trust can be foreclosed
a period of three years beginning by a private sale, a mortgage can- 0
not. m
November 1, 1990. On November 5, 7C1
1993, Tilly was still in possession of
Brownacres without obtaining Larry's 102. Bernice agreed in writing to buy
prior consent to her continued presence Sarah's land. Since Bernice did not
on the property. As of that date, she has have enough cash to pay the purchase
not paid Larry any money for her pos- price immediately, they entered into
session of Brownacres after October the following land security device
31, 1993. Before Larry decides what agreement. Their agreement stated that
legal action to take with respect to Bernice would pay Sarah a monthly
Tilly's continued presence on Browna- percentage of the total purchase price
cres, she may best be describe as a: plus interest each month until the total
price and accrued interest was paid.
A. Periodic tenant. Bernice had the legal right to posses-
sion of the property and the obligation
B. Tenant for years. to pay taxes, etc. on it. As security for
the money owed, Sarah would transfer
C. Tenant at sufferance. legal title to Bernice when the total
price was paid. If at any time Bernice
D. Trespasser. defaulted on her payments, Sarah
could cancel the agreement, retain all
101. One of the principal differences be- moneys paid, and retake possession.
tween a mortgage and a deed of trust This land security device is most prop-
is: erly described as:

A. A mortgage can be foreclosed A. A deed of trust.


by a judicial sale, a deed of trust
cannot. B. An installment land contract.

B. A deed of trust can be foreclosed C. A mortgage.


by a judicial sale, a mortgage
cannot. D. A present transfer of a leasehold
coupled with a future interest
C. A mortgage can be foreclosed in fee simple absolute when
by a private sale, a deed of trust Bernice pays the full purchase
cannot. price.

PROPERTY 429
103. Aaron borrowed $100,000 from Barry C. Both Ellen and Frank. Even af-
to purchase Blackacre. In return, he ter the transfer, unless there is a
gave Barry a mortgage on Blackacre novation, Ellen is still personally
for 30 years. After 15 years, Aaron liable on the mortgage.
sold Blackacre to Carla who began
paying the mortgage debt to Barry. D. Frank. He is substituted for Ellen
However, after 5 years of payments, with respect to the mortgage.
Carla moved from Blackacre and
stopped paying rent. Which of the fol- 105. Gayle sold Blueacre to Hector, who
lowing people are personally liable on gave her a 30 year mortgage as security
the mortgage to Barry? for the purchase price. Fifteen years
later, Hector sold Blueacre to Ignatz,
A. Aaron. who assumed the mortgage. Ignatz
defaulted and Blueacre was sold at a
B. Carla. judicial sale for less than the remaining
debt. From whom can Gayle collect the
C. Aaron and Carla. deficiency on the debt?

D. Neither Aaron nor Carla. A. Hector.

104. Dilbert bought Greenacre from Ellen B. Ignatz.


who took a mortgage from him in
return for lending him the money for C. Hector and Ignatz.
the purchase price. Ellen transferred
her interest in the mortgage to Frank. D. Neither Hector nor Ignatz.
The Ellen-to-Frank agreement was
silent as to whether Frank assumed the
mortgage along with Ellen's interest in
it. After the Ellen-Frank transfer, the
mortgagee's rights and responsibilities
lie with:

A. Ellen. If a transfer is silent, it is


presumed to be taken "subject
to" the mortgage.

B. Frank. If a transfer is silent, the


transferee is presumed to be "as-
suming" the mortgage.

430 PROPERTY
Q
106. Jerrold sold Goldacre to Kyle, who A. Yes. Della took the property sub-
gave him a 30 year mortgage as se- ject to the mortgage and cannot
curity for the purchase price. Fifteen prevent the sale, even though she
years later, Lyle sold Goldacre to is not liable on the debt.
Lilly, who assumed the mortgage. As
a result of the transfer, Jerrold agreed B. No. Della took the property
to release Kyle from the debt since subject to the mortgage and she
the property was worth more than is not liable on the debt. The
the outstanding mortgage. However, property cannot be foreclosed
property values fell and the mortgage upon without her consent.
exceeded the value of the land. Lilly
defaulted and Goldacre was sold at a C. Yes. Marcy and Bob did not
judicial sale for less than the remaining pay Adam in 1970 when they
debt. From whom can Jerrold collect received $200,000 from Della.
the deficiency on the debt? Della should have insisted that
they do so and, therefore, is not
A. Kyle. a BFP.

B. Lilly. D. No. Della paid $200,000 to the


original mortgagor, more money
C. Kyle and Lilly. than the outstanding debt, there-
fore, she has been equitably
D. Neither Kyle nor Lilly. discharged from the mortgage.

107. In 1960, Marcy and Bob owned Sil- 108. In 1980, Nancy mortgaged Orangeacre
veracre, which they mortgaged to to Orville for $100,000. In 1985, Nan-
Adam for 30 years in order to borrow cy borrowed $35,000 from Paula and
$150,000 from him. In 1970, Marcy gave another mortgage on Orangeacre.
and Bob sold Silveracre to Della for Paula was aware of Orville's mortgage.
$200,000. Della did not agree to as- In 1990, Nancy defaulted on Orville's
sume the mortgage and she moved loan on which there was an outstanding
onto the property. In 1980, Marcy and balance of $80,000, although she did
Bob defaulted on the mortgage. Adam not default on her loan from Paula. Af-
now wishes to sell the property at a ter giving notice to all relevant parties,
judicial sale without Della's consent. Orville foreclosed and had the property
May he do so? sold at a judicial sale for $120,000 after
court costs, etc. The proper distribution
of the foreclosure proceeds is:

PROPERTY 431
Q
A. $80,000 to Orville first, then 109. In 1970, Rhetta mortgaged Redacre
LLI
0-
$40,000 to Nancy, since she is to Tom for $100,000. In 1980, Rhetta
0 not in default to Paula. borrowed $45,000 from Sally and
gave another mortgage on Redacre.
B. $35,000 to Paula first, because Sally was aware of Tom's mortgage.
she has had the security for her In 1985, Rhetta defaulted on Tom's
mortgage taken away without loan on which there was an outstand-
any action on her part, then ing balance of $80,000, and she also
$80,000 to Orville, and last defaulted on her loan from Sally ow-
$5,000 to Nancy. ing $40,000. After giving notice to all
relevant parties, Tom foreclosed and
C. $80,000 to Orville by first in had the property sold at a judicial sale
time, first in right, then $35,000 for $90,000 after court costs, etc. The
to Paula, and last $5,000 to proper distribution of the foreclosure
Nancy. proceeds is:

D. $80,000 to Orville by first in A. $60,000 to Tom (2/3 of $90,000),


time, first in right, then $35,000 $30,000 to Sally (1/3 of $90,000)
to Paula, and the remaining — since Tom is owed twice as
$5,000 to be divided by Or- much as Sally by Rhetta.
ville and Paula according to
their mortgage percentages be- B. $60,000 to Tom (75% of
cause Nancy is the person in $80,000), $30,000 to Sally (75%
default and cannot profit from of 40,000) — since $90,000
her wrongdoing. is 75% of $120,000 the total
amount owed.

C. $50,000 to Tom, $40,000 to Sally


since she is the junior mortgag-
ee.

D. $80,000 to Tom, $10,000 to Sally


since she is the junior mortgag-
ee.

432 PROPERTY
Q
110. In 1980, Ursala mortgaged Copperacre 111. In 1970, Xenia mortgaged Purpleacre
to Vern for $100,000. In 1985, Ursala to Zack for $100,000. In 1980, Xenia 0
m
borrowed $50,000 from Wilma and borrowed $45,000 from Yvetta and
gave Wilma a mortgage on Copper- gave another mortgage on Purpleacre.
acre. Wilma was aware of Vern's Yvetta was aware of Zack's mortgage.
mortgage. In 1990, Ursala defaulted on In 1985, Xenia defaulted on Zack's
her loan to Vern, owing him $80,000. loan, on which there was an outstand-
She did not default on her loan to ing balance of $90,000, but she was
Wilma, of which $40,000 remained. not in default on the second mortgage
Vern foreclosed on his mortgage, but to Yvetta. After giving notice to all
did not give Wilma notice of the fore- relevant parties, Zack foreclosed and
closure or the resulting sale. That sale had the property sold to Arthur at a
netted $80,000. Wilma's rights in her judicial sale for $90,000 after court
mortgage after Vern's foreclosure: costs, etc. What is the state of the title
to Purpleacre after the judicial sale to
A. Are extinguished; she was the Arthur?
junior claimant and there is no
money left from the sale. A. Title is free and clear in Arthur.

B. Are extinguished, but only if B. Title is in Arthur, subject to


the buyer at the foreclosure sale Yvetta's mortgage.
was unaware that Wilma was not
given notice of the sale. C. Title is free and clear in Arthur,
but only if he pays the balance
C. Are not extinguished; since she on Yvetta's mortgage.
was not given notice, the prop-
erty remains subject to her mort- D. Title is in Yvetta, but held in trust
gage and Ursala is still liable to for Arthur.
her personally.

D. Are not extinguished; since she


was not given notice, she is
entitled to be fully paid before
Vern.

PROPERTY 433
>-
Questions 112 — 113 are based on C. $40,000 to Cynthia, $50,000 to
LLI

the following facts: Alice.
0
ce In 1970, Alice mortgaged Yellowacre to D. None of the above.
Benjamin for $100,000. In 1980, Alice
borrowed $45,000 from Cynthia and gave 114. In 1980, Fiona mortgaged Browna-
another mortgage on Yellowacre. Cynthia cre to Gus for $100,000. Gus did not
was aware of Benjamin's mortgage. In record his mortgage although he was
1985, Alice defaulted on Cynthia's loan on permitted to do so by the state's record-
which there was an outstanding balance of ing act. In 1985, Fiona sold Browna-
$40,000, but she was not in default on the cre by warranty deed to Howard for
first mortgage to Benjamin on which there $150,000. Howard had no actual
was still $85,000 owing. After giving notice knowledge of Gus's mortgage. What
to all relevant parties, Cynthia foreclosed is the state of the title to Brownacre in
and had the property sold to Ed at a judicial 1986?
sale for $90,000 after court costs, etc.
A. Free and clear in Howard.
112. What is the state of the title to Yellow-
acre after the judicial sale to Ed? B. In Howard, subject to Gus's
mortgage.
A. Title is free and clear in Ed.
C. In Howard, who has a claim for
B. Title is in Ed, subject to Ben- breach of the covenant of seisin
jamin's mortgage. against Fiona.

C. Title would be free and clear in D. In Howard, who has a claim of


Ed, but only if the proceeds had breach of the covenant against
been enough to pay the balance encumbrances against Fiona.
on Benjamin's mortgage.
115. In 1980, Julie mortgaged Whiteacre to
D. Title is in Benjamin, but held in Mel for $100,000. Mel did not record
trust for Ed. his mortgage, since the state's record-
ing act did not encompass property
113. How should the proceeds of the judi- liens. In 1985, Julie sold Whiteacre by
cial sale be distributed? warranty deed to Kurt for $150,000.
Kurt had no actual knowledge of Mel's
A. $85,000 to Benjamin, $5,000 to mortgage. What is the state of the title
Cynthia. in 1986?

B. $40,000 to Cynthia, $50,000 to A. Free and clear in Kurt.


Benjamin.

434 PROPERTY
Q
B. In Kurt, subject to Mel's mort- 117. In 1970, Pete mortgaged Greyacre
gage. to Quentin. Quentin did not record,
although the state's recording act per-
C. In Kurt, who has a claim for mitted the recording of liens. In 1975,
breach of the covenant of seisin Pete transferred his mortgage to Ruby,
against Julie. who assumed that mortgage. In 1980,
the mortgage went into default and
D. In Kurt, who has a claim of Greyacre was sold for less than the
breach of the covenant of war- outstanding balance on the mortgage.
ranty against Julie. What are Quentin's rights?

116. In 1980, Oprah mortgaged Pinkacre to A. He may sue Pete, but not Ruby,
Lou for $100,000. Lou did not record since the mortgage was not re-
his mortgage since the state's record- corded.
ing act did not encompass property
liens. In 1985, Oprah sold Pinkacre B. He may sue Ruby, but not Pete,
by warranty deed to Ned for $150,000. since Ruby assumed the mort-
Ned had no actual knowledge of Lou's gage.
mortgage. If Ned's property is now
burdened with Lou's mortgage, al- C. He may sue both Ruby and
though not yet in default, Ned can sue Pete.
Oprah:
D. He may not sue either Ruby or
A. For breach of the covenant of Pete, since he did not record his
quiet use and enjoyment. mortgage.

B. For breach of the covenant


against encumbrances.

C. For breach of the covenant of


seisin against Oprah.

D. For breach of the covenant of


warranty against Oprah.

PROPERTY 435
118. In 1975, Samantha mortgaged Violet- 119. In 1970, Al sold Peachacre to Betty for
acre to Ted for $100,000. Ted did not $80,000. Betty recorded her warranty
record although the state's recording deed. In 1970, however, Peachacre was
act permitted the recording of liens. actually owned by Oscar. In 1980, Os-
In 1980, Ted transferred his interest car sold Peachacre to Al for $90,000.
in the mortgage to Umberto. Umberto That deed was recorded. In 1990,
recorded that transfer. In 1985, Saman- Al gave a mortgage on Peachacre to
tha gave a second mortgage to Velma Chuck for $70,000. That mortgage was
for $75,000. Velma was unaware of the recorded. What is the state of the title
prior mortgage and she recorded her after 1990?
mortgage. In 1990, Samantha default-
ed on both mortgages with $90,000 A. Title is in Betty, free and clear.
remaining to Umberto and $70,000
remaining to Velma. The property was B. Title is in Betty, subject to
sold at a foreclosure sale for $80,000 Chuck's mortgage.
after expenses. How should the court
award the proceeds? C. Title is in Al, subject to Chuck's
mortgage.
A. $80,000 to Umberto, since his is
the senior mortgage. D. Title is in Al, free and clear.

B. $70,000 to Velma, $10,000 to


Umberto, since Velma is the
junior interest.

C. $80,000 to Umberto, since his


is the first document to be re-
corded.

D. $70,000 to Velma, $10,000 to


Umberto, since Umberto's mort-
gage is not within her chain of
title.

436 PROPERTY
Q
120. In 1980, Dwane gave a mortgage on 121. Gwen and Hilda owned Oliveacre as
Aquaacres to Edna for $75,000. That joint tenants in 1980. In 1985, Gwen 0
-0
m
document was not recorded, although gave a mortgage on Oliveacre to
recoramg or liens. in i u, IJWUlle lJI1VeilLIC 10 is.erty. VV Hal IS tile stale 1/1

gave another mortgage on Aquaacres the title after Gwen's death?


to Felicia for $75,000. Felicia did not
have actual knowledge of the prior A. In Hilda, free and clear.
mortgage. Felicia recorded her mort-
gage. In 1995, Dwane defaulted on B. In Hilda, subject to Jeremy's
both mortgages, owing $65,000 on mortgage.
each. Both Edna and Felicia discovered
and gave notice to the other, and both C. In Hilda and Kelly, as tenants in
foreclosed on the property. Aquaacres common.
was sold for $65,000 after expenses.
Under which types of recording acts D. In Hilda and Kelly, as tenants
will Felicia get the $65,000 from the in common, subject to Jeremy's
foreclosure? mortgage on Kelly's share.

I. Race. 122. Lance, Mike and Norris all owned


Beigeacre as joint tenants in 1960. In
II. Race-Notice. 1970, Lance sold his undivided 1/3
to Orene. In 1975, Mike mortgaged
III. Notice his share to Perry. In 1980, Norris
died, willing his share to Quigley. If
A. I and II. Beigeacre is located in a state follow-
ing the title theory of secured land
B. H and III. devices, what is the state of the title
to Beigeacre in 1981?
C. I and III.
A. In Mike, free and clear.
D. I, II and III.
B. In Mike for 2/3 subject to Perry's
E. None. She is the junior mort- mortgage, and Orene for 1/3 as
gagee. tenants in common.

C. In Mike for 2/3, 1/3 subject to


Perry's mortgage, and Orene for
1/3 as tenants in common.

PROPERTY 437
D. In Mike for 1/3 subject to Perry's 124. In 1960, Beulah gave a deed of trust
mortgage, in Orene for 1/3, and to Sandacre to Carlos for $150,000.
in Quigley for 1/3 — all tenants In 1965, Beulah borrowed $75,000
in common. from Delilah and gave Delilah a deed
of trust on Sandacre. Delilah was
123. Renee and Sal were husband and wife aware of Carlos's deed of trust. In
who owned Taupeacre as concurrent 1980 Beulah defaulted on her loan to
owners in 1985. In 1990, Renee se- Carlos, owing him $130,000. She also
cretly gave a mortgage on her share in defaulted on her loan to Delilah, of
the property to Tony in exchange for which $60,000 remained. Carlos fore-
$100,000. In 1995, Renee died, leav- closed on his deed of trust, but did not
ing the mortgage in default. If Tony give Delilah notice of the foreclosure
wishes to sell Taupeacre at a judicial or the resulting sale. Delilah did not
foreclosure sale, he will most likely: foreclose. The judicial sale at Carlos's
foreclosure netted $100,000. What are
A. Be able to foreclose on Renee's Delilah's rights in her deed of trust
heirs who own her former ten- after Carlos's foreclosure?
ancy in common interest.
A. They are not extinguished. Since
B. Be able to foreclose on Sal, who she was not given notice, the
owns her former joint tenancy in- property remains subject to her
terest subject to the mortgage. deed of trust and Beulah is still
liable to her personally.
C. Not be able to foreclose on Sal,
because Renee's interest passed B. They are extinguished. She was
to him by right of survivorship. the junior claimant and there is
no money left from the sale.
D. Not be able to foreclose on Sal,
because a tenancy by the entirety C. They are not extinguished. Since
cannot be severed unilaterally she was not given notice, she is
by one member of a married entitled to be fully paid before
couple. Carlos.

D. They are extinguished, but only


if the buyer at the foreclosure
sale was unaware that Delilah
was not given notice of the
sale.

438 PROPERTY
Q
125. In 1970, Loni mortgaged Wheatacre END OF QUESTIONS
to Maria for $250,000. That mortgage
was recorded. In 1975, Loni borrowed
$175,000 from Nash and gave Nash
a mortgage on Wheatacre. Nash re-
corded his mortgage. In 1980, Loni
defaulted on her loan to Maria, owing
her $200,000. She did not default on
her loan to Nash, of which $160,000
remained. Maria foreclosed on her
mortgage, but did not give Nash notice
of the foreclosure or the resulting sale.
The property was sold at foreclosure to
Otto for $200,000. Since the foreclo-
sure sale, Loni has not paid anything
on her mortgage to Nash. If Nash seeks
to force a judicial foreclosure sale of
Wheatacre out from under Otto, he
will:

A. Prevail, but only if Otto had


actual knowledge of Nash's
mortgage.

B. Not prevail, once a senior interest


forecloses at a judicial sale, all
unpaid junior claims are extin-
guished.

C. Prevail, but only because she


was not given notice or joined
in Maria's foreclosure sale.

D. Not prevail, because Otto was a


BFP who purchased at a judicial
sale without Nash present and
without notice of Nash's inter-
est.

PROPERTY 439
440 PROPERTY
PROPERTY ANSWERS 4. D is the correct answer. This problem
depends on whether the jurisdiction
requires that a title searcher look in
1. C is the correct answer. The deed states the entire grantor-grantee index for all
that it is "subject to the understanding conveyances made by a grantor, even
that...", which states a contractual obli- those beyond the years the record shows
gation and not a limitation on the estate title in that person. If so, then the wild
granted. Either a fee simple determin- deed from Mary to Nancy will be within
able, B, or by a fee subject to condition Orene's chain of title and she is bound
subsequent, D, limits the estate. More- by constructive notice of it. Thus, es-
over, A is incorrect because although toppel by deed. C will not apply under
an equitable charge or servitude is that type of recording scheme and C is
enforceable in equity, it too needs to be incorrect, as well as A and B.
created with words of limitation on the
use of the property. 5. D is the correct answer. The question
asks for legal title to the fodder area
2. C is the correct answer and is really (land). Therefore, since the document of
the only correct statement of law. A which A speaks is inadequate to transfer
is incorrect because a co-tenant may title to land, A cannot be correct. B is
assign his share of the leasehold bar- also wrong since title to the land is not
ring other contractual provisions. B is an incident to the easement or profit
incorrect because a "no assignment" to cut fodder which are rights to use
clause grants rights to the landlord, but property. Similarly, C is also incorrect
not to the co-tenant as co-signatory. D for the same reason as B. Thus, D is the
is incorrect because although restraints only answer which correctly speaks of
on alienation are disfavored, they are title to land.
sometimes upheld when public policy
dictates, and "no assignment" clauses 6. A is the correct answer. Xavier had been
are examples of this policy choice. in adverse possession of Purpleacres
under color of title for 14 years when
3. B is the correct answer. The Rule the statute of limitations is 10 years.
Against Perpetuities is not violated. Title is in him from that point forward.
Since this is a will, at Oscar's death All subsequent events (i.e. the lease, the
we will know who all of his surviving transfer of rights to Yves, and the deed
children are regardless of age. Using to Norris are irrelevant to this problem.)
those children as measuring lives, we B, C and D are therefore incorrect.
must know, at each child's life, whether
or not that child attained age 30. Thus,
A, C and D are incorrect.

PROPERTY 441
7. B is the correct answer. Delivery of the 10. B is the correct answer. A covenant runs
deed requires acceptance by the grantee. with the land to bind successive owners
Here, Darla was unaware of the deed of the property regardless of their use
when made and when she became aware of the facilities. An easement, A grants
of the deed's existence, she did not ac- the right to use someone's property,
cept it. A is incorrect because even if a and does not pertain to the payment
grantee was unaware of the deed at the of money. Both a mortgage, C, and a
time of recording, the grantee's subse- personal obligation, D, are personal to
quent actions at the time of learning of the signatory of the promise and do not
the existence of the deed may constitute automatically pass to successor own-
valid delivery. C and D are incorrect ers.
statements of law.
11. C is the correct answer. The courts
8. C is the correct answer. The right to are reluctant to recognize affirmative
remove timber is incident to a fee and obligations to pay money indefinitely
Ben has a defeasible fee. Carl does have per covenants. A and B are incorrect
a future interest in Silveracres, so A is statements of law. D, while potentially
wrong. Bad faith is irrelevant to the correct as a matter of constitutional
right to take timber — ownership, and law, does not necessarily mean that
not possession, is the relevant factor; the provisions would in fact violate
thus, D and C are incorrect. the Constitution; in fact, they probably
would not.
9. B is the correct answer. The Statute of
Frauds requires a writing to transfer
real property, so despite the fact that
John and Joan orally agreed to a right
of survivorship, Joan died as a tenant in
common with John. Thus, the property
goes to their heirs equally. A states a
correct result for the wrong reason. C
and D are wrong because the Statute of
Frauds controls the result.

442 PROPERTY
12. C is the correct answer. The life ten- 14. D is the correct answer since it is the
ant is responsible for payment of the only answer that makes sense. Retalia-
mortgage principal during that estate tory eviction would prevent a landlord
in this instance. Although the general from raising rent in response to the as-
rule is that the life tenant is responsible sertion of a tenant's legal rights. C states
for the interest and the remainderman a rule of law which does not exist and
is responsible for the principal, in this B may or may not be true depending on
fact pattern the mortgage is for 10 what, if anything, this jurisdiction's law
years. Thus, the mortgage should be on unconscionability states. Although
completely satisfied within the life the payment of monthly rental may in
tenant's estate and Amanda should be some circumstances imply a periodic
responsible for the principal as well. tenancy, that does not directly affect the
However, the remainderman is respon- amount of rent due under that tenancy;
sible for any remaining portion and may therefore A is incorrect.
have his future interest affected if the
life tenant does not pay. A and B are 15. D is the correct answer. The answer
therefore incorrect. Bailey could try depends on the type of tenancy created.
to compel partition, D, but it is not his B and C state absolutely what the own-
only protection as the answer states; ership of Indigoacres is; however, the
therefore, it is wrong. question does not make that clear. Thus,
there are situations in which B and C
13. D is the correct answer. Otto entered might be wrong. D is the only answer
into a lease with Beatrice, even if it was which states the conditions under which
rent-free. The subsequent conveyance it will be true, and those conditions
to Aaron only gave Aaron rights in the make the answer true under all readings
property if he obtained a role in a Hol- of the facts in the question. Therefore,
lywood movie by age 35, something D is correct. A is wrong; constructive
he had not done. Thus, Otto is still knowledge is irrelevant.
the landlord of Beatrice. A is incorrect
because the key to the question lies in 16. C is the correct answer. Agnes began us-
recognizing the fact that Aaron's inter- ing the property with 011ie's permission
est was a future interest. B is wrong as and, therefore, no easement by prescrip-
a matter of law. C is the right result for tion could be gained. Since Paul never
the wrong reason. gained title to the property, his posses-
sion cannot affect Agnes since he was
never the owner of the property. Thus,
answers A, B and D, which all depend
on Paul's involvement in the problem
in some way, are incorrect.

PROPERTY 443
17. A is the correct answer. B is not a cor- 20. A is the correct answer. The restrictive
rect statement of law. The joint tenancy covenant in the deed from Oliver Wen-
cannot be severed by an oral agree- dell to Beth runs with the land to Della
ment. Whatever fiduciary obligations, even if the deed from Beth to Della did
if any, Betty had by being named as not physically contain that covenant.
executrix of Alice's estate cannot affect Thus, D and C are both incorrect.
title to land; C is incorrect. D is also Whether or not the apartment building
the Statute or Frauds tor ciose family value is irrelevant ro me answer ro 11115
relationships. Thus, although A is not question. Therefore, B is incorrect.
obviously always correct, it is the only
likely possibility. 21. C is the correct answer. D is an incorrect
statement of law: Equitable servitudes
18. B is the correct answer. The Statute of do not require privity with the original
Frauds demands a writing not present covenanting parties. A is a nonsense
in this problem. Neither A nor C are answer. It may sound good, but means
correct since a joint tenancy can be sev- nothing and is not even correct as a
ered unilaterally by one tenant through statement of constitutional law. Al-
a valid conveyance which need not be though B is true as an abstract statement
recorded. D, while true, does not have of principle, it has little to do with this
much to do with whether Alice severed question which specifically depends on
the joint tenancy during her lifetime whether Abby can say that it was origi-
— which is what the problem is about. nally contemplated that Oliver Wendell
intended to develop the 20-acre parcel
19. A is the correct answer. The question in the same manner as the 40 lots. Thus,
asks for Baker's best argument for C is the best choice.
immediate possession; therefore, C is
wrong since a month-to-month tenant
would need to give 30 days notice,
which Baker did not provide. Since
Selwyn was, in fact, once lawfully in
possession, he could not be a trespasser,
answer D. Only calling Selwyn a licens-
ee, A gives Baker the automatic right to
terminate Selwyn's use of the property
immediately and unequivocally. Calling
him a tenant at sufferance, B provides
Baker with the option of ejectment, but
still requires termination of the tenancy
by Baker.

444 PROPERTY
22. D is the correct answer. The question 25. D is the correct answer. A tenancy by
asks for the probable outcome of the the entirety absolutely requires a grant
litigation. The school has breached the to a validly married couple. The con-
condition on the land and, thus, dam- veyance granted a tenancy which can
ages and an injunction are proper. B be partitioned by the petition of one of
and C are, therefore, incorrect. The real the tenants. B and C are incorrect state-
choice is between A and D. The problem ments of law. Although A is true in the
with A is that it is not as specific as D abstract, it is not true on these facts.
as to the outcome of the suit. D states
that damages need to be impounded, 26. B is the correct answer. At the end of
presumably to await the determination Fred's life estate, the gift to his children
of who Juan's grandchildren will be at takes immediate effect. It is, therefore,
the time of Juanita's death. This answer a remainder and not an executory inter-
is more accurate than A because it is est, answer A. It vests now, but will be
more complete. divested "provided that...."; thus, it is
vested subject to divestment, answer
23. B is the correct answer. C is not correct, B, and is not contingent, answer D, nor
as a matter of law, because recording indefeasibly vested, answer C.
cannot insulate an invalid deed. D calls
upon a general equitable principle that 27. B is the correct answer. At the time
the most innocent person not bear the the land sale contract was executed,
loss, but that principle cannot control equitable conversion applies and the
over a legal reason like agency. Simi- purchase price is treated as the personal
larly, A states a general public policy property of the seller, and title to the
for certainty in land records, but that land is treated as real property of the
policy also cannot control over a legal buyer. Therefore, when Bernie died, the
reason like agency. Therefore, B is cor- proceeds of the sale were his personal
rect since, if true in this situation, Maria property and pass to Peter. A is, thus,
will lose the suit. incorrect because B is correct. C and
D are incorrect statements of law.
24. C is the correct answer. The Rule
Against Perpetuities is not violated
and so B is wrong. Using Dolly as the
measuring life, we must know who her
surviving children are within 21 years
of her death. Only the restrictions on
mortgage or sale are invalid as restraints
on alienation. Thus, A and D are incor-
rect.

PROPERTY 445
28. A is the correct answer. At the time 31. D is the correct answer. A is incorrect
the land sale contract was executed, because a landowner has an absolute
equitable conversion applies and the right to lateral support for unimproved
purchase price is treated as personal land. B is incorrect because even if
property of the seller and title to the the weight of the barn contributed to
land is treated as the real property of the the damage, he may still recover if he
buyer. Therefore, when Carla died, the can prove that Ethel was negligent in
title to the land was in her and passes her excavation. C is incorrect, for the
to Ricky, her heir who can demand reasons given for A and B, even if she
specific performance of the contract. B is negligent, Fred has an absolute right
is incorrect because A is correct. C and to lateral support for unimproved land.
D are incorrect statements of law. Therefore, D correctly states the full
legal principle.
29. A is the correct answer. The legal
description of land in a deed must be 32. B is the correct answer. The interest in
precise enough to determine which Helen's children immediately follows
property is being conveyed. This de- the preceding life estate; thus, it is a
scription is insufficient. The covenant of remainder and C is incorrect. Moreover,
seisin, answer B, consideration, answer since there is no child of Helen alive
D, and lack of facial ambiguity, answer at the time of the conveyance, the re-
C, are all irrelevant. mainder is contingent and A is incorrect.
Since the Rule Against Perpetuities is
30. D is the correct answer. Although there not violated (use Helen as the measur-
is a discrepancy in acreage B, the legal ing life), D is incorrect.
description of land given by the address
is sufficient to adequately identify the
property, especially where, as here,
the discrepancy is minor. There is no
requirement that the deed contain a
metes and bounds description, answer
A. C is incorrect because, although a
grantor cannot convey more than she
owns, the problem here is with the land
description in the deed, not the amount
of the property conveyed.

446 PROPERTY
33. D is the correct answer. An easement 35. C is the correct answer. The fact that
created by express grant is not de- the grantor retains possession of the
pendent on continued necessity for its deed creates a rebuttable presumption
continuation. Thus, A is incorrect. B of an invalid delivery and there is no
is incorrect since a change in circum- evidence which would rebut that pre-
stances does not necessarily terminate sumption here. As between the parties
an easement, and, in any event, this is to a transaction, recording of the deed
an express easement which is imposed is not necessary for a deed's validity.
and perpetuated despite any diminution Therefore, B and D are incorrect. A is
in market value. C is incorrect because incorrect because there is no conditional
this is an easement and not an incorpo- delivery on these facts.
real hereditament (which is an interest
in land incapable of being possessed 36. C is the correct answer. If Wilma rebuts
and only consisting of a right to use the presumption of invalidity which
something). This sewer was not only arises when the deed remains in the
used but possessed by the parties. grantor's possession by showing that
her mother intended to part with the
34. C is the correct answer. There is no property irrevocably, Victoria would
natural right to an easement for light, air lose. A is incorrect because there is no
or view. A states otherwise and is incor- oral condition on these facts which is
rect. B is incorrect because Quintin was being used to rebut delivery. B and D
under no obligation to inform anyone are incorrect because they state prin-
of his plans to lawfully develop his ciples of law which are irrelevant to this
property and, thus, under no obligation problem.
to "preserve" that right by complain-
ing about Susan's pool. Similarly, D is 37. D is the correct answer. Since repay-
incorrect because his right to lawfully ment of the principal goes to protect the
develop his property is not dependent future value of the remainderman's fee,
on informing his neighbors. Xerxes is responsible for that amount.
The interest represents the current use
of the money borrowed and is, there-
fore, payable by the current user of the
property, Will. A, B and C are incorrect
for the same reasons that D is correct.

PROPERTY 447
38. A is the correct answer. Zerlina's status 40. B is the correct answer. The question
as a bona fide purchaser is irrelevant as states that a junior claimant must be
between the covenanting parties, Zer- in good faith. Therefore, he must be
lina and Yvette. Therefore, for this rea- without notice and a Race statute is
son, D is incorrect. Yvette gave Zerlina inapplicable. The question does not tell
a warranty deed, which by definition us that the claimant must record first (or
contains all the title covenants. There- at all), and so the recording act could
fore, B is inapplicable here. Moreover, be either Race-Notice or Notice. A, C
the suit for breach of title covenants is and D are, therefore, incorrect.
a suit for breach by the original cov-
enantee, Zerlina, and thus, answer C is 41. C is the correct answer. Since the re-
inapplicable here, too. The suit for title cording act must be a Race-Notice or a
covenant breaches is dependent on the Notice statute (see answer above), Hal
original Yvette-Zerlina deed's original must prevail if he paid value. When
validity. Hal received his deed, Gus' deed was
not recorded (Hal wins under a Notice
39. A is the correct answer. If Carla and statute), and when Hal recorded first, he
Dan were legally married at the time wins under a Race-Notice statute. The
of Benjy's conveyance and remained only scenario under which Hal does
so when Carla conveyed to Esther, not prevail is if he is not a purchaser
the title was held by Carla and Dan and therefore cannot claim protection
as tenants by the entirety, and was under the act. A and B are inaccurate
inalienable by either of them without statements of law. D is incorrect for
the other spouse's consent. B is incor- the same reasons that C is the correct
rect because if Carla and Dan were not answer.
married during the relevant period, the
tenancy might have been a joint ten- 42. C is the correct answer. The interest
ancy which was severable by Carla's in UNICEF followed a defeasible fee
intervivos conveyance. Dan would not in Randy. Therefore, it could not have
be the sole owner. D is incorrect for the been a remainder of any sort. A and
same reason A is the correct answer, i.e., B are incorrect. The interest does not
Carla and Dan's possible marriage. C is violate the Rule Against Perpetuities
incorrect because notice is irrelevant to because the residential-purposes condi-
this problem. tion only applies until 1970. Therefore,
using anyone alive in 1950 as the mea-
suring life, at their death plus 21 years,
we must know whether the property has
been used for residential purposes until
1970. D is, therefore, incorrect.

448 PROPERTY
43. D is the correct answer. This is a notice 46. D is the correct answer. Max prevails
statute, giving priority to purchasers for because of the common law rule of
value who take without notice. A and B "first in time, first in right." His status
are incorrect because a mortgagee pays as purchaser, or taker under a quitclaim
consideration (the loan proceeds) for are all irrelevant. B and C are, thus, in-
the interest in land received (mortgage) correct. A is incorrect since it refers to
and would qualify under the statute as priority under the recording act which
an "interest in land." C is incorrect, is irrelevant here.
because under a notice statute, priority
in recording is not required to prevail. 47. B is the correct answer. Covenants in
a lease against assignment or sublet-
44. C is the correct answer. Since Max ting are strictly construed against the
did not record his deed until 1985, his landlord. C and D are incorrect since
deed is outside of Pedro's chain of title they state erroneous conclusions of law.
because it cannot be found under the Although they are disfavored, those
traditional method of searching title. lease covenants are not always invalid.
Therefore, Pedro is without notice of A is therefore incorrect.
Max's interest in Pineacres and entitled
to prevail under a Notice statute. A is an 48. D is the correct answer. Kelly's interest
erroneous statement of law. B and D are would have been an executory inter-
incorrect because Pedro's rights derive est since it follows a fee — had it not
from his being without notice and are violated the Rule Against Perpetuities.
not dependent on the equities regarding It violates the Rule since there is no
his grantor, Oliver (who may not be able life in being such that at that person's
to prevail himself in an action against death, we must know whether or not the
Max — see below). property will cease being used for ranch
purposes. Thus, A and B are wrong even
45. A is the correct answer. Oliver is a donee before the application of the Rule, and
and not a purchaser for value. Thus, the C is incorrect after the application of
recording act is inapplicable to him. The the Rule.
property will be awarded by common
law "first in time, first in right," to Max.
B is incorrect because the reason Max
prevails is that he was the first convey-
ance from Lana. His knowledge and
purchase price are irrelevant. C and
D are incorrect since these statements
refer to priority under the recording act
which is inapplicable to Oliver.

PROPERTY 449
49. D is the correct answer. Since the book- 51. D is the correct answer. If common law
cases were permanently affixed to the ameliorative waste is still the law of
realty, they are treated as fixtures. Thus, this jurisdiction, a tenant cannot make
C is incorrect. Whether or not they are alterations to the premises — even those
considered trade fixtures and remov- which improve the property. The book-
able at the end of the lease (answer A), cases are an alteration to the premises.
or non-trade fixtures and removable A and B are incorrect because, although
per the modern trend (answer B), the modernly a long-term or life tenant may
removing tenant must remove them be permitted to set off the value of the
without doing damage to the realty — a improvements against any money owed
requirement contradicted by the facts the landlord, at common law he could
here. A and B are, therefore, incorrect. not. Neither A nor B make that distinc-
tion. C is incorrect because permissive
50. D is the correct answer. If common law waste (failure to make repairs), does not
ameliorative waste is still the law of apply to this situation.
this jurisdiction, a tenant cannot make
alteration to the premises — even those 52. D is the correct answer. The interest
which improve the property. A and B in UNICEF violates the Rule Against
are incorrect because, although mod- Perpetuities since there is no life in be-
ernly a long-term or life tenant may ing such that at that person's death, we
be permitted to set off the value of the must know whether the property has
improvements against any money owed ceased being used for residential pur-
the landlord, at common law he could poses. Therefore, the executory interest
not. Neither A nor B make that distinc- in UNICEF fails and is replaced by a
tion. C is incorrect because permissive possibility of reverter in Laura, which
waste (failure to make repairs) does not is willed to Nella in 1974. Therefore,
apply to this situation. A, B and C are all incorrect.

53. D is the correct answer. The interest


in UNICEF followed a defeasible fee
in Marcy. Therefore, it could not have
been a remainder of any sort. A and B
are thus incorrect. The interest violates
the Rule Against Perpetuities (see an-
swer above), and, thus, C is incorrect.

450 PROPERTY
54. A is the correct answer. Zedidiah has 57. A is the correct answer. As between
a written document from the owner of the original parties to a covenant, there
Wheatacres entitling him to enter the is no requirement that the covenant
property and remove wheat from it. (burden or benefit) run with the land.
This describes a protectable property The arrangement need only be in writ-
interest under the due process clause ing as this one is. Similarly, there is no
of the federal Constitution (called a requirement of horizontal privity either;
profit a prendre). B and D are incorrect it is irrelevant to this problem that Boris
since this interest is not a license. C is and Adam do not have an interest in the
incorrect because a profit is an interest same land at the same time (horizontal
in land, the taking of which requires privity). C is therefore incorrect. D is in-
compensation by the state. correct since the difference in remedies
is relevant to whether the agreement can
55. A is the correct answer. As between he be enforced as a covenant (damages)
original parties to a covenant, there is or servitude (injunction). This promise
no requirement that the covenant (bur- could be enforced as either.
den or benefit) run with the land. The
arrangement need only be in writing, as 58. C is the correct answer. Carole, a
this one is. Therefore, B, C and D are grantee from the original covenantee,
incorrect. Adam is seeking to enforce the cov-
enant against Della, a grantee from the
56. C is the correct answer. Since this cov- original covenantor. The burden and
enant is sought to be enforced by Car- benefit of the covenant must touch and
ole, the grantee of the covenantee, only concern land. However, there is also a
the benefit side of the promise must run requirement that the original covenant-
with the land. If it does so, Carole may ing parties be in horizontal privity with
enforce this covenant against Boris, the each other. Therefore, A and B are
original covenantor. B and D are incor- incorrect. D is incorrect because the
rect for the same reasons that C is the validity of a covenant is not affected
correct answer. A is incorrect because solely by economic efficiency or the
the requirements for the benefit of a zoning of adjacent properties.
covenant to run are met on these facts.
59. B is the correct answer. A landowner has
the absolute right to subjacent support
for land and all buildings existing on
the date of severance of the underlying
strata. In order to recover for damage
to subsequently erected buildings, a
landowner must prove negligence.
Therefore, A, C and D are incorrect.

PROPERTY 451
60. C is the correct answer. Ellen's son's 63. D is the correct answer. The class gift
interest is an executory interest. There- to the grandchildren is void under the
fore, A is incorrect since it is not vested. Rule Against Perpetuities. Presently, no
The Rule Against Perpetuities applies person exists as a "measuring life" such
to all executory interests, and, thus, B that, at that person(s)' death, plus 21
is incorrect. However, the Rule is not years, you will know who all the grand-
violated since using Ellen as the mea- children are. Since it is possible for
suring life, one must know at her death Francisco to have children born after the
plus 21 years whether or not she has a date of the instrument, the class of his
son. D is, therefore, incorrect. children cannot be used as measuring
lives. Therefore, by the "all or nothing"
61. D is the correct answer. The 1980 con- rule, since it is possible for one member
veyance did not expressly create a joint of the class (grandchildren) to vest (be
tenancy and thus was a tenancy in com- born) after the perpetuities period, the
mon. Therefore, when Benny died, his gift to the class must completely fail,
1/2 interest passed to Della, who shared and the interest is void. Therefore, A
the property with Carl. Consequently, is incorrect. B is incorrect because the
A and B are incorrect since they state date of Isabella's birth, per se, is not rel-
sole interests. C is incorrect because the evant. C is incorrect, because although
interest shared by Carl and Della is not in a per stirpes distribution the children
a joint tenancy. of a deceased parent do step up to take
their parent's share, that principle is
62. B is the correct answer. A co-tenant irrelevant to this problem.
owes a proportionate share of taxes
and other expenses of the property. 64. C is the correct answer. The will
However, since each co-tenant has the originally gave Kermit a life estate, his
exclusive right to possess the entire children a vested remainder subject to
property, Carl does not owe the fair open (one child is alive in 1960), and a
market rental value of the property to contingent remainder in Kermit's heirs.
Della. Therefore, since B is the only The Rule in Shelly's case changes the
answer to contain the correct combina- remainder in Kermit's heirs to a vested
tion of those two principles, A, C and remainder in Kermit which cannot
D are incorrect. merge with Kermit's life estate because
of the vested interest in Kermit's chil-
dren. Therefore, A is incorrect because
it ignores the Rule in Shelly's case. B is
incorrect because it improperly applies
merger. D is incorrect for the reasons
stated above.

452 PROPERTY
65. B is the correct answer. Maxilla has hos- 67. A is the correct answer. The parking
tilely, openly, actually and continuously garage was not yet in existence when
adversely possessed Greenacre for the a promise was made concerning the
statutory period of 20 years. However, maintenance of the spaces. That prom-
the statute of limitations for adverse ise did not expressly mention assigns,
possession begins in 1983 when Larry's and therefore does not bind successors.
disability of infancy is terminated. A B is incorrect because there is hori-
is incorrect because it ignores the fact zontal privity in that Laurette and Ted
that Larry is disabled by his age from were landlord and tenant respectively.
bringing an action in ejectment. C is C and D are incorrect since the benefit
incorrect because there is no tacking and burden do touch and concern land
of disabilities. D is incorrect because in that the parking spaces would be
of the reasons stated above. maintained, causing a benefit to the
landlord's reversion and a detriment
66. C is the correct answer. The Rule in through the expenditure of time/labor/
Dumpor's case states that once the money by the tenant.
landlord gives consent to one assign-
ment, any covenant in a lease requiring 68. D is the correct answer. Tina possessed
consent is terminated. Because Laurette the office space for nine months; thus,
already consented to the assignment constituting an obligation of $200 per
to Tina, she did not need to give her month under privity of estate, for that
consent to any other assignment to To- time. Since she never signed a lease
masso. A is incorrect because although agreement with Laurette, she is not
lease covenants may be construed bound by privity of contract for any
against the landlord, that principle is amount of money. Thus, C is incorrect.
merely a generalized policy statement Since Tomasso never signed a lease, nor
and would not necessarily determine the moved into the building, he is bound by
outcome of a case, like answer C. B is neither privity of contract nor privity of
a correct statement of principle, but is estate, respectively. Therefore, A and B
inapplicable to these facts. D refers to are incorrect.
the Rule in Spencer's case, which states
that a covenant relating to a thing not
yet in existence must expressly bind
successors. That rule is also inappli-
cable to the facts.

PROPERTY 453
69. B is the correct answer. In adverse 72. B is the correct answer. The measure
possession, a possessor does not have of damages for a breach of one of the
to constantly live on the land for the title covenants, regarding title defects,
statutory period, but need only make is the purchase price received by the
such use of the property consistent with person making the promise (plus inter-
its nature and character, e.g., using a ski est). Since Owen was only an owner
cabin during ski season. A is incorrect of 1/2 of the interest promised in the
because it assumes the essential issue deed, the damages here would be 1/2
to be resolved, i.e., whether intermit- the purchase price received by Albert
tent use is consistent with the nature of ($50,000/2= $25,000). A, C and D are
the property. C and D state conclusions all incorrect figures.
which contradict the legal principle in
this problem. 73. B is the correct answer. The measure of
damages for a breach of one of the title
70. B is the correct answer. The measure of covenants, regarding title defects, is the
damages for a breach of the covenant of purchase price received by the person
seisin is the purchase price (plus inter- making the promise (plus interest).
est). Since Owen was only an owner of Since Owen was only an owner of 1/2
1/2 of the interest promised in the deed, of the interest promised in the deed, the
the damages here would be 1/2 the pur- damages here would be 1/2 the purchase
chase price ($40,000/2= $20,000). A, C price received by Owen ($40,000/2 =
and D are all incorrect figures. $20,000). C and D are incorrect fig-
ures. A is incorrect because although
71. B is the correct answer. The measure of the Albert-Bea deed was a quitclaim,
damages for breach of a title insurance Bea can sue under the future covenants
contract is normally the diminution in given in the Owen-Albert deed.
fair market value of the land caused by
the defect. Since the value of the land 74. C is the correct answer. In adverse
as promised was $40,000 and the value possession, the possessor gains the es-
of the land as a tenancy in common is tate in existence at the date she began
$15,000, the damages equal $40,000 possession. In 1951, when Sara began
— $15,000, i.e., = $25,000. A, C and D possession of Purpleacres, the estate in
are all incorrect figures. existence was a life estate in Quintin.
After his death in 1966, the remainder in
Rula went into effect and a new period
of adverse possession began. That pe-
riod would be complete in 1986. Thus,
A, B and D are incorrect.

454 PROPERTY
75. A is the correct answer. In adverse 78. C is the correct answer. A wife received
possession, the possessor gains the es- a dower interest of 1/3 in all the land in
tate in existence at the date she began which her husband was seised at his
possession. In 1949, when Sara began death. At Harry's death, his life estate
possession of Purpleacres, the estate in in Violetacres ended. Thus, answers A
existence was a fee simple in Paulette. and B, which state Wendy has an inter-
The subsequent conveyance and divi- est in that property, are incorrect. The
cinn of thr. ■ci-atir• wac irt-FdPArant tri t1ii riPfplacihlta fPPj n Indiana•rt.c and thg. fgla

session. That period was completed in (or his heirs) after his death, and Wendy
1969. B, C and D are incorrect for the had dower in those lands. Therefore,
same reasons that A is correct. answer D is incorrect.

76. C is the correct answer. Since Wilbur's 79. D is the correct answer. A wife received
land is not located adjacent or beneath a dower interest in 1/3 in all land in
the Jefferson River, it is not riparian which her husband was seized at his
land. Therefore, A and B, which are death. A, B and C are, therefore, incor-
riparian theories, only permit water to rect.
be taken for use on riparian land and
are not available for Wilbur's use. Prior 80. D is the correct answer. In order to
appropriation gives rights in water to satisfy the statute of frauds, a written
the first taker, regardless of his or her memorandum must contain the price
ownership or use on riparian land. It is term. A and B are incorrect, as an es-
the only water rights theory available sential term price may never be implied
to Wilbur. D is incorrect for the same by the court in a land sales contract. C
reason that C is correct. is incorrect. Whether or not Bella intro-
duced evidence of fair market value, the
77. C is the correct answer. The statute of written memorandum must contain the
frauds requires that contracts for the actual price agreed upon.
sale of land, and also agency contracts,
be in writing. Therefore, although the 81. C is the correct answer. The 1950 con-
land sales contract and the Sam-Theresa veyance created a fee tail in Agnes,
contract are in writing, the Beth-Carlos and, therefore, left a reversion in Lee.
contract was not. Thus, A and B are A, B and D are incorrect since they
incorrect. D is incorrect since there state other interests in grantors — none
is no requirement that the parties to a of which are present here.
contract personally sign the agreement.
An agent's signature is binding on the
principal.

PROPERTY 455
82. A is the correct answer. Since Agnes 85. A is the correct answer. The natural-
had a fee tail, after her death the land flow riparian rights theory gives a
must transfer to her lineal descendants downstream owner the right not to have
or, if none, it must revert to the grantor. the flow of water diminished in either
Therefore, Bruno's interest must end quality or quantity from its natural state.
at Agnes' death. Moreover, since she This is true even if the downstream
gave him a life estate, it must also end owner had sufficient water for her
at his death. Consequently, C and D are needs. The reasonable-use theory only
incorrect. B is incorrect since the death permits injunction if the downstream
of either Agnes or Bruno immediately owner has insufficient water for her
ends the estate. Thus, it is the first to die, needs. Since answer A is the only option
not the last, which terminates Bruno's which presents the natural-flow theory
estate. alone, it is the only correct answer.

83. D is the correct answer. Agnes had a 86. C is the correct answer. A cumulative
fee tail which descends to her lineal zoning ordinance is one in which higher,
heirs, here Carlotta, the child of Agnes. but not lower, uses are permitted. Since
A, B and C are all estates which may single-family residential is a higher use
approximate a fee tail, but are not the than multi-family residential, both those
estate granted to Agnes and inherited uses are permitted in a multi-family
by Carlotta. zone. A, B and D are incorrect since
they do not state that combination.
84. C is the correct answer. After Agnes
died, her fee tail passed to her daughter, 87. B is the correct answer. The grant in
Carlotta. Lee still retains a reversion writing of the right to enter Juan's land
which will go into possession, if ever, and install and maintain telephone
after Agnes' lineal heirs all die. A, B and equipment is an easement. A is incorrect
D are incorrect since they all state in- since a license only grants permission to
terests in the grantor not present here. enter the land of another. If a license had
been granted, Juan could have forced
the telephone company to remove its
equipment by withdrawing his permis-
sion. C incorrectly applies the law, and
D misstates the law.

456 PROPERTY
88. A is the correct answer. Juan expressly 89. D is the correct answer. If the agree-
granted the telephone company the right ment was not in writing, it could not be
to enter the land; therefore, it could an easement. Therefore, A is incorrect.
not have been granted an easement by Moreover, since Juan gave his permis-
prescription. D is incorrect. The type sion for the telephone company to
of easement granted is an easement install and maintain its equipment, the
in gross, since the telephone company company could not gain a prescriptive
owns no land benefited by the ease- easement (which is adverse to the owner
ment. However, C is incorrect since the of the servient estate). C is, thus, incor-
fact that the easement is in gross is not rect. If the license were revocable, then
relevant to whether that easement can Juan could force the telephone com-
be apportioned/transferred to the cable pany to remove its equipment. Since
television company. A non-exclusive the question states that Juan loses, B
easement is one in which the owner cannot be the correct answer. The oral
of the servient estate (Juan) retains agreement must have been a license,
the right to make the same use of his which was made irrevocable through
land that was granted by the easement. the expenditure of money and labor by
Therefore, the owner of a non-exclusive the telephone company in installing and
easement (but not the owner of an ex- maintaining its equipment. Therefore, D
clusive one) cannot apportion/transfer is correct.
it to another, since to do so would be in
derogation of the rights of the owner of
the property to sell another easement
to the cable company. B is, therefore,
incorrect.

PROPERTY 457
90. A is the correct answer. Although the 91. C is the correct answer. A and B state the
conveyance is somewhat ambiguous, it same interest in land — the future inter-
does not state the limitation for school est in the grantor after he or she conveys
purposes in such a way as to cut down an estate on condition subsequent. That
the estate granted (the length of time interest must be affirmatively exercised
that the school board is given title to by the grantor before it goes into effect.
the property). Therefore, given the Answer C correctly states the interest
courts' policy preference for the estate in the grantor, which automatically fol-
that will least result in forfeitures, the lows the divesting event in a determin-
most likely result of the conveyance is able estate. Thus, C is better than A or
that Chad granted the school board a fee B. D is not possible on this set of facts,
simple absolute with a covenant that the since a reversion cannot follow a fee
property would be used for school pur- (the present interest here).
poses. In that event, in 1993, the school
may be liable in damages for breach of 92. D is the correct answer. The lease states
the covenant, but its estate continues. that it is to run for 24 months. Therefore,
B refers to the school board's having a it has a definite ending date, December
fee simple on condition subsequent, fol- 31, 1993, and, consequently, must be a
lowed by a right of entry in Chad. That tenancy for years. The provision of an-
answer is not as good as A, but better nual rent or monthly rental installments
than C. C refers to the school board as cannot contradict the fixed term in the
an owner of a fee simple determinable, lease. Periodic tenancies continue for
which contains an automatic possibility period-to-period because they do not
of reverter in Chad. The language of the have a definite ending point. A and B
grant did not so specify. D is simply are, thus, incorrect. D states a possible
wrong — economic inefficiency cannot scenario in a holdover tenant situation,
change the estate granted. which is not present in the Louisa-Tilly
agreement.

93. A is the correct answer. Modernly, the


landlord has a duty to deliver actual
possession of the leasehold to the tenant
at the beginning of the lease. C and D
state the common law rule of delivery
of the legal right to possession only,
and are accordingly incorrect. B may
be correct as a factual statement, but it
does not answer the question posed.

458 PROPERTY
94. A is the correct answer. This scenario 96. A is the correct answer. Since Peter is
meets the elements for both an ease- suing the original promisor, Minny, only
ment by implication/quasi easement the benefit side of this covenant needs
(common owner, quasi-easement, ap- to run with the land. All those elements
parent, continuous and strict necessity) are met: Intent, the parties intended the
and an easement by necessity (common promise to run "forever"; Notice, the
grantor, strict necessity). Since there promise was recorded; Touch and Con-
have only been 19 years of use by Bo cern, the promise will raise the value
against the use by his grantee, Celia, an of Peter's land by limiting the uses to
easement by prescription is not possible be made on Minny's land; and Vertical
here (such requires 20 years). There- Privity, Peter is Nora's grantee. Thus,
fore, A is the only answer which states B and D are incorrect. Since only the
the correct combination; B, C and D do benefit needs to run, horizontal privity
not. (choice C), is not required.

95. B is the correct answer. As stated above, 97. C is the correct answer. Unlike #96,
in the answer to #94, an easement by when Peter sues Otto, both the benefit
necessity and an easement by implica- and the burden of the covenant need to
tion/quasi-easement are possible here. run. As stated above, the benefit does
However, if an easement by necessity run with the land. However, the burden
is chosen, the easement will terminate cannot since Minny and Nora did not
whenever the necessity does. That limi- have an interest in the same land at the
tation does not apply to the easement by same time that the promise was made
implication/quasi-easement. Here, the (horizontal privity). A is incorrect since
necessity ends when the county puts in the covenant does not run with the land.
the new road in 1988. Therefore, C will C and D are incorrect since there is ver-
not give Bo what he wants. A is incor- tical privity (choice C) and the covenant
rect since there could not be an ease- does touch and concern the land (choice
ment by prescription here. D is possible, D).
but a license is normally revocable by
the licensor. Thus, Bo's best argument
is B.

PROPERTY 459
98. D is the correct answer. This is a suit 100. C is the correct answer. A tenant who
for specific enforcement, in equity. wrongfully holds over on her lease is
Therefore, the question is whether the a tenant at sufferance until such time
promise can be enforced as an equitable as the landlord chooses to treat her as
servitude. This servitude will run be- a trespasser and evict her, or as a ten-
cause the elements of intent, notice and ant for a new term. Since Tilly legally
touch and concern (see above) are all entered possession of Brownacres
met. Of all the answers, only D can be initially, she is not a trespasser; D
correct. A is wrong because horizontal is incorrect. Until Tilly pays a new
privity is not required for the servitude month's rent and Larry accepts that
to run. B and C are wrong since those rent, she cannot be a periodic tenant;
elements are present here. D is correct A is incorrect. B is also incorrect since,
because should notice be insufficient, unless there is an express agreement
the servitude cannot run. between the landlord and the tenant
to fix the ending date of the new term
99. B is the correct answer. An equitable of Tilly's holdover tenancy, she cannot
servitude needs neither horizontal priv- be a tenant for years.
ity (choice A), nor a writing (choice C),
to run with the land. Moreover, since 101. D is the correct answer. On default of a
there was an actual promise made in deed of trust the lender/beneficiary has
1970 binding Minny's property, there is the right to sell the property at either
no need to imply one via a negative re- a judicial or private sale. A mortgage
ciprocal easement, and no need to have permits only a judicial sale. Therefore,
a common plan or scheme for develop- D is correct, and A, B, and C are incor-
ment of all lots (choice D). Therefore, rect.
A, C and D are all incorrect. However,
since there is no other possibility of
gaining notice of this restriction through
actual notice (personal knowledge) or
constructive notice (recording), only
if the character of the property is such
that Otto should have inquired as to the
single-family restriction on Minny's
former land, would there be sufficient
notice to make the servitude run with
the land. Therefore, B is an accurate
statement.

460 PROPERTY
102. B is the correct answer. In a mortgage, 104. D is the correct answer. Ellen is the
the title to the property transfers im- lender/mortgagee in this transaction.
mediately to the mortgagor, thus C is When she transfers her interest in the
incorrect. In a deed of trust, the legal mortgage to Frank, he steps into her
title would be vested in a third party shoes. The phrases "subject to" or "as-
(trustee) by the debtor (Bernice) for suming" a mortgage have to do with
the benefit of the lender (Sarah). No the debtor's/mortgagor's interest. A, B
such arrangement exists here, so A is and C are incorrect.
incorrect. D is also incorrect, since
neither a present transfer of leasehold 105. C is the correct answer. When a person
nor a future interest in a fee simple ap- takes property and assumes a mort-
pears contemplated. Rather, the parties gage, he or she becomes personally
entered into a contractual arrangement liable on the debt. The original debtor
leading to transfer of title. The agree- is not released from personal liability
ment states that payments will be unless there has been a novation. Thus,
made in installments with title passing A, B and D are incorrect.
to Bernice when the full price plus
interest is paid. That arrangement is 106. B is the correct answer. When a person
characteristic of an installment sale. takes property and assumes a mort-
gage, he or she becomes personally li-
103. A is the correct answer. The general able on the debt. The original debtor is
rule is that when property burdened released from personal liability when
with a mortgage is sold, the property there has been a novation (release from
is transferred "subject to" the mort- liability). Thus, A, C and D are incor-
gage. Therefore, Aaron is still person- rect.
ally liable to Barry for the debt as the
original mortgagor. The transfer does 107. A is the correct answer. Even though
not release him from liability. There- Della did not assume the mortgage
fore, A is correct and D is not. Carla is (and is therefore, not personally li-
not personally liable on the mortgage able on the debt), she still took land
unless she "assumes" the mortgage, a burdened with the mortgage. Con-
fact not present here. Thus, B and C sequently, if the original mortgagors
are incorrect. default, the land can be sold out from
under Della. B is incorrect, Della's
consent is irrelevant. The fact that
Della paid more than the amount of
the outstanding debt to the original
mortgagors is similarly irrelevant to
this question. C and D are incorrect.

PROPERTY 461
108. C is the correct answer. On sale, the 111. A is the correct answer. After a fore-
proceeds are distributed according to closure sale by a senior interest, any
the priority of the mortgages with se- properly noticed junior interests are
nior interests paid before junior ones. extinguished. The property can be sold
Any excess is given to the mortgagor. free and clear. The junior mortgagee
Thus, $80,000 to Orville, $35,000 to has a personal right to recover against
Paula, and $5,000 to Nancy. A, B and the mortgagor. Thus, B, C and D are
D are incorrect. incorrect.

109. D is the correct answer. On sale, the 112. B is the correct answer. A foreclosure
proceeds are distributed according sale by a junior interest cannot affect
to the priority of the mortgages with the rights of a senior interest. The
senior interests paid before junior property can be sold, but it is taken by
ones. If there is a shortfall, the senior the buyer subject to the senior interest.
claims are paid in full before the junior Thus, A and D are incorrect. The buyer
who may have a right to go against the is under no obligation to pay that debt
mortgagor personally. C is incorrect. A of the original mortgagor. Thus, C is
and B divide the proceeds among the incorrect.
various claimants proportionally. That
is not correct. 113. C is the correct answer. A foreclosure
sale by a junior interest cannot affect
110. C is the correct answer. If a junior inter- the rights of a senior interest. The
est is not given notice at a foreclosure property can be sold, but it is taken by
sale, that interest is not extinguished. the buyer subject to the senior interest.
The property is taken by the buyer Therefore, the proceeds are paid to the
subject to the mortgage. A and B are junior claimant who foreclosed, with
incorrect. However, despite no notice, the balance given to the mortgagor.
on sale, the proceeds are distributed ac- The senior's interests are still secured
cording to the priority of the mortgages by the mortgage on the property and
with senior interests paid before junior the personal debt of the mortgagor.
ones. If there is a shortfall, the senior Thus, A, B and D are incorrect.
claims are paid in full before the junior
who may have a right to go against the 114. A is the correct answer. If a mortgage is
mortgagor personally. D is incorrect. a recordable interest in land and if it is
not duly recorded, it is void as against a
subsequent BFP. Answers, B, C and D
assume that Gus's mortgage survives
as against Howard. It does not. They
are incorrect.

462 PROPERTY
115. B is the correct answer. If a mortgage 118. D is the correct answer. Umberto's
is not a recordable interest in land then mortgage, although recorded, will not
priority cannot be determined by the be found by a proper search of the
recording act, but by the common law records because the Samantha-Ted
rule of first in time, first in right. Kurt mortgage was not recorded. Thus,
takes the land, subject to Mel's interest. between Umberto and Velma, Velma
A is incorrect. C and D are incorrect has priority. C is incorrect because he
since the covenants of seisin or war- needed to be within the chain of title to
ranty concern defects in title, and a claim recording act priority. A is incor-
mortgage is an encumbrance/lien. rect, because, although he is the senior
interest, he lost his seniority because
116. B is the correct answer. A mortgage is, of Ted's non-recordation. B is the right
in most states, considered to be a lien, conclusion, for the wrong reason.
not affecting title to property. Since
the covenant of quiet use and enjoy- 119. B is the correct answer. This is an af-
ment is a future covenant, it required ter-acquired title problem/estoppel by
a dispossession before breach. That deed issue. Since the Al-Betty deed is
has not occurred on these facts. A is wild, Chuck, as a BFP, should be the
incorrect. C and D are incorrect since person with first priority in Peachacre.
the covenants of seisin or warranty Therefore, title is in Betty, subject to
concern defects in title, and a mortgage Chuck's mortgage. Therefore, B is cor-
is an encumbrance/lien. rect and A is not. As between Al and
Betty, Betty should prevail, therefore,
117. C is the correct answer. Recording, title cannot go to Al. C and D are incor-
or lack of it is only used to determine rect.
priority among claimants to the same
property. As between the transacting 120. D is the correct answer. Although
parties it is not relevant. A and D are Felicia is the junior mortgagee, she is
incorrect. Since Ruby assumed the the only one to record. Recording then
mortgage, she agreed to be personally determines priority, not the common
bound by the debt. Pete is not released law first in time, first in right. Under a
from liability by her assumption. Thus, race statute, she is the first to record.
B is incorrect. Under a race-notice statute, she is the
first to record without notice. Under a
notice statute she is the last person to
take without notice. Therefore, A, B,
and C are incomplete/incorrect.

PROPERTY 463
121. B is the correct answer. The majority 124. A is the correct answer. If a junior inter-
rule is that the grant of a mortgage est is not given notice at a foreclosure
by one joint tenant does not sever the sale, that interest is not extinguished.
joint tenancy, it merely places a lien The property is taken by the buyer
on it to the extent of the joint tenant's subject to the deed of trust. B and D are
share. Therefore, when Gwen died, her incorrect. However, despite no notice,
undivided 1/2 passed to Hilda by right on sale, the proceeds are distributed ac-
of survivorship — subject to Jeremy's cording to the priority of the deeds of
mortgage. A is incorrect. Gwen's inter- trust with senior interests paid before
est could not pass to Kelly. C and D junior ones. If there is a shortfall, the
are incorrect. senior claims are paid in full before
the junior who may have a right to
122. D is the correct answer. In a title theory go against the trustor personally. C is
state, the conveyance of a mortgage af- incorrect.
fects title and severs the joint tenancy.
Therefore, when Lance sells his share, 125. C is the correct answer. If a junior inter-
it is severed; when Mike mortgages est is not given notice at a foreclosure
his share, it is severed; and when Nor- sale, that interest is not extinguished.
ris dies, he has a tenancy in common The property is taken by the buyer
share, so it passes to Quigley. A, B and subject to the mortgage. B is incor-
C are incorrect. rect. Although Otto may not have had
actual notice of Nash's mortgage, he
123. A is the correct answer. The facts do had constructive notice via the record-
not state which concurrent estate Re- ing act. A and D are incorrect.
nee and Sal have. The law modernly
presumes that a tenancy in common
was created. Therefore, Renee can
mortgage her interest and on her death
it goes to her heirs subject to the mort-
ikliilLatiall§....-
gage. B, C and D are incorrect.

464 PROPERTY
MULTISTATE
EXAMINATION WORKBOOK
VOLUME II

The Multistate Workbook IS DESIGNED FOR ALL MULTISTATE EXAMINATIONS

Fleming's Fundamentals Multistate Examination Workbook is a complete self instruction resource


book that permits you to effectively prepare for the Multistate Examination at your own pace.

WHY YOU NEED The Multistate Examination. Workbook

With Fleming's Fundamentals Multistate Examination Workbook, you will learn the proper test-
taking techniques and methodology that you must implement when analyzing this type of testing
format. The methodology discussed herein can be applied to any multiple choice testing format,
both in law school and for the Bar Examination.

WHAT YOU RECEIVE WITH The Multistate Examination Workbook

♦ The "Rules Of The Road," and introduction to the Multistate Method that explains
examination techniques with specific examples and illustrations describing each "rule"

♦ 100 Questions and explanatory answers in Civil Procedure

♦ 101 Questions and explanatory answers in Constitutional Law

♦ 100 Questions and explanatory answers in Criminal Procedure

♦ 110 Questions and explanatory answers in Evidence

♦ 125 Questions and explanatory answers in Real Property

•••••••••••••••• •♦ LEARN TO
WRITE .
THE RIGHT WAY®
♦••••••••••••••♦

You might also like